Location via proxy:   [ UP ]  
[Report a bug]   [Manage cookies]                

Financial Management Brigham 13th Edition

FREQUENTLY USED SYMBOLS ACP ADR APR AR b bL bU BEP BVPS CAPM CCC CF CFPS CR CV Δ Dps Dt DCF D/E DPS DRIP DRP DSO EAR EBIT EBITDA EPS EVA F FCF FVN FVAN g I I/YR INT IP IPO IRR LP M M/B MIRR MRP MVA n N N(di) NOPAT NOWC NPV P Pc Average collection period American Depository Receipt Annual percentage rate Accounts receivable Beta coefficient, a measure of an asset’s market risk Levered beta Unlevered beta Basic earning power Book value per share Capital Asset Pricing Model Cash conversion cycle Cash flow; CFt is the cash flow in Period t Cash flow per share Conversion ratio Coefficient of variation Difference, or change (uppercase delta) Dividend of preferred stock Dividend in Period t Discounted cash flow Debt-to-equity ratio Dividends per share Dividend reinvestment plan Default risk premium Days sales outstanding Effective annual rate, EFF% Earnings before interest and taxes; net operating income Earnings before interest, taxes, depreciation, and amortization Earnings per share Economic Value Added (1) Fixed operating costs (2) Flotation cost Free cash flow Future value for Year N Future value of an annuity for N years Growth rate in earnings, dividends, and stock prices Interest rate; also denoted by r Interest rate key on some calculators Interest payment in dollars Inflation premium Initial public offering Internal rate of return Liquidity premium (1) Maturity value of a bond (2) Margin (profit margin) Market-to-book ratio Modified Internal Rate of Return Maturity risk premium Market Value Added Number of shares outstanding Calculator key denoting number of periods Represents area under a standard normal distribution function Net operating profit after taxes Net operating working capital Net present value (1) Price of a share of stock in Period t; P0 = price of the stock today (2) Sales price per unit of product sold Conversion price Pf Ph PN P/E PMT PPP PV PVAN Q QBE r r̄ ^r r* rd re rf rh ri rM rNOM rps rPER rRF rs ρ ROA ROE RP RPM RR S SML ∑ σ σ2 t T TVN TIE V VB VL Vop Vps VU VC w wd wps ws wce WACC X YTC YTM Price of good in foreign country Price of good in home country A stock’s horizon, or terminal, value Price/earnings ratio Payment of an annuity Purchasing power parity Present value Present value of an annuity for N years Quantity produced or sold Breakeven quantity (1) A percentage discount rate, or cost of capital; also denoted by i (2) Nominal risk-adjusted required rate of return “r bar,” historic, or realized, rate of return “r hat,” an expected rate of return Real risk-free rate of return Before-tax cost of debt Cost of new common stock (outside equity) Interest rate in foreign country Interest rate in home country Required return for an individual firm or security Return for “the market” or for an “average” stock Nominal rate of interest; also denoted by iNOM (1) Cost of preferred stock (2) Portfolio’s return Periodic rate of return Rate of return on a risk-free security (1) Required return on common stock (2) Cost of old common stock (inside equity) Correlation coefficient (lowercase rho); also denoted by R when using historical data Return on assets Return on equity Risk premium Market risk premium Retention rate (1) Sales (2) Estimated standard deviation for sample data (3) Intrinsic value of stock (i.e., all common equity) Security Market Line Summation sign (uppercase sigma) Standard deviation (lowercase sigma) Variance Time period Marginal income tax rate A stock’s horizon, or terminal, value Times interest earned Variable cost per unit Bond value Total market value of a levered firm Value of operations Value of preferred stock Total market value of an unlevered firm Total variable costs Proportion or weight Weight of debt Weight of preferred stock Weight of common equity raised internally by retaining earnings Weight of common equity raised externally by issuing stock Weighted averaged cost of capital Exercise price of option Yield to call Yield to maturity Financial Management: Theory and Practice THIRTEENTH EDITION MICHAEL C. EHRHARDT University of Tennessee EUGENE F. BRIGHAM University of Florida Australia • Brazil • Japan • Korea • Mexico • Singapore • Spain • United Kingdom • United States Financial Management: Theory and Practice, Thirteen Edition Michael C. Ehrhardt and Eugene F. Brigham VP/Editorial Director: Jack W. Calhoun Publisher: Joe Sabatino Executive Editor: Mike Reynolds Developmental Editor: Michael Guendelsberger Senior Editorial Assistant: Adele Scholtz Marketing Manager: Nathan Anderson Marketing Coordinator: Suellen Ruttkay Content Project Manager: Jacquelyn K Featherly Technology Production Analyst: Starratt Alexander Senior Manufacturing Coordinator: Kevin Kluck Production House/Compositor: Integra Software Services Pvt. Ltd. Senior Art Director: Michelle Kunkler Cover and Internal Designer: Rokusek Design Cover Images: © Lael Henderson/Stock Illustration Source/Getty Images, Inc. ©2011, 2009 South-Western, a part of Cengage Learning ALL RIGHTS RESERVED. No part of this work covered by the copyright herein may be reproduced, transmitted, stored or used in any form or by any means graphic, electronic, or mechanical, including but not limited to photocopying, recording, scanning, digitizing, taping, Web distribution, information networks, or information storage and retrieval systems, except as permitted under Section 107 or 108 of the 1976 United States Copyright Act, without the prior written permission of the publisher. For product information and technology assistance, contact us at Cengage Learning Customer & Sales Support, 1-800-354-9706 For permission to use material from this text or product, submit all requests online at www.cengage.com/permissions Further permissions questions can be emailed to permissionrequest@cengage.com ExamView® and ExamView Pro® are registered trademarks of FSCreations, Inc. Windows is a registered trademark of the Microsoft Corporation used herein under license. Macintosh and Power Macintosh are registered trademarks of Apple Computer, Inc. used herein under license. Library of Congress Control Number: 2010920554 Student Edition ISBN 13: 978-1-4390-7810-5 Student Edition ISBN 10: 1-4390-7810-6 Package Student Edition ISBN 13: 978-1-4390-7809-9 Package Student Edition ISBN 10: 1-4390-7809-2 South-Western Cengage Learning 5191 Natorp Boulevard Mason, OH 45040 USA Cengage Learning products are represented in Canada by Nelson Education, Ltd. For your course and learning solutions, visit www.cengage.com Purchase any of our products at your local college store or at our preferred online store www.CengageBrain.com Printed in the United States of America 1 2 3 4 5 6 7 14 13 12 11 10 Brief Contents Preface PART 1 xix Fundamental Concepts of Corporate Finance 1 CHAPTER 1 An Overview of Financial Management and the Financial Environment 3 Web Extensions PART 3 Web Extensions 1B: A Closer Look at the Stock Markets Web Extensions 2A: The Federal Income Tax System for Individuals CHAPTER 3 Analysis of Financial Statements 87 PART 2 123 CHAPTER 7 Stocks, Stock Valuation, and Stock Market Equilibrium 267 7A: Derivation of Valuation Equations CHAPTER 8 Financial Options and Applications in Corporate Finance 305 Web Extensions PART 4 4A: The Tabular Approach 4B: Derivation of Annuity Formulas 4C: Continuous Compounding CHAPTER 5 Bonds, Bond Valuation, and Interest Rates 173 Web Extensions 6A: Continuous Probability Distributions 6B: Estimating Beta with a Financial Calculator Fixed Income Securities 121 CHAPTER 4 Time Value of Money Web Extensions 215 CHAPTER 6 Risk, Return, and the Capital Asset Pricing Model 217 1A: An Overview of Derivatives CHAPTER 2 Financial Statements, Cash Flow, and Taxes 47 Stocks and Options 5A: A Closer Look at Zero Coupon Bonds 5B: A Closer Look at TIPS: Treasury Inflation-Protected Securities 5C: A Closer Look at Bond Risk: Duration 5D: The Pure Expectations Theory and Estimation of Forward Rates Projects and Their Valuation 333 CHAPTER 9 The Cost of Capital 335 9A: The Required Return Assuming Nonconstant Dividends and Stock Repurchases CHAPTER 10 The Basics of Capital Budgeting: Evaluating Cash Flows 379 Web Extensions 10A: The Accounting Rate of Return (ARR) CHAPTER 11 Cash Flow Estimation and Risk Analysis 423 Web Extensions Web Extensions 11A: Certainty Equivalents and Risk-Adjusted Discount Rates iii iv Brief Contents PART 5 Corporate Valuation and Governance 471 CHAPTER 12 Financial Planning and Forecasting Financial Statements 473 12A: Advanced Techniques for Forecasting Financial Statements Accounts CHAPTER 13 Corporate Valuation, Value-Based Management and Corporate Governance 511 Web Extensions PART 6 Cash Distributions and Capital Structure 557 CHAPTER 14 Distributions to Shareholders: Dividends and Repurchases 559 CHAPTER 15 Capital Structure Decisions 599 Web Extensions PART 7 15A: Degree of Leverage Managing Global Operations 639 CHAPTER 19 Hybrid Financing: Preferred Stock, Warrants, and Convertibles 759 Web Extensions CHAPTER 20 Initial Public Offerings, Investment Banking, and Financial Restructuring 787 Web Extensions 20A: Rights Offerings PART 9 Special Topics 825 CHAPTER 21 Mergers, LBOs, Divestitures, and Holding Companies 827 Web Extensions 21A: Projecting Consistent Debt and Interest Expenses CHAPTER 22 Bankruptcy, Reorganization, and Liquidation 869 Web Extensions 22A: Multiple Discriminant Analysis CHAPTER 23 Derivatives and Risk Management 899 Web Extensions CHAPTER 16 Working Capital Management 641 16A: Secured Short-Term Financing CHAPTER 17 Multinational Financial Management 691 19A: Calling Convertible Issues 23A: Risk Management with Insurance Web Extensions PART 8 Tactical Financing Decisions 731 CHAPTER 18 Lease Financing Web Extensions 733 18A: Leasing Feedback 18B: Percentage Cost Analysis 18C: Leveraged Leases PART 10 Advanced Issues 929 CHAPTER 24 Portfolio Theory, Asset Pricing Models, and Behavioral Finance 931 CHAPTER 25 Real Options Web Extensions 971 25A: The Abandonment Real Option 25B: Risk-Neutral Valuation CHAPTER 26 Analysis of Capital Structure Theory 995 Brief Contents Appendixes Appendix A Solutions to Self-Test Problems 1029 Appendix B Answers to End-of-Chapter Problems 1063 Appendix C Selected Equations and Data 1071 Appendix D Values of the Areas under the Standard Normal Distribution Function 1085 Glossary 1087 Name Index 1113 Subject Index 1119 Web Chapters CHAPTER 27 Providing and Obtaining Credit CHAPTER 28 Advanced Issues in Cash Management and Inventory Control CHAPTER 29 Pension Plan Management CHAPTER 30 Financial Management in Not-for-Profit Businesses v Contents Preface . . . . . . . . . . . . . . . . . . . . . . . . . . . . . . . . . . . . . . . . . . . . . . . . . . . . . . . . . . . . . . . . . xix PART 1 Fundamental Concepts of Corporate Finance 1 CHAPTER 1 An Overview of Financial Management and the Financial Environment The Five-Minute MBA 4 5 Box: Say Hello to the Global Economic Crisis! The Corporate Life Cycle 5 Box: Columbus Was Wrong—the World Is Flat! And Hot, and Crowded! The Primary Objective of the Corporation: Value Maximization Box: Ethics for Individuals and Businesses 10 Box: Corporate Scandals and Maximizing Stock Price 13 An Overview of the Capital Allocation Process Financial Securities 15 The Cost of Money 19 Financial Institutions Financial Markets 23 27 Trading Procedures in Financial Markets Types of Stock Market Transactions Box: Rational Exuberance? Box: Measuring the Market Stock Market Returns Summary 31 33 34 The Global Economic Crisis e-Resources 29 30 31 The Secondary Stock Markets The Big Picture 13 36 42 43 44 Web Extensions 1A: An Overview of Derivatives 1B: A Closer Look at the Stock Markets CHAPTER 2 Financial Statements, Cash Flow, and Taxes 47 Box: Intrinsic Value, Free Cash Flow, and Financial Statements Financial Statements and Reports The Balance Sheet 49 Box: Let’s Play Hide-and-Seek! vi 51 48 48 9 6 3 Contents The Income Statement 52 Statement of Stockholders’ Equity Net Cash Flow 53 54 Statement of Cash Flows 55 Box: Financial Analysis on the WEB 56 Modifying Accounting Data for Managerial Decisions MVA and EVA 67 Box: Sarbanes-Oxley and Financial Fraud The Federal Income Tax System Summary 59 63 Box: Financial Bamboozling: How to Spot It 70 71 76 Web Extensions 2A: The Federal Income Tax System for Individuals CHAPTER 3 Analysis of Financial Statements 87 88 Box: Intrinsic Value and Analysis of Financial Statements Financial Analysis 88 Liquidity Ratios 89 Asset Management Ratios 92 Box: The Price is Right! (Or Wrong!) Debt Management Ratios Profitability Ratios 93 95 98 Box: The World Might be Flat, but Global Accounting is Bumpy! The Case of IFRS versus FASB 99 Market Value Ratios 100 Trend Analysis, Common Size Analysis, and Percentage Change Analysis Tying the Ratios Together: The Du Pont Equation Comparative Ratios and Benchmarking 107 Uses and Limitations of Ratio Analysis 108 Box: Ratio Analysis on the Web Looking beyond the Numbers Summary 106 109 110 110 PART 2 Fixed Income Securities 121 CHAPTER 4 Time Value of Money 123 Box: Corporate Valuation and the Time Value of Money 124 Time Lines 125 Future Values 125 Box: Hints on Using Financial Calculators 129 Box: The Power of Compound Interest 132 Present Values 133 102 vii viii Contents Finding the Interest Rate, I 136 Finding the Number of Years, N Annuities 137 138 Future Value of an Ordinary Annuity Future Value of an Annuity Due 138 141 Present Value of Ordinary Annuities and Annuities Due Finding Annuity Payments, Periods, and Interest Rates Perpetuities 141 144 Box: Variable Annuities: Good or Bad? 144 146 Box: Using the Internet for Personal Financial Planning Uneven, or Irregular, Cash Flows 147 148 Future Value of an Uneven Cash Flow Stream Solving for I with Irregular Cash Flows 151 152 Semiannual and Other Compounding Periods 153 Box: Truth in Lending: What Loans Really Cost 156 Fractional Time Periods Amortized Loans 157 158 Growing Annuities 159 Box: An Accident Waiting to Happen: Option Reset Adjustable Rate Mortgages 160 Summary 162 Web Extensions 4A: The Tabular Approach 4B: Derivation of Annuity Formulas 4C: Continuous Compounding CHAPTER 5 Bonds, Bond Valuation, and Interest Rates Box: Intrinsic Value and the Cost of Debt Who Issues Bonds? 173 174 174 Key Characteristics of Bonds 175 Box: Betting With or Against the U.S. Government: The Case of Treasury Bond Credit Default Swaps 176 Bond Valuation 180 Changes in Bond Values over Time Box: Drinking Your Coupons Bonds with Semiannual Coupons Bond Yields 184 187 187 188 The Pre-Tax Cost of Debt: Determinants of Market Interest Rates The Real Risk-Free Rate of Interest, r* The Inflation Premium (IP) 192 193 The Nominal, or Quoted, Risk-Free Rate of Interest, rRF The Default Risk Premium (DRP) 195 195 Box: Insuring with Credit Default Swaps: Let the Buyer Beware! 197 191 Contents Box: Might the U.S. Treasury Bond Be Downgraded? Box: Are Investors Rational? 201 The Maturity Risk Premium (MRP) 201 The Term Structure of Interest Rates 204 205 Bankruptcy and Reorganization Summary 199 201 The Liquidity Premium (LP) Financing with Junk Bonds ix 206 207 Web Extensions 5A: A Closer Look at Zero Coupon Bonds 5B: A Closer Look at TIPS: Treasury Inflation-Protected Securities 5C: A Closer Look at Bond Risk: Duration 5D: The Pure Expectations Theory and Estimation of Forward Rates PART 3 Stocks and Options 215 CHAPTER 6 Risk, Return, and the Capital Asset Pricing Model Box: Intrinsic Value, Risk, and Return Returns on Investments Stand-Alone Risk 217 219 219 220 Box: What Does Risk Really Mean? 227 229 Box: The Trade-off between Risk and Return Risk in a Portfolio Context 231 236 Box: How Risky Is a Large Portfolio of Stocks? Box: The Benefits of Diversifying Overseas Calculating Beta Coefficients 239 243 The Relationship between Risk and Return 246 Box: Another Kind of Risk: The Bernie Madoff Story Some Concerns about Beta and the CAPM 252 253 Some Concluding Thoughts: Implications for Corporate Managers and Investors Summary 255 Web Extensions 6A: Continuous Probability Distributions 6B: Estimating Beta with a Financial Calculator CHAPTER 7 Stocks, Stock Valuation, and Stock Market Equilibrium Box: Corporate Valuation and Stock Prices 268 Legal Rights and Privileges of Common Stockholders Types of Common Stock 269 The Market Stock Price versus Intrinsic Value Stock Market Reporting 272 270 268 267 253 x Contents Valuing Common Stocks 273 Valuing a Constant Growth Stock 276 Expected Rate of Return on a Constant Growth Stock Valuing Nonconstant Growth Stocks 281 Stock Valuation by the Free Cash Flow Approach Market Multiple Analysis Preferred Stock 279 285 285 286 Stock Market Equilibrium 287 The Efficient Markets Hypothesis 290 Box: Rational Behavior versus Animal Spirits, Herding, and Anchoring Bias 293 Summary 294 Web Extensions 7A: Derivation of Valuation Equations CHAPTER 8 Financial Options and Applications in Corporate Finance Box: The Intrinsic Value of Stock Options Overview of Financial Options 305 306 306 Box: Financial Reporting for Employee Stock Options 309 The Single-Period Binomial Option Pricing Approach 310 The Single-Period Binomial Option Pricing Formula The Multi-Period Binomial Option Pricing Model The Black-Scholes Option Pricing Model (OPM) Box: Taxes and Stock Options The Valuation of Put Options 314 316 319 324 325 Applications of Option Pricing in Corporate Finance Summary 326 328 PART 4 Projects and Their Valuation 333 CHAPTER 9 The Cost of Capital 335 Box: Corporate Valuation and the Cost of Capital The Weighted Average Cost of Capital Basic Definitions 336 337 338 Cost of Debt, rd(1 − T) 340 Cost of Preferred Stock, rps 342 Box: GE and Warren Buffett: The Cost of Preferred Stock Cost of Common Stock, rs The CAPM Approach 343 344 345 Dividend-Yield-Plus-Growth-Rate, or Discounted Cash Flow (DCF), Approach Over-Own-Bond-Yield-Plus-Judgmental-Risk-Premium Approach 353 355 Comparison of the CAPM, DCF, and Over-Own-Bond-Yield-Plus-Judgmental-RiskPremium Methods 356 Contents Adjusting the Cost of Equity for Flotation Costs 357 Composite, or Weighted Average, Cost of Capital, WACC 358 Box: Global Variations in the Cost of Capital 361 Factors That Affect the WACC 361 Adjusting the Cost of Capital for Risk 363 Privately Owned Firms and Small Businesses Four Mistakes to Avoid Summary 366 367 368 Web Extensions 9A: The Required Return Assuming Nonconstant Dividends and Stock Repurchases CHAPTER 10 The Basics of Capital Budgeting: Evaluating Cash Flows 381 Box: Corporate Valuation and Capital Budgeting An Overview of Capital Budgeting Net Present Value (NPV) 381 383 Internal Rate of Return (IRR) 387 Box: Why NPV Is Better Than IRR Multiple Internal Rates of Return Reinvestment Rate Assumptions 389 390 392 Modified Internal Rate of Return (MIRR) NPV Profiles 396 Profitability Index (PI) Payback Period 400 401 Conclusions on Capital Budgeting Methods Decision Criteria Used in Practice 405 Other Issues in Capital Budgeting 405 Summary 393 403 411 Web Extensions 10A: The Accounting Rate of Return (ARR) CHAPTER 11 Cash Flow Estimation and Risk Analysis 423 Box: Corporate Valuation, Cash Flows, and Risk Analysis Conceptual Issues 424 Analysis of an Expansion Project Risk Analysis in Capital Budgeting Measuring Stand-Alone Risk Sensitivity Analysis Scenario Analysis 424 429 435 436 436 439 Monte Carlo Simulation 442 Box: Are Bank Stress Tests Stressful Enough? 445 Project Risk Conclusions 446 Box: Capital Budgeting Practices in the Asian/Pacific Region 447 379 xi xii Contents Replacement Analysis 448 Real Options 449 Phased Decisions and Decision Trees 451 Summary 454 Appendix 11A Tax Depreciation 468 Web Extensions 11A: Certainty Equivalents and Risk-Adjusted Discount Rates PART 5 Corporate Valuation and Governance 471 CHAPTER 12 Financial Planning and Forecasting Financial Statements Box: Corporate Valuation and Financial Planning Overview of Financial Planning 474 Sales Forecast 476 Additional Funds Needed (AFN) Method 473 474 478 Forecasted Financial Statements Method 482 Forecasting When the Ratios Change 496 Summary 499 Web Extensions 12A: Advanced Techniques for Forecasting Financial Statements Accounts CHAPTER 13 Corporate Valuation, Value-Based Management and Corporate Governance 511 Box: Corporate Valuation: Putting the Pieces Together Overview of Corporate Valuation The Corporate Valuation Model Value-Based Management 513 514 521 Managerial Behavior and Shareholder Wealth Corporate Governance 512 530 531 Box: Let’s Go to Miami! IBM’s 2009 Annual Meeting 533 Box: Would the U.S. Government Be an Effective Board Director? Box: Shareholder Reactions to the Crisis 538 Box: The Sarbanes-Oxley Act of 2002 and Corporate Governance Box: International Corporate Governance 540 542 Employee Stock Ownership Plans (ESOPs) Summary 536 543 546 PART 6 Cash Distributions and Capital Structure 557 CHAPTER 14 Distributions to Shareholders: Dividends and Repurchases Box: Uses of Free Cash Flow: Distributions to Shareholders An Overview of Cash Distributions 560 560 559 Contents Procedures for Cash Distributions 562 Cash Distributions and Firm Value Clientele Effect 564 567 Information Content, or Signaling, Hypothesis Implications for Dividend Stability 568 569 Box: Will Dividends Ever Be the Same? 570 Setting the Target Distribution Level: The Residual Distribution Model The Residual Distribution Model in Practice 572 A Tale of Two Cash Distributions: Dividends versus Stock Repurchases The Pros and Cons of Dividends and Repurchases Box: Dividend Yields around the World 584 Other Factors Influencing Distributions 584 Summarizing the Distribution Policy Decision Stock Splits and Stock Dividends 587 Dividend Reinvestment Plans Summary 585 590 591 CHAPTER 15 Capital Structure Decisions 599 Box: Corporate Valuation and Capital Structure A Preview of Capital Structure Issues Business Risk and Financial Risk Capital Structure Theory 600 609 611 Capital Structure Evidence and Implications 618 Box: Taking a Look at Global Capital Structures Estimating the Optimal Capital Structure Anatomy of a Recapitalization Box: Deleveraging 600 603 Box: Yogi Berra on the MM Proposition Summary 582 588 Box: Talk about a Split Personality! 620 621 625 630 630 Web Extensions 15A: Degree of Leverage PART 7 Managing Global Operations 639 CHAPTER 16 Working Capital Management 641 Box: Corporate Valuation and Working Capital Management Current Asset Holdings 642 643 Current Assets Financing Policies The Cash Conversion Cycle 644 648 Box: Some Firms Operate with Negative Working Capital! 653 The Cash Budget 654 570 573 xiii xiv Contents Cash Management and the Target Cash Balance Box: The CFO Cash Management Scorecard Cash Management Techniques Inventory Management 658 659 661 662 Box: Supply Chain Management Receivables Management 663 665 Box: Supply Chain Finance Accruals and Accounts Payable (Trade Credit) Short-Term Marketable Securities Short-Term Financing 670 672 676 Use of Security in Short-Term Financing Summary 667 672 Short-Term Bank Loans Commercial Paper 657 677 678 Web Extensions 16A: Secured Short-Term Financing CHAPTER 17 Multinational Financial Management 691 Box: Corporate Valuation in a Global Context 692 Multinational, or Global, Corporations 692 Multinational versus Domestic Financial Management Exchange Rates 693 694 Exchange Rates and International Trade 698 The International Monetary System and Exchange Rate Policies Trading in Foreign Exchange Interest Rate Parity 703 704 Purchasing Power Parity 706 708 Box: Hungry for a Big Mac? Go To Malaysia! Inflation, Interest Rates, and Exchange Rates International Money and Capital Markets 709 710 Box: Greasing the Wheels of International Business Box: Stock Market Indices around the World Multinational Capital Budgeting Box: Consumer Finance in China International Capital Structures 713 714 715 718 Multinational Working Capital Management Summary 711 720 723 PART 8 Tactical Financing Decisions 731 CHAPTER 18 Lease Financing Types of Leases 733 734 699 Contents Tax Effects 736 Financial Statement Effects 738 Box: Off–Balance Sheet Financing: Is It Going to Disappear? Evaluation by the Lessee 740 Evaluation by the Lessor 745 Other Issues in Lease Analysis 747 Box: What You Don’t Know Can Hurt You! 748 750 Box: Lease Securitization Other Reasons for Leasing Summary 740 751 753 Web Extensions 18A: Leasing Feedback 18B: Percentage Cost Analysis 18C: Leveraged Leases CHAPTER 19 Hybrid Financing: Preferred Stock, Warrants, and Convertibles Preferred Stock 761 Box: The Romance Had No Chemistry, But It Had a Lot of Preferred Stock! Warrants 762 765 Convertible Securities 770 A Final Comparison of Warrants and Convertibles 777 Reporting Earnings When Warrants or Convertibles Are Outstanding Summary 759 778 779 Web Extensions 19A: Calling Convertible Issues CHAPTER 20 Initial Public Offerings, Investment Banking, and Financial Restructuring 787 The Financial Life Cycle of a Start-up Company The Decision to Go Public 788 789 The Process of Going Public: An Initial Public Offering Equity Carve-outs: A Special Type of IPO 791 799 Other Ways to Raise Funds in the Capital Markets Box: Bowie Bonds Ch-Ch-Change Asset Securitization 800 803 Investment Banking Activities and Their Role in the Global Economic Crisis Box: Investment Banks and the Global Economic Crisis The Decision to Go Private 806 Managing the Maturity Structure of Debt Refunding Operations 805 808 810 Box: TVA Ratchets Down Its Interest Expenses 813 Managing the Risk Structure of Debt with Project Financing Summary 817 Web Extensions 20A: Rights Offerings 815 803 xv xvi Contents PART 9 Special Topics 825 CHAPTER 21 Mergers, LBOs, Divestitures, and Holding Companies Rationale for Mergers Types of Mergers 827 828 830 Level of Merger Activity 831 Hostile versus Friendly Takeovers Merger Regulation 832 833 Overview of Merger Analysis 834 The Adjusted Present Value (APV) Approach 835 The Free Cash Flow to Equity (FCFE) Approach 838 Illustration of the Three Valuation Approaches for a Constant Capital Structure Setting the Bid Price 845 Analysis When There Is a Permanent Change in Capital Structure Taxes and the Structure of the Takeover Bid 850 Box: Tempest in a Teapot? Financial Reporting for Mergers 852 Analysis for a “True Consolidation” The Role of Investment Bankers Corporate Alliances Leveraged Buyouts Divestitures 857 858 858 859 859 Holding Companies Summary 855 855 Who Wins: The Empirical Evidence Box: Merger Mistakes 849 860 862 Web Extensions 21A: Projecting Consistent Debt and Interest Expenses CHAPTER 22 Bankruptcy, Reorganization, and Liquidation Financial Distress and Its Consequences 869 870 Issues Facing a Firm in Financial Distress 871 Settlements without Going through Formal Bankruptcy Federal Bankruptcy Law 874 Reorganization in Bankruptcy Liquidation in Bankruptcy Box: A Nation of Defaulters? 875 885 888 Other Motivations for Bankruptcy Some Criticisms of Bankruptcy Laws Summary 890 Web Extensions 22A: Multiple Discriminant Analysis 889 889 872 847 840 Contents CHAPTER 23 Derivatives and Risk Management 899 900 Box: Corporate Valuation and Risk Management Reasons to Manage Risk 901 Background on Derivatives Derivatives in the News 903 904 Other Types of Derivatives 907 Corporate Risk Management 913 Box: Enterprise Risk Management and Value at Risk 916 Using Derivatives to Reduce Risks 917 Box: Risk Management in the Cyber Economy Summary 920 924 Web Extensions 23A: Risk Management with Insurance PART 10 Advanced Issues 929 CHAPTER 24 Portfolio Theory, Asset Pricing Models, and Behavioral Finance 932 Box: Corporate Valuation and Risk Efficient Portfolios 932 Choosing the Optimal Portfolio 936 The Basic Assumptions of the Capital Asset Pricing Model The Capital Market Line and the Security Market Line Calculating Beta Coefficients Box: Skill or Luck? 939 940 944 945 Empirical Tests of the CAPM Arbitrage Pricing Theory 952 954 The Fama-French Three-Factor Model 957 An Alternative Theory of Risk and Return: Behavioral Finance Summary 963 CHAPTER 25 Real Options 971 Valuing Real Options 972 The Investment Timing Option: An Illustration The Growth Option: An Illustration Concluding Thoughts on Real Options Summary 989 Web Extensions 25A: The Abandonment Real Option 25B: Risk-Neutral Valuation 982 986 973 961 931 xvii xviii Contents CHAPTER 26 Analysis of Capital Structure Theory 995 Box: Corporate Valuation and Capital Structure Decisions 996 Capital Structure Theory: Arbitrage Proofs of the Modigliani-Miller Models Introducing Personal Taxes: The Miller Model Criticisms of the MM and Miller Models 1010 An Extension of the MM Model: Nonzero Growth and a Risky Tax Shield Risky Debt and Equity as an Option 1015 Capital Structure Theory: Our View 1019 Summary 996 1006 1011 1021 Appendix Appendix Appendix Appendix A Solutions to Self-Test Problems . . . . . . . . . . . . . . . . . . . . . . . . . . 1029 B Answers to End-of-Chapter Problems . . . . . . . . . . . . . . . . . . . . . 1063 C Selected Equations and Data . . . . . . . . . . . . . . . . . . . . . . . . . . . . . 1071 D Values of the Areas under the Standard Normal Distribution Function . . . . . . . . . . . . . . . . . . . . . . . . . . . . . . . . . . . 1085 Glossary . . . . . . . . . . . . . . . . . . . . . . . . . . . . . . . . . . . . . . . . . . . . . . . . . . . . . . . . . . . 1087 Name Index . . . . . . . . . . . . . . . . . . . . . . . . . . . . . . . . . . . . . . . . . . . . . . . . . . . . . . . . 1113 Subject Index . . . . . . . . . . . . . . . . . . . . . . . . . . . . . . . . . . . . . . . . . . . . . . . . . . . . . . 1119 WEB CHAPTERS CHAPTER 27 Providing and Obtaining Credit CHAPTER 28 Advanced Issues in Cash Management and Inventory Control CHAPTER 29 Pension Plan Management CHAPTER 30 Financial Management in Not-for-Profit Businesses Preface resource Be sure to visit the Financial Management: Theory and Practice (13th Edition) Web site at www.cengage.com/ brigham. This site provides access for instructors and students. When we wrote the first edition of Financial Management: Theory and Practice, we had four goals: (1) to create a text that would help students make better financial decisions; (2) to provide a book that could be used in the introductory MBA course, but one that was complete enough for use as a reference text in follow-on case courses and after graduation; (3) to motivate students by demonstrating that finance is both interesting and relevant; and (4) to make the book clear enough so that students could go through the material without wasting either their time or their professor’s time trying to figure out what we were saying. The collapse of the sub-prime mortgage market, the financial crisis, and the global economic crisis make it more important than ever for students and managers to understand the role that finance plays in a global economy, in their own companies, and in their own lives. So in addition to the four goals listed above, this edition has a fifth goal, to prepare students for a changed world. INTRINSIC VALUATION AS A UNIFYING THEME Our emphasis throughout the book is on the actions that a manager can and should take to increase the intrinsic value of the firm. Structuring the book around intrinsic valuation enhances continuity and helps students see how various topics are related to one another. As its title indicates, this book combines theory and practical applications. An understanding of finance theory is absolutely essential for anyone developing and/or implementing effective financial strategies. But theory alone isn’t sufficient, so we provide numerous examples in the book and the accompanying Excel spreadsheets to illustrate how theory is applied in practice. Indeed, we believe that the ability to analyze financial problems using Excel is absolutely essential for a student’s successful job search and subsequent career. Therefore, many exhibits in the book come directly from the accompanying Excel spreadsheets. Many of the spreadsheets also provide brief “tutorials” by way of detailed comments on Excel features that we have found to be especially useful, such as Goal Seek, Tables, and many financial functions. The book begins with fundamental concepts, including background on the economic and financial environment, financial statements (with an emphasis on cash flows), the time value of money, bond valuation, risk analysis, and stock valuation. With this background, we go on to discuss how specific techniques and decision rules can be used to help maximize the value of the firm. This organization provides four important advantages: 1. Managers should try to maximize the intrinsic value of a firm, which is determined by cash flows as revealed in financial statements. Our early coverage of financial statements thus helps students see how particular financial decisions affect the various parts of the firm and the resulting cash flow. Also, financial statement analysis provides an excellent vehicle for illustrating the usefulness of spreadsheets. 2. Covering time value of money early helps students see how and why expected future cash flows determine the value of the firm. Also, it takes time for students to digest TVM concepts and to learn how to do the required calculations, so it is good to cover TVM concepts early and often. xix xx Preface 3. Most students—even those who do not plan to major in finance—are interested in investments. The ability to learn is a function of individual interest and motivation, so Financial Management’s early coverage of securities and security markets is pedagogically sound. 4. Once basic concepts have been established, it is easier for students to understand both how and why corporations make specific decisions in the areas of capital budgeting, raising capital, working capital management, mergers, and the like. INTENDED MARKET AND USE Financial Management is designed primarily for use in the introductory MBA finance course and as a reference text in follow-on case courses and after graduation. There is enough material for two terms, especially if the book is supplemented with cases and/or selected readings. The book can also be used as an undergraduate introductory text with exceptionally good students, or where the introductory course is taught over two terms. IMPROVEMENTS IN THE 13TH EDITION As in every revision, we updated and clarified materials throughout the text, reviewing the entire book for completeness, ease of exposition, and currency. We made hundreds of small changes to keep the text up-to-date, with particular emphasis on updating the real world examples and including the latest changes in the financial environment and financial theory. In addition, we made a number of larger changes. Some affect all chapters, some involve reorganizing sections among chapters, and some modify material covered within specific chapters. CHANGES THAT AFFECT ALL CHAPTERS Reorganization to better accommodate one-semester and two-semester sequences. Finance is taught as a one-semester course at many schools, so we moved the essential material into the first 17 chapters. The remaining chapters cover additional topics and provide more advanced treatment of the essential material in the first 17 chapters. This makes it easy for a professor teaching a one-semester course to cover the essential materials and then pick and choose from the remaining topics if time permits. If finance is taught in a two-semester sequence, the first semester can focus on the essential materials in the first 17 chapters and the second semester can focus on advanced materials in the remaining chapters, perhaps supplemented with cases. The global economic crisis. In virtually every chapter we use real world examples to show how the chapter’s topics are related to some aspect of the global economic crisis. In addition, many chapters have new “Global Economic Crisis” boxes that focus on particularly important issues related to the crisis. The big picture. Students often fail to see the forest for the trees, and this is especially true in finance because students must learn new vocabularies and analytical tools. To help students understand the big picture and integrate the different parts into an overall framework, we have added a graphic at the beginning of each chapter (and in the PowerPoint shows) that clearly illustrates where the chapter’s topics fit into the big picture. Following is an example from Chapter 9: Preface xxi Determinants of Intrinsic Value: The Weighted Average Cost of Capital Net operating profit after taxes Free cash flow (FCF) Value = Required investments in operating capital − FCF1 + (1 + WACC)1 FCF2 = + …+ (1 + WACC)2 FCF∞ (1 + WACC)∞ Weighted average cost of capital (WACC) Market interest rates Market risk aversion Cost of debt Cost of equity Firm’s debt/equity mix Firm’s business risk Additional integration of the textbook and the accompanying Excel Tool Kit spreadsheet models for each chapter. Many figures in the textbook are actually screen shots from the chapter’s Excel Tool Kit model. This makes the analysis more transparent to the students and better enables them to follow the analysis in the Excel model. Signficant Reorganization of Some Chapters Financial markets and performance measures. Chapter 1 still addresses the financial environment, but now is followed by two chapters focused on measuring the firm’s performance in the financial environment by understanding financial statements, calculating free cash flow, and analyzing ratios. Time value of money and bond valuation. Chapter 4 covers the time value of money and Chapter 5 applies these concepts to bond pricing. Thus, students learn a tool and then immediately use the tool. Dividends and stock repurchases before capital structure decisions. We now cover dividends and stock repurchases in Chapter 14 so that students will already understand stock repurchases when we discuss recapitalizations in Chapter 15. Notable Changes within Selected Chapters We made too many small improvements within each chapter to mention them all, but some of the more notable ones are discussed below. Chapter 1: An Overview of Financial Management and the Financial Environment. We added a new box on globalization, “Columbus Was Wrong— xxii Preface the World Is Flat! And Hot, and Crowded,” and a new box on the global economic crisis, “Say Hello to the Global Economic Crisis!” We completely rewrote the section on financial securities, including a discussion of securitization, and added a new section on the global crisis. New figures showing the national debt, trade balances, federal budget deficits and the Case-Shiller real estate index help us better illustrate different aspects of the global crisis. Chapter 2: Financial Statements, Cash Flow, and Taxes. A new opening vignette shows the cash that several different companies generated and the different ways that they used the cash flow. We added a new box on the global economic crisis that explains the problems associates with off-balance-sheet assets, “Let’s Play Hideand-Seek!” We added a new figure illustrating the uses of free cash flow. We now have two end-of-chapter spreadsheet problems, one focusing on the articulation between the income statement and statement of cash flows, and one focusing on free cash flow. Chapter 3: Analysis of Financial Statements. We added a new box on marking to market, “The Price is Right! (Or Wrong!),” and a new box on international accounting standards, “The World Might be Flat, but Global Accounting is Bumpy! The Case of IFRS versus FASB.” We have included discussion of the price/EBITDA ratio, gross profit margin, and operating profit margin; we also explain how to use the statement of cash flows in financial analysis. Chapter 4: Time Value of Money. We added three new boxes: (1) “Hints on Using Financial Calculators,” (2) “Variable Annuities: Good or Bad?”, and (3) “An Accident Waiting to Happen: Option Reset Adjustable Rate Mortgages.” Chapter 5: Bonds, Bond Valuation, and Interest Rates. We added four new boxes related to the global economic crisis: (1) “Betting With or Against the U.S. Government: The Case of Treasury Bond Credit Default Swaps,” (2) “Insuring with Credit Default Swaps: Let the Buyer Beware!” (3) “Might the U.S. Treasury Bond Be Downgraded?” and (4) “Are Investors Rational?” We also added a new table summarizing corporate bond default rates and annual changes in ratings. Chapter 6: Risk, Return, and the Capital Asset Pricing Model. The new opening vignette discusses the recent stock market and compares the market’s returns to GE’s returns. We added a new box on the risk that remains even for long-term investors, “What Does Risk Really Mean?” We added two additional boxes on risk, “How Risky Is a Large Portfolio of Stocks?” and “Another Kind of Risk: The Bernie Madoff Story.” Chapter 7: Stocks, Stock Valuation, and Stock Market Equilibrium. A new opening vignette discusses buy- and sell-side analysts. We added a new box on behavioral issues, “Rational Behavior vs. Animal Spirits, Herding, and Anchoring Bias.” We added a new section, “The Market Stock Price vs. Intrinsic Value.” Chapter 8: Financial Options and Applications in Corporate Finance. We completely rewrote the description of the binomial option pricing model. In addition to the hedge portfolio, we also discuss replicating portfolios. We now provide the binomial formula and we show the complete solution to the 2-period model. To provide greater continuity, the company used to illustrate the binomial example is now the same company used to illustrate the Black-Scholes model. Our discussion of put options now includes the Black-Scholes put formula. Chapter 9: The Cost of Capital. We added a new figure to highlight the similarities and differences among capital structure weights based on book values, market Preface xxiii values, and target values. We added a new box, “GE and Warren Buffett: The Cost of Preferred Stock.” We completely rewrote our discussion of the market risk premium, which now includes the impact of stock repurchases on estimating the market risk premium. We also present data from surveys identifying the market risk premia used by CFOs and professors. Chapter 10: The Basics of Capital Budgeting: Evaluating Cash Flows. We added a new box, “Why NPV is Better than IRR.” Chapter 11: Cash Flow Estimation and Risk Analysis. We now show how to use tornado diagrams in sensitivity analysis. We rewrote our discussion of Monte Carlo simulation and show how to conduct a simulation analysis without using addins but instead using only Excel’s built-in features (Data Tables and random number generators). We have included an example of replacement analysis and an example of a decision tree showing abandonment. We added a new box, “Are Bank Stress Tests Stressful Enough?” Chapter 12: Financial Planning and Forecasting Financial Statements. It is difficult to do financial planning without using spreadsheet software, so we completely rewrote the chapter and explicitly integrated the text and the Excel Tool Kit model. We illustrate the ways that financial policies (i.e., dividend payout and capital structure choices) affect financial projections, including ways to ensure that balance sheets balance. The Excel Tool Kit model now shows a very simple way to incorporate financing feedback effects. Chapter 13: Corporate Valuation, Value-Based Management, and Corporate Governance. The new opening vignette discusses the role of corporate governance in the global economic crisis. We also added three new boxes. The first describes corporate governance issues at IBM, “Let’s Go to Miami! IBM’s 2009 Annual Meeting.” The second discusses leadership at bailout recipients, “Would the U.S. Government be an Effective Board Director?” The third discusses the 2009 proxy season, “Shareholder Reactions to the Crisis.” Chapter 14: Distributions to Shareholders: Dividends and Repurchases. We consolidated the coverage of stock repurchases that had been spread over two chapters and located it here, which now precedes our discussion of capital structure in Chapter 15. We also use the FCF valuation model to illustrate the different impacts of stock repurchases versus dividend payments. We added two new boxes. The first discusses recent dividend cuts, “Will Dividends Ever Be the Same?” and the second discusses Sun Microsystem’s stock splits and recent reverse split, “Talk About a Split Personality!” Chapter 15: Capital Structure Decisions. The new opening vignette discusses recent bankruptcies and Black & Decker efforts to reduce liquidity risk by refinancing short-term debt with long-term debt. Because the stock repurchases are now covered in the preceding chapter, we were able to improve our discussion of recapitalizations within the context of the FCF valuation model. We added a new box, “Deleveraging” that discusses the changes in leverage many companies and individuals are making in light of the global economic crisis. Chapter 16: Working Capital Management. We reorganized the chapter so that we now discuss working capital holdings and financing before discussing the cash conversion cycle. We rewrote our coverage of the cash conversion cycle to explain the general concepts and then apply them to actual financial statement data. xxiv Preface We added the box “Some Firms Operate with Negative Working Capital!” and a new section on the cost of cost of bank loans. Chapter 17: Multinational Financial Management. We added a new opening vignette on the global economic crisis and its impact on world economies, foreign direct investment, and cross-border M&As. We added two new boxes, the first on regulating international bribery and taxation, “Greasing the Wheels of International Business.” The second new box discusses the wave of foreign companies partnering with Chinese banks to provide consumer finance services, “Consumer Finance in China.” Chapter 18: Lease Financing. The new opening vignette discusses Virgin Atlantic’s order of 10 Airbus jets to be leased from AerCap. A new box addresses the FASB/IASB movement to capitalize all leases, “Off-Balance Sheet Financing: Is it Going to Disappear?” Chapter 19: Hybrid Financing: Preferred Stock, Warrants, and Convertibles. The new opening vignette discusses the Treasury Department’s use of preferred stock and warrants to support troubled companies. A new box discusses the use of paymentin-kind preferred stock in the merger of Dow Chemical Company and Rohm & Haas, “The Romance Had No Chemistry, But It Had a Lot of Preferred Stock!” Chapter 20: Initial Public Offerings, Investment Banking, and Financial Restructuring. The new opening vignette discusses three companies that recently raised capital via an initial public offering, a seasoned stock offering, and a debt offering. We added a new section on investment banking activities. We added a new box on “Investment Banks and the Global Economic Crisis.” Chapter 21: Mergers, LBOs, Divestitures, and Holding Companies. We added a section explaining how the stock-swap ratio is determined for mergers where the payment is in the form of the acquiring company’s stock. Chapter 22: Bankruptcy, Reorganization, and Liquidation. The new opening vignette discusses the bankruptcies of Lehman Brothers, Washington Mutual, Chrysler, and General Motors. We added a new box on personal and small business bankruptcies, “A Nation of Defaulters?”. Chapter 23: Derivatives and Risk Management. The new opening vignette discusses risk management at Koch Industries, Navistar, and Pepsi. We added a new box on “Value at Risk and Enterprise Risk Management.” Throughout the chapter we discuss the failure of risk management during the global economic crisis. Chapter 24: Portfolio Theory, Asset Pricing Models, and Behavioral Finance. We added a box on the WSJ contest between dart-throwers and investors, “Skill or Luck?”. We expanded our discussion of the Fama-French 3-factor model and included a table showing returns of portfolios formed by sorting on size and the bookto-market ratio. Chapter 25: Real Options. The new opening vignette discusses Honda’s flexible manufacturing plants. Aplia Finance Aplia Finance, an interactive learning system, engages students in course concepts, ensures they practice on a regular basis, and helps them prepare to learn finance through a series of tutorials. Created by an instructor to help students excel, book- Preface xxv specific problem sets have instant grades and detailed feedback, ensuring students have the opportunity to learn from and improve with every question. Chapter assignments use the same language and tone of the course textbook, giving students a seamless experience in and out of the classroom. Problems are automatically graded and offer detailed explanations, helping students learn from every question. Aplia Finance offers: • • • • • Problem Sets: Chapter-specific problem sets ensure that students are completing finance assignments on a regular basis. Preparing for Finance Tutorials: Hands-on tutorials solve math, statistics, economics, and accounting roadblocks before they become a problem in the course, and financial calculator tutorials help students learn to use the tools needed in a finance course. News Analyses: Students connect course theories to real-world events by reading relevant news articles and answering graded questions about the article. Course Management System Digital Textbook For more information, visit http://www.aplia.com/finance. Thomson ONE—Business School Edition WWW To access Thomson ONE— BSE, go to http://tobsefin .swlearning.com and follow the instructions shown there. You will need the serial number that came on the card in your textbook. Thomson ONE—Business School Edition is an online database that draws from the world acclaimed Thomson Financial data sources, including the SEC Disclosure, Datastream, First Call, and Worldscope databases. Now you can give your students the opportunity to practice with a business school version of the same Internet-based database that brokers and analysts around the world use every day. Thomson ONE— BSE provides (1) one-click download of financial statements to Excel, (2) data from domestic and international companies, (3) 10 years of financial data; and (4) oneclick Peer Set analyses. Many chapters have suggested problems based on data available at Thomson ONE—BSE. Here is a description of the data provided by Thomson ONE—BSE: I/B/E/S Consensus Estimates. Includes consensus estimates—averages, means, and medians; analyst-by-analyst earnings coverage; analysts’ forecasts based on 15 industry standard measures; current and historic coverage for the selected 500 companies. Current coverage is five years forward plus historic data from 1976 for U.S. companies and from 1987 for international companies, with current data updated daily and historic data updated monthly. Worldscope. Includes company profiles, financials, accounting results, and market per-share data for the selected 500 companies going back to 1980, all updated daily. Disclosure SEC Database. Includes company profiles, annual and quarterly company financials, pricing information, and earnings estimates for selected U.S. and Canadian companies, annually from 1987, quarterly for the last 10 years, and monthly for prices, all updated weekly. DataStream Pricing. Daily international pricing, including share price information (open, high, low, close, P/E) plus index and exchange rate data, for the last 10 years. xxvi Preface ILX Systems Delayed Quotes. Includes 20-minute delayed quotes of equities and indices from U.S. and global tickers covering 130 exchanges in 25 developed countries. Comtex Real-Time News. Includes current news releases. SEC Edgar Filings and Global Image Source Filings. Includes regulatory and nonregulatory filings for both corporate and individual entities. Edgar filings are real-time and go back 10 years; image filings are updated daily and go back 7 years. MAKE IT YOURS Your course is unique; create a casebook that reflects it. Let us help you put together a quality casebook simply, quickly, and affordably. We want to help you focus on the most important thing – teaching. That’s why we have made this as simple as possible for you. We have aligned best-selling cases from our Klein/Brigham and Brigham/Buzzard series at the chapter level to Brigham/Ehrhardt. We encourage you to visit http://www.cengage.com/custom/makeityours/ BrighamEhrhardt and select the cases to include in a custom case book. The cases are listed under each chapter title. To review cases, simply click on “view abstract” next to each case title. If you would like to review the full case, contact your Cengage Learning representative or fill out the form and we will contact you. For more information about custom publishing options, visit www.cengage.com/ custom/. THE INSTRUCTIONAL PACKAGE: AN INTEGRATED LEARNING SYSTEM Financial Management includes a broad range of ancillary materials designed to enhance students’ learning and to make it easier for instructors to prepare for and conduct classes. All resources available to students are of course also available to instructors, and instructors also have access to the course management tools. Learning Tools Available for Students and Instructors The Cengage Global Economic Watch (GEW) Resource Center. This is your source for turning today’s challenges into tomorrow’s solutions. This online portal, available for free when bundled with the text, houses the most current and upto-date content concerning the economic crisis. Organized by discipline, the GEC Resource Center offers the solutions instructors and students need in an easy-to-use format. Included are an overview and timeline of the historical events leading up to the crisis; links to the latest news and resources; discussion and testing content; an instructor feedback forum; and a Global Issues Database. Study Guide. This supplement outlines the key sections of each chapter, and it provides students with a set of questions and problems similar to those in the text and in the Test Bank, along with worked-out solutions. Instructors seldom use the Study Guide themselves, but students often find it useful, so we recommend that instructors ask their bookstores to have copies available. Our bookstores generally have to reorder it, which attests to its popularity with students. In addition to these printed resources and the items noted above, many other resources are available on the Web at Financial Management’s Web site. These ancillaries include: Preface xxvii Excel Tool Kits. Proficiency with spreadsheets is an absolute necessity for all MBA students. With this in mind, we created Excel spreadsheets, called “Tool Kits,” for each chapter to show how the calculations used in the chapter were actually done. The Tool Kit models include explanations and screen shots that show students how to use many of the features and functions of Excel, enabling the Tool Kits to serve as selftaught tutorials. An e-Library: Web Extensions and Web Chapters. Many chapters have Adobe PDF “appendices” that provide more detailed coverage of topics that were addressed in the chapter. In addition, these four specialized topics are covered in PDF web chapters: Banking Relationships, Working Capital Management Extensions, Pension Plan Management, and Financial Management in Not-for-Profit Businesses. End-of-Chapter Spreadsheet Problems. Each chapter has a “Build a Model” problem, where students start with a spreadsheet that contains financial data plus general instructions relating to solving a specific problem. The model is partially completed, with headings but no formulas, so the student must literally build a model. This structure guides the student through the problem, minimizes unnecessary typing and data entry, and also makes it easy to grade the work, since all students’ answers are in the same locations on the spreadsheet. The partial spreadsheets for the “Build a Model” problems are available to students on the book’s Web site, while the completed models are in files on the Instructor’s portion of the Web site. Thomson ONE—BSE Problem Sets. The book’s Web site has a set of problems that require accessing the Thomson ONE—Business School Edition Web data. Using real world data, students are better able to develop the skills they will need in the real world. Interactive Study Center. The textbook’s Web site contains links to all Web sites that are cited in each chapter. Course Management Tools Available only to Instructors Instructors have access to all of the materials listed above, plus additional course management tools. These are available at Financial Management’s Instructor companion Web site and on the Instructor’s Resource CD. These materials include: Solutions Manual. This comprehensive manual contains worked-out solutions to all end-of-chapter materials. It is available in both print and electronic forms at the Instructor’s Web site. PowerPoint Slides. There is a Mini Case at the end of each chapter. These cases cover all the essential issues presented in the chapter, and they provide the structure for our class lectures. For each Mini Case, we developed a set of PowerPoint slides that present graphs, tables, lists, and calculations for use in lectures. Although based on the Mini Cases, the slides are completely self-contained in the sense that they can be used for lectures regardless of whether students are required to read the mini cases. Also, instructors can easily customize the slides, and they can be converted quickly into any PowerPoint Design Template.1 Copies of these files are on the Instructor’s Web site. To convert into PowerPoint, select Format, Apply Design Template, and then pick any template. Always double-check the conversion; some templates use differently sized fonts, which can cause some slide titles to run over their allotted space. 1 xxviii Preface Mini Case Spreadsheets. In addition to the PowerPoint slides, we also provide Excel spreadsheets that do the calculations required in the Mini Cases. These spreadsheets are similar to the Tool Kits except (a) the numbers correspond to the Mini Cases rather than the chapter examples, and (b) we added some features that make it possible to do what-if analysis on a real-time basis in class. We usually begin our lectures with the PowerPoint presentation, but after we have explained a basic concept we “toggle” to the mini case Excel file and show how the analysis can be done in Excel.2 For example, when teaching bond pricing, we begin with the PowerPoint show and cover the basic concepts and calculations. Then we toggle to Excel and use a sensitivity-based graph to show how bond prices change as interest rates and time to maturity vary. More and more students are bringing their laptops to class, and they can follow along, doing the what-if analysis for themselves. Solutions to End-of-Chapter Spreadsheet Problems. The partial spreadsheets for the “Build a Model” problems are available to students, while the completed models are in files on the Instructor’s Web site. Solutions to Thomson ONE—BSE Problem Sets. The Thomson ONE—BSE problems set require students to use real world data. Although the solutions change daily as the data change, we provide instructors with “representative” answers. Test Bank. The Test Bank contains more than 1,200 class-tested questions and problems. Information regarding the topic and degree of difficulty, along with the complete solution for all numerical problems, is provided with each question. The Test Bank is available in three forms: (1) in a printed book; (2) in Microsoft Word files; and (3) in a computerized test bank software package, Exam View, which has many features that make test preparation, scoring, and grade recording easy, including the ability to generate different versions of the same problem. Exam View is easily able to export pools into Blackboard and WebCT. Textchoice, the Cengage Learning Online Case Library. More than 100 cases written by Eugene F. Brigham, Linda Klein, and Chris Buzzard are now available via the Internet, and new cases are added every year. These cases are in a database that allows instructors to select cases and create their own customized casebooks. Most of the cases have accompanying spreadsheet models that, while not essential for working the case, do reduce number crunching and thus leave more time for students to consider conceptual issues. The models also illustrate how computers can be used to make better financial decisions. Cases that we have found particularly useful for the different chapters are listed in the end-of-chapter references. The cases, case solutions, and spreadsheet models can be previewed and ordered by instructors at http://www.textchoice2.com. Cengage/South-Western will provide complimentary supplements or supplement packages to those adopters qualified under Cengage’s adoption policy. Please contact your sales representative to learn how you may qualify. If, as an adopter or potential user, you receive supplements you do not need, please return them to your sales representative. 2 Note: To toggle between two open programs, such as Excel and PowerPoint, hold the Alt key down and hit the Tab key until you have selected the program you want to show. Preface xxix ACKNOWLEDGMENTS This book reflects the efforts of a great many people over a number of years. First, we would like to thank the following reviewers of the Twelfth Edition for their suggestions: ANNE ANDERSON Lehigh University SHARON H. GARRISON University of Arizona OMAR M. BENKATO Ball State University HASSAN MOUSSAWI Wayne State University RAHUL BISHNOI Hofstra University A. JON SAXON Loyola Marymount University JONATHAN CLARKE Georgia Institute of Technology JOSEPH VU DePaul University-Lincoln In addition, we appreciate the many helpful comments and suggestions that we incorporated into this edition made by Richard M. Burns, Greg Faulk, John Harper, Robert Irons, Joe Walker, Barry Wilbratte, and Serge Wind. Many professors and professionals who are experts on specific topics reviewed earlier versions of individual chapters or groups of chapters and we are grateful for their insights; in addition, we would like to thank those whose reviews and comments on earlier editions and companion books have contributed to this edition: Mike Adler, Syed Ahmad, Sadhana M. Alangar, Ed Altman, Mary Schary Amram, Bruce Anderson, Ron Anderson, Bob Angell, Vince Apilado, Henry Arnold, Nasser Arshadi, Bob Aubey, Abdul Aziz, Gil Babcock, Peter Bacon, Kent Baker, Tom Bankston, Les Barenbaum, Charles Barngrover, Michael Barry, Bill Beedles, Moshe Ben-Horim, Bill Beranek, Tom Berry, Bill Bertin, Roger Bey, Dalton Bigbee, John Bildersee, Eric Blazer, Russ Boisjoly, Keith Boles, Gordon R. Bonner, Geof Booth, Kenneth Boudreaux, Helen Bowers, Oswald Bowlin, Don Boyd, G. Michael Boyd, Pat Boyer, Ben S. Branch, Joe Brandt, Elizabeth Brannigan, Greg Brauer, Mary Broske, Dave Brown, Kate Brown, Bill Brueggeman, Kirt Butler, Robert Button, Chris Buzzard, Bill Campsey, Bob Carleson, Severin Carlson, David Cary, Steve Celec, Don Chance, Antony Chang, Susan Chaplinsky, Jay Choi, S. K. Choudhury, Lal Chugh, Maclyn Clouse, Margaret Considine, Phil Cooley, Joe Copeland, David Cordell, John Cotner, Charles Cox, David Crary, John Crockett, Roy Crum, Brent Dalrymple, Bill Damon, Joel Dauten, Steve Dawson, Sankar De, Miles Delano, Fred Dellva, Anand Desai, Bernard Dill, Greg Dimkoff, Les Dlabay, Mark Dorfman, Gene Drycimski, Dean Dudley, David Durst, Ed Dyl, Dick Edelman, Charles Edwards, John Ellis, Dave Ewert, John Ezzell, Richard Fendler, Michael Ferri, Jim Filkins, John Finnerty, Susan Fischer, Mark Flannery, Steven Flint, Russ Fogler, E. Bruce Frederickson, Dan French, Tina Galloway, Partha Gangopadhyay, Phil Gardial, Michael Garlington, Jim Garvin, Adam Gehr, Jim Gentry, Stuart Gillan, Philip Glasgo, Rudyard Goode, Myron Gordon, Walt Goulet, Bernie Grablowsky, Theoharry Grammatikos, Ed Grossnickle, John Groth, Alan Grunewald, Manak Gupta, Sam Hadaway, Don Hakala, Janet Hamilton, Sally Hamilton, Gerald Hamsmith, William Hardin, John Harris, Paul Hastings, Patty Hatfield, Bob Haugen, Steve Hawke, Del Hawley, Hal Heaton, Robert Hehre, John Helmuth, George Hettenhouse, Hans Heymann, Kendall Hill, Roger Hill, Tom Hindelang, Linda Hittle, Ralph Hocking, J. Ronald Hoffmeister, Jim Horrigan, John Houston, John Howe, Keith Howe, Hugh Hunter, Steve Isberg, Jim Jackson, Vahan Janjigian, Kurt Jesswein, Kose John, Craig Johnson, xxx Preface Keith Johnson, Steve Johnson, Ramon Johnson, Ray Jones, Manuel Jose, Gus Kalogeras, Mike Keenan, Bill Kennedy, Joe Kiernan, Robert Kieschnick, Rick Kish, Linda Klein, Don Knight, Dorothy Koehl, Theodor Kohers, Jaroslaw Komarynsky, Duncan Kretovich, Harold Krogh, Charles Kroncke, Lynn Phillips Kugele, Joan Lamm, P. Lange, Howard Lanser, Martin Laurence, Ed Lawrence, Richard LeCompte, Wayne Lee, Jim LePage, Ilene Levin, Jules Levine, John Lewis, James T. Lindley, Chuck Linke, Bill Lloyd, Susan Long, Judy Maese, Bob Magee, Ileen Malitz, Phil Malone, Terry Maness, Chris Manning, Terry Martell, D. J. Masson, John Mathys, John McAlhany, Andy McCollough, Tom McCue, Bill McDaniel, Robin McLaughlin, Jamshid Mehran, Ilhan Meric, Larry Merville, Rick Meyer, Stuart E. Michelson, Jim Millar, Ed Miller, John Mitchell, Carol Moerdyk, Bob Moore, Barry Morris, Gene Morris, Fred Morrissey, Chris Muscarella, Stu Myers, David Nachman, Tim Nantell, Don Nast, Bill Nelson, Bob Nelson, Bob Niendorf, Tom O’Brien, Dennis O’Connor, John O’Donnell, Jim Olsen, Robert Olsen, Frank O’Meara, David Overbye, R. Daniel Pace, Coleen Pantalone, Jim Pappas, Stephen Parrish, Pam Peterson, Glenn Petry, Jim Pettijohn, Rich Pettit, Dick Pettway, Hugo Phillips, John Pinkerton, Gerald Pogue, Ralph A. Pope, R. Potter, Franklin Potts, R. Powell, Chris Prestopino, Jerry Prock, Howard Puckett, Herbert Quigley, George Racette, Bob Radcliffe, Allen Rappaport, Bill Rentz, Ken Riener, Charles Rini, John Ritchie, Jay Ritter, Pietra Rivoli, Fiona Robertson, Antonio Rodriguez, E. M. Roussakis, Dexter Rowell, Mike Ryngaert, Jim Sachlis, Abdul Sadik, Thomas Scampini, Kevin Scanlon, Frederick Schadler, James Schallheim, Mary Jane Scheuer, Carl Schweser, John Settle, Alan Severn, Sol Shalit, Elizabeth Shields, Frederic Shipley, Dilip Shome, Ron Shrieves, Neil Sicherman, J. B. Silvers, Clay Singleton, Joe Sinkey, Stacy Sirmans, Jaye Smith, Steve Smith, Don Sorenson, David Speairs, Ken Stanly, John Stansfield, Ed Stendardi, Alan Stephens, Don Stevens, Jerry Stevens, G. Bennett Stewart, Mark Stohs, Glen Strasburg, Robert Strong, Philip Swensen, Ernie Swift, Paul Swink, Eugene Swinnerton, Robert Taggart, Gary Tallman, Dennis Tanner, Craig Tapley, Russ Taussig, Richard Teweles, Ted Teweles, Jonathan Tiemann, Sheridan Titman, Andrew Thompson, George Trivoli, George Tsetsekos, Alan L. Tucker, Mel Tysseland, David Upton, Howard Van Auken, Pretorious Van den Dool, Pieter Vanderburg, Paul Vanderheiden, David Vang, Jim Verbrugge, Patrick Vincent, Steve Vinson, Susan Visscher, John Wachowicz, Mark D. Walker, Mike Walker, Sam Weaver, Kuo Chiang Wei, Bill Welch, Gary R. Wells, Fred Weston, Norm Williams, Tony Wingler, Ed Wolfe, Larry Wolken, Don Woods, Thomas Wright, Michael Yonan, Zhong-guo Zhou, David Ziebart, Dennis Zocco, and Kent Zumwalt. Special thanks are due to Dana Clark, Susan Whitman, Amelia Bell, Stephanie Hodge, and Kirsten Benson, who provided invaluable editorial support; to Joel Houston and Phillip Daves, whose work with us on other books is reflected in this text; and to Lou Gapenski, our past coauthor, for his many contributions. Our colleagues and our students at the Universities of Florida and Tennessee gave us many useful suggestions, and the Cengage/South-Western staff—especially Mike Guendelsberger, Scott Fidler, Jacquelyn Featherly, Nate Anderson, and Mike Reynolds—helped greatly with all phases of text development, production, and marketing. Preface ERRORS IN THE xxxi TEXT At this point, authors generally say something like this: “We appreciate all the help we received from the people listed above, but any remaining errors are, of course, our own responsibility.” And in many books, there are plenty of remaining errors. Having experienced difficulties with errors ourselves, both as students and as instructors, we resolved to avoid this problem in Financial Management. As a result of our error detection procedures, we are convinced that the book is relatively free of mistakes. Partly because of our confidence that few such errors remain, but primarily because we want to detect any errors in the textbook that may have slipped by so we can correct them in subsequent printings, we decided to offer a reward of $10 per error to the first person who reports a textbook error to us. For purposes of this reward, errors in the textbook are defined as misspelled words, nonrounding numerical errors, incorrect statements, and any other error that inhibits comprehension. Typesetting problems such as irregular spacing and differences in opinion regarding grammatical or punctuation conventions do not qualify for this reward. Also, given the ever-changing nature of the Internet, changes in Web addresses do not qualify as errors, although we would appreciate reports of changed Web addresses. Finally, any qualifying error that has follow-through effects is counted as two errors only. Please report any errors to Michael C. Ehrhardt at the e-mail address given below. CONCLUSION Finance is, in a real sense, the cornerstone of the free enterprise system. Good financial management is therefore vitally important to the economic health of business firms, hence to the nation and the world. Because of its importance, corporate finance should be thoroughly understood. However, this is easier said than done—the field is relatively complex, and it is undergoing constant change in response to shifts in economic conditions. All of this makes corporate finance stimulating and exciting, but also challenging and sometimes perplexing. We sincerely hope that Financial Management: Theory and Practice will help readers understand and solve the financial problems faced by businesses today. Michael C. Ehrhardt University of Tennessee Ehrhardt@utk.edu Eugene F. Brigham University of Florida Gene.Brigham@cba.ufl.edu January 2010 PART 1 Fundamental Concepts of Corporate Finance Chapter 1 An Overview of Financial Management and the Financial Environment Chapter 2 Financial Statements, Cash Flow, and Taxes Chapter 3 Analysis of Financial Statements 1 This page intentionally left blank CHAPTER 1 An Overview of Financial Management and the Financial Environment WWW See http://money.cnn.com/ magazines/fortune/ for updates on the ranking. In a global beauty contest for companies, the winner is … Apple. Or at least Apple is the most admired company in the world, according to Fortune magazine’s annual survey. The others in the global top ten are Berkshire Hathaway, Toyota, Google, Johnson & Johnson, Procter & Gamble, FedEx, Southwest Airlines, General Electric, and Microsoft. What do these companies have that separates them from the rest of the pack? According to a survey of executives, directors, and security analysts, these companies have very high average scores across nine attributes: (1) innovativeness, (2) quality of management, (3) long-term investment value, (4) social responsibility, (5) employee talent, (6) quality of products and services, (7) financial soundness, (8) use of corporate assets, and (9) effectiveness in doing business globally. After culling weaker companies, the final rankings are then determined by over 3,700 experts from a wide variety of industries. What do these companies have in common? First, they have an incredible focus on using technology to understand their customers, reduce costs, reduce inventory, and speed up product delivery. Second, they continually innovate and invest in ways to differentiate their products. Some are known for game-changing products, such as Apple’s touch screen iPhone or Toyota’s hybrid Prius. Others continually introduce small improvements, such as Southwest Airline’s streamlined boarding procedures. In addition to their acumen with technology and customers, they are also on the leading edge when it comes to training employees and providing a workplace in which people can thrive. In a nutshell, these companies reduce costs by having innovative production processes, they create value for customers by providing highquality products and services, and they create value for employees by training and fostering an environment that allows employees to utilize all of their skills and talents. Do investors benefit from this focus on processes, customers, and employees? During the most recent 5-year period, these ten companies posted an average annual stock return of 6.9%, which is not too shabby when compared with the −4.1% average annual return of the S&P 500. These superior returns are due to superior cash flow generation. But, as you will see throughout this book, a company can generate cash flow only if it also creates value for its customers, employees, and suppliers. 3 4 Part 1: Fundamental Concepts of Corporate Finance resource The textbook’s Web site has tools for teaching, learning, and conducting financial research. This chapter should give you an idea of what financial management is all about, including an overview of the financial markets in which corporations operate. Before going into details, let’s look at the big picture. You’re probably in school because you want an interesting, challenging, and rewarding career. To see where finance fits in, here’s a five-minute MBA. 1.1 THE FIVE-MINUTE MBA Okay, we realize you can’t get an MBA in five minutes. But just as an artist quickly sketches the outline of a picture before filling in the details, we can sketch the key elements of an MBA education. The primary objective of an MBA program is to provide managers with the knowledge and skills they need to run successful companies, so we start our sketch with some common characteristics of successful companies. In particular, all successful companies are able to accomplish two main goals: 1. All successful companies identify, create, and deliver products or services that are highly valued by customers—so highly valued that customers choose to purchase from them rather than from their competitors. 2. All successful companies sell their products/services at prices that are high enough to cover costs and to compensate owners and creditors for the use of their money and their exposure to risk. It’s easy to talk about satisfying customers and investors, but it’s not so easy to accomplish these goals. If it were, then all companies would be successful, and you wouldn’t need an MBA! The Key Attributes of Successful Companies First, successful companies have skilled people at all levels inside the company, including leaders, managers, and a capable workforce. Second, successful companies have strong relationships with groups outside the company. For example, successful companies develop win–win relationships with suppliers and excel in customer relationship management. Third, successful companies have enough funding to execute their plans and support their operations. Most companies need cash to purchase land, buildings, equipment, and materials. Companies can reinvest a portion of their earnings, but most growing companies must also raise additional funds externally by some combination of selling stock and/or borrowing in the financial markets. Just as a stool needs all three legs to stand, a successful company must have all three attributes: skilled people, strong external relationships, and sufficient capital. WWW Consult http://www .careers-in-finance.com for an excellent site containing information on a variety of business career areas, listings of current jobs, and other reference materials. The MBA, Finance, and Your Career To be successful, a company must meet its first main goal: identifying, creating, and delivering highly valued products and services to its customers. This requires that it possess all three of the key attributes mentioned above. Therefore, it’s not surprising that most of your MBA courses are directly related to these attributes. For example, courses in economics, communication, strategy, organizational behavior, and human resources should prepare you for a leadership role and enable you to effectively manage your company’s workforce. Other courses, such as marketing, operations management, and information technology, increase your knowledge of specific disciplines, enabling you to develop the efficient business processes and strong external relationships your company needs. Portions of this finance course will address Chapter 1: An Overview of Financial Management and the Financial Environment 5 THE GLOBAL ECONOMIC CRISIS Say Hello to the Global Economic Crisis! Imagine a story of greed and reckless daring, of fortunes made and fortunes lost, of enormous corporations failing and even governments brought to the brink of ruin. No, this isn’t a box-office blockbuster, but instead is the situation facing the world’s financial markets and economies as we write this in mid-2009. What exactly is the crisis? At the risk of oversimplification, many of the world’s individuals, financial institutions, and governments borrowed too much money and used those borrowed funds to make speculative investments. As those investments are turning out to be worth less than the amounts owed by the borrowers, widespread bankruptcies, buyouts, and restructurings for both borrowers and lenders are occurring. This in turn is reducing the supply of available funds that finan- cial institutions normally lend to creditworthy individuals, manufacturers, and retailers. Without access to credit, consumers are buying less, manufacturers are producing less, and retailers are selling less—all of which leads to layoffs. Because of falling consumption, shrinking production, and higher unemployment, the National Bureau of Economic Research declared that the United States entered a recession in December 2007. In fact, this is a global downturn, and most economists expect it to be severe and lengthy. As we progress through this chapter and the rest of the book, we will discuss different aspects of the crisis. For real-time updates, go to the Global Economic Crisis (GEC) Resource Center at http://www.cengage.com/gec and log in. raising the capital your company needs to implement its plans. In short, your MBA courses will give you the skills you need to help a company achieve its first goal: producing goods and services that customers want. Recall, though, that it’s not enough just to have highly valued products and satisfied customers. Successful companies must also meet their second main goal, which is generating enough cash to compensate the investors who provided the necessary capital. To help your company accomplish this second goal, you must be able to evaluate any proposal, whether it relates to marketing, production, strategy, or any other area, and implement only the projects that add value for your investors. For this, you must have expertise in finance, no matter your major. Thus, finance is a critical part of an MBA education, and it will help you throughout your career. Self-Test What are the goals of successful companies? What are the three key attributes common to all successful companies? How does expertise in finance help a company become successful? 1.2 THE CORPORATE LIFE CYCLE Many major corporations, including Apple Computer and Hewlett-Packard, began life in a garage or basement. How is it possible for such companies to grow into the giants we see today? No two companies develop in exactly the same way, but the following sections describe some typical stages in the corporate life cycle. Starting Up as a Proprietorship Many companies begin as a proprietorship, which is an unincorporated business owned by one individual. Starting a business as a proprietor is easy—one merely begins business operations after obtaining any required city or state business licenses. The proprietorship has three important advantages: (1) it is easily and inexpensively 6 Part 1: Fundamental Concepts of Corporate Finance Columbus Was Wrong—the World Is Flat! And Hot, and Crowded! In his best-selling book The World Is Flat, Thomas L. Friedman argues that many of the barriers that long protected businesses and employees from global competition have been broken down by dramatic improvements in communication and transportation technologies. The result is a level playing field that spans the entire world. As we move into the information age, any work that can be digitized will flow to those able to do it at the lowest cost, whether they live in San Jose’s Silicon Valley or Bangalore, India. For physical products, supply chains now span the world. For example, raw materials might be extracted in South America, fabricated into electronic components in Asia, and then used in computers assembled in the United States, with the final product being sold in Europe. Similar changes are occurring in the financial markets, as capital flows across the globe to those who can best use it. Indeed, China raised more money through initial public offerings than any other country in 2006, and the euro is becoming the currency of choice for denominating global bond issues. Unfortunately, a dynamic world can bring runaway growth, which can lead to significant environmental problems and energy shortages. Friedman describes these problems in another bestseller, Hot, Flat, and Crowded. In a flat world, the keys to success are knowledge, skills, and a great work ethic. In a flat, hot, and crowded world, these factors must be combined with innovation and creativity to deal with truly global problems. formed, (2) it is subject to few government regulations, and (3) its income is not subject to corporate taxation but is taxed as part of the proprietor’s personal income. However, the proprietorship also has three important limitations: (1) it may be difficult for a proprietorship to obtain the capital needed for growth; (2) the proprietor has unlimited personal liability for the business’s debts, which can result in losses that exceed the money invested in the company (creditors may even be able to seize a proprietor’s house or other personal property!); and (3) the life of a proprietorship is limited to the life of its founder. For these three reasons, sole proprietorships are used primarily for small businesses. In fact, proprietorships account for only about 13% of all sales, based on dollar values, even though about 80% of all companies are proprietorships. More Than One Owner: A Partnership Some companies start with more than one owner, and some proprietors decide to add a partner as the business grows. A partnership exists whenever two or more persons or entities associate to conduct a noncorporate business for profit. Partnerships may operate under different degrees of formality, ranging from informal, oral understandings to formal agreements filed with the secretary of the state in which the partnership was formed. Partnership agreements define the ways any profits and losses are shared between partners. A partnership’s advantages and disadvantages are generally similar to those of a proprietorship. Regarding liability, the partners can potentially lose all of their personal assets, even assets not invested in the business, because under partnership law, each partner is liable for the business’s debts. Therefore, in the event the partnership goes bankrupt, if any partner is unable to meet his or her pro rata liability then the remaining partners must make good on the unsatisfied claims, drawing on their personal assets to the extent necessary. To avoid this, it is possible to limit the liabilities of some of the partners by establishing a limited partnership, wherein certain partners are designated general partners and others limited partners. In a limited partnership, the limited partners can lose only the amount of their investment in the partnership, Chapter 1: An Overview of Financial Management and the Financial Environment 7 while the general partners have unlimited liability. However, the limited partners typically have no control—it rests solely with the general partners—and their returns are likewise limited. Limited partnerships are common in real estate, oil, equipment leasing ventures, and venture capital. However, they are not widely used in general business situations because usually no one partner is willing to be the general partner and thus accept the majority of the business’s risk, and none of the others are willing to be limited partners and give up all control. In both regular and limited partnerships, at least one partner is liable for the debts of the partnership. However, in a limited liability partnership (LLP), sometimes called a limited liability company (LLC), all partners enjoy limited liability with regard to the business’s liabilities, and their potential losses are limited to their investment in the LLP. Of course, this arrangement increases the risk faced by an LLP’s lenders, customers, and suppliers. Many Owners: A Corporation Most partnerships have difficulty attracting substantial amounts of capital. This is generally not a problem for a slow-growing business, but if a business’s products or services really catch on, and if it needs to raise large sums of money to capitalize on its opportunities, then the difficulty in attracting capital becomes a real drawback. Thus, many growth companies, such as Hewlett-Packard and Microsoft, began life as a proprietorship or partnership, but at some point their founders decided to convert to a corporation. On the other hand, some companies, in anticipation of growth, actually begin as corporations. A corporation is a legal entity created under state laws, and it is separate and distinct from its owners and managers. This separation gives the corporation three major advantages: (1) unlimited life—a corporation can continue after its original owners and managers are deceased; (2) easy transferability of ownership interest—ownership interests are divided into shares of stock, which can be transferred far more easily than can proprietorship or partnership interests; and (3) limited liability—losses are limited to the actual funds invested. To illustrate limited liability, suppose you invested $10,000 in a partnership that then went bankrupt and owed $1 million. Because the owners are liable for the debts of a partnership, you could be assessed for a share of the company’s debt, and you could be held liable for the entire $1 million if your partners could not pay their shares. On the other hand, if you invested $10,000 in the stock of a corporation that went bankrupt, your potential loss on the investment would be limited to your $10,000 investment.1 Unlimited life, easy transferability of ownership interest, and limited liability make it much easier for corporations than proprietorships or partnerships to raise money in the financial markets and grow into large companies. The corporate form offers significant advantages over proprietorships and partnerships, but it also has two disadvantages: (1) Corporate earnings may be subject to double taxation—the earnings of the corporation are taxed at the corporate level, and then earnings paid out as dividends are taxed again as income to the stockholders. (2) Setting up a corporation involves preparing a charter, writing a set of bylaws, and filing the many required state and federal reports, which is more complex and timeconsuming than creating a proprietorship or a partnership. The charter includes the following information: (1) name of the proposed corporation, (2) types of activities it will pursue, (3) amount of capital stock, (4) number of In the case of very small corporations, the limited liability may be fiction because lenders frequently require personal guarantees from the stockholders. 1 8 Part 1: Fundamental Concepts of Corporate Finance directors, and (5) names and addresses of directors. The charter is filed with the secretary of the state in which the firm will be incorporated, and when it is approved, the corporation is officially in existence.2 After the corporation begins operating, quarterly and annual employment, financial, and tax reports must be filed with state and federal authorities. The bylaws are a set of rules drawn up by the founders of the corporation. Included are such points as (1) how directors are to be elected (all elected each year or perhaps one-third each year for 3-year terms); (2) whether the existing stockholders will have the first right to buy any new shares the firm issues; and (3) procedures for changing the bylaws themselves, should conditions require it. There are actually several different types of corporations. Professionals such as doctors, lawyers, and accountants often form a professional corporation (PC) or a professional association (PA). These types of corporations do not relieve the participants of professional (malpractice) liability. Indeed, the primary motivation behind the professional corporation was to provide a way for groups of professionals to incorporate and thus avoid certain types of unlimited liability yet still be held responsible for professional liability. Finally, if certain requirements are met, particularly with regard to size and number of stockholders, owners can establish a corporation but elect to be taxed as if the business were a proprietorship or partnership. Such firms, which differ not in organizational form but only in how their owners are taxed, are called S corporations. Growing and Managing a Corporation Once a corporation has been established, how does it evolve? When entrepreneurs start a company, they usually provide all the financing from their personal resources, which may include savings, home equity loans, or even credit cards. As the corporation grows, it will need factories, equipment, inventory, and other resources to support its growth. In time, the entrepreneurs usually deplete their own resources and must turn to external financing. Many young companies are too risky for banks, so the founders must sell stock to outsiders, including friends, family, private investors (often called angels), or venture capitalists. If the corporation continues to grow, it may become successful enough to attract lending from banks, or it may even raise additional funds through an initial public offering (IPO) by selling stock to the public at large. After an IPO, corporations support their growth by borrowing from banks, issuing debt, or selling additional shares of stock. In short, a corporation’s ability to grow depends on its interactions with the financial markets, which we describe in much more detail later in this chapter. For proprietorships, partnerships, and small corporations, the firm’s owners are also its managers. This is usually not true for a large corporation, which means that large firms’ stockholders, who are its owners, face a serious problem. What is to prevent managers from acting in their own best interests, rather than in the best interests of the stockholder/owners? This is called an agency problem, because managers are hired as agents to act on behalf of the owners. Agency problems can be addressed by a company’s corporate governance, which is the set of rules that control the company’s behavior towards its directors, managers, employees, shareholders, creditors, customers, competitors, and community. We will have much more to say about More than 60% of major U.S. corporations are chartered in Delaware, which has, over the years, provided a favorable legal environment for corporations. It is not necessary for a firm to be headquartered, or even to conduct operations, in its state of incorporation, or even in its country of incorporation. 2 Chapter 1: An Overview of Financial Management and the Financial Environment 9 agency problems and corporate governance throughout the book, especially in Chapters 13 and 14.3 Self-Test What are the key differences between proprietorships, partnerships, and corporations? Describe some special types of partnerships and corporations, and explain the differences among them. 1.3 THE PRIMARY OBJECTIVE VALUE MAXIMIZATION OF THE CORPORATION: Shareholders are the owners of a corporation, and they purchase stocks because they want to earn a good return on their investment without undue risk exposure. In most cases, shareholders elect directors, who then hire managers to run the corporation on a day-to-day basis. Because managers are supposed to be working on behalf of shareholders, they should pursue policies that enhance shareholder value. Consequently, throughout this book we operate on the assumption that management’s primary objective is stockholder wealth maximization. The market price is the stock price that we observe in the financial markets. We later explain in detail how stock prices are determined, but for now it is enough to say that a company’s market price incorporates the information available to investors. If the market price reflects all relevant information, then the observed price is also the intrinsic, or fundamental, price. However, investors rarely have all relevant information. For example, companies report most major decisions, but they sometimes withhold selected information to prevent competitors from gaining strategic advantages. Unfortunately, some managers deliberately mislead investors by taking actions to make their companies appear more valuable than they truly are. Sometimes these actions are illegal, such as those taken by the senior managers at Enron. Sometimes the actions are legal but are taken to push the current market price above its fundamental price in the short term. For example, suppose a utility’s stock price is equal to its fundamental price of $50 per share. What would happen if the utility substantially reduced its tree-trimming program but didn’t tell investors? This would lower current costs and thus boost current earnings and current cash flow, but it would also lead to major expenditures in the future when falling limbs damage the lines. If investors were told about the major repair costs facing the company, the market price would immediately drop to a new fundamental value of $45. But if investors were kept in the dark, they might misinterpret the higher-than-expected current earnings, and the market price might go up to $52. Investors would eventually understand the situation when the company later incurred large costs to repair the damaged lines; when that happened, the price would fall to its fundamental value of $45. Consider this hypothetical sequence of events. A company’s managers deceived investors, and the price rose to $52 when it would have fallen to $45 if not for the deception. Of course, this benefited those who owned the stock at the time of the deception, including managers with stock options. But when the deception came to light, those The classic work on agency theory is Michael C. Jensen and William H. Meckling’s “Theory of the Firm, Managerial Behavior, Agency Costs, and Ownership Structure,” Journal of Financial Economics, October 1976, 305–360. Another article by Jensen specifically addresses these issues; see “Value Maximization, Stakeholder Theory, and the Corporate Objective Function,” Journal of Applied Corporate Finance, Fall 2001, 8–21. For an overview of corporate governance, see Stuart Gillan, “Recent Developments in Corporate Governance: An Overview,” Journal of Corporate Finance, June 2006, 381–402. 3 10 Part 1: Fundamental Concepts of Corporate Finance Ethics for Individuals and Businesses A firm’s commitment to business ethics can be measured by the tendency of its employees, from the top down, to adhere to laws, regulations, and moral standards relating to product safety and quality, fair employment practices, fair marketing and selling practices, the use of confidential information for personal gain, community involvement, and illegal payments to obtain business. Ethical Dilemmas When conflicts arise between profits and ethics, sometimes legal and ethical considerations make the choice obvious. At other times the right choice isn’t clear. For example, suppose Norfolk Southern’s managers know that its trains are polluting the air, but the amount of pollution is within legal limits and further reduction would be costly, causing harm to their shareholders. Are the managers ethically bound to reduce pollution? Aren’t they also ethically bound to act in their shareholders’ best interests? This is clearly a dilemma. Ethical Responsibility Over the past few years, illegal ethical lapses have led to a number of bankruptcies, which have raised this question: Were the companies unethical, or was it just a few of their employees? Arthur Andersen, an accounting firm, audited Enron, WorldCom, and several other companies that committed accounting fraud. The U.S. Justice Department concluded that Andersen itself was guilty because it fostered a climate in which unethical behavior was permitted, and it built an incentive system that made such behavior profitable to both the perpetrators and the firm itself. As a result, Andersen went out of business. Anderson was later judged to be not guilty, but by the time the judgment was rendered the company was already out of business. People simply did not want to deal with a tainted accounting firm. Protecting Ethical Employees If employees discover questionable activities or are given questionable orders, should they obey their bosses’ orders, refuse to obey those orders, or report the situation to a higher authority, such as the company’s board of directors, its auditors, or a federal prosecutor? In 2002 Congress passed the SarbanesOxley Act, with a provision designed to protect “whistle-blowers.” If an employee reports corporate wrongdoing and later is penalized, he or she can ask the Occupational Safety and Health Administration to investigate the situation, and if the employee was improperly penalized, the company can be required to reinstate the person, along with back pay and a sizable penalty award. Several big awards have been handed out since the act was passed. stockholders who still owned the stock suffered a significant loss, ending up with stock worth less than its original fundamental value. If the managers cashed in their stock options prior to this, then only the stockholders were hurt by the deception. Because the managers were hired to act in the interests of stockholders, their deception was a breach of their fiduciary responsibility. In addition, the managers’ deception would damage the company’s reputation, making it harder to raise capital in the future. Therefore, when we say management’s objective should be to maximize stockholder wealth, we really mean it is to maximize the fundamental price of the firm’s common stock, not just the current market price. Firms do, of course, have other objectives; in particular, the managers who make the actual decisions are interested in their own personal satisfaction, in their employees’ welfare, and in the good of their communities and of society at large. Still, for the reasons set forth in the following sections, maximizing intrinsic stock value is the most important objective for most corporations. Intrinsic Stock Value Maximization and Social Welfare If a firm attempts to maximize its intrinsic stock value, is this good or bad for society? In general, it is good. Aside from such illegal actions as fraudulent accounting, ex- Chapter 1: An Overview of Financial Management and the Financial Environment WWW The Security Industry Association’s Web site, http:// www.sifma.org, is a great source of information. To find data on stock ownership, go to its Web page, click on Research, choose Surveys, then Equity/Bond Ownership in America. You can purchase the most recent data, or look at the prior year for free. 11 ploiting monopoly power, violating safety codes, and failing to meet environmental standards, the same actions that maximize intrinsic stock values also benefit society. Here are some of the reasons: 1. To a large extent, the owners of stock are society. Seventy-five years ago this was not true, because most stock ownership was concentrated in the hands of a relatively small segment of society consisting of the wealthiest individuals. Since then, there has been explosive growth in pension funds, life insurance companies, and mutual funds. These institutions now own more than 61% of all stock, which means that most individuals have an indirect stake in the stock market. In addition, more than 47% of all U.S. households now own stock or bonds directly, as compared with only 32.5% in 1989. Thus, most members of society now have an important stake in the stock market, either directly or indirectly. Therefore, when a manager takes actions to maximize the stock price, this improves the quality of life for millions of ordinary citizens. 2. Consumers benefit. Stock price maximization requires efficient, low-cost businesses that produce high-quality goods and services at the lowest possible cost. This means that companies must develop products and services that consumers want and need, which leads to new technology and new products. Also, companies that maximize their stock price must generate growth in sales by creating value for customers in the form of efficient and courteous service, adequate stocks of merchandise, and well-located business establishments. People sometimes argue that firms, in their efforts to raise profits and stock prices, increase product prices and gouge the public. In a reasonably competitive economy, which we have, prices are constrained by competition and consumer resistance. If a firm raises its prices beyond reasonable levels, it will simply lose its market share. Even giant firms such as Dell and Coca-Cola lose business to domestic and foreign competitors if they set prices above the level necessary to cover production costs plus a “normal” profit. Of course, firms want to earn more, and they constantly try to cut costs, develop new products, and so on, and thereby earn above-normal profits. Note, though, that if they are indeed successful and do earn above-normal profits, those very profits will attract competition, which will eventually drive prices down. So again, the main long-term beneficiary is the consumer. 3. Employees benefit. There are situations where a stock increases when a company announces plans to lay off employees, but viewed over time this is the exception rather than the rule. In general, companies that successfully increase stock prices also grow and add more employees, thus benefiting society. Note too that many governments across the world, including U.S. federal and state governments, are privatizing some of their state-owned activities by selling these operations to investors. Perhaps not surprisingly, the sales and cash flows of recently privatized companies generally improve. Moreover, studies show that newly privatized companies tend to grow and thus require more employees when they are managed with the goal of stock price maximization. One of Fortune magazine’s key criteria in determining its list of most-admired companies is a company’s ability to attract, develop, and retain talented people. The results consistently show high correlations among admiration for a company, its ability to satisfy employees, and its creation of value for shareholders. Employees find that it is both fun and financially rewarding to work for successful companies. Thus, successful companies get the cream of the employee crop, and skilled, motivated employees are one of the keys to corporate success. 12 Part 1: Fundamental Concepts of Corporate Finance Managerial Actions to Maximize Shareholder Wealth What types of actions can managers take to maximize shareholder wealth? To answer this question, we first need to ask, “What determines a firm’s value?” In a nutshell, it is a company’s ability to generate cash flows now and in the future. We address different aspects of this in detail throughout the book, but we can lay out three basic facts now: (1) any financial asset, including a company’s stock, is valuable only to the extent that it generates cash flows; (2) the timing of cash flows matters—cash received sooner is better; and (3) investors are averse to risk, so all else equal, they will pay more for a stock whose cash flows are relatively certain than for one whose cash flows are more risky. Because of these three facts, managers can enhance their firm’s value by increasing the size of the expected cash flows, by speeding up their receipt, and by reducing their risk. The cash flows that matter are called free cash flows (FCF), not because they are free, but because they are available (or free) for distribution to all of the company’s investors, including creditors and stockholders. You will learn how to calculate free cash flows in Chapter 2, but for now you should know that free cash flows depend on three factors: (1) sales revenues, (2) operating costs and taxes, and (3) required new investments in operating capital. In particular, free cash flow is equal to: FCF = Sales revenues − Operating costs − Operating taxes − Required new investments in operating capital Brand managers and marketing managers can increase sales (and prices) by truly understanding their customers and then designing goods and services that customers want. Human resource managers can improve productivity through training and employee retention. Production and logistics managers can improve profit margins, reduce inventory, and improve throughput at factories by implementing supply chain management, just-in-time inventory management, and lean manufacturing. In fact, all managers make decisions that can increase free cash flows. One of the financial manager’s roles is to help others see how their actions affect the company’s ability to generate cash flow and, hence, its intrinsic value. Financial managers also must decide how to finance the firm. In particular, they must choose the mix of debt and equity that should be used and the specific types of debt and equity securities that should be issued. They must also decide what percentage of current earnings should be retained and reinvested rather than paid out as dividends. Along with these financing decisions, the general level of interest rates in the economy, the risk of the firm’s operations, and stock market investors’ overall attitude toward risk determine the rate of return that is required to satisfy a firm’s investors. This rate of return from an investor’s perspective is a cost from the company’s point of view. Therefore, the rate of return required by investors is called the weighted average cost of capital (WACC). The relationship between a firm’s fundamental value, its free cash flows, and its cost of capital is defined by the following equation: Value ¼ FCF1 FCF2 FCF3 FCF∞ …þ (1-1) 1 þ 2 þ 3þ ð1 þ WACCÞ∞ ð1 þ WACCÞ ð1 þ WACCÞ ð1 þ WACCÞ We will explain how to use this equation in later chapters, but for now note that (1) a growing firm often needs to raise external funds in the financial markets and Chapter 1: An Overview of Financial Management and the Financial Environment 13 Corporate Scandals and Maximizing Stock Price The list of corporate scandals seems to go on forever: Sunbeam, Enron, ImClone, WorldCom, Tyco, Adelphia …. At first glance, it’s tempting to say, “Look what happens when managers care only about maximizing stock price.” But a closer look reveals a much different story. In fact, if these managers were trying to maximize stock price, they failed dismally, given the resulting values of these companies. Although details vary from company to company, a few common themes emerge. First, managerial compensation was linked to the short-term performance of the stock price via poorly designed stock option and stock grant programs. This provided managers with a powerful incentive to drive up the stock price at the option vesting date without worrying about the future. Second, it is virtually impossible to take legal and ethical actions that drive up the stock price in the short term without harming it in the long term because the value of a company is based on all of its future free cash flows and not just cash flows in the immediate future. Because legal and ethical actions to quickly drive up the stock price didn’t exist (other than the old-fashioned ones, such as increasing sales, cutting costs, or reducing capital requirements), these managers began bending a few rules. Third, as they initially got away with bending rules, it seems that their egos and hubris grew to such an extent that they felt they were above all rules, so they began breaking even more rules. Stock prices did go up, at least temporarily, but as Abraham Lincoln said, “You can’t fool all of the people all of the time.” As the scandals became public, the stocks’ prices plummeted, and in some cases the companies were ruined. There are several important lessons to be learned from these examples. First, people respond to incentives, and poorly designed incentives can cause disastrous results. Second, ethical violations usually begin with small steps, so if stockholders want managers to avoid large ethical violations, then they shouldn’t let them make the small ones. Third, there is no shortcut to creating lasting value. It takes hard work to increase sales, cut costs, and reduce capital requirements, but this is the formula for success. (2) the actual price of a firm’s stock is determined in those markets. Therefore, the rest of this chapter focuses on financial markets. Self-Test What should be management’s primary objective? How does maximizing the fundamental stock price benefit society? Free cash flow depends on what three factors? How is a firm’s fundamental value related to its free cash flows and its cost of capital? 1.4 AN OVERVIEW OF ALLOCATION PROCESS THE CAPITAL Businesses often need capital to implement growth plans; governments require funds to finance building projects; and individuals frequently want loans to purchase cars, homes, and education. Where can they get this money? Fortunately, there are some individuals and firms with incomes greater than their expenditures. In contrast to William Shakespeare’s advice, most individuals and firms are both borrowers and lenders. For example, an individual might borrow money with a car loan or a home mortgage but might also lend money through a bank savings account. In the aggregate, individuals are net savers and provide most of the funds ultimately used by nonfinancial corporations. Although most nonfinancial corporations own some financial securities, such as short-term Treasury bills, nonfinancial corporations are net borrowers in the aggregate. It should be no surprise to you that in the United States 14 Part 1: Fundamental Concepts of Corporate Finance federal, state, and local governments are also net borrowers in the aggregate (although many foreign governments, such as those of China and oil-producing countries, are actually net lenders). Banks and other financial corporations raise money with one hand and invest it with the other. For example, a bank might raise money from individuals in the form of a savings account and then lend most of that money to business customers. In the aggregate, financial corporations borrow slightly more than they lend. Transfers of capital between savers and those who need capital take place in three different ways. Direct transfers of money and securities, as shown in Panel 1 of Figure 1-1, occur when a business (or government) sells its securities directly to savers. The business delivers its securities to savers, who in turn provide the firm with the money it needs. For example, a privately held company might sell shares of stock directly to a new shareholder, or the U.S. government might sell a Treasury bond directly to an individual investor. As shown in Panel 2, indirect transfers may go through an investment banking house such as Goldman Sachs, which underwrites the issue. An underwriter serves as a middleman and facilitates the issuance of securities. The company sells its stocks or bonds to the investment bank, which in turn sells these same securities to savers. Because new securities are involved and the corporation receives the proceeds of the sale, this is a “primary” market transaction. Transfers can also be made through a financial intermediary such as a bank or mutual fund, as shown in Panel 3. Here the intermediary obtains funds from savers in exchange for its own securities. The intermediary then uses this money to purchase and then hold businesses’ securities. For example, a saver might give dollars to a bank and receive a certificate of deposit, and then the bank might lend the money to a small business, receiving in exchange a signed loan. Thus, intermediaries literally create new types of securities. FIGURE 1-1 Diagram of the Capital Allocation Process 1. Direct Transfers Business’s Securities Business Savers Dollars 2. Indirect Transfers through Investment Bankers Business’s Securities Business Dollars Business’s Securities Investment Banking Houses Dollars Savers 3. Indirect Transfers through a Financial Intermediary Intermediary’s Securities Business’s Securities Business Dollars Financial Intermediary Dollars Savers Chapter 1: An Overview of Financial Management and the Financial Environment 15 There are three important characteristics of the capital allocation process. First, new financial securities are created. Second, financial institutions are often involved. Third, allocation between providers and users of funds occurs in financial markets. The following sections discuss each of these characteristics. Self-Test WWW You can access current and historical interest rates and economic data from the Federal Reserve Economic Data (FRED) site at http://www.stls.frb.org/ fred/. Identify three ways that capital is transferred between savers and borrowers. Distinguish between the roles played by investment banking houses and financial intermediaries. 1.5 FINANCIAL SECURITIES The variety of financial securities is limited only by human creativity, ingenuity, and governmental regulations. At the risk of oversimplification, we can classify most financial securities by the type of claim and the time until maturity. In addition, some securities actually are created from packages of other securities. We discuss the key aspects of financial securities in this section. Type of Claim: Debt, Equity, or Derivatives resource For an overview of derivatives, see Web Extension 1A on the textbook’s Web site. Financial securities are simply pieces of paper with contractual provisions that entitle their owners to specific rights and claims on specific cash flows or values. Debt instruments typically have specified payments and a specified maturity. For example, an Alcoa bond might promise to pay 10% interest for 30 years, at which time it promises to make a $1,000 principal payment. If debt matures in more than a year, it is called a capital market security. Thus, the Alcoa bond in this example is a capital market security. If the debt matures in less than a year, it is a money market security. For example, Home Depot might expect to receive $300,000 in 75 days, but it needs cash now. Home Depot might issue commercial paper, which is essentially an IOU. In this example, Home Depot might agree to pay $300,000 in 75 days in exchange for $297,000 today. Thus, commercial paper is a money market security. Equity instruments are a claim upon a residual value. For example, Alcoa’s stockholders are entitled to the cash flows generated by Alcoa after its bondholders, creditors, and other claimants have been satisfied. Because stock has no maturity date, it is a capital market security. Notice that debt and equity represent claims upon the cash flows generated by real assets, such as the cash flows generated by Alcoa’s factories and operations. In contrast, derivatives are securities whose values depend on, or are derived from, the values of some other traded assets. For example, options and futures are two important types of derivatives, and their values depend on the prices of other assets. An option on Alcoa stock or a futures contract to buy pork bellies are examples of derivatives. We discuss options in Chapter 8 and in Web Extension 1A, which provides a brief overview of options and other derivatives. Some securities are a mix of debt, equity, and derivatives. For example, preferred stock has some features like debt and some like equity, while convertible debt has both debt-like and option-like features. We discuss these and other financial securities in detail later in the book, but Table 1-1 provides a summary of the most important conventional financial securities. We discuss rates of return later in this chapter, but notice now in Table 1-1 that interest rates tend to increase with the maturity and risk of the security. 16 Part 1: Fundamental Concepts of Corporate Finance T AB L E 1 - 1 Summary of Major Financial Instruments INSTRUMENT U.S. Treasury bills Bankers’ acceptances Commercial paper Negotiable certificates of deposit (CDs) Money market mutual funds Eurodollar market time deposits Consumer credit loans Commercial loans U.S. Treasury notes and bonds Mortgages Municipal bonds Corporate bonds Leases Preferred stocks Common stocksd MAJOR P A R TI C I P A N TS Sold by U.S. Treasury A firm’s promise to pay, guaranteed by a bank Issued by financially secure firms to large investors Issued by major banks to large investors Invest in short-term debt; held by individuals and businesses Issued by banks outside U.S. Loans by banks/credit unions/finance companies Loans by banks to corporations Issued by U.S. government Loans secured by property Issued by state and local governments to individuals and institutions Issued by corporations to individuals and institutions Similar to debt; firms lease assets rather than borrow and then buy them Issued by corporations to individuals and institutions Issued by corporations to individuals and institutions ORIGINAL MATU RITY RATES OF RETURN ON 1/ 08/09 a Default-free Low if strong bank guarantees Low default risk 91 days to 1 year Up to 180 days 0.41% 1.5% Up to 270 days 0.28% Depends on strength of issuer Low degree of risk Up to 1 year 1.58% 1.27% Depends on strength of issuer Risk is variable No specific maturity (instant liquidity) Up to 1 year 2.60% Variable Variable Depends on borrower Up to 7 years No default risk, but price falls if interest rates rise Risk is variable Riskier than U.S. government bonds, but exempt from most taxes Riskier than U.S. government debt; depends on strength of issuer Risk similar to corporate bonds 2 to 30 years Tied to prime rate (3.25%) or LIBOR (2.02%)b 3.04% Up to 30 years Up to 30 years 5.02% 5.02% Up to 40 yearsc 5.03% Generally 3 to 20 years Similar to bond yields Riskier than corporate bonds Unlimited 6% to 9% Riskier than preferred stocks Unlimited 9% to 15% RISK a Data are from The Wall Street Journal (http://online.wsj.com) or the Federal Reserve Statistical Release (http://www.federalreserve .gov/releases/H15/update). Bankers’ acceptances assume a 3-month maturity. Money market rates are for the Merrill Lynch Ready Assets Trust. The corporate bond rate is for AAA-rated bonds. b The prime rate is the rate U.S. banks charge to good customers. LIBOR (London Interbank Offered Rate) is the rate that U.K. banks charge one another. c A few corporations have issued 100-year bonds; however, most have issued bonds with maturities of less than 40 years. d Common stocks are expected to provide a “return” in the form of dividends and capital gains rather than interest. Of course, if you buy a stock, your actual return may be considerably higher or lower than your expected return. Some securities are created from packages of other securities, a process called securitization. The misuse of securitized assets is one of the primary causes of the global financial crisis, so we discuss securitization next. Chapter 1: An Overview of Financial Management and the Financial Environment 17 The Process of Securitization Many types of assets can be securitized, but we will focus on mortgages because they played such an important role in the global financial crisis. At one time, most mortgages were made by savings and loan associations (S&Ls), which took in the vast majority of their deposits from individuals who lived in nearby neighborhoods. The S&Ls pooled these deposits and then lent money to people in the neighborhood in the form of fixed-rate mortgages, which were pieces of paper signed by borrowers promising to make specified payments to the S&L. The new homeowners paid principal and interest to the S&L, which then paid interest to its depositors and reinvested the principal repayments in other mortgages. This was clearly better than having individuals lend directly to aspiring homeowners, because a single individual might not have enough money to finance an entire house nor the expertise to know if the borrower was creditworthy. Note that the S&Ls were government-chartered institutions, and they obtained money in the form of immediately withdrawable deposits and then invested most of it in the form of mortgages with fixed interest rates and on individual homes. Also, initially the S&Ls were not permitted to have branch operations—they were limited to one office so as to maintain their local orientation. These restrictions had important implications. First, in the 1950s there was a massive migration of people to the west, so there was a strong demand for funds in that area. However, the wealthiest savers were in the east. That meant that mortgage interest rates were much higher in California and other western states than in New York and the east. This created disequilibrium, something that can’t exist forever in financial markets. Second, note that the S&Ls’ assets consisted mainly of long-term, fixed-rate mortgages, but their liabilities were in the form of deposits that could be withdrawn immediately. The combination of long-term assets and short-term liabilities created another problem. If the overall level of interest rates increased, the S&Ls would have to increase the rates they paid on deposits or else savers would take their money elsewhere. However, the S&Ls couldn’t increase the rates on their outstanding mortgages because these mortgages had fixed interest rates. This problem came to a head in the 1960s, when the Vietnam War led to inflation, which pushed up interest rates. At this point, the “money market fund” industry was born, and it literally sucked money out of the S&Ls, forcing many of them into bankruptcy. The government responded by giving the S&Ls broader lending powers, permitting nationwide branching, and allowing them to obtain funds as long-term debt in addition to immediately withdrawable deposits. Unfortunately, these changes had another set of unintended consequences. S&L managers who had previously dealt with a limited array of investments and funding choices in local communities were suddenly allowed to expand their scope of operations. Many of these inexperienced S&L managers made poor business decisions and the result was disastrous—virtually the entire S&L industry collapsed, with many S&Ls going bankrupt or being acquired in shotgun mergers with commercial banks. The demise of the S&Ls created another financial disequilibrium—a higher demand for mortgages than the supply of available funds from the mortgage lending industry. Savings were accumulating in pension funds, insurance companies, and other institutions, not in S&Ls and banks, the traditional mortgage lenders. This situation led to the development of “mortgage securitization,” a process whereby banks, the remaining S&Ls, and specialized mortgage originating firms would originate mortgages and then sell them to investment banks, which would bundle them into packages and then use these packages as collateral for bonds that could be sold to 18 Part 1: Fundamental Concepts of Corporate Finance pension funds, insurance companies, and other institutional investors. Thus, individual loans were bundled and then used to back a bond—a “security”—that could be traded in the financial markets. Congress facilitated this process by creating two stockholder-owned but governmentsponsored entities, the Federal National Mortgage Association (Fannie Mae) and the Federal Home Loan Mortgage Corporation (Freddie Mac). Fannie Mae and Freddie Mac were financed by issuing a relatively small amount of stock and a huge amount of debt. To illustrate the securitization process, suppose an S&L or bank is paying its depositors 5% but is charging its borrowers 8% on their mortgages. The S&L can take hundreds of these mortgages, put them in a pool, and then sell the pool to Fannie Mae. The mortgagees can still make their payments to the original S&L, which will then forward the payments (less a small handling fee) to Fannie Mae. Consider the S&L’s perspective. First, it can use the cash it receives from selling the mortgages to make additional loans to other aspiring homeowners. Second, the S&L is no longer exposed to the risk of owning mortgages. The risk hasn’t disappeared—it has been transferred from the S&L (and its federal deposit insurers) to Fannie Mae. This is clearly a better situation for aspiring homeowners and perhaps also for taxpayers. Fannie Mae can take the mortgages it just bought, put them into a very large pool, and sell bonds backed by the pool to investors. The homeowner will pay the S&L, the S&L will forward the payment to Fannie Mae, and Fannie Mae will use the funds to pay interest on the bonds it issued, to pay dividends on its stock, and to buy additional mortgages from S&Ls, which can then make additional loans to aspiring homeowners. Notice that the mortgage risk has been shifted from Fannie Mae to the investors who now own the mortgage-backed bonds. How does the situation look from the perspective of the investors who own the bonds? In theory, they own a share in a large pool of mortgages from all over the country, so a problem in a particular region’s real estate market or job market won’t affect the whole pool. Therefore, their expected rate of return should be very close to the 8% rate paid by the home-owning mortgagees. (It will be a little less due to handling fees charged by the S&L and Fannie Mae and to the small amount of expected losses from the homeowners who could be expected to default on their mortgages.) These investors could have deposited their money at an S&L and earned a virtually risk-free 5%. Instead, they chose to accept more risk in hopes of the higher 8% return. Note too that mortgage-backed bonds are more liquid than individual mortgage loans, so the securitization process increases liquidity, which is desirable. The bottom line is that risk has been reduced by the pooling process and then allocated to those who are willing to accept it in return for a higher rate of return. Thus, in theory it is a win–win–win situation: More money is available for aspiring homeowners, S&Ls (and taxpayers) have less risk, and there are opportunities for investors who are willing to take on more risk to obtain higher potential returns. Although the securitization process began with mortgages, it is now being used with car loans, student loans, credit card debt, and other loans. The details vary for different assets, but the processes and benefits are similar to those with mortgage securitization: (1) increased supplies of lendable funds; (2) transfer of risk to those who are willing to bear it; and (3) increased liquidity for holders of the debt. Mortgage securitization was a win–win situation in theory, but as practiced in the last decade it has turned into a lose–lose situation. We will have more to say about securitization and the global economic crisis later in this chapter, but first let’s take a look at the cost of money. Chapter 1: An Overview of Financial Management and the Financial Environment 1.6 THE COST OF 19 MONEY In a free economy, capital from those with available funds is allocated through the price system to users who have a need for funds. The interaction of the providers’ supply and the users’ demand determines the cost (or price) of money, which is the rate users pay to providers. For debt, we call this price the interest rate. For equity, we call it the cost of equity, and it consists of the dividends and capital gains stockholders expect. Keep in mind that the “price” of money is a cost from a user’s perspective but a return from the provider’s point of view. Notice in Table 1-1 that a financial instrument’s rate of return generally increases as its maturity and risk increase. We will have much more to say about the relationships among an individual security’s features, risk, and return later in the book, but there are some fundamental factors and economic conditions that affect all financial instruments. Fundamental Factors That Affect the Cost of Money The four most fundamental factors affecting the cost of money are (1) production opportunities, (2) time preferences for consumption, (3) risk, and (4) inflation. By production opportunities, we mean the ability to turn capital into benefits. If a business raises capital, the benefits are determined by the expected rates of return on its production opportunities. If a student borrows to finance his or her education, the benefits are higher expected future salaries (and, of course, the sheer joy of learning!). If a homeowner borrows, the benefits are the pleasure from living in his or her own home, plus any expected appreciation in the value of the home. Observe that the expected rates of return on these “production opportunities” put an upper limit on how much users can pay to providers. Providers can use their current funds for consumption or saving. By saving, they give up consumption now in the expectation of having more consumption in the future. If providers have a strong preference for consumption now, then it takes high interest rates to induce them to trade current consumption for future consumption. Therefore, the time preference for consumption has a major impact on the cost of money. Notice that the time preference for consumption varies for different individuals, for different age groups, and for different cultures. For example, people in Japan have a lower time preference for consumption than those in the United States, which partially explains why Japanese families tend to save more than U.S. families even though interest rates are lower in Japan. If the expected rate of return on an investment is risky, then providers require a higher expected return to induce them to take the extra risk, which drives up the cost of money. As you will see later in this book, the risk of a security is determined by market conditions and the security’s particular features. Inflation also leads to a higher cost of money. For example, suppose you earned 10% one year on your investment but inflation caused prices to increase by 20%. This means you can’t consume as much at the end of the year as when you originally invested your money. Obviously, if you had expected 20% inflation, you would have required a higher rate of return than 10%. Economic Conditions and Policies That Affect the Cost of Money Economic conditions and policies also affect the cost of money. These include: (1) Federal Reserve policy; (2) the federal budget deficit or surplus; (3) the level of 20 Part 1: Fundamental Concepts of Corporate Finance WWW The home page for the Board of Governors of the Federal Reserve System can be found at http:// www.federalreserve.gov. You can access general information about the Federal Reserve, including press releases, speeches, and monetary policy. business activity; and (4) international factors, including the foreign trade balance, the international business climate, and exchange rates. Federal Reserve Policy. If the Federal Reserve Board wants to stimulate the economy, it most often uses open market operations to purchases Treasury securities held by banks. Because banks are selling some of their securities, the banks will have more cash, which increases their supply of loanable funds, which in turn makes banks willing to lend more money at lower interest rates. In addition, the Fed’s purchases represent an increase in the demand for Treasury securities. As for anything that is for sale, increased demand causes Treasury securities’ prices to go up and interest rates to go down (we explain the mathematical relationship between higher prices and lower interest rates in Chapter 4; for now, just trust us when we say that a security’s price and its interest rate move in opposite directions). The net result is a reduction in interest rates, which stimulates the economy by making it less costly for companies to borrow for new projects or for individuals to borrow for major purchases or other expenditures. When banks sell their holdings of Treasury securities to the Fed, the banks’ reserves go up, which increases the money supply. A larger money supply ultimately leads to an increase in expected inflation, which eventually pushes interest rates up. Thus, the Fed can stimulate the economy in the short term by driving down interest rates and increasing the money supply, but this creates longer-term inflationary pressures. This is exactly the dilemma facing the Fed in early 2009 as it attempts to stimulate the economy to prevent another great depression. If the Fed wishes to slow down the economy and reduce inflation, the Fed reverses the process. Instead of purchasing Treasury securities, the Fed sells Treasury securities to banks, which causes an increase in short-term interest rates but a decrease in long-term inflationary pressures. Budget Deficits or Surpluses. If the federal government spends more than it takes in from tax revenues then it’s running a deficit, and that deficit must be covered either by borrowing or by printing money (increasing the money supply). The government borrows by issuing new Treasury securities. All else held equal, this creates a greater supply of Treasury securities, which leads to lower security prices and higher interest rates. Other federal government actions that increase the money supply also increase expectations for future inflation, which drives up interest rates. Thus, the larger the federal deficit, other things held constant, the higher the level of interest rates. As shown in Figure 1-2, the federal government has run large budget deficits for 12 of the past 16 years, and even larger deficits are predicted for at least several years into the future. These deficits contributed to the cumulative federal debt, which stood at over $11 trillion at the beginning of 2009. Business Activity. Figure 1-3 shows interest rates, inflation, and recessions. Notice that interest rates and inflation typically rise prior to a recession and fall afterward. There are several reasons for this pattern. Consumer demand slows during a recession, keeping companies from increasing prices, which reduces price inflation. Companies also cut back on hiring, which reduces wage inflation. Less disposable income causes consumers to reduce their purchases of homes and automobiles, reducing consumer demand for loans. Companies reduce investments in new operations, which reduces their demand for funds. The cumulative effect is downward pressure on inflation and interest rates. The Federal Reserve is also active during recessions, trying to stimulate the economy by driving down interest rates. Chapter 1: An Overview of Financial Management and the Financial Environment FIGURE 1-2 21 Federal Budget Surplus/Deficits and Trade Balances (Billions of Dollars) Surplus or Deficit Federal Budget Surplus/Deficit 400 200 0 –200 –400 Trade Balance 2008 2007 2006 2005 2004 2003 2002 2001 2000 1999 1998 1997 1996 1995 1994 –800 1993 –600 Notes: 1. Years are for federal government fiscal years, which end on September 30. 2. Federal budget surplus/deficit data are from the Congressional Budget Office, http://www.cbo.gov/. 3. Data for international trade balances are from the St. Louis Federal Reserve Web site known as FRED: http://research.stlouisfed.org/fred/. International Trade Deficits or Surpluses. Businesses and individuals in the United States buy from and sell to people and firms in other countries. If we buy more than we sell (that is, if we import more than we export), we are said to be running a foreign trade deficit. When trade deficits occur, they must be financed, and the main source of financing is debt. In other words, if we import $200 billion of goods but export only $90 billion, we run a trade deficit of $110 billion, and we will probably borrow the $110 billion.4 Therefore, the larger our trade deficit, the more we must borrow, and increased borrowing drives up interest rates. Also, international investors are willing to hold U.S. debt if and only if the risk-adjusted rate paid on this debt is competitive with interest rates in other countries. Therefore, if the Federal Reserve attempts to lower interest rates in the United States, causing our rates to fall below rates abroad (after adjustments for expected changes in the exchange rate), then international investors will sell U.S. bonds, which will depress bond prices and result in higher U.S. rates. Thus, if the trade deficit is large relative to the size of The deficit could also be financed by selling assets, including gold, corporate stocks, entire companies, and real estate. The United States has financed its massive trade deficits by all of these means in recent years, but the primary method has been by borrowing from foreigners. 4 22 Part 1: Fundamental Concepts of Corporate Finance FIGURE 1-3 Business Activity, Interest Rates, and Inflation Interest Rate (%) 16 Interest Rates 14 12 10 8 Inflation 6 4 2009 2007 2005 2003 2001 1999 1997 1995 1993 1991 1989 1987 1985 1983 1981 1979 1977 1975 1971 0 1973 2 Notes: 1. Tick marks represent January 1 of the year. 2. The shaded areas designate business recessions as defined by the National Bureau of Economic Research; see http://www.nber.org/cycles. 3. Interest rates are for AAA corporate bonds; see the St. Louis Federal Reserve Web site: http://research.stlouisfed.org/fred/. These rates reflect the average rate during the month ending on the date shown. 4. Inflation is measured by the annual rate of change for the Consumer Price Index (CPI) for the preceding 12 months; see http://research.stlouisfed.org/fred/. WWW Transparency International provides a ranking of countries based on their levels of perceived corruption. See http://www .transparency.org/policy_ research/surveys_indices/ cpi/2008. The U.S. Department of State provides thorough descriptions of countries’ business climates at http://www.state .gov/e/eeb/ifd/2008/. the overall economy, it will hinder the Fed’s ability to reduce interest rates and combat a recession. The United States has been running annual trade deficits since the mid-1970s; see Figure 1-2 for recent years. The cumulative effect of trade deficits and budget deficits is that the United States has become the largest debtor nation of all time. As noted earlier, this federal debt has exceeded $11 trillion! As a result, our interest rates are very much influenced by interest rates in other countries around the world. International Country Risk. International risk factors may increase the cost of money that is invested abroad. Country risk is the risk that arises from investing or doing business in a particular country, and it depends on the country’s economic, political, and social environment. Countries with stable economic, social, political, and regulatory systems provide a safer climate for investment and therefore have less country risk than less stable nations. Examples of country risk include the risk associated with changes in tax rates, regulations, currency conversion, and exchange rates. Country risk also includes the risk that (1) property will be expropriated without adequate compensation; (2) the host country will impose new stipulations concerning local production, sourcing, or hiring practices; and (3) there might be damage or destruction of facilities due to internal strife. Chapter 1: An Overview of Financial Management and the Financial Environment 23 Exchange Rate Risk. International securities frequently are denominated in a currency other than the dollar, which means that the value of an investment depends on what happens to exchange rates. This is known as exchange rate risk. For example, if a U.S. investor purchases a Japanese bond, interest will probably be paid in Japanese yen, which must then be converted to dollars if the investor wants to spend his or her money in the United States. If the yen weakens relative to the dollar, then the yen will buy fewer dollars when it comes time for the investor to convert the Japanese bond’s payout. Alternatively, if the yen strengthens relative to the dollar, the investor will earn higher dollar returns. It therefore follows that the effective rate of return on a foreign investment will depend on both the performance of the foreign security in its home market and on what happens to exchange rates over the life of the investment. We discuss exchange rates in detail in Chapter 17. Self-Test What four fundamental factors affect the cost of money? Name some economic conditions that influence interest rates and explain their effects. 1.7 FINANCIAL INSTITUTIONS When raising capital, direct transfers of funds from individuals to businesses are most common for small businesses or in economies where financial markets and institutions are not well developed. Businesses in developed economies usually find it more efficient to enlist the services of one or more financial institutions to raise capital. Most financial institutions don’t compete in a single line of business but instead provide a wide variety of services and products, both domestically and globally. The following sections describe the major types of financial institutions and services, but keep in mind that the dividing lines among them are often blurred. Also, note that the global financial crisis we are now going through is changing the structure of our financial institutions, and new regulations are certain to affect those that remain. Finance today is dynamic, to say the least! Investment Banks and Brokerage Activities Investment banking houses help companies raise capital. Such organizations underwrite security offerings, which means they (1) advise corporations regarding the design and pricing of new securities, (2) buy these securities from the issuing corporation, and (3) resell them to investors. Although the securities are sold twice, this process is really one primary market transaction, with the investment banker acting as a facilitator to help transfer capital from savers to businesses. An investment bank often is a division or subsidiary of a larger company. For example, JPMorgan Chase & Co. is a very large financial services firm, with over $2 trillion in managed assets. One of its holdings is J.P. Morgan, an investment banking house. In addition to security offerings, investment banks also provide consulting and advisory services, such as merger and acquisition (M&A) analysis and investment management for wealthy individuals. Most investment banks also provide brokerage services for institutions and individuals (called “retail” customers). For example, Merrill Lynch (acquired in 2008 by Bank of America) has a large retail brokerage operation that provides advice and executes trades for its individual clients. Similarly, J.P. Morgan helps execute trades for institutional customers, such as pension funds. At one time, most investment banks were partnerships, with income generated primarily by fees from their underwriting, M&A consulting, asset management, and 24 Part 1: Fundamental Concepts of Corporate Finance brokering activities. When business was good, investment banks generated high fees and paid big bonuses to their partners. When times were tough, investment banks paid no bonuses and often fired employees. In the 1990s, however, most investment banks were reorganized into publicly traded corporations (or were acquired and then operated as subsidiaries of public companies). For example, in 1994 Lehman Brothers sold some of its own shares of stock to the public via an IPO. Like most corporations, Lehman Brothers was financed by a combination of equity and debt. A relaxation of regulations in the 2000s allowed investment banks to undertake much riskier activities than at any time since the Great Depression. Basically, the new regulations allowed investment banks to use an unprecedented amount of debt to finance their activities—Lehman used roughly $30 of debt for every dollar of equity. In addition to their fee-generating activities, most investment banks also began trading securities for their own accounts. In other words, they took the borrowed money and invested it in financial securities. If you are earning 12% on your investments while paying 8% on your borrowings, then the more money you borrow, the more profit you make. But if you are leveraged 30 to 1 and your investments decline in value by even 3.33%, your business will fail. This is exactly what happened to Bear Stearns, Lehman Brothers, and Merrill Lynch in the fall of 2008. In short, they borrowed money, used it to make risky investments, and then failed when the investments turned out to be worth less than the amount they owed. Notice that it was not their traditional investment banking activities that caused the failure, but the fact that they borrowed so much and used those funds to speculate in the market. Deposit-Taking Financial Intermediaries Some financial institutions take deposits from savers and then lend most of the deposited money to borrowers. Following is a brief description of such intermediaries. Savings and Loan Associations (S&Ls). As we explained in Section 1.5, S&Ls originally accepted deposits from many small savers and then loaned this money to home buyers and consumers. Later, they were allowed to make riskier investments, such as real estate development. Mutual savings banks (MSBs) are similar to S&Ls, but they operate primarily in the northeastern states. Today, most S&Ls and MSBs have been acquired by banks. Credit Unions. Credit unions are cooperative associations whose members have a common bond, such as being employees of the same firm or living in the same geographic area. Members’ savings are loaned only to other members, generally for auto purchases, home improvement loans, and home mortgages. Credit unions are often the cheapest source of funds available to individual borrowers. Commercial Banks. Commercial banks raise funds from depositors and by issuing stock and bonds to investors. For example, someone might deposit money in a checking account. In return, that person can write checks, use a debit card, and even receive interest on the deposits. Those who buy the banks’ stocks expect to receive dividends and interest payments. Unlike nonfinancial corporations, most commercial banks are highly leveraged in the sense that they owe much more to their depositors and creditors than they raised from stockholders. For example, a typical bank has about $90 of debt for every $10 of stockholders’ equity. If the bank’s assets are worth $100, we can calculate its equity capital by subtracting the $90 of liabilities from the $100 of assets: Equity capital = $100 − $90 = $10. But if the assets drop in value by 5% to $95, the equity drops to $5 = $95 − $90, a 50% decline. Chapter 1: An Overview of Financial Management and the Financial Environment 25 Banks are critically important to a well-functioning economy, and their high leverage makes them risky. As a result, banks are more highly regulated than nonfinancial firms. Given the high risk, banks might have a hard time attracting and retaining deposits unless the deposits were insured, so the Federal Deposit Insurance Corporation (FDIC), which is backed by the U.S. government, insures up to $250,000 per depositor. As a result of the global economic crisis, this insured amount was increased from $100,000 in 2008 to reassure depositors. Without such insurance, if depositors believed that a bank was in trouble, they would rush to withdraw funds. This is called a “bank run,” which is exactly what happened in the United States during the Great Depression, causing many bank failures and leading to the creation of the FDIC in an effort to prevent future bank runs. Not all countries have their own versions of the FDIC, so international bank runs are still possible. In fact, a bank run occurred in September 2008 at the U.K. bank Northern Rock, leading to its nationalization by the government. Most banks are small and locally owned, but the largest banks are parts of giant financial services firms. For example, JPMorgan Chase Bank, commonly called Chase Bank, is owned by JPMorgan Chase & Co., and Citibank is owned by Citicorp (at the time we write this, but perhaps not when you read this—the financial landscape is changing daily). Investment Funds At some financial institutions, savers have an ownership interest in a pool of funds rather than owning a deposit account. Examples include mutual funds, hedge funds, and private equity funds. Mutual Funds. Mutual funds are corporations that accept money from savers and then use these funds to buy financial instruments. These organizations pool funds, which allows them to reduce risks by diversification and achieve economies of scale in analyzing securities, managing portfolios, and buying/selling securities. Different funds are designed to meet the objectives of different types of savers. Hence, there are bond funds for those who desire safety and stock funds for savers who are willing to accept risks in the hope of higher returns. There are literally thousands of different mutual funds with dozens of different goals and purposes. Some funds are actively managed, with their managers trying to find undervalued securities, while other funds are passively managed and simply try to minimize expenses by matching the returns on a particular market index. Money market funds invest in short-term, low-risk securities, such as Treasury bills and commercial paper. Many of these funds offer interest-bearing checking accounts with rates that are greater than those offered by banks, so many people invest in mutual funds as an alternative to depositing money in a bank. Note, though, that money market funds are not required to be insured by the FDIC and so are riskier than bank deposits. Most traditional mutual funds allow investors to redeem their share of the fund only at the close of business. A special type of mutual fund, the exchange traded fund (ETF), allows investors to sell their share at any time during normal trading hours. ETFs usually have very low management expenses and are rapidly gaining in popularity. Hedge Funds. Hedge funds raise money from investors and engage in a variety of investment activities. Unlike typical mutual funds, which can have thousands of investors, hedge funds are limited to institutional investors and a relatively small 26 Part 1: Fundamental Concepts of Corporate Finance number of high–net-worth individuals. Because these investors are supposed to be sophisticated, hedge funds are much less regulated than mutual funds. The first hedge funds literally tried to hedge their bets by forming portfolios of conventional securities and derivatives in such a way as to limit their potential losses without sacrificing too much of their potential gains. Recently, though, most hedge funds began to lever their positions by borrowing heavily. Many hedge funds had spectacular rates of return during the 1990s. This success attracted more investors, and thousands of new hedge funds were created. Much of the low-hanging fruit had already been picked, however, so the hedge funds began pursuing much riskier (and unhedged) strategies. Perhaps not surprisingly (at least in retrospect), some funds have produced spectacular losses. For example, many hedge fund investors suffered huge losses in 2007 and 2008 when large numbers of sub-prime mortgages defaulted. Private Equity Funds. Private equity funds are similar to hedge funds in that they are limited to a relatively small number of large investors, but they differ in that they own stock (equity) in other companies and often control those companies, whereas hedge funds usually own many different types of securities. In contrast to a mutual fund, which might own a small percentage of a publicly traded company’s stock, a private equity fund typically owns virtually all of a company’s stock. Because the company’s stock is not traded in the public markets, it is called “private equity.” In fact, private equity funds often take a public company (or subsidiary) and turn it private, such as the 2007 privatization of Chrysler by Cerberus. The general partners who manage the private equity funds usually sit on the boards of the companies the funds owns and guide the firms’ strategies with the goal of later selling them for a profit. For example, The Carlyle Group, Clayton Dubilier & Rice, and Merrill Lynch Global Private Equity bought Hertz from Ford on December 22, 2005, and then sold shares of Hertz in an IPO less than a year later. Chapter 15 provides additional discussion of private equity funds, but it is important to note here that many private equity funds experienced high rates of return in the last decade, and those returns attracted enormous sums from investors. A few funds, most notably The Blackstone Group, actually went public themselves through an IPO. Just as with hedge funds, the performance of many private equity funds faltered. For example, shortly after its IPO in June 2007, Blackstone’s stock price was over $31 per share; by early 2009, it had fallen to about $4. Life Insurance Companies and Pension Funds Life insurance companies take premiums, invest these funds in stocks, bonds, real estate, and mortgages, and then make payments to beneficiaries. Life insurance companies also offer a variety of tax-deferred savings plans designed to provide retirement benefits. Traditional pension funds are retirement plans funded by corporations or government agencies. Pension funds invest primarily in bonds, stocks, mortgages, hedge funds, private equity, and real estate. Most companies now offer selfdirected retirement plans, such as 401(k) plans, as an addition to or substitute for traditional pension plans. In traditional plans, the plan administrators determine how to invest the funds; in self-directed plans, all individual participants must decide how to invest their own funds. Many companies are switching from traditional plans to self-directed plans, partly because this shifts the risk from the company to the employee. Chapter 1: An Overview of Financial Management and the Financial Environment 27 Regulation of Financial Institutions With the notable exception of investment banks, hedge funds, and private equity funds, financial institutions have been heavily regulated to ensure their safety and thus protect investors and depositors. Historically, many of these regulations—which have included a prohibition on nationwide branch banking, restrictions on the types of assets the institutions could buy, ceilings on the interest rates they could pay, and limitations on the types of services they could provide—tended to impede the free flow of capital and thus hurt the efficiency of our capital markets. Recognizing this fact, policymakers took several steps from the 1970s to the 1990s to deregulate financial services companies. For example, the barriers that restricted banks from expanding nationwide were eliminated. Likewise, regulations that once forced a strict separation of commercial and investment banking were relaxed. The result of the ongoing regulatory changes has been a blurring of the distinctions between the different types of institutions. Indeed, the trend in the United States was toward huge financial services corporations, which own banks, S&Ls, investment banking houses, insurance companies, pension plan operations, and mutual funds and which have branches across the country and around the world. For example, Citigroup combined one of the world’s largest commercial banks (Citibank), a huge insurance company (Travelers), and a major investment bank (Smith Barney), along with numerous other subsidiaries that operate throughout the world. This structure was similar to that of major institutions in Europe, Japan, and elsewhere around the globe. Among the world’s largest world banking companies, only one (Citigroup) is based in the United States. While U.S. banks have grown dramatically as a result of recent mergers, they are still relatively small by global standards. However, the global economic crisis is causing regulators and financial institutions to rethink the wisdom of conglomerate financial services corporations. For example, in late 2008 Merrill Lynch sold itself to Bank of America to avoid bankruptcy. That was supposed to strengthen BofA, but Merrill brought with it billions of “toxic” loans, and now BofA is in danger of bankruptcy. Then, in early 2009 Citigroup was reorganizing itself in preparation for spinning off several lines of business into separate companies, again with the bankruptcy gun pointed straight at its head. Thus, the two largest U.S. banks are in danger of failure, and their continued survival is due primarily to support from the U.S. government. Congress and the new Obama administration are currently (mid-2009) considering new regulations on a variety of financial institutions, and more bank failures are a certainty. As the crisis unfolds, it will be interesting to see how regulations and the structure of financial institutions evolve to reshape our financial infrastructure, both in the U.S. and around the globe. Self-Test What is the difference between a pure commercial bank and a pure investment bank? List the major types of financial institutions, and briefly describe the original purpose of each. What are some important differences between mutual funds and hedge funds? How are they similar? 1.8 FINANCIAL MARKETS Financial markets bring together people and organizations needing money with those having surplus funds. There are many different financial markets in a developed economy. Each market deals with a somewhat different type of instrument, customer, or geographic location. Here are some ways to classify markets: 28 Part 1: Fundamental Concepts of Corporate Finance 1. Physical asset markets (also called “tangible” or “real” asset markets) are those for such products as wheat, autos, real estate, computers, and machinery. Financial asset markets, on the other hand, deal with stocks, bonds, notes, mortgages, derivatives, and other financial instruments. 2. Spot markets and futures markets are markets where assets are being bought or sold for “on-the-spot” delivery (literally, within a few days) or for delivery at some future date, such as 6 months or a year into the future. 3. Money markets are the markets for short-term, highly liquid debt securities, while capital markets are the markets for corporate stocks and debt maturing more than a year in the future. The New York Stock Exchange is an example of a capital market. When describing debt markets, “short term” generally means less than 1 year, “intermediate term” means 1 to 5 years, and “long term” means more than 5 years. 4. Mortgage markets deal with loans on residential, agricultural, commercial, and industrial real estate, while consumer credit markets involve loans for autos, appliances, education, vacations, and so on. 5. World, national, regional, and local markets also exist. Thus, depending on an organization’s size and scope of operations, it may be able to borrow or lend all around the world, or it may be confined to a strictly local, even neighborhood, market. 6. Primary markets are the markets in which corporations raise new capital. If Microsoft were to sell a new issue of common stock to raise capital, this would be a primary market transaction. The corporation selling the newly created stock receives the proceeds from such a transaction. The initial public offering (IPO) market is a subset of the primary market. Here firms “go public” by offering shares to the public for the first time. Microsoft had its IPO in 1986. Previously, Bill Gates and other insiders owned all the shares. In many IPOs, the insiders sell some of their shares and the company sells newly created shares to raise additional capital. Secondary markets are markets in which existing, already outstanding securities are traded among investors. Thus, if you decided to buy 1,000 shares of AT&T stock, the purchase would occur in the secondary market. The New York Stock Exchange is a secondary market, since it deals in outstanding (as opposed to newly issued) stocks. Secondary markets also exist for bonds, mortgages, and other financial assets. The corporation whose securities are being traded is not involved in a secondary market transaction and, thus, does not receive any funds from such a sale. 7. Private markets, where transactions are worked out directly between two parties, are differentiated from public markets, where standardized contracts are traded on organized exchanges. Bank loans and private placements of debt with insurance companies are examples of private market transactions. Since these transactions are private, they may be structured in any manner that appeals to the two parties. By contrast, securities that are issued in public markets (for example, common stock and corporate bonds) are ultimately held by a large number of individuals. Public securities must have fairly standardized contractual features because public investors cannot afford the time to study unique, nonstandardized contracts. Hence private market securities are more tailor-made but less liquid, whereas public market securities are more liquid but subject to greater standardization. The distinctions among markets are often blurred. For example, it makes little difference if a firm borrows for 11, 12, or 13 months and thus whether such borrowing Chapter 1: An Overview of Financial Management and the Financial Environment 29 is a “money” or “capital” market transaction. You should recognize the big differences among types of markets, but don’t get hung up trying to distinguish them at the boundaries. Self-Test Distinguish between (1) physical asset markets and financial asset markets, (2) spot and futures markets, (3) money and capital markets, (4) primary and secondary markets, and (5) private and public markets. 1.9 TRADING PROCEDURES IN FINANCIAL MARKETS A huge volume of trading occurs in the secondary markets. Although there are many secondary markets for a wide variety of securities, we can classify their trading procedures along two dimensions: location and method of matching orders. Physical Location versus Electronic Network A secondary market can be either a physical location exchange or a computer/ telephone network. For example, the New York Stock Exchange, the American Stock Exchange (AMEX), the Chicago Board of Trade (the CBOT trades futures and options), and the Tokyo Stock Exchange are all physical location exchanges. In other words, the traders actually meet and trade in a specific part of a specific building. In contrast, Nasdaq, which trades a number of U.S. stocks, is a network of linked computers. Other network examples are the markets for U.S. Treasury bonds and foreign exchange, which are conducted via telephone and/or computer networks. In these electronic markets, the traders never see one another except maybe for cocktails after work. By their very nature, networks are less transparent than physical location exchanges. For example, credit default swaps are traded directly between buyers and sellers, and there is no easy mechanism for recording, aggregating, and reporting the transactions or the net positions of the buyers and sellers. Matching Orders: Auctions, Dealers, and ECNs The second dimension is the way orders from sellers and buyers are matched. This can occur through an open outcry auction system, through dealers, or by automated order matching. An example of an outcry auction is the CBOT, where traders actually meet in a pit and sellers and buyers communicate with one another through shouts and hand signals. In a dealer market, there are “market makers” who keep an inventory of the stock (or other financial instrument) in much the same way that any merchant keeps an inventory. These dealers list bid and ask quotes, which are the prices at which they are willing to buy or sell. Computerized quotation systems keep track of all bid and asked prices, but they don’t actually match buyers and sellers. Instead, traders must contact a specific dealer to complete the transaction. Nasdaq (U.S. stocks) is one such market, as are the London SEAQ (U.K. stocks) and the Neuer Market (stocks of small German companies). The third method of matching orders is through an electronic communications network (ECN). Participants in an ECN post their orders to buy and sell, and the ECN automatically matches orders. For example, someone might place an order to buy 1,000 shares of IBM stock—this is called a “market order” since it is to buy the stock at the current market price. Suppose another participant had placed an order to sell 1,000 shares of IBM, but only at a price of $91 per share, and this was the lowest 30 Part 1: Fundamental Concepts of Corporate Finance price of any “sell” order. The ECN would automatically match these two orders, execute the trade, and notify both participants that the trade has occurred. The $91 sell price was a “limit order” as opposed to a market order because the action was limited by the seller. Note that orders can also be limited with regard to their duration. For example, someone might stipulate that they are willing to buy 1,000 shares of IBM at $90 per share if the price falls that low during the next two hours. In other words, there are limits on the price and/or the duration of the order. The ECN will execute the limit order only if both conditions are met. Two of the largest ECNs for trading U.S. stocks are Instinet (now owned by Nasdaq) and Archipelago (now owned by the NYSE). Other large ECNs include Eurex, a Swiss–German ECN that trades futures contracts, and SETS, a U.K. ECN that trades stocks. Self-Test What are the major differences between physical location exchanges and computer/ telephone networks? What are the differences among open outcry auctions, dealer markets, and ECNs? 1.10 TYPES resource For more on issuing stock, see Web Extension 1B on the textbook’s Web site. WWW For updates on IPO activity, see http://www.ipohome .com/IPOHome/Review/ 2008main.aspx or http:// www.hoovers.com/global/ ipoc/index.xhtml. The Wall Street Journal also provides IPO data in its YearEnd Review of Markets & Finance at http://online .wsj.com. See Professor Jay Ritter’s Web site for additional IPO data and analysis, http://bear.cba .ufl.edu/ritter/ipodata.htm. OF STOCK MARKET TRANSACTIONS Because the primary objectives of financial management are to maximize the firm’s intrinsic value and then help ensure that the current stock price equals that value, knowledge of the stock market is important to anyone involved in managing a business. We can classify stock market transactions into three distinct types: (1) initial public offerings, (2) seasoned equity offerings, and (3) secondary market transactions. Whenever stock is offered to the public for the first time, the company is said to be going public. This primary market transaction is called the initial public offering (IPO) market. If a company later decides to sell (i.e., issue) additional shares to raise new equity capital, this is still a primary market, but it is called a seasoned equity offering. Trading in the outstanding shares of established, publicly owned companies are secondary market transactions. For example, if the owner of 100 shares of publicly held stock sells his or her stock, the trade is said to have occurred in the secondary market. Thus, the market for outstanding shares, or used shares, is the secondary market. The company receives no new money when sales occur in this market. Here is a brief description of recent IPO activity. The 662 total global IPOs in 2008 was a huge decline from the 1,711 in 2007. Proceeds also plummeted, to $77 billion from $279 billion. The Americas raised more money than any other region in the world, with the United States having 33 IPOs that raised a total of $26.4 billion. Visa’s IPO was the largest in the world, bringing in over $19 billion. In the United States, the average first-day return was around 5.3% in 2008. However, some firms had spectacular first-day price run-ups, such as Intrepid Potash’s 57% gain on its first day of trading and Grand Canyon Education’s 59.7% gain for the year. However, not all companies fared so well—indeed, Intrepid Potash fell 30% for the year, despite its great first-day return. Some lost even more, including GT Solar International, which lost 11.6% on its first day and a total of 82.5% for the year. Even if you are able to identify a “hot” issue, it is often difficult to purchase shares in the initial offering. In strong markets, these deals are generally oversubscribed, which means that the demand for shares at the offering price exceeds the number of shares issued. In such instances, investment bankers favor large institutional investors (who are their best customers), and small investors find it hard, if not impossible, to get in on the ground floor. They can buy the stock in the aftermarket, but evidence Chapter 1: An Overview of Financial Management and the Financial Environment 31 Rational Exuberance? The Daily Planet Ltd. made history on May 1, 2003, by becoming the world’s first publicly traded brothel. Technically, the Daily Planet owns only property, including a hotel with 18 rooms, each with a different theme, but all have multi-person showers and very large beds. The Daily Planet charges guests a room fee of 115 Australian dollars (A$) per hour; clients also pay a fee of A$115 directly to individual members of the staff. The IPO was for 7.5 million shares of stock, initially priced at A$0.50. However, the price ended the first day of trading at A$1.09 for a first-day return of 118%. The price closed the second day at A$1.56 for a two-day return of 212%, one of the largest returns since the days of the dot-com boom. Institutional investors normally buy about 60% to 70% of the stock in an IPO, but they didn’t participate in this offering. The company is named after the fictitious newspaper for which comic strip character Clark Kent was a reporter. All receptionists have “Lois Lane” nametags, and there is a telephone booth in the lobby. What would Superman think! suggests that if you do not get in on the ground floor, the average IPO underperforms the overall market over the long run.5 Before you conclude that it isn’t fair to let only the best customers have the stock in an initial offering, think about what it takes to become a best customer. Best customers are usually investors who have done lots of business in the past with the investment banking firm’s brokerage department. In other words, they have paid large sums as commissions in the past, and they are expected to continue doing so in the future. As is so often true, there is no free lunch—most of the investors who get in on the ground floor of an IPO have, in fact, paid for this privilege. Self-Test Differentiate between an IPO, a seasoned equity offering, and a secondary transaction. Why is it often difficult for the average investor to make money during an IPO? 1.11 THE SECONDARY STOCK MARKETS The two leading U.S. stock markets today are the New York Stock Exchange and the Nasdaq stock market. The New York Stock Exchange WWW You can access the home pages of the major U.S. stock markets at http:// www.nyse.com or http:// www.nasdaq.com. These sites provide background information as well as the opportunity to obtain individual stock quotes. Before March of 2006, the New York Stock Exchange (NYSE) was a privately held firm owned by its members. It then merged with Archipelago, a publicly traded company that was one of the world’s largest ECNs. NYSE members received approximately 70% of the shares in the combined firm, with Archipelago shareholders receiving 30%. The combined firm, which also owned the Pacific Exchange, was known as The NYSE Group, Inc., and was traded publicly under the ticker symbol NYX. It continued to operate the New York Stock Exchange (a physical location exchange located on Wall Street) and Arca (comprising the Pacific Exchange and the ECN formerly known as Archipelago). In 2007 The NYSE Group merged with Euronext, a European company that operates stock exchanges (called bourses) in Paris, Amsterdam, Brussels, and Lisbon. The combined company is called NYSE Euronext. 5 See Jay R. Ritter, “The Long-Run Performance of Initial Public Offerings,” Journal of Finance, March 1991, pp. 3–27. 32 Part 1: Fundamental Concepts of Corporate Finance resource For more on stock markets, see Web Extension 1B on the textbook’s Web site. The NYSE still has over 300 member organizations, which are corporations, partnerships, or LLCs. Membership prices were as high as $4 million in 2005, and the last sale before the Euronext merger was $3.5 million. Member organizations are registered broker-dealers, but they may not conduct trading on the floor of the exchange unless they also hold a trading license issued by the NYSE. Before going public, the equivalent to the trading license was called a “seat,” although there was very little sitting on the floor of the exchange. Trading licenses are now leased by member organizations from the exchange, with an annual fee of $40,000 for 2009. The NYSE has leased most of its 1,500 available trading licenses. Most of the larger investment banking houses operate brokerage departments and are members of the NYSE with leased trading rights. The NYSE is open on all normal working days, and members meet in large rooms equipped with electronic equipment that enables each member to communicate with his or her firm’s offices throughout the country. For example, Merrill Lynch (now owned by Bank of America) might receive an order in its Atlanta office from a customer who wants to buy shares of Procter & Gamble stock. Simultaneously, Edward Jones’ St. Louis office might receive an order from a customer wishing to sell shares of P&G. Each broker communicates electronically with the firm’s representative on the NYSE. Other brokers throughout the country also communicate with their own exchange members. The exchange members with sell orders offer the shares for sale, and they are bid for by the members with buy orders. Thus, the NYSE operates as an auction market.6 The Nasdaq Stock Market The National Association of Securities Dealers (NASD) is a self-regulatory body that licenses brokers and oversees trading practices. The computerized network used by the NASD is known as the NASD Automated Quotation System, or Nasdaq. Nasdaq started as just a quotation system, but it has grown to become an organized securities market with its own listing requirements. Nasdaq lists about 5,000 stocks, although not all trade through the same Nasdaq system. For example, the Nasdaq National Market lists the larger Nasdaq stocks, such as Microsoft and Intel, while the Nasdaq SmallCap Market lists smaller companies with the potential for high growth. Nasdaq also operates the Nasdaq OTC Bulletin Board, which lists quotes The NYSE is actually a modified auction market, wherein people (through their brokers) bid for stocks. Originally—about 200 years ago—brokers would literally shout, “I have 100 shares of Erie for sale; how much am I offered?” and then sell to the highest bidder. If a broker had a buy order, he or she would shout, “I want to buy 100 shares of Erie; who’ll sell at the best price?” The same general situation still exists, although the exchanges now have members known as specialists who facilitate the trading process by keeping an inventory of shares of the stocks in which they specialize. If a buy order comes in at a time when no sell order arrives, the specialist will sell off some inventory. Similarly, if a sell order comes in, the specialist will buy and add to inventory. The specialist sets a bid price (the price the specialist will pay for the stock) and an asked price (the price at which shares will be sold out of inventory). The bid and asked prices are set at levels designed to keep the inventory in balance. If many buy orders start coming in because of favorable developments or sell orders come in because of unfavorable events, the specialist will raise or lower prices to keep supply and demand in balance. Bid prices are somewhat lower than asked prices, with the difference, or spread, representing the specialist’s profit margin. Special facilities are available to help institutional investors such as mutual funds or pension funds sell large blocks of stock without depressing their prices. In essence, brokerage houses that cater to institutional clients will purchase blocks (defined as 10,000 or more shares) and then resell the stock to other institutions or individuals. Also, when a firm has a major announcement that is likely to cause its stock price to change sharply, it will ask the exchanges to halt trading in its stock until the announcement has been made and digested by investors. See Web Extension 1B on the textbook’s Web site for more on specialists and trading off the exchange floor. 6 Chapter 1: An Overview of Financial Management and the Financial Environment 33 Measuring the Market A stock index is designed to show the performance of the stock market. Here we describe some leading indexes. Dow Jones Industrial Average Begun in 1896, the Dow Jones Industrial Average (DJIA) now includes 30 widely held stocks that represent almost a fifth of the market value of all U.S. stocks. See http://www.dowjones.com for more information. value-weighted index based on just over 2,000 stocks that represent 77% of the total market capitalization of all publicly traded companies in the United States. See http://www.nyse.com for more information. Trading the Market Created in 1926, the S&P 500 Index is widely regarded as the standard for measuring large-cap U.S. stocks’ market performance. It is value weighted, so the largest companies (in terms of value) have the greatest influence. The S&P 500 Index is used as a comparison benchmark by 97% of all U.S. money managers and pension plan sponsors. See http://www2.standardand poors.com for more information. Through the use of exchange traded funds (ETFs), it is now possible to buy and sell the market in much the same way as an individual stock. For example, the Standard & Poor’s depository receipt (SPDR) is a share of a fund that holds the stocks of all the companies in the S&P 500. SPDRs trade during regular market hours, making it possible to buy or sell the S&P 500 any time during the day. There are hundreds of other ETFs, including ones for the Nasdaq, the Dow Jones Industrial Average, gold stocks, utilities, and so on. Nasdaq Composite Index Recent Performance The Nasdaq Composite Index measures the performance of all common stocks listed on the Nasdaq stock market. Currently, it includes more than 3,200 companies, many of which are in the technology sector. Microsoft, Cisco Systems, and Intel account for a high percentage of the index’s value-weighted market capitalization. For this reason, substantial movements in the same direction by these three companies can move the entire index. See http://www.nasdaq.com for more information. Go to the Web site http://finance.yahoo.com/. Enter the symbol for any of the indexes (^DJI for the Dow Jones, ^SPC for the S&P 500, ^IXIC for the Nasdaq, and ^NYA for the NYSE) and then click GO. This will bring up the current value of the index, shown in a table. Click Basic Chart in the panel on the left, which will bring up a chart showing the historical performance of the index. Directly above the chart is a series of buttons that allows you to choose the number of years and to plot the relative performance of several indexes on the same chart. You can even download the historical data in spreadsheet form by clicking Historical Prices in the left panel. S&P 500 Index NYSE Composite Index The NYSE Composite Index measures the performance of all common stocks listed on the NYSE. It is a for stocks that are registered with the Securities and Exchange Commission (SEC) but are not listed on any exchange, usually because the company is too small or not sufficiently profitable.7 Finally, Nasdaq operates the Pink Sheets, which provide quotes on companies that are not registered with the SEC. OTC stands for over-the-counter. Before Nasdaq, the quickest way to trade a stock that was not listed at a physical location exchange was to find a brokerage firm that kept shares of that stock in inventory. The stock certificates were actually kept in a safe and were literally passed over the counter when bought or sold. Nowadays the certificates for almost all listed stocks and bonds in the United States are stored in a vault, beneath Manhattan, that is operated by the Depository Trust and Clearing Corporation (DTCC). Most brokerage firms have an account with the DTCC, and most investors leave their stocks with their brokers. Thus, when stocks are sold, the DTCC simply adjusts the accounts of the brokerage firms that are involved, and no stock certificates are actually moved. 7 34 Part 1: Fundamental Concepts of Corporate Finance "Liquidity” is the ability to trade quickly at a net price (i.e., after any commissions) that is close to the security’s recent market price. In a dealer market, such as Nasdaq, a stock’s liquidity depends on the number and quality of the dealers who make a market in the stock. Nasdaq has more than 400 dealers, most of whom make markets in a large number of stocks. The typical stock has about 10 market makers, but some stocks have more than 50 market makers. Obviously, there are more market makers, and hence there is more liquidity, for the Nasdaq National Market than for the SmallCap Market. Stocks listed on the OTC Bulletin Board or the Pink Sheets have much less liquidity. Competition in the Secondary Markets WWW For updates, see http:// www.world-exchanges .org/statistics/time-series/ market-capitalization at the World Federation of Exchanges. Self-Test There is intense competition between the NYSE, Nasdaq, and other international stock exchanges—they all want the larger, more profitable companies to list on their exchange. Since most of the largest U.S. companies trade on the NYSE, the market capitalization of NYSE-traded stocks is much higher than for stocks traded on Nasdaq (about $15.7 trillion compared with $4.0 trillion at the end of 2007). However, reported volume (number of shares traded) is often larger on Nasdaq, and more companies are listed on Nasdaq.8 For comparison, the market capitalizations for global exchanges are $4.3 trillion in Tokyo, $3.9 trillion in London, $3.7 trillion in Shanghai, $2.7 trillion in Hong Kong, $2.1 trillion in Germany, and $1.8 trillion in Bombay. Interestingly, many high-tech companies such as Microsoft and Intel have remained on Nasdaq even though they easily meet the listing requirements of the NYSE. At the same time, however, other high-tech companies such as Gateway and Iomega have left Nasdaq for the NYSE. Despite these defections, Nasdaq’s growth over the past decade has been impressive. In an effort to become even more competitive with the NYSE and with international markets, Nasdaq acquired one of the leading ECNs, Instinet, in 2005. Moreover, in early 2006 Nasdaq made an offer to acquire the London Stock Exchange (LSE), was rejected by the LSE, withdrew the offer but retained the right to make a subsequent offer, and busily acquired additional shares of stock in the LSE. In late 2006, Nasdaq made a second offer for the LSE and again was rejected. Nasdaq ultimately ended up by selling most of its LSE shares to Bourse Dubai, which owns about 28% of the LSE. Nasdaq did acquire the Nordic exchange OMX, giving it an international presence. The combined company is now known as the NASDAQ OMX Group. Despite all the shifting ownerships of exchanges, one thing is clear—there will be a continued consolidation in the securities exchange industry, with a blurring of the lines between physical location exchanges and electronic exchanges. What are some major differences between the NYSE and the Nasdaq stock market? 1.12 STOCK MARKET RETURNS During the period 1968–2008, the average annual return for the stock market, as measured by total returns (dividends plus capital gains) on the S&P 500 index, was about 10.6%, but this average does not reflect the considerable annual variation. Notice in Panel A of Figure 1-4 that the market was relatively flat in the 1970s, increased somewhat in the 1980s, and has been a roller coaster ever since. In fact, the market in early 2009 dipped to a level last seen in 1995. Panel B highlights the One transaction on Nasdaq generally shows up as two separate trades (the buy and the sell). This “double counting” makes it difficult to compare the volume between stock markets. 8 Chapter 1: An Overview of Financial Management and the Financial Environment S&P 500 Stock Index Performance Panel A: End-of-Month Index Value 1,800 1,600 1,400 1,200 1,000 800 600 400 Dec-08 Dec-04 Dec-00 Dec-96 Dec-92 Dec-88 Dec-84 Dec-80 Dec-76 Dec-68 0 Dec-72 200 Panel B: Total Annual Returns: Dividend Yield + Capital Gain or Loss Percent 50 40 30 20 10 0 –10 –20 –30 2008 2004 2000 1996 1992 1988 1984 1980 1976 –50 1972 –40 1968 FIGURE 1-4 35 Sources: Returns data are from various issues of The Wall Street Journal, “Investment Scoreboard” section; the index level is from http://finance.yahoo.com. year-to-year risk by showing annual returns. Notice that stocks have had positive returns in most years, but there have been several years with large losses. Stocks lost more than 40% of their value during 1973–1974 and again during 2000–2002, and they lost 37% of their value in 2008 alone. We will examine risk in more detail later in the book, but even a cursory glance at Figure 1-4 shows just how risky stocks can be! U.S. stocks amount to only about 40% of the world’s stocks, and this is prompting many U.S. investors to also hold foreign stocks. Analysts have long touted the benefits of investing overseas, arguing that foreign stocks improve diversification and provide good growth opportunities. This has been true for many years, but it wasn’t the 36 Part 1: Fundamental Concepts of Corporate Finance T AB L E 1 - 2 20 0 8 P e r f o r m a n c e o f S e l e c t e d D o w Jo n es G l o b a l S t o c k I n d e x e s , R an k e d Highest to Lowest COUNTRY Morocco Japan Switzerland Colombia Israel United States Mexico South Africa Spain France Germany Taiwan Canada U.K. Italy U.S. DO LLARS −17.5% −29.3 −31.0 −31.4 −36.2 −38.6 −39.8 −42.8 −43.4 −45.5 −46.3 −47.6 −49.1 −51.2 −52.5 LO C A L CURRENCY −14.1% −42.6 −35.0 −23.5 −37.4 −38.6 −23.6 −22.6 −40.5 −42.6 −43.5 −46.9 −36.3 −32.5 −50.0 COUNTRY Singapore Sweden China Hong Kong Australia South Korea Argentina Brazil Egypt Indonesia India Ireland Russia Cyprus Iceland U.S. DO LLARS −53.1% −53.2 −53.3 −53.9 −54.5 −55.6 −56.1 −57.0 −57.2 −63.0 −66.7 −68.7 −73.1 −79.0 −96.2 L OCAL CURRENCY −53.0% −42.7 −53.6 −54.2 −42.7 −40.3 −51.9 −43.7 −57.3 −57.1 −58.9 −67.1 −66.6 −77.9 −92.6 Source: Adapted from The Wall Street Journal Online, http://online.wsj.com. case in 2008 and 2009. Table 1-2 shows returns in selected countries. Notice that all the countries had negative returns. The table shows how each country’s stocks performed in its local currency and in terms of the U.S. dollar. For example, in 2008 British (U.K.) stocks had a −32.5% return in their own currency, but that translated into a −51.2% return to a U.S. investor; the difference was due to depreciation in the British pound relative to the U.S. dollar. As this example shows, the results of foreign investments depend in part on what happens in the foreign economy and in part on movements in exchange rates. Indeed, when you invest overseas, you face two risks: (1) that foreign stocks will decrease in their local markets and (2) that the currencies in which you will be paid will fall relative to the dollar. Even though foreign stocks have exchange rate risk, this by no means suggests that investors should avoid them. Foreign investments do improve diversification, and it is inevitable that there will be years when foreign stocks outperform U.S. domestic stocks. When this occurs, U.S. investors will be glad they put some of their money in overseas markets. Self-Test Explain how exchange rates affect the rate of return on international investments. 1.13 THE GLOBAL ECONOMIC CRISIS Although the global economic crisis has many causes, mortgage securitization in the 2000s is certainly one culprit, so we begin with it. The Globalization of Mortgage Market Securitization A national TV program ran a documentary on the travails of Norwegian retirees resulting from defaults on Florida mortgages. Your first reaction might be to wonder how Norwegian retirees became financially involved with risky Florida mortgages. Chapter 1: An Overview of Financial Management and the Financial Environment 37 We will break the answer to that question into two parts. First, we will identify the different links in the financial chain between the retirees and mortgagees. Second, we will explain why there were so many weak links. In the movie Jerry Maguire, Tom Cruise said “Show me the money!” That’s a good way to start identifying the financial links, starting with a single home purchase in Florida. 1. Home Purchase. In exchange for cash, a seller in Florida turned over ownership of a house to a buyer. 2. Mortgage Origination. To get the cash used to purchase the house, the home buyer signed a mortgage loan agreement and gave it to an “originator.” Years ago the originator would probably have been an S&L or a bank, but more recently the originators have been specialized mortgage brokers, which was true in this case. The broker gathered and examined the borrower’s credit information, arranged for an independent appraisal of the house’s value, handled the paperwork, and received a fee for these services. 3. Securitization and Resecuritization. In exchange for cash, the originator sold the mortgage to a securitizing firm. For example, Merrill Lynch’s investment banking operation was a major player in securitizing loans. It would bundle large numbers of mortgages into pools and then create new securities that had claims on the pools’ cash flows. Some claims were simple, such as a proportional share of a pool, and some claims were more complex, such as a claim on all interest payments during the first five years or a claim on only principal payments. More complicated claims were entitled to a fixed payment, while other claims would receive payments only after the “senior” claimants had been paid. These slices of the pool were called “tranches,” which comes from a French word for slice. Some of the tranches were themselves re-combined and then re-divided into securities called “collateralized debt obligations (CDOs)”, some of which were themselves combined and subdivided into other securities, commonly called CDOs-squared. For example, Lehman Brothers often bought different tranches, split them into CDOs of differing risk, and then had the different CDOs rated by an agency like Moody’s or Standard & Poor’s. There are two very important points to notice. First, the process didn’t change the total amount of risk embedded in the mortgages, but it did make it possible to create some securities that were less risky than average and some that were more risky. Second, each time a new security was created or rated, fees were being earned by the investment banks and rating agencies. 4. The Investors. In exchange for cash, the securitizing firms sold the newly created securities to individual investors, hedge funds, college endowments, insurance companies, and other financial institutions, including a pension fund in Norway. Keep in mind that financial institutions are themselves funded by individuals, so cash begins with individuals and flows through the system until it is eventually received by the seller of the home. If all goes according to plan, payments on the mortgages eventually return to the individuals who originally provided the cash. But in this case, the chain was broken by a wave of mortgage defaults, resulting in problems for Norwegian retirees. Students and managers often ask us, “What happened to all the money?” The short answer is “It went from investors to home sellers, with fees being skimmed off all along the way.” 38 Part 1: Fundamental Concepts of Corporate Finance Although the process is complex, in theory there is nothing inherently wrong with it. In fact, it should, in theory, provide more funding for U.S. home purchasers, and it should allow risk to be shifted to those best able to bear it. Unfortunately, this isn’t the end of the story. The Dark Side of Securitization: The Sub-Prime Mortgage Meltdown What caused the financial crisis? Entire books are now being written on this subject, but we can identify a few of the culprits. Regulators Approved Sub-Prime Standards. In the 1980s and early 1990s, regulations did not permit a nonqualifying mortgage to be securitized, so most originators mandated that borrowers meet certain requirements, including having at least a certain minimum level of income relative to the mortgage payments and a minimum down payment relative to the size of the mortgage. But in the mid-1990s, Washington politicians wanted to extend home ownership to groups that traditionally had difficulty obtaining mortgages. To accomplish this, regulations were relaxed so that nonqualifying mortgages could be securitized. Such loans are commonly called subprime or Alt-A mortgages. Thus, riskier mortgages were soon being securitized and sold to investors. Again, there was nothing inherently wrong, provided the two following questions were being answered in the affirmative: One, were home buyers making sound decisions regarding their ability to repay the loans? And two, did the ultimate investors recognize the additional risk? We now know that the answer to both questions is a resounding “no.” Homeowners were signing mortgages that they could not hope to repay, and investors treated these mortgages as if they were much safer than they actually were. The Fed Helped Fuel the Real Estate Bubble. With more people able to get a mortgage, including people who should not have obtained one, the demand for homes increased. This alone would have driven up house prices. However, the Fed also slashed interest rates to historic lows after 9/11 to prevent a recession, and it kept them low for a long time. These low rates made mortgage payments lower, which made home ownership seem even more affordable, again contributing to an increase in the demand for housing. Figure 1-5 shows that the combination of lower mortgage qualifications and lower interest rates caused house prices to skyrocket. Thus, the Fed contributed to an artificial bubble in real estate. Home Buyers Wanted More for Less. Even with low interest rates, how could sub-prime borrowers afford the mortgage payments, especially with house prices rising? First, most sub-prime borrowers chose an adjustable rate mortgage (ARM) with an interest rate based on a short-term rate, such as that on 1-year Treasury bonds, to which the lender added a couple of percentage points. Because the Fed had pushed short-term rates so low, the initial rates on ARMs were very low. With a traditional fixed-rate mortgage, the payments remain fixed over time. But with an ARM, an increase in market interest rates triggers higher monthly payments, so an ARM is riskier than a fixed-rate mortgage. However, many borrowers chose an even riskier mortgage, the “option ARM,” where the borrower can choose to make such low payments during the first couple of years that they don’t even cover the interest, causing the loan balance to actually increase each month! At a later date, the payments would be reset to reflect both the current market interest rate and the higher loan balance. For example, in some cases a monthly payment of $948 for the first Chapter 1: An Overview of Financial Management and the Financial Environment FIGURE 1-5 39 The Real Estate Boom: Housing Prices and Mortgage Rates Real Estate Index Mortgage Rate (%) 12 250 Mortgage Rate 10 200 8 150 6 100 4 50 2 Real Estate Index 2009 2007 2005 2003 2001 1999 1997 1995 1993 1991 1989 0 1987 0 Notes: 1. The real estate index is the Case-Shiller composite index for house prices in 10 real estate markets, available at http://www2.standardandpoors.com/spf/pdf/index/CSHomePrice_History_012724.xls. 2. Interest rates are for 30-year conventional fixed rate mortgages, available from the St. Louis Federal Reserve: http://research.stlouisfed.org/fred/. 32 months was reset to $2,454 for the remaining 328 months (we provide the calculations for this example in Chapter 4). Why would anyone who couldn’t afford to make a $2,454 monthly payment choose an option ARM? Here are three possible reasons. First, some borrowers simply didn’t understand the situation and were victims of predatory lending practices by brokers eager to earn fees regardless of the consequences. Second, some borrowers thought that the home price would go up enough to allow them to sell at a profit or else refinance with another low-payment loan. Third, some people were simply greedy and shortsighted, and they wanted to live in a better home than they could afford. Mortgage Brokers Didn’t Care. Years ago, S&Ls and banks had a vested interest in the mortgages they originated because they held them for the life of the loan— up to 30 years. If a mortgage went bad, the bank or S&L would lose money, so they were careful to verify that the borrower would be able to repay the loan. In the bubble years, though, over 80% of mortgages were arranged by independent mortgage brokers who received a commission. Thus, the broker’s incentive was to complete deals even if the borrowers couldn’t make the payments after the soon-to-come reset. So it’s easy to understand (but not to approve!) why brokers pushed deals onto borrowers who were almost certain to default eventually. Real Estate Appraisers Were Lax. The relaxed regulations didn’t require the mortgage broker to verify the borrower’s income, so these loans were called “liar 40 Part 1: Fundamental Concepts of Corporate Finance loans” because the borrowers could overstate their income. But even in these cases the broker had to get an appraisal showing that the house’s value was greater than the loan amount. Many real estate appraisers simply assumed that house prices would keep going up, so they were willing to appraise houses at unrealistically high values. Like the mortgage brokers, they were paid at the time of their service. Other than damage to their reputations, they weren’t concerned if the borrower later defaulted and the value of the house turned out to be less than the remaining loan balance, causing a loss for the lender. Originators and Securitizers Wanted Quantity, not Quality. Originating institutions like Countrywide Financial and New Century Mortgage made money when they sold the mortgages, long before any of the mortgages defaulted. The same is true for securitizing firms such as Bear Stearns, Merrill Lynch, and Lehman Brothers. Their incentives were to generate volume originating loans, not to make sure the loans should have been made. This started at the top—CEOs and other top executives received stock options and bonuses based on their firms’ profits, and profits depended on volume. Thus, the top officers pushed their subordinates to generate volume, those subordinates pushed the originators to write more mortgages, and the originators pushed the appraisers to come up with high values. Rating Agencies Were Lax. Investors who purchased the complicated mortgage backed securities wanted to know how risky they were, so they insisted on seeing the bonds’ “ratings.” Rating agencies were paid to investigate the details of each bond and to assign a rating which reflected the security’s risk. The securitizing firms paid the rating agencies to do the ratings. For example, Lehman Brothers hired Moody’s to rate some of their CDOs. Indeed, the investment banks would actually pay for advice from the rating agencies as they were designing the securities. The rating and consulting activities were extremely lucrative for the agencies, which ignored the obvious conflict of interest: The investment bank wanted a high rating, the rating agency got paid to help design securities that would qualify for a high rating, and high ratings led to continued business for the raters. Insurance Wasn’t Insurance. To provide a higher rating and make these mortgage-backed securities look even more attractive to investors, the issuers would frequently purchase a type of insurance policy on the security called a credit default swap. For example, suppose you had wanted to purchase a CDO from Lehman Brothers but were worried about the risk. What if Lehman Brothers had agreed to pay an annual fee to an insurance company like AIG, which would guarantee the CDO’s payments if the underlying mortgages defaulted? You probably would have felt confident enough to buy the CDO. But any similarity to a conventional insurance policy ends here. Unlike home insurance, where there is a single policyholder and a single insurer, totally uninvolved speculators can also make bets on your CDO by either selling or purchasing credit default swaps on the CDO. For example, a hedge fund could buy a credit default swap on your CDO if it thinks the CDO will default; or an investment bank like Bear Stearns could sell a swap, betting that the CDO won’t default. In fact, the International Swaps and Derivatives Association estimates that in mid-2008 there was about $54 trillion in credit default swaps. This staggering amount is approximately 7 times the value of all U.S. mortgages, over 4 times the level of the U.S. national debt, and over twice the value of the entire U.S. stock market. Another big difference is that home insurance companies are highly regulated, but there was virtually no regulation in the credit default swap market. The players Chapter 1: An Overview of Financial Management and the Financial Environment 41 traded directly among themselves, with no central clearinghouse. It was almost impossible to tell how much risk any of the players had taken on, making it impossible to know whether or not counterparties like AIG would be able to fulfill their obligations in the event of a CDO default. And that made it impossible to know the value of CDOs held by many banks, which in turn made it impossible to judge whether or not those banks were de facto bankrupt. Rocket Scientists Had Poor Rearview Mirrors. Brilliant financial experts, often trained in physics and hired from rocket science firms, built elegant models to determine the value of these new securities. Unfortunately, a model is only as good as its inputs. The experts looked at the high growth rates of recent real estate prices (see Figure 1-5) and assumed that future growth rates also would be high. These high growth rates caused models to calculate very high CDO prices, at least until the real estate market crumbled. Investors Wanted More for Less. In the early 2000s, low-rated debt (including mortgage-backed securities), hedge funds, and private equity funds produced great rates of return. Many investors jumped into this debt to keep up with the Joneses. As shown in Chapter 5 when we discuss bond ratings and bond spreads, investors began lowering the premium they required for taking on extra risk. Thus, investors focused primarily on returns and largely ignored risk. In fairness, some investors assumed the credit ratings were accurate, and they trusted the representatives of the investment banks selling the securities. In retrospect, however, Warren Buffett’s maxim that “I only invest in companies I understand” seems wiser than ever. The Emperor Has No Clothes. In 2006, many of the option ARMs began to reset, borrowers began to default, and home prices first leveled off and then began to fall. Things got worse in 2007 and 2008, and by early 2009, almost 1 out of 10 mortgages was in default or foreclosure, resulting in displaced families and virtual ghost towns of new subdivisions. As homeowners defaulted on their mortgages, so did the CDOs backed by the mortgages. That brought down the counterparties like AIG who had insured the CDOs via credit default swaps. Virtually overnight, investors realized that mortgage-backed security default rates were headed higher and that the houses used as collateral were worth less than the mortgages. Mortgage-backed security prices plummeted, investors quit buying newly securitized mortgages, and liquidity in the secondary market disappeared. Thus, the investors who owned these securities were stuck with pieces of paper that were substantially lower than the values reported on their balance sheets. From Sub-Prime Meltdown to Liquidity Crisis to Economic Crisis Like the Andromeda strain, the sub-prime meltdown went viral, and it ended up infecting almost all aspects of the economy. Financial institutions were the first to fall. Many originating firms had not sold all of their sub-prime mortgages, and they failed. For example, New Century declared bankruptcy in 2007, IndyMac was placed under FDIC control in 2008, and Countrywide was acquired by Bank of America in 2008 to avoid bankruptcy. Securitizing firms also crashed, partly because they kept some of the new securities they created. For example, Fannie Mae and Freddie Mac had huge losses on their portfolio assets, causing them to be virtually taken over by the Federal Housing 42 Part 1: Fundamental Concepts of Corporate Finance Finance Agency in 2008. In addition to big losses on their own sub-prime portfolios, many investment banks also had losses related to their positions in credit default swaps. Thus, Lehman Brothers was forced into bankruptcy, Bear Stearns was sold to JPMorgan Chase, and Merrill Lynch was sold to Bank of America, with huge losses to their stockholders. Because Lehman Brothers defaulted on some of its commercial paper, investors in the Reserve Primary Fund, a big money market mutual fund, saw the value of its investments “break the buck,” dropping to less than a dollar per share. To avoid panic and a total lockdown in the money markets, the U.S. Treasury agreed to insure some investments in money market funds. AIG was the number one backer of credit default swaps, and it operated worldwide. In 2008 it became obvious that AIG could not honor its commitments as a counterparty, so the Fed effectively nationalized AIG to avoid a domino effect in which AIG’s failure would topple hundreds of other financial institutions. In normal times, banks provide liquidity to the economy and funding for creditworthy businesses and individuals. These activities are absolutely crucial for a wellfunctioning economy. However, the financial contagion spread to commercial banks because some owned mortgage-backed securities, some owned commercial paper issued by failing institutions, and some had exposure to credit default swaps. As banks worried about their survival in the fall of 2008, they stopped providing credit to other banks and businesses. The market for commercial paper dried up to such an extent that the Fed began buying new commercial paper from issuing companies. Banks also began hoarding cash rather than lending it. The Fed requires banks to keep 10% of the funds they raise from depositors on “reserve.” Banks use the other 90% to make loans or to buy securities. In aggregate, there usually has been about $9 billion in excess reserves—that is, reserves over and above the 10% they are required to keep on hand. However, at the end of 2008, banks held over $770 billion in excess reserves compared to $75 billion in required reserves. This hoarding may have reduced the banks’ risk, but it deprived the economy of a much needed capital. Consequently, there has been a reduction in construction, manufacturing, retailing, and consumption, all of which caused job losses in 2008 and 2009, with more expected in the future. In short, this has led to a serious recession in the United States and most of the developed world, a recession that brings back memories of the Great Depression of the 1930s. Self-Test Briefly describe some of the mistakes that were made by participants in the subprime mortgage process. 1.14 THE BIG PICTURE Finance has a lot of vocabulary and tools that might be new to you. To help you avoid getting bogged down in the trenches, Figure 1-6 presents the “big picture.” A manager’s primary job is to increase the company’s intrinsic value, but how exactly does one go about doing that? The equation in the center of Figure 1-6 shows that intrinsic value is the present value of the firm’s expected free cash flows, discounted at the weighted average cost of capital. Thus, there are two approaches for increasing intrinsic value: Improve FCF or reduce the WACC. Observe that several factors affect FCF and several factors affect the WACC. In the rest of the book’s chapters, we will typically focus on only one of these factors, systematically building the vocabu- Chapter 1: An Overview of Financial Management and the Financial Environment FIGURE 1-6 43 The Determinants of Intrinsic Value: The Big Picture Sales revenues − Operating costs and taxes − Required investments in operating capital Free cash flow (FCF) FCF1 Value = (1 + WACC)1 + FCF2 (1 + WACC)2 = +…+ FCF∞ (1 + WACC)∞ Weighted average cost of capital (WACC) Market interest rates Market risk aversion Cost of debt Cost of equity Firm’s debt/equity mix Firm’s business risk lary and tools that you will use after graduation to improve your company’s intrinsic value. It is true that every manager needs to understand financial vocabulary and be able to apply financial tools, but really successful managers also understand how their decisions affect the big picture. So as you read this book, keep in mind where each topic fits into the big picture. e-RESOURCES The textbook’s Web site contains several types of files that will be helpful to you: resource 1. It contains Excel files, called Tool Kits, that provide well-documented models for almost all of the text’s calculations. Not only will these Tool Kits help you with this finance course, they also will serve as tool kits for you in other courses and in your career. 2. There are problems at the end of the chapters that require spreadsheets, and the Web site contains the models you will need to begin work on these problems. When we think it might be helpful for you to look at one of the Web site’s files, we’ll show an icon in the margin like the one shown here. Other resources are also on the Web site, including Cyberproblems and problems that use the Thomson ONE—Business School Edition Web site. The textbook’s Web site also contains an electronic library that contains Adobe PDF 44 Part 1: Fundamental Concepts of Corporate Finance files for “extensions” to many chapters that cover additional useful material related to the chapter. Summary • • • • • The three main forms of business organization are the proprietorship, the partnership, and the corporation. Although each form of organization offers advantages and disadvantages, corporations conduct much more business than the other forms. The primary objective of management should be to maximize stockholders’ wealth, and this means maximizing the company’s fundamental, or intrinsic, stock price. Legal actions that maximize stock prices usually increase social welfare. Free cash flows (FCFs) are the cash flows available for distribution to all of a firm’s investors (shareholders and creditors) after the firm has paid all expenses (including taxes) and has made the required investments in operations to support growth. The weighted average cost of capital (WACC) is the average return required by all of the firm’s investors. It is determined by the firm’s capital structure (the firm’s relative amounts of debt and equity), interest rates, the firm’s risk, and the market’s attitude toward risk. The value of a firm depends on the size of the firm’s free cash flows, the timing of those flows, and their risk. A firm’s fundamental, or intrinsic, value is defined by Value ¼ • • • • • • • • FCF1 FCF2 FCF3 FCF∞ …þ 1þ 2þ 3þ ð1 þ WACCÞ∞ ð1 þ WACCÞ ð1 þ WACCÞ ð1 þ WACCÞ Transfers of capital between borrowers and savers take place (1) by direct transfers of money and securities; (2) by transfers through investment banking houses, which act as go-betweens; and (3) by transfers through financial intermediaries, which create new securities. Four fundamental factors affect the cost of money: (1) production opportunities, (2) time preferences for consumption, (3) risk, and (4) inflation. Derivatives, such as options, are claims on other financial securities. In securitization, new securities are created from claims on packages of other securities. Major financial institutions include commercial banks, savings and loan associations, mutual savings banks, credit unions, pension funds, life insurance companies, mutual funds, money market funds, hedge funds, and private equity funds. Spot markets and futures markets are terms that refer to whether the assets are bought or sold for “on-the-spot” delivery or for delivery at some future date. Money markets are the markets for debt securities with maturities of less than a year. Capital markets are the markets for long-term debt and corporate stocks. Primary markets are the markets in which corporations raise new capital. Secondary markets are markets in which existing, already outstanding securities are traded among investors. Orders from buyers and sellers can be matched in one of three ways: (1) in an open outcry auction, (2) through dealers, and (3) automatically through an electronic communications network (ECN). Chapter 1: An Overview of Financial Management and the Financial Environment • • 45 There are two basic types of markets—the physical location exchanges (such as the NYSE) and computer/telephone networks (such as Nasdaq). Web Extension 1A discusses derivatives, and Web Extension 1B provides additional coverage of stock markets. Questions (1–1) Define each of the following terms: a. Proprietorship; partnership; corporation b. Limited partnership; limited liability partnership; professional corporation c. Stockholder wealth maximization d. Money market; capital market; primary market; secondary market e. Private markets; public markets; derivatives f. Investment banker; financial services corporation; financial intermediary g. Mutual fund; money market fund h. Physical location exchanges; computer/telephone network i. Open outcry auction; dealer market; electronic communications network (ECN) j. Production opportunities; time preferences for consumption k. Foreign trade deficit (1–2) What are the three principal forms of business organization? What are the advantages and disadvantages of each? (1–3) What is a firm’s fundamental, or intrinsic, value? What might cause a firm’s intrinsic value to be different than its actual market value? (1–4) Edmund Enterprises recently made a large investment to upgrade its technology. Although these improvements won’t have much of an impact on performance in the short run, they are expected to reduce future costs significantly. What impact will this investment have on Edmund Enterprises’s earnings per share this year? What impact might this investment have on the company’s intrinsic value and stock price? (1–5) Describe the different ways in which capital can be transferred from suppliers of capital to those who are demanding capital. (1–6) What are financial intermediaries, and what economic functions do they perform? (1–7) Is an initial public offering an example of a primary or a secondary market transaction? (1–8) Differentiate between dealer markets and stock markets that have a physical location. (1–9) Identify and briefly compare the two leading stock exchanges in the United States today. Mini Case Assume that you recently graduated and have just reported to work as an investment advisor at the brokerage firm of Balik and Kiefer Inc. One of the firm’s clients is Michelle DellaTorre, a professional tennis player who has just come to the United States from Chile. DellaTorre is a highly ranked tennis player who would like to start a company to produce and market apparel she designs. She also expects to invest substantial amounts of money through Balik and Kiefer. 46 Part 1: Fundamental Concepts of Corporate Finance DellaTorre is very bright, and she would like to understand in general terms what will happen to her money. Your boss has developed the following set of questions you must answer to explain the U.S. financial system to DellaTorre. a. Why is corporate finance important to all managers? b. Describe the organizational forms a company might have as it evolves from a start-up to a major corporation. List the advantages and disadvantages of each form. c. How do corporations go public and continue to grow? What are agency problems? What is corporate governance? d. What should be the primary objective of managers? (1) (2) (3) e. f. g. h. i. j. k. l. m. n. o. Do firms have any responsibilities to society at large? Is stock price maximization good or bad for society? Should firms behave ethically? What three aspects of cash flows affect the value of any investment? What are free cash flows? What is the weighted average cost of capital? How do free cash flows and the weighted average cost of capital interact to determine a firm’s value? Who are the providers (savers) and users (borrowers) of capital? How is capital transferred between savers and borrowers? What do we call the price that a borrower must pay for debt capital? What is the price of equity capital? What are the four most fundamental factors that affect the cost of money, or the general level of interest rates, in the economy? What are some economic conditions (including international aspects) that affect the cost of money? What are financial securities? Describe some financial instruments. List some financial institutions. What are some different types of markets? How are secondary markets organized? (1) (2) List some physical location markets and some computer/telephone networks. Explain the differences between open outcry auctions, dealer markets, and electronic communications networks (ECNs). p. Briefly explain mortgage securitization and how it contributed to the global economic crisis. CHAPTER 2 Financial Statements, Cash Flow, and Taxes E ven in today’s era of financial crises, $14.6 billion is a lot of money. This is the amount of cash flow that Hewlett-Packard’s (HP) operations generated in 2008, up from $9.6 billion in 2007, despite the recession. The ability to generate cash flow is the lifeblood of a company and the basis for its fundamental value. How did HP use this cash flow? HP invested for the future by making over $11 billion in acquisitions. Other companies also generated large cash flows from operations in 2008, but they used the money differently. For example, Walgreens generated over $3 billion from its operations and used over $2 billion for capital expenditures, much of it on new stores and the purchase of worksite health centers. Procter & Gamble generated $15.8 billion. P&G made relatively small capital expenditures (abut $3 billion) and returned the lion’s share (over $12 billion) to shareholders as dividends or through stock repurchases. Apple generated about $9.6 billion (up from $5.5 billion the previous year) but made relatively small capital expenditures, acquisitions, or distributions to shareholders. Instead, it put about $9.1 billion into shortterm financial securities like T-bills. These four well-managed companies used their operating cash flows in four different ways: HP made acquisitions, Walgreens spent on a mix of internal and external growth, P&G returned cash to shareholders, and Apple saved for a rainy day. Which company made the right choice? Only time will tell, but keep these companies and their different cash flow strategies in mind as you read this chapter. 47 48 Part 1: Fundamental Concepts of Corporate Finance Intrinsic Value, Free Cash Flow, and Financial Statements In Chapter 1, we told you that managers should strive to make their firms more valuable and that the intrinsic value of a firm is determined by the present value of its free cash flows (FCF) discounted at the weighted av- erage cost of capital (WACC). This chapter focuses on FCF, including its calculation from financial statements and its interpretation when evaluating a company and manager. Sales revenues – Operating costs and taxes – Required investments in operating capital Free cash flow (FCF) Value = FCF1 1 (1 + WACC) + = FCF2 2 +...+ (1 + WACC) FCF∞ (1 + WACC)∞ Weighted average cost of capital (WACC) resource The textbook’s Web site contains an Excel file that will guide you through the chapter’s calculations. The file for this chapter is Ch02 Tool Kit.xls, and we encourage you to open the file and follow along as you read the chapter. WWW A source for links to the annual reports of many companies is http://www .annualreportservice.com. Market interest rates Cost of debt Firm’s debt/equity mix Market risk aversion Cost of equity Firm’s business risk A manager’s primary goal is to maximize the fundamental, or intrinsic, value of the firm’s stock. This value is based on the stream of cash flows the firm is expected to generate in the future. But how does an investor go about estimating future cash flows, and how does a manager decide which actions are most likely to increase cash flows? The first step is to understand the financial statements that publicly traded firms must provide to the public. Thus, we begin with a discussion of financial statements, including how to interpret them and how to use them. Because value depends on usable, after-tax cash flows, we highlight the difference between accounting income and cash flow. In fact, it is after-tax cash flow that is important, so we also provide an overview of the federal income tax system. 2.1 FINANCIAL STATEMENTS AND REPORTS A company’s annual report usually begins with the chairman’s description of the firm’s operating results during the past year and a discussion of new developments that will affect future operations. The annual report also presents four basic financial statements—the balance sheet, the income statement, the statement of stockholders’ equity, and the statement of cash flows.1 Firms also provide less comprehensive quarterly reports. Larger firms file even more detailed statements, giving breakdowns for each major division or subsidiary, with the Securities and Exchange Commission (SEC). These reports, called 10-K reports, are available on the SEC’s Web site at http://www.sec.gov under the heading “EDGAR.” 1 Chapter 2: Financial Statements, Cash Flow, and Taxes 49 The quantitative and written materials are equally important. The financial statements report what has actually happened to assets, earnings, dividends, and cash flows during the past few years, whereas the written materials attempt to explain why things turned out the way they did. For illustrative purposes, we use a hypothetical company, MicroDrive Inc., which produces hard drives for microcomputers. Formed in 1982, MicroDrive has grown steadily and has a reputation as one of the best firms in the microcomputer components industry. Self-Test What is the annual report, and what two types of information are given in it? What four types of financial statements are typically included in the annual report? 2.2 THE BALANCE SHEET resource See Ch02 Tool Kit.xls for details. T AB L E 2 - 1 Table 2-1 shows MicroDrive’s most recent balance sheets, which represent “snapshots” of its financial position on the last day of each year. Although most companies report their balance sheets only on the last day of a given period, the “snapshot” actually changes daily as inventories are bought and sold, as fixed assets are added or retired, or as loan balances are increased or paid down. Moreover, a retailer will have much larger inventories before Christmas than later in the spring, so balance sheets for the same company can look quite different at different times during the year. The left side of a balance sheet lists assets, which are the “things” the company owns. They are listed in order of “liquidity,” or length of time it typically takes to convert them to cash at fair market values. The right side lists the claims that various groups have against the company’s value, listed in the order in which they must be paid. For example, suppliers may have a claim called “accounts payable” that is due within 30 days, banks may have claims called “notes payable” that are due within 90 days, and bondholders may have claims that are not due for 20 years or more. Stockholders come last, for two reasons. First, their claim represents ownership (or equity) and need never be “paid off.” Second, they have a residual claim in the sense that they may receive payments only if the other claimants have already been paid. The nonstockholder claims are liabilities from the stockholders’ perspective. The amounts shown on the balance sheets are called book values because they are based on the amounts recorded by bookkeepers when assets are purchased or liabilities are issued. As you will see throughout this textbook, book values may be very different from market values, which are the current values as determined in the marketplace. M i c r oD r i v e I n c . : D ec e m b e r 3 1 B a l a nce She e t s ( M il l io ns o f D ol l ar s ) ASSETS 2010 2009 L I A B I L I TI E S A N D E Q U I T Y 2 0 10 2 0 09 Cash and equivalents Short-term investments Accounts receivable Inventories Total current assets Net plant and equipment $ 10 0 375 615 $1,000 1,000 $ 15 65 315 415 $ 810 870 $ $ $2,000 $1,680 Accounts payable Notes payable Accruals Total current liabilities Long-term bonds Total liabilities Preferred stock (400,000 shares) Common stock (50,000,000 shares) Retained earnings Total common equity Total liabilities and equity Total assets 60 110 140 $ 310 754 $1,064 40 130 766 $ 896 $2,000 30 60 130 $ 220 580 $ 800 40 130 710 $ 840 $1,680 50 Part 1: Fundamental Concepts of Corporate Finance The following sections provide more information about specific asset, liability, and equity accounts. Assets Cash, short-term investments, accounts receivable, and inventories are listed as current assets because MicroDrive is expected to convert them into cash within a year. All assets are stated in dollars, but only cash represents actual money that can be spent. Some marketable securities mature very soon, and these can be converted quickly into cash at prices close to their book values. Such securities are called “cash equivalents” and are included with cash. Therefore, MicroDrive could write checks for a total of $10 million. Other types of marketable securities have a longer time until maturity, and their market values are less predictable. These securities are classified as “short-term investments.” When MicroDrive sells its products to a customer but doesn’t demand immediate payment, the customer then has an obligation called an “account receivable.” The $375 million shown in accounts receivable is the amount of sales for which MicroDrive has not yet been paid. Inventories show the dollars MicroDrive has invested in raw materials, workin-process, and finished goods available for sale. MicroDrive uses the FIFO (firstin, first-out) method to determine the inventory value shown on its balance sheet ($615 million). It could have used the LIFO (last-in, first-out) method. During a period of rising prices, by taking out old, low-cost inventory and leaving in new, high-cost items, FIFO will produce a higher balance sheet inventory value but a lower cost of goods sold on the income statement. (This is strictly used for accounting; companies actually use older items first.) Because MicroDrive uses FIFO and because inflation has been occurring: (1) its balance sheet inventories are higher than they would have been had it used LIFO, (2) its cost of goods sold is lower than it would have been under LIFO, and (3) its reported profits are therefore higher. In MicroDrive’s case, if the company had elected to switch to LIFO, then its balance sheet would have inventories of $585 million rather than $615 million and its earnings (discussed in the next section) would have been reduced by $18 million. Thus, the inventory valuation method can have a significant effect on financial statements, which is important to know when comparing different companies. Rather than treat the entire purchase price of a long-term asset (such as a factory, plant, or equipment) as an expense in the purchase year, accountants “spread” the purchase cost over the asset’s useful life.2 The amount they charge each year is called the depreciation expense. Some companies report an amount called “gross plant and equipment,” which is the total cost of the long-term assets they have in place, and another amount called “accumulated depreciation,” which is the total amount of depreciation that has been charged on those assets. Some companies, such as MicroDrive, report only net plant and equipment, which is gross plant and equipment less accumulated depreciation. Chapter 11 provides a more detailed explanation of depreciation methods. Liabilities and Equity Accounts payable, notes payable, and accruals are listed as current liabilities because MicroDrive is expected to pay them within a year. When MicroDrive purchases supplies but doesn’t immediately pay for them, it takes on an obligation called an account payable. Similarly, when MicroDrive takes out a loan that must be repaid within a year, it signs an IOU called a note payable. MicroDrive doesn’t pay its taxes This is called accrual accounting, which attempts to match revenues to the periods in which they are earned and expenses to the periods in which the effort to generate income occurred. 2 Chapter 2: Financial Statements, Cash Flow, and Taxes 51 THE GLOBAL ECONOMIC CRISIS Let’s Play Hide-and-Seek! In a shameful lapse of regulatory accountability, banks and other financial institutions were allowed to use “structured investment vehicles” (SIVs) to hide assets and liabilities off their balance sheets and simply not report them. Here’s how SIVs worked and why they subsequently failed. The SIV was set up as a separate legal entity that the bank owned and managed. The SIV would borrow money in the short-term market (backed by the credit of the bank) and then invest in long-term securities. As you might guess, many SIVs invested in mortgage-backed securities. When the SIV paid only 3% on its borrowings but earned 10% on its investments, the managing bank was able to report fabulous earnings, especially if it also earned fees for creating the mortgage securities that went into the SIV. But this game of hide-and-seek doesn’t have a happy ending. Mortgage-backed securities began defaulting in 2007 and 2008, causing the SIVs to pass losses through to the banks. SunTrust, Citigroup, Bank of America, and Northern Rock are just a few of the many banks that reported enormous losses in the SIV game. Investors, depositors, and the government eventually found the hidden assets and liabilities, but by then the assets were worth a lot less than the liabilities. In a case of too little and too late, regulators are closing these loopholes, and it doesn’t look like there will be any more SIVs in the near future. But the damage has been done, and the entire financial system is at risk in large part because of this high-stakes game of hideand-seek. or its employees’ wages daily, and the amount it owes on these items at any point in time is called an “accrual” or an “accrued expense.” Long-term bonds are also liabilities because they, too, reflect a claim held by someone other than a stockholder. Preferred stock is a hybrid, or a cross between common stock and debt. In the event of bankruptcy, preferred stock ranks below debt but above common stock. Also, the preferred dividend is fixed, so preferred stockholders do not benefit if the company’s earnings grow. Most firms do not use much, or even any, preferred stock, so “equity” usually means “common equity” unless the words “total” or “preferred” are included. When a company sells shares of stock, the proceeds are recorded in the common stock account.3 Retained earnings are the cumulative amount of earnings that have not been paid out as dividends. The sum of common stock and retained earnings is called “common equity,” or sometimes just equity. If a company’s assets could actually be sold at their book value, and if the liabilities and preferred stock were actually worth their book values, then a company could sell its assets, pay off its liabilities and preferred stock, and the remaining cash would belong to common stockholders. Therefore, common equity is sometimes called net worth—it’s the assets net of the liabilities. Self-Test What is the balance sheet, and what information does it provide? What determines the order of the information shown on the balance sheet? Why might a company’s December 31 balance sheet differ from its June 30 balance sheet? A firm has $8 million in total assets. It has $3 million in current liabilities, $2 million in long-term debt, and $1 million in preferred stock. What is the total value of common equity? ($2 million) Companies sometimes break the total proceeds into two parts, one called “par” and the other called “paid-in capital” or “capital surplus.” For example, if a company sells shares of stock for $10, it might record $1 of par and $9 of paid-in capital. For most purposes, the distinction between par and paid-in capital is not important, and most companies use no-par stock. 3 52 Part 1: Fundamental Concepts of Corporate Finance 2.3 THE INCOME STATEMENT resource See Ch02 Tool Kit.xls for details. TABLE 2- 2 Table 2-2 shows the income statements for MicroDrive. Income statements can cover any period of time, but they are usually prepared monthly, quarterly, and annually. Unlike the balance sheet, which is a snapshot of a firm at a point in time, the income statement reflects performance during the period. Subtracting operating costs from net sales but excluding depreciation and amortization results in EBITDA, which stands for earnings before interest, taxes, depreciation, and amortization. Depreciation and amortization are annual charges that reflect the estimated costs of the assets used up each year. Depreciation applies to tangible M i c r oD r i v e I n c . : I n c o m e S t a t e m e n t s f or Ye a r s E n d i n g D e c e m b e r 3 1 ( M i l l i o n s o f D o l l a rs , E x c ep t f o r P er Sh ar e D a t a ) 2010 2009 $3,000.0 2,616.2 $ 383.8 $2,850.0 2,497.0 $ 353.0 $ 100.0 0.0 100.0 283.8 88.0 195.8 78.3 117.5 4.0 113.5 $ 90.0 0.0 90.0 263.0 60.0 203.0 81.2 121.8 4.0 117.8 Additional Information Common dividends Addition to retained earnings $ $ 57.5 56.0 $ $ 53.0 64.8 Per Share Data Common stock price Earnings per share (EPS) Dividends per share (DPS) Book value per share (BVPS) Cash flow per share (CFPS) $ $ $ $ $ 23.00 2.27 1.15 17.92 4.27 $ $ $ $ $ 26.00 2.36 1.06 16.80 4.16 Net sales Operating costs excluding depreciation and amortization Earnings before interest, taxes, depreciation, and amortization (EBITDA) Depreciation Amortization Depreciation and amortization Earnings before interest and taxes (EBIT, or operating income) Less interest Earnings before taxes (EBT) Taxes (40%) Net income before preferred dividends Preferred dividends Net income $ $ $ $ $ $ $ $ Notes: There are 50,000,000 shares of common stock outstanding. Note that EPS is based on earnings after preferred dividends—that is, on net income available to common stockholders. Calculations of the most recent EPS, DPS, BVPS, and CFPS values are as follows: Earnings per share ¼ EPS ¼ Dividends per share ¼ DPS ¼ Book value per share ¼ BVPS ¼ Cash flow per share ¼ CFPS ¼ Net income Common shares outstanding ¼ $113;500;000 ¼ $ 2:27 50;000;000 Dividends paid to common stockholders $57;500;000 ¼ Common shares outstanding 50;000;000 Total common equity Common shares outstanding ¼ $ 1:15 $896;000;000 50;000;000 ¼ $17:92 Net income þ Depreciation þ Amortization $213;500;000 ¼ Common shares outstanding 50;000;000 ¼ $ 4:27 ¼ Chapter 2: Financial Statements, Cash Flow, and Taxes 53 assets, such as plant and equipment, whereas amortization applies to intangible assets such as patents, copyrights, trademarks, and goodwill.4 Because neither depreciation nor amortization is paid in cash, some analysts claim that EBITDA is a better measure of financial strength than is net income. However, as we show later in the chapter, EBITDA is not as important as free cash flow. In fact, some financial wags have stated that EBITDA really stands for “earnings before anything bad happens.” The net income available to common shareholders, which is revenues less expenses, taxes, and preferred dividends (but before paying common dividends), is generally referred to as net income, although it is also called profit or earnings, particularly in the news or financial press. Dividing net income by the number of shares outstanding gives earnings per share (EPS), which is often called “the bottom line.” Throughout this book, unless otherwise indicated, net income means net income available to common stockholders.5 Self-Test What is an income statement, and what information does it provide? What is often called “the bottom line?” What is EBITDA? Regarding the time period reported, how does the income statement differ from the balance sheet? A firm has $2 million in earnings before taxes. The firm has an interest expense of $300,000 and depreciation of $200,000; it has no amortization. What is its EBITDA? ($2.5 million) 2.4 STATEMENT resource See Ch02 Tool Kit.xls for details. OF STOCKHOLDERS’ EQUITY Changes in stockholders’ equity during the accounting period are reported in the statement of stockholders’ equity. Table 2-3 shows that MicroDrive earned $113.5 million during 2010, paid out $57.5 million in common dividends, and plowed $56 million back into the business. Thus, the balance sheet item “Retained earnings” increased from $710 million at year-end 2009 to $766 million at year-end 2010.6 The last column shows the beginning stockholders’ equity, any changes, and the endof-year stockholders’ equity. Note that “retained earnings” does not represent assets but is instead a claim against assets. In 2010, MicroDrive’s stockholders allowed it to reinvest $56 million instead of distributing the money as dividends, and management spent this money The accounting treatment of goodwill resulting from mergers has changed in recent years. Rather than an annual charge, companies are required to periodically evaluate the value of goodwill and reduce net income only if the goodwill’s value has decreased materially (“become impaired,” in the language of accountants). For example, in 2002 AOL Time Warner wrote off almost $100 billion associated with the AOL merger. It doesn’t take too many $100 billion expenses to really hurt net income! 4 5 Companies also report “comprehensive income,” which is the sum of net income and any “comprehensive” income item, such as unrealized gain or loss when an asset is marked-to-market. For our examples, we assume that there are no comprehensive income items. Some companies also choose to report “pro forma income.” For example, if a company incurs an expense that it doesn’t expect to recur, such as the closing of a plant, it might calculate pro forma income as though it had not incurred the one-time expense. There are no hard-and-fast rules for calculating pro forma income, so many companies find ingenious ways to make pro forma income higher than traditional income. The SEC and the Public Company Accounting Oversight Board (PCAOB) are taking steps to reduce deceptive uses of pro forma reporting. If they had been applicable, then columns would have been used to show “Additional Paid-in Capital” and “Treasury Stock.” Also, additional rows would have contained information on such things as new issues of stock, treasury stock acquired or reissued, stock options exercised, and unrealized foreign exchange gains or losses. 6 54 Part 1: Fundamental Concepts of Corporate Finance TABLE 2-3 M i c r o D r i v e I n c . : S t a t e m e n t o f S t o c k h o l d e r s ’ Eq ui ty, D ec em b er 3 1, 20 1 0 ( M i l l i o n s of Do l l a r s ) COM MON STOCK (MILLIONS) Balances, Dec. 31, 2009 Net income Cash dividends Issuance of common stock Balances, Dec. 31, 2010 SHARES AMOUNT 50 $130.0 0 50 0.0 $130.0 RETAINED EAR NINGS TOTAL EQUITY $710.0 $113.5 (57.5) $840.0 $113.5 (57.5) $766.0 $896.0 Note: Here and throughout the book, parentheses are used to denote negative numbers. on new assets. Thus, retained earnings, as reported on the balance sheet, does not represent cash and is not “available” for the payment of dividends or anything else.7 Self-Test What is the statement of stockholders’ equity, and what information does it provide? Why do changes in retained earnings occur? Explain why the following statement is true: “The retained earnings reported on the balance sheet does not represent cash and is not available for the payment of dividends or anything else.” A firm had a retained earnings balance of $3 million in the previous year. In the current year, its net income is $2.5 million. If it pays $1 million in common dividends in the current year, what is its resulting retained earnings balance? ($4.5 million) 2.5 NET CASH FLOW A business’s net cash flow generally differs from its accounting profit because some of the revenues and expenses listed on the income statement were not received or paid in cash during the year. The relationship between net cash flow and net income is: Net cash flow ¼ Net income − Noncash revenues þ Noncash charges (2-1) The primary examples of noncash charges are depreciation and amortization. These items reduce net income but are not paid out in cash, so we add them back to net income when calculating net cash flow. Another example of a noncash charge is deferred taxes. In some instances, companies are allowed to defer tax payments to a later date even though the tax payment is reported as an expense on the income statement. Therefore, deferred tax payments are added to net income when calculatThe amount reported in the retained earnings account is not an indication of the amount of cash the firm has. Cash (as of the balance sheet date) is found in the cash account, an asset account. A positive number in the retained earnings account indicates only that in the past the firm earned some income, but its dividends paid were less than its earnings. Even though a company reports record earnings and shows an increase in its retained earnings account, it still may be short of cash. The same situation holds for individuals. You might own a new BMW (no loan), lots of clothes, and an expensive stereo—and hence have a high net worth—but if you have only 23 cents in your pocket plus $5 in your checking account, you will still be short of cash. 7 Chapter 2: Financial Statements, Cash Flow, and Taxes 55 ing net cash flow.8 Sometimes a customer will purchase services or products that extend beyond the reporting date, such as iPhone subscriptions at Apple. Even if the company collects the cash at the time of the purchase, the company will spread the reported revenues over the life of the purchase. This causes income to be lower than cash flow in the first year and higher in any subsequent years, so adjustments are made when calculating net cash flow. Depreciation and amortization usually are the largest noncash items, and in many cases the other noncash items roughly net out to zero. For this reason, many analysts assume that net cash flow equals net income plus depreciation and amortization: Net cash flow ¼ Net income þ Depreciation and amortization (2-2) We will generally assume that Equation 2-2 holds. However, you should remember that Equation 2-2 will not accurately reflect net cash flow when there are significant noncash items other than depreciation and amortization. We can illustrate Equation 2-2 with 2010 data for MicroDrive taken from Table 2-2: Net cash flow ¼ $113:5 þ $100:0 ¼ $213:5 million To illustrate depreciation’s effect, suppose a machine with a life of 5 years and zero expected salvage value was purchased in late 2009 for $100,000 and placed into service in early 2010. This $100,000 cost is not expensed in the purchase year; rather, it is charged against production over the machine’s 5-year depreciable life. If the depreciation expense were not taken, then profits would be overstated and taxes would be too high. Therefore, the annual depreciation charge is deducted from sales revenues, along with such other costs as labor and raw materials, to determine income. However, because the $100,000 was actually expended back in 2009, the depreciation charged against income in 2010 and subsequent years is not a cash outflow. Depreciation is a noncash charge, so it must be added back to net income to obtain the net cash flow. If we assume that all other noncash items (including amortization) sum to zero, then net cash flow is simply equal to net income plus depreciation. Self-Test Differentiate between net cash flow and accounting profit. A firm has net income of $5 million. Assuming that depreciation of $1 million is its only noncash expense, what is the firm’s net cash flow? ($6 million) 2.6 STATEMENT OF CASH FLOWS Even if a company reports a large net income during a year, the amount of cash reported on its year-end balance sheet may be the same or even lower than its beginning cash. The reason is that its net income can be used in a variety of ways, not just kept as cash in the bank. For example, the firm may use its net income to pay dividends, to increase inventories, to finance accounts receivable, to invest in fixed assets, to reduce debt, or to buy back common stock. Indeed, the company’s cash position as reported on its balance sheet is affected by a great many factors, which include the following. 1. Net income before preferred dividends. Other things held constant, a positive net income will lead to more cash in the bank. However, as we shall discuss, other things generally are not held constant. Deferred taxes may arise, for example, if a company uses accelerated depreciation for tax purposes but straight-line depreciation for reporting its financial statements to investors. If deferred taxes are increasing, then the company is paying less in taxes than it reports to the public. 8 56 Part 1: Fundamental Concepts of Corporate Finance Financial Analysis on the WEB A wide range of valuable financial information is available on the Web. With just a couple of clicks, an investor can easily find the key financial statements for most publicly traded companies. Here’s a partial (by no means a complete) list of places you can go to get started. ◆ One of the very best sources of financial in- formation is Thomson Financial. Go to the textbook’s Web site and follow the directions to access Thomson ONE—Business School Edition. An especially useful feature is the ability to download up to 10 years of financial statements in spreadsheet form. First, enter the ticker for a company and click Go. From the top tab (in dark blue), select Financials. This will show a second row of items (in light blue). Selecting More from this row will reveal a drop-down menu. Select SEC Database Reports & Charts. This will bring up another drop-down menu that includes the 10-year balance sheets, income statements, and statement of cash flows. To download the financial statements into a spreadsheet, first select one of the statements, such as the 10YR Balance Sheet. The balance sheets will then be displayed on your browser page. To download, click on the Excel icon toward the right of the light blue row at the top of the Thomson ONE panel. This will bring up a dialog box that lets you download the Excel file to your computer. ◆ Try Yahoo! Finance’s Web site, http://finance .yahoo.com. Here you will find updated market information along with links to a variety of interesting research sites. Enter a stock’s ticker symbol, click GO, and you will see the stock’s current price along with recent news about the company. The panel on the left has links to key statistics and to the company’s income statement, balance sheet, statement ◆ ◆ ◆ ◆ ◆ of cash flows, and more. The Web site also has a list of insider transactions, so you can tell if a company’s CEO and other key insiders are buying or selling their company’s stock. In addition, there is a message board where investors share opinions about the company, and there is a link to the company’s filings with the SEC. Note that, in most cases, a more complete list of the SEC filings can be found at http://www.sec.gov. Other sources for up-to-date market information are http://money.cnn.com and http://www.zacks.com. These sites also provide financial statements in standardized formats. Both http://www.bloomberg.com and http://www.marketwatch.com have areas where you can obtain stock quotes along with company financials, links to Wall Street research, and links to SEC filings. If you are looking for charts of key accounting variables (for example, sales, inventory, depreciation and amortization, and reported earnings) as well as financial statements, take a look at http://www.smartmoney.com. Another good place to look is http://www .investor.reuters.com. Here you can find links to analysts’ research reports along with the key financial statements. Zacks (already mentioned) and http://www .hoovers.com have free research available along with more detailed information provided to subscribers. In addition to this information, you may be looking for sites that provide opinions regarding the direction of the overall market and views regarding individual stocks. Two popular sites in this category are The Motley Fool’s Web site, http://www.fool.com, and the Web site for The Street.com, http://www.thestreet.com. 2. Noncash adjustments to net income. To calculate cash flow, it is necessary to adjust net income to reflect noncash revenues and expenses, such as depreciation and deferred taxes, as shown previously in the calculation of net cash flow. 3. Changes in working capital. Increases in current assets other than cash (such as inventories and accounts receivable) decrease cash, whereas decreases in Chapter 2: Financial Statements, Cash Flow, and Taxes 57 these accounts increase cash. For example, if inventories are to increase, then the firm must use some of its cash to acquire the additional inventory. Conversely, if inventories decrease, this generally means the firm is selling inventories and not replacing all of them, hence generating cash. On the other hand, if payables increase then the firm has received additional credit from its suppliers, which saves cash, but if payables decrease, this means it has used cash to pay off its suppliers. Therefore, increases in current liabilities such as accounts payable increase cash, whereas decreases in current liabilities decrease cash. 4. Investments. If a company invests in fixed assets or short-term financial investments, this will reduce its cash position. On the other hand, if it sells some fixed assets or short-term investments, this will increase cash. 5. Security transactions and dividend payments. If a company issues stock or bonds during the year, the funds raised will increase its cash position. On the other hand, if the company uses cash to buy back outstanding stock or to pay off debt, or if it pays dividends to its shareholders, this will reduce cash. Each of these five factors is reflected in the statement of cash flows, which summarizes the changes in a company’s cash position. The statement separates activities into three categories, plus a summary section, as follows. 1. Operating activities, which includes net income, depreciation, changes in current assets and liabilities other than cash, short-term investments, and short-term debt. 2. Investing activities, which includes investments in or sales of fixed assets and short-term financial investments. 3. Financing activities, which includes raising cash by issuing short-term debt, long-term debt, or stock. Also, because dividend payments, stock repurchases, and principal payments on debt reduce a company’s cash, such transactions are included here. Accounting texts explain how to prepare the statement of cash flows, but the statement is used to help answer questions such as these: Is the firm generating enough cash to purchase the additional assets required for growth? Is the firm generating any extra cash that can be used to repay debt or to invest in new products? Such information is useful both for managers and investors, so the statement of cash flows is an important part of the annual report. Table 2-4 shows MicroDrive’s statement of cash flows as it would appear in the company’s annual report. The top section shows cash generated by and used in operations—for MicroDrive, operations provided net cash flows of minus $2.5 million. This subtotal, the minus $2.5 million net cash flow provided by operating activities, is in many respects the most important figure in any of the financial statements. Profits as reported on the income statement can be “doctored” by such tactics as depreciating assets too slowly, not recognizing bad debts promptly, and the like. However, it is far more difficult to simultaneously doctor profits and the working capital accounts. Therefore, it is not uncommon for a company to report positive net income right up to the day it declares bankruptcy. In such cases, however, the net cash flow from operations almost always began to deteriorate much earlier, and analysts who kept an eye on cash flow could have predicted trouble. Therefore, if you are ever analyzing a company and are pressed for time, look first at the trend in net cash flow provided by operating activities, because it will tell you more than any other number. 58 Part 1: Fundamental Concepts of Corporate Finance T AB L E 2 - 4 M i c r oD r i v e I n c . : S t a t e m e n t of Ca s h F l o w s f o r 2 0 10 (Millions of Dollars) resource See Ch02 Tool Kit.xls for details. CASH PROVIDED OR USED Operating Activities Net income before preferred dividends Adjustments: Noncash adjustments: Depreciationa Due to changes in working capital:b Increase in accounts receivable Increase in inventories Increase in accounts payable Increase in accruals Net cash provided (used) by operating activities (60.0) (200.0) 30.0 10.0 ($ 2.5) Investing Activities Cash used to acquire fixed assetsc Sale of short-term investments Net cash provided (used) by investing activities ($230.0) $ 65.0 ($165.0) Financing Activities Increase in notes payable Increase in bonds outstanding Payment of preferred and common dividends Net cash provided (used) by financing activities $ 50.0 174.0 (61.5) $162.5 Summary Net change in cash Cash at beginning of year Cash at end of year $117.5 100.0 ($ 5.0) 15.0 $ 10.0 a Depreciation is a noncash expense that was deducted when calculating net income. It must be added back to show the correct cash flow from operations. b An increase in a current asset decreases cash. An increase in a current liability increases cash. For example, inventories increased by $200 million and therefore reduced cash by a like amount. c The net increase in fixed assets is $130 million; however, this net amount is after a deduction for the year’s depreciation expense. Depreciation expense would have to be added back to find the increase in gross fixed assets. From the company’s income statement, we see that the 2010 depreciation expense is $100 million; thus, expenditures on fixed assets were actually $230 million. The second section shows investing activities. MicroDrive purchased fixed assets totaling $230 million and sold $65 million of short-term investments, for a net cash flow from investing activities of minus $165 million. The third section, financing activities, includes borrowing from banks (notes payable), selling new bonds, and paying dividends on common and preferred stock. MicroDrive raised $224 million by borrowing, but it paid $61.5 million in preferred and common dividends. Therefore, its net inflow of funds from financing activities was $162.5 million. In the summary, when all of these sources and uses of cash are totaled, we see that MicroDrive’s cash outflows exceeded its cash inflows by $5 million during 2010; that is, its net change in cash was a negative $5 million. MicroDrive’s statement of cash flows should be worrisome to its managers and to outside analysts. The company had a $2.5 million cash shortfall from operations, it spent Chapter 2: Financial Statements, Cash Flow, and Taxes 59 an additional $230 million on new fixed assets, and it paid out another $61.5 million in dividends. It covered these cash outlays by borrowing heavily and by liquidating $65 million of short-term investments. Obviously, this situation cannot continue year after year, so something will have to be done. In Chapter 12, when we discuss financial planning, we consider some of the actions that MicroDrive’s financial staff might recommend.9 Self-Test What types of questions does the statement of cash flows answer? Identify and briefly explain the three different categories of activities shown in the statement of cash flows. A firm has inventories of $2 million for the previous year and $1.5 million for the current year. What impact does this have on net cash provided by operations? (Increase of $500,000) 2.7 MODIFYING ACCOUNTING DATA DECISIONS FOR MANAGERIAL Thus far in the chapter we have focused on financial statements as they are presented in the annual report. When you studied income statements in accounting, the emphasis was probably on the firm’s net income, which is its accounting profit. However, the intrinsic value of a company’s operations is determined by the stream of cash flows that the operations will generate now and in the future. To be more specific, the value of operations depends on all the future expected free cash flows (FCF), defined as aftertax operating profit minus the amount of new investment in working capital and fixed assets necessary to sustain the business. Therefore, the way for managers to make their companies more valuable is to increase free cash flow now and in the future. Notice that FCF is the cash flow available for distribution to all the company’s investors after the company has made all investments necessary to sustain ongoing operations. How well have MicroDrive’s managers done in generating FCF? In this section, we will calculate MicroDrive’s FCF and evaluate the performance of MicroDrive’s managers. Figure 2-1 shows the five steps in calculating free cash flow. As we explain each individual step in the following sections, refer back to Figure 2-1 to keep the big picture in mind. Net Operating Profit after Taxes (NOPAT) If two companies have different amounts of debt and hence different amounts of interest charges, they could have identical operating performances but different net incomes—the one with more debt would have a lower net income. Net income is certainly important, but it does not always reflect the true performance of a company’s operations or the effectiveness of its operating managers. A better measurement for comparing managers’ performance is net operating profit after taxes, or NOPAT, which is the amount of profit a company would generate if it had no debt and held no financial assets. NOPAT is defined as follows:10 For a more detailed discussion of financial statement analysis, see Lyn M. Fraser and Aileen Ormiston, Understanding Financial Statements, 9th ed. (Upper Saddle River, NJ: Prentice-Hall, 2010). 9 For firms with a more complicated tax situation, it is better to define NOPAT as follows: NOPAT = (Net income before preferred dividends) + (Net interest expense)(1 − Tax rate). Also, if firms are able to defer paying some of their taxes, perhaps by the use of accelerated depreciation, then NOPAT should be adjusted to reflect the taxes that the company actually paid on its operating income. See P. Daves, M. Ehrhardt, and R. Shrieves, Corporate Valuation: A Guide for Managers and Investors (Mason, OH: Thomson South-Western, 2004) for a detailed explanation of these and other adjustments. Also see Tim Koller, Marc Goedhart, and David Wessels, Valuation: Measuring and Managing the Value of Companies (Hoboken, NJ: Wiley, 2005), and G. Bennett Stewart, The Quest for Value (New York: Harper Collins, 1991). 10 60 Part 1: Fundamental Concepts of Corporate Finance FIGURE 2-1 Calculating Free Cash Flow Step 2 Step 1 Operating current assets Earning before interest and taxes X – (1 – Tax rate) Operating current liabilities Net operating working capital Net operating profit after taxes Step 3 Net operating working capital + Operating long-term assets Total net operating capital Step 5 Step 4 Net operating profit after taxes Total net operating capital this year – Net investment in operating capital Free cash flow – Total net operating capital last year Net investment in operating capital NOPAT ¼ EBITð1 − Tax rateÞ (2-3) Using data from the income statements of Table 2-2, MicroDrive’s 2010 NOPAT is NOPAT ¼ $283:8ð1 − 0:4Þ ¼ $283:8ð0:6Þ ¼ $170:3 million This means MicroDrive generated an after-tax operating profit of $170.3 million, a little better than its previous NOPAT of $263(0.6) = $157.8 million. However, the income statements in Table 2-2 show that MicroDrive’s earnings per share actually declined. This decrease in EPS was caused by an increase in interest expense, and not by a decrease in operating profit. Net Operating Working Capital Most companies need some current assets to support their operating activities. For example, all companies must carry some cash to “grease the wheels” of their operations. Companies continuously receive checks from customers and write checks to suppliers, employees, and so on. Because inflows and outflows do not coincide perfectly, a company must keep some cash in its bank account. In other words, some cash is required to conduct operations. The same is true for most other current assets, such as inventory and accounts receivable, which are required for normal operations. The short-term assets normally used in a company’s operating activities are called operating current assets. Not all current assets are operating current assets. For example, holdings of short-term securities generally result from investment decisions made by the Chapter 2: Financial Statements, Cash Flow, and Taxes 61 treasurer and not as a natural consequence of operating activities. Therefore, short-term investments are nonoperating assets and normally are excluded when calculating operating current assets.11 A useful rule of thumb is that if an asset pays interest, it should not be classified as an operating asset. Some current liabilities—especially accounts payable and accruals—arise in the normal course of operations. Such short-term liabilities are called operating current liabilities. Not all current liabilities are operating current liabilities. For example, consider the current liability shown as notes payable to banks. The company could have raised an equivalent amount as long-term debt or could have issued stock, so the choice to borrow from the bank was a financing decision and not a consequence of operations. Again, the rule of thumb is that if a liability charges interest, it is not an operating liability. If you are ever uncertain about whether an item is an operating asset or operating liability, ask yourself whether the item is a natural consequence of operations or if it is a discretionary choice, such as a particular method of financing or an investment in a particular financial asset. If it is discretionary, then the item is not an operating asset or liability. Notice that each dollar of operating current liabilities is a dollar that the company does not have to raise from investors in order to conduct its short-term operating activities. Therefore, we define net operating working capital (NOWC) as operating current assets minus operating current liabilities. In other words, net operating working capital is the working capital acquired with investor-supplied funds. Here is the definition in equation form: Net operating Operating current Operating current ¼ − working capital assets liabilities (2-4) We can apply these definitions to MicroDrive, using the balance sheet data given in Table 2-1. Here is its net operating working capital at year-end 2010: NOWC ¼ Operating current assets  Operating current liabilities ¼ ðCash þ Accounts receivable þ InventoriesÞ − ðAccounts payable þ AccrualsÞ ¼ ð$10 þ $375 þ $615Þ − ð$60 þ $140Þ ¼ $800 million For the previous year, net operating working capital was NOWC ¼ ð$15 þ $315 þ $415Þ − ð$30 þ $130Þ ¼ $585 million Total Net Operating Capital In addition to working capital, most companies also use long-term assets to support their operations. These include land, buildings, factories, equipment, and the like. Total net operating capital is the sum of NOWC and operating long-term assets: Total net operating capital ¼ NOWC þ operating long-term assets (2-5) If the marketable securities are held as a substitute for cash and therefore reduce the cash requirements, then they may be classified as part of operating working capital. Generally, though, large holdings of marketable securities are held as a reserve for some contingency or else as a temporary “parking place” for funds prior to an acquisition, a major capital investment program, or the like. 11 62 Part 1: Fundamental Concepts of Corporate Finance Because MicroDrive’s operating long-term assets consist only of net plant and equipment, its total net operating capital at year-end 2010 was Total net operating capital ¼ $800 þ $1;000 ¼ $1;800 million For the previous year, its total net operating capital was Total net operating capital ¼ $585 þ $870 ¼ $1;455 million Notice that we have defined total net operating capital as the sum of net operating working capital and operating long-term assets. In other words, our definition is in terms of operating assets and liabilities. However, we can also calculate total net operating capital by adding up the funds provided by investors, such as notes payable, long-term bonds, preferred stock, and common equity. For MicroDrive, the total capital provided by investors at year-end 2009 was $60 + $580 + $40 + $840 = $1,520 million. Of this amount, $65 million was tied up in short-term investments, which are not directly related to MicroDrive’s operations. Therefore, only $1,520 − $65 = $1,455 million of investor-supplied capital was used in operations. Notice that this is exactly the same value as calculated before. This shows that we can calculate total net operating capital either from net operating working capital and operating long-term assets or from the investor-supplied funds. We usually base our calculations on operating data because this approach allows us to analyze a division, factory, or work center, whereas the approach based on investor-supplied capital is applicable only for the entire company. The expression “total net operating capital” is a mouthful, so we often call it operating capital or even just capital. Also, unless we specifically say “investor-supplied capital,” we are referring to total net operating capital. Net Investment in Operating Capital As calculated previously, MicroDrive had $1,455 million of total net operating capital at the end of 2009 and $1,800 million at the end of 2010. Therefore, during 2010, it made a net investment in operating capital of Net investment in operating capital ¼ $1;800 − $1;455 ¼ $345 million Most of this investment was made in net operating working capital, which rose from $585 million to $800 million, or by $215 million. This 37% increase in net operating working capital, in view of a sales increase of only 5% (from $2,850 to $3,000 million), should set off warning bells in your head: Why did MicroDrive tie up so much additional cash in working capital? Is the company gearing up for a big increase in sales, or are inventories not moving and receivables not being collected? We will address these questions in detail in Chapter 3, when we cover ratio analysis. Calculating Free Cash Flow Free cash flow is defined as FCF ¼ NOPAT − Net investment in operating capital MicroDrive’s free cash flow in 2010 was FCF ¼ $170:3 − ð$1;800 − $1;455Þ ¼ $170:3 − $345 ¼ −$174:7 million (2-6) Chapter 2: Financial Statements, Cash Flow, and Taxes 63 Financial Bamboozling: How to Spot It Recent accounting frauds by Enron, WorldCom, Xerox, Merck, Arthur Andersen, Tyco, and many others have shown that analysts can no longer blindly assume that a firm’s published financial statements are the best representation of its financial position. Clearly, many companies were “pushing the envelope” if not outright lying in an effort to make their companies look better. A recent Fortune article points out that there are only three basic ways to manipulate financial statements: moving earnings from the future to the present, avoiding taxes, or hiding debt. For example, suppose one telecom firm (think WorldCom or Global Crossing) sold the right to use parts of its fiber-optic network for 10 years to another telecom firm for $100 million. The seller would immediately record revenues of $100 million. The buyer, however, could spread the expense over 10 years and report an expense of only $10 million this year. The buyer would simultaneously sell similar rights to the original seller for $100 million. This way, no cash changes hands, both companies report an extra $100 million in revenue, but each reports a cost of only $10 million. Thus, both companies “created” an extra $90 million in pre-tax profits without actually doing anything. Of course, both companies will have to report an extra $10 million expense each year for the remaining 9 years, but they have each boosted shortterm profits and thus this year’s executive bonuses. To boost earnings next year, all they have to do is play the same game, but on a bigger scale. For hiding debt, it’s hard to beat Enron’s special purpose entities (SPEs). These SPEs owed hundreds of millions of dollars, and it turned out that Enron was responsible for this debt, even though it never showed up on Enron’s financial statements. How can you spot bamboozling? Here are some tips. When companies have lots of write-offs or charges for restructuring, it could be that they are planning on managing earnings in the future. In other words, they sandbag this year to pad next year’s earnings. Beware of serial acquirers, especially if they use their own stock to buy other companies. This can increase reported earnings, but it often erodes value since the acquirer usually pays a large premium for the target. Watch out for companies that depreciate their assets much more slowly than others in the industry (this is shown in the financial statements’ footnotes). This causes their current earnings to look larger than their competitors’, even though they aren’t actually performing any better. Perhaps the best evidence of bamboozling is if earnings are consistently growing faster than cash flows, which almost always indicates a financial scam. Sources: Geoffrey Colvin, “Bamboozling: A Field Guide,” Fortune, July 8, 2002, 51; and Shawn Tully, “Don’t Get Burned,” Fortune, February 18, 2002, 87–90. Although we prefer this approach to calculating FCF, sometimes the financial press calculates FCF with a different approach. The results are the same either way, but you should be aware of this alternative approach. The difference lies in how depreciation is treated. To see this, notice that net fixed assets rose from $870 to $1,000 million, or by $130 million. However, MicroDrive reported $100 million of depreciation, so its gross investment in fixed assets was $130 + $100 = $230 million for the year. With this background, the gross investment in operating capital is Net investment Gross investment ¼ þ Depreciation in operating capital in operating capital For MicroDrive, the gross investment in operating capital was: Gross investment ¼ $345 þ $100 ¼ $445 million in operating capital (2-7) 64 Part 1: Fundamental Concepts of Corporate Finance Because depreciation is a noncash expense, some analysts calculate operating cash flow as Operating cash flow ¼ NOPAT þ Depreciation (2-8) MicroDrive’s most recent operating cash flow is Operating cash flow ¼ NOPAT þ Depreciation ¼ $170:3 þ $100 ¼ $270:3 An algebraically equivalent expression for free cash flow in terms of operating cash flow and gross investment in operating capital is  FCF ¼ NOPAT þ Depreciation  0 1 Net investment − @ in operating capital A þ Depreciation Operating Gross investment ¼ cash flow − in operating capital (2-9) Gross investment Investment Operating − in NOWC ¼ cash flow − in long-term operating assets For MicroDrive, this definition produces FCF of −$174.7, the same value as found earlier: FCF ¼ ð$170:3 þ $100Þ − $445 ¼ −$174:7 million Equations 2-6 and 2-9 are equivalent because depreciation is added to both NOPAT and net investment in Equation 2-6 to arrive at Equation 2-9. We usually use Equation 2-6, because it saves us this step, but you should be aware of this alternative approach. The Uses of FCF Recall that free cash flow (FCF) is the amount of cash that is available for distribution to all investors, including shareholders and debtholders. There are five good uses for FCF: 1. Pay interest to debtholders, keeping in mind that the net cost to the company is the after-tax interest expense. 2. Repay debtholders; that is, pay off some of the debt. 3. Pay dividends to shareholders. 4. Repurchase stock from shareholders. 5. Buy short-term investments or other nonoperating assets. Consider MicroDrive, with its FCF of −$174.7 million in 2010. How did MicroDrive use the FCF? MicroDrive’s income statement shows an interest expense of $88 million. With a tax rate of 40%, the after-tax interest payment for the year is After-tax interest payment ¼ $88ð1 − 40%Þ ¼ $52:8 million The net amount of debt that is repaid is equal to the amount at the beginning of the year minus the amount at the end of the year. This includes notes payable and long-term debt. If the amount of ending debt is less than the beginning debt, the company paid down Chapter 2: Financial Statements, Cash Flow, and Taxes 65 some of its debt. But if the ending debt is greater than the beginning debt, the company actually borrowed additional funds from creditors. In that case, it would be a negative use of FCF. For MicroDrive, the net debt repayment for 2010 is Net reduction in debt ¼ ð$60 þ $580Þ − ð$754 − $110Þ ¼ −$224 million This is a “negative use” of FCF because it increased the debt balance. This is typical of most companies because growing companies usually add debt each year. MicroDrive paid $4 million in preferred dividends and $57.5 in common dividends for a total of Dividend payments ¼ $4 þ $57:5 ¼ $61:5 million The net amount of stock that is repurchased is equal to the amount at the beginning of the year minus the amount at the end of the year. This includes preferred stock and common stock. If the amount of ending stock is less than the beginning stock, then the company made net repurchases. But if the ending stock is greater than the beginning stock, the company actually made net issuances. In that case, it would be a negative use of FCF. Even though MicroDrive neither issued nor repurchased stock during the year, many companies use FCF to repurchase stocks as a replacement for or supplement to dividends, as we discuss in Chapter 14. The amount of net purchases of short-term investments is equal to the amount at the end of the year minus the amount at the beginning of the year. If the amount of ending investments is greater than the beginning investments, then the company made net purchases. But if the ending investments are less than the beginning investments, the company actually sold investments. In that case, it would be a negative use of FCF. MicroDrive’s net purchases of short-term investments in 2010 is: Net purchases of short-term investments ¼ $0 − $65 ¼ −$65 million Notice that this is a “negative use” because MicroDrive sold short-term investments instead of purchasing them. We combine these individual uses of FCF to find the total uses. 1. 2. 3. 4. 5. After-tax interest: $ 52.8 Net debt repayments: −224.0 Dividends: 61.5 Net stock repurchases: 0.0 −65.0 Net purchases of ST investments: Total uses of FCF: −$174.7 The −$174.7 total for uses of FCF is identical to the value of FCF from operations that we calculated previously. If it were not equal, then we would have made an error somewhere in our calculations. Observe that a company does not use FCF to acquire operating assets, because the calculation of FCF already takes into account the purchase of operating assets needed to support growth. Unfortunately, there is evidence to suggest that some companies with high FCF tend to make unnecessary investments that don’t add value, such as paying too much to acquire another company. Thus, high FCF can cause waste if managers fail to act in the best interests of shareholders. As discussed in Chapter 1, this is called an agency cost, since managers are hired as agents to act on behalf of stockholders. We discuss agency costs and ways to control them in Chapter 13, where we discuss value-based management and corporate governance, and in Chapter 15, where we discuss the choice of capital structure. 66 Part 1: Fundamental Concepts of Corporate Finance FCF and Corporate Value Free cash flow is the amount of cash available for distribution to investors; so the fundamental value of a company to its investors depends on the present value of its expected future FCFs, discounted at the company’s weighted average cost of capital (WACC). Subsequent chapters will develop the tools needed to forecast FCFs and evaluate their risk. Chapter 13 ties all this together with a model that is used to calculate the value of a company. Even though you do not yet have all the tools to apply the model, it’s important that you understand this basic concept: FCF is the cash flow available for distribution to investors. Therefore, the fundamental value of a firm primarily depends on its expected future FCF. Evaluating FCF, NOPAT, and Operating Capital Even though MicroDrive had a positive NOPAT, its very high investment in operating assets resulted in a negative FCF. Because free cash flow is the cash flow available for distribution to investors, MicroDrive’s negative FCF meant that MicroDrive had to sell short-term investments and so investors actually had to provide additional money to keep the business going. Is a negative free cash flow always bad? The answer is, “Not necessarily; it depends on why the free cash flow was negative.” It’s a bad sign if FCF was negative because NOPAT was negative, since then the company is probably experiencing operating problems. However, many high-growth companies have positive NOPAT but negative FCF because they are making large investments in operating assets to support growth. There is nothing wrong with profitable growth, even if it causes negative cash flows. One way to determine whether growth is profitable is by examining the return on invested capital (ROIC), which is the ratio of NOPAT to total operating capital. If the ROIC exceeds the rate of return required by investors, then a negative free cash flow caused by high growth is nothing to worry about. Chapter 13 discusses this in detail. To calculate the ROIC, we first calculate NOPAT and operating capital. The return on invested capital is a performance measure that indicates how much NOPAT is generated by each dollar of operating capital: ROIC ¼ NOPAT Operating capital (2-10) If ROIC is greater than the rate of return that investors require, which is the weighted average cost of capital (WACC), then the firm is adding value. As noted previously, a negative FCF is not necessarily bad, provided it is due to high, profitable growth.12 For example, Qualcomm’s sales grew by 26% in 2008, which led to large capital investments and a FCF of negative $4.6 billion. However, its ROIC was about 29%, so the growth was profitable. At some point Qualcomm’s growth will slow and will not require large capital investments. If Qualcomm maintains a high ROIC, then its FCF will become positive and very large as growth slows. MicroDrive had an ROIC in 2010 of 9.46% ($170.3/$1,800 = 0.0946). Is this enough to cover its cost of capital? We’ll answer that question in the next section. 12 If g is the growth rate in capital, then with a little (or a lot of!) algebra, free cash flow is   g FCF ¼ Capital ROIC  1þg This shows that when the growth rate gets almost as high as ROIC, then FCF will be negative. Chapter 2: Financial Statements, Cash Flow, and Taxes Self-Test 67 What is net operating working capital? Why does it exclude most short-term investments and also notes payable? What is total net operating capital? Why is it important for managers to calculate a company’s capital requirements? Why is NOPAT a better performance measure than net income? What is free cash flow? Why is it important? A firm’s total net operating capital for the previous year was $2 million. For the current year, its total net operating capital is $2.5 million and its NOPAT is $1.2 million. What is its free cash flow for the current year? ($700,000) 2.8 MVA AND EVA Neither traditional accounting data nor the modified data discussed in the preceding section incorporates stock prices, even though the primary goal of management is to maximize the firm’s stock price. Financial analysts have therefore developed two additional performance measures, Market Value Added (MVA) and Economic Value Added (EVA). These concepts are discussed in this section.13 Market Value Added (MVA) The primary goal of most firms is to maximize shareholders’ wealth. This goal obviously benefits shareholders, but it also helps to ensure that scarce resources are allocated efficiently, which benefits the economy. Shareholder wealth is maximized by maximizing the difference between the market value of the firm’s stock and the amount of equity capital that was supplied by shareholders. This difference is called the Market Value Added (MVA): WWW For an updated estimate of Coca-Cola’s MVA, go to http://finance.yahoo.com, enter KO, and click GO. This shows the market value of equity, called Mkt Cap. To get the book value of equity, select Balance Sheet from the left panel. MVA ¼ Market value of stock − Equity capital supplied by shareholders ¼ ðShares outstandingÞðStock priceÞ − Total common equity (2-11) To illustrate, consider Coca-Cola. In January 2009, its total market equity value was $103.2 billion while its balance sheet showed that stockholders had put up only $23.7 billion. Thus, Coca-Cola’s MVA was $103.2 − $23.7 = $79.5 billion. This $79.5 billion represents the difference between the money that Coca-Cola’s stockholders have invested in the corporation since its founding—including indirect investment by retaining earnings—and the cash they could get if they sold the business. The higher its MVA, the better the job management is doing for the firm’s shareholders. Sometimes MVA is defined as the total market value of the company minus the total amount of investor-supplied capital: MVA ¼ Total market value − Total investor-supplied capital ¼ ðMarket value of stock þ Market value of debtÞ − Total investor-supplied capital (2-11a) The concepts of EVA and MVA were developed by Joel Stern and Bennett Stewart, co-founders of the consulting firm Stern Stewart & Company. Stern Stewart copyrighted the terms “EVA” and “MVA,” so other consulting firms have given other names to these values. Still, EVA and MVA are the terms most commonly used in practice. 13 68 Part 1: Fundamental Concepts of Corporate Finance For most companies, the total amount of investor-supplied capital is the sum of equity, debt, and preferred stock. We can calculate the total amount of investorsupplied capital directly from their reported values in the financial statements. The total market value of a company is the sum of the market values of common equity, debt, and preferred stock. It is easy to find the market value of equity, since stock prices are readily available, but it is not always easy to find the market value of debt. Hence, many analysts use the value of debt that is reported in the financial statements, which is the debt’s book value, as an estimate of its market value. For Coca-Cola, the total amount of reported debt was about $24.4 billion, and Coca-Cola had no preferred stock. Using this as an estimate of the market value of debt, Coke’s total market value was $103.2 + $24.4 = $127.6 billion. The total amount of investor-supplied funds was $23.7 + $24.4 = $48.1 billion. Using these total values, the MVA was $127.6 − $48.1 = $79.5 billion. Note that this is the same answer as when we used the previous definition of MVA. Both methods will give the same result if the market value of debt is approximately equal to its book value. Economic Value Added (EVA) Whereas MVA measures the effects of managerial actions since the very inception of a company, Economic Value Added (EVA) focuses on managerial effectiveness in a given year. The basic EVA formula is: EVA ¼ Net operating profit after taxes ðNOPATÞ  After-tax dollar cost of capital used to support operations ¼ EBITð1  Tax rateÞ  ðTotal net operating capitalÞðWACCÞ (2-12) We can also calculate EVA in terms of ROIC: EVA ¼ ðOperating capitalÞðROIC  WACCÞ (2-13) As this equation shows, a firm adds value—that is, has a positive EVA—if its ROIC is greater than its WACC. If WACC exceeds ROIC, then new investments in operating capital will reduce the firm’s value. Economic Value Added is an estimate of a business’s true economic profit for the year, and it differs sharply from accounting profit.14 EVA represents the residual income that remains after the cost of all capital, including equity capital, has been deducted, whereas accounting profit is determined without imposing a charge for equity capital. As we discuss in Chapter 9, equity capital has a cost because shareholders give up the opportunity to invest and earn returns elsewhere when they provide capital to the firm. This cost is an opportunity cost rather than an accounting cost, but it is quite real nevertheless. Note that when calculating EVA we do not add back depreciation. Although it is not a cash expense, depreciation is a cost because worn-out assets must be replaced, and it is therefore deducted when determining both net income and EVA. Our calculation of The most important reason EVA differs from accounting profit is that the cost of equity capital is deducted when EVA is calculated. Other factors that could lead to differences include adjustments that might be made to depreciation, to research and development costs, to inventory valuations, and so on. These other adjustments also can affect the calculation of investor-supplied capital, which affects both EVA and MVA. See Stewart, The Quest for Value, cited in footnote 10. 14 Chapter 2: Financial Statements, Cash Flow, and Taxes 69 EVA assumes that the true economic depreciation of the company’s fixed assets exactly equals the depreciation used for accounting and tax purposes. If this were not the case, adjustments would have to be made to obtain a more accurate measure of EVA. Economic Value Added measures the extent to which the firm has increased shareholder value. Therefore, if managers focus on EVA, this will help to ensure that they operate in a manner that is consistent with maximizing shareholder wealth. Note too that EVA can be determined for divisions as well as for the company as a whole, so it provides a useful basis for determining managerial performance at all levels. Consequently, EVA is being used by an increasing number of firms as the primary basis for determining managerial compensation. Table 2-5 shows how MicroDrive’s MVA and EVA are calculated. The stock price was $23 per share at year-end 2010, down from $26 per share the previous year. Its WACC, which is the percentage after-tax cost of capital, was 10.8% in 2009 and 11.0% in 2010, and its tax rate was 40%. Other data in Table 2-5 were given in the basic financial statements provided earlier in the chapter. Note first that the lower stock price and the higher book value of equity (due to retaining earnings during 2010) combined to reduce the MVA. The 2010 MVA is still positive, but $460 − $254 = $206 million of stockholders’ value was lost during the year. Economic Value Added for 2009 was just barely positive, and in 2010 it was negative. Operating income (NOPAT) rose, but EVA still declined, primarily because the amount of capital rose more sharply than NOPAT—by about 26% versus 8%—and the cost of this additional capital pulled EVA down. Recall also that net income fell, but not nearly so dramatically as the decline in EVA. Net income does not reflect the amount of equity capital employed, but EVA TABLE 2-5 resource See Ch02 Tool Kit.xls for details. M V A a nd E VA fo r M i c r oD r i v e I nc . ( M i l l i on s o f Do l l a r s ) 2010 MVA Calculation Price per share Number of shares (millions) Market value of equity = Share price × Number of shares Book value of equity MVA = Market value − Book value EVA Calculation EBIT Tax rate NOPAT = EBIT(1 − T ) Total investor-supplied operating capitala Weighted average cost of capital, WACC (%) Dollar cost of capital = Operating capital × WACC EVA = NOPAT − Dollar cost of capital ROIC = NOPAT ÷ Operating capital ROIC − Cost of capital = ROIC − WACC EVA = Operating capital × (ROIC − WACC) a 2 00 9 $ 23.0 50.0 $1,150.0 $ $ 896.0 $ 254.0 $ 840.0 $ 460.0 $ 283.8 40.0% $ 170.3 $1,800.0 11.0% $ 198.0 ($ 27.7) 9.46% (1.54%) ($ 27.7) 26.0 50.0 $1,300.0 $ 263.0 40.0% $ 157.8 $1,455.0 10.8% $ 157.1 $ 0.7 10.85% 0.05% $ 0.7 Investor-supplied operating capital equals the sum of notes payable, long-term debt, preferred stock, and common equity, less short-term investments. It could also be calculated as total liabilities and equity minus accounts payable, accruals, and short-term investments. It is also equal to total net operating capital. 70 Part 1: Fundamental Concepts of Corporate Finance Sarbanes-Oxley and Financial Fraud Investors need to be cautious when they review financial statements. Although companies are required to follow generally accepted accounting principles (GAAP), managers still have quite a lot of discretion in deciding how and when to report certain transactions. Consequently, two firms in exactly the same operating situation may report financial statements that convey different impressions about their financial strength. Some variations may stem from legitimate differences of opinion about the correct way to record transactions. In other cases, managers may choose to report numbers in a way that helps them present either higher earnings or more stable earnings over time. As long as they follow GAAP, such actions are not illegal, but these differences make it harder for investors to compare companies and gauge their true performances. Unfortunately, there have also been cases where managers overstepped the bounds and reported fraudulent statements. Indeed, a number of high-profile executives have faced criminal charges because of their misleading accounting practices. For example, in June 2002 it was discovered that WorldCom (now called MCI) had committed the most massive accounting fraud of all time by recording over $7 billion of ordinary operating costs as capital expenditures, thus overstating net income by the same amount. WorldCom’s published financial statements fooled most investors—investors bid the stock price up to $64.50, and banks and other lenders provided the company with more than $30 billion of loans. Arthur Andersen, the firm’s auditor, was faulted for not detecting the fraud. WorldCom’s CFO and CEO were convicted, and Arthur Andersen went bankrupt. But that didn’t help the investors who relied on the published financial statements. In response to these and other abuses, Congress passed the Sarbanes-Oxley Act of 2002. One of its provisions requires both the CEO and the CFO to sign a statement certifying that the “financial statements and disclosures fairly represent, in all material respects, the operations and financial condition” of the company. This will make it easier to haul off in handcuffs a CEO or CFO who has been misleading investors. Whether this will prevent future financial fraud remains to be seen. does. Because of this omission, net income is not as useful as EVA for setting corporate goals and measuring managerial performance. We will have more to say about both MVA and EVA later in the book, but we can close this section with two observations. First, there is a relationship between MVA and EVA, but it is not a direct one. If a company has a history of negative EVAs, then its MVA will probably be negative; conversely, its MVA probably will be positive if the company has a history of positive EVAs. However, the stock price, which is the key ingredient in the MVA calculation, depends more on expected future performance than on historical performance. Therefore, a company with a history of negative EVAs could have a positive MVA, provided investors expect a turnaround in the future. The second observation is that when EVAs or MVAs are used to evaluate managerial performance as part of an incentive compensation program, EVA is the measure that is typically used. The reasons are: (1) EVA shows the value added during a given year, whereas MVA reflects performance over the company’s entire life, perhaps even including times before the current managers were born; and (2) EVA can be applied to individual divisions or other units of a large corporation, whereas MVA must be applied to the entire corporation. Self-Test Define “Market Value Added (MVA)” and “Economic Value Added (EVA).” How does EVA differ from accounting profit? A firm has $100 million in total net operating capital. Its return on invested capital is 14%, and its weighted average cost of capital is 10%. What is its EVA? ($4 million) Chapter 2: Financial Statements, Cash Flow, and Taxes 71 2.9 THE FEDERAL INCOME TAX SYSTEM The value of any financial asset (including stocks, bonds, and mortgages), as well as most real assets such as plants or even entire firms, depends on the after-tax stream of cash flows produced by the asset. The following sections describe the key features of corporate and individual taxation. Corporate Income Taxes The corporate tax structure, shown in Table 2-6, is relatively simple. The marginal tax rate is the rate paid on the last dollar of income, while the average tax rate is the average rate paid on all income. To illustrate, if a firm had $65,000 of taxable income, its tax bill would be Taxes ¼ $7;500 þ 0:25ð$65;000 − $50;000Þ ¼ $7;500 þ $3;750 ¼ $11;250 Its marginal rate would be 25%, and its average tax rate would be $11,250/$65,000 = 17.3%. Note that corporate income above $18,333,333 has an average and marginal tax rate of 35%.15 TABLE 2-6 C or p o r at e T a x R a t es a s o f Ja n u a r y 2 0 0 8 I F A C O R PO R A T IO N’ S TAXABLE INCOME IS IT P AYS THIS AMOU NT ON T H E B A SE O F TH E B RACKET Up to $50,000 $50,000–$75,000 $75,000–$100,000 $100,000–$335,000 $335,000–$10,000,000 $10,000,000–$15,000,000 $15,000,000–$18,333,333 Over $18,333,333 $0 $7,500 $13,750 $22,250 $113,900 $3,400,000 $5,150,000 $6,416,667 P L U S T H IS PERCEN TA GE O N T H E EXC E S S OVER THE BASE 15% 25 34 39 34 35 38 35 AVER AGE TAX RATE AT TOP OF BRACKET 15.0% 18.3 22.3 34.0 34.0 34.3 35.0 35.0 Prior to 1987, many large, profitable corporations such as General Electric and Boeing paid no income taxes. The reasons for this were as follows: (1) expenses, especially depreciation, were defined differently for calculating taxable income than for reporting earnings to stockholders, so some companies reported positive profits to stockholders but losses—hence no taxes—to the Internal Revenue Service; and (2) some companies that did have tax liabilities used various tax credits to offset taxes that would otherwise have been payable. This situation was effectively eliminated in 1987. The principal method used to eliminate this situation is the Alternative Minimum Tax (AMT). Under the AMT, both corporate and individual taxpayers must figure their taxes in two ways, the “regular” way and the AMT way, and then pay the higher of the two. The AMT is calculated as follows: (1) Figure your regular taxes. (2) Take your taxable income under the regular method and then add back certain items, especially income on certain municipal bonds, depreciation in excess of straight-line depreciation, certain research and drilling costs, itemized or standard deductions (for individuals), and a number of other items. (3) The income determined in (2) is defined as AMT income, and it must then be multiplied by the AMT tax rate to determine the tax due under the AMT system. An individual or corporation must then pay the higher of the regular tax or the AMT tax. In 2008, there were two AMT tax rates for individuals (26% and 28%, depending on the level of AMT income and filing status). Most corporations have an AMT of 20%. However, there is no AMT for very small companies, defined as those that have had average sales of less than $7.5 million for the past 3 years. 15 72 Part 1: Fundamental Concepts of Corporate Finance Interest and Dividend Income Received by a Corporation. Interest income received by a corporation is taxed as ordinary income at regular corporate tax rates. However, 70% of the dividends received by one corporation from another is excluded from taxable income, while the remaining 30% is taxed at the ordinary tax rate.16 Thus, a corporation earning more than $18,333,333 and paying a 35% marginal tax rate would pay only (0.30)(0.35) = 0.105 = 10.5% of its dividend income as taxes, so its effective tax rate on dividends received would be 10.5%. If this firm had $10,000 in pre-tax dividend income, then its after-tax dividend income would be $8,950: After-tax income ¼ Before-tax income − Taxes ¼ Before-tax income − ðBefore-tax incomeÞ ðEffective tax rateÞ ¼ Before-tax income ð1 − Effective tax rateÞ ¼ $10;000½1 − ð0:30Þð0:35Þ ¼ $10;000ð1 − 0:105Þ ¼ $10;000ð0:895Þ ¼ $8;950: resource See Ch02 Tool Kit.xls for details. If the corporation pays its own after-tax income out to its stockholders as dividends, then the income is ultimately subjected to triple taxation: (1) the original corporation is first taxed, (2) the second corporation is then taxed on the dividends it received, and (3) the individuals who receive the final dividends are taxed again. This is the reason for the 70% exclusion on intercorporate dividends. If a corporation has surplus funds that can be invested in marketable securities, the tax treatment favors investment in stocks, which pay dividends, rather than in bonds, which pay interest. For example, suppose GE had $100,000 to invest, and suppose it could buy either bonds that paid interest of $8,000 per year or preferred stock that paid dividends of $7,000. GE is in the 35% tax bracket; therefore, its tax on the interest, if it bought bonds, would be 0.35($8,000) = $2,800, and its after-tax income would be $5,200. If it bought preferred (or common) stock, its tax would be 0.35[(0.30)($7,000)] = $735, and its after-tax income would be $6,265. Other factors might lead GE to invest in bonds, but the tax treatment certainly favors stock investments when the investor is a corporation.17 Interest and Dividends Paid by a Corporation. A firm’s operations can be financed with either debt or equity capital. If the firm uses debt then it must pay interest on this debt, but if the firm uses equity then it is expected to pay dividends to the equity investors (stockholders). The interest paid by a corporation is deducted from its operating income to obtain its taxable income, but dividends paid are not deductible. Therefore, a firm needs $1 of pre-tax income to pay $1 of interest, but if it is in The size of the dividend exclusion actually depends on the degree of ownership. Corporations that own less than 20% of the stock of the dividend-paying company can exclude 70% of the dividends received; firms that own more than 20% but less than 80% can exclude 80% of the dividends; and firms that own more than 80% can exclude the entire dividend payment. We will, in general, assume a 70% dividend exclusion. 16 This illustration demonstrates why corporations favor investing in lower-yielding preferred stocks over higher-yielding bonds. When tax consequences are considered, the yield on the preferred stock, [1 − 0.35(0.30)](7.0%) = 6.265%, is higher than the yield on the bond, (1 − 0.35)(8.0%) = 5.2%. Also, note that corporations are restricted in their use of borrowed funds to purchase other firms’ preferred or common stocks. Without such restrictions, firms could engage in tax arbitrage, whereby the interest on borrowed funds reduces taxable income on a dollar-for-dollar basis while taxable income is increased by only $0.30 per dollar of dividend income. Thus, current tax laws reduce the 70% dividend exclusion in proportion to the amount of borrowed funds used to purchase the stock. 17 Chapter 2: Financial Statements, Cash Flow, and Taxes 73 the 40% federal-plus-state tax bracket, it must earn $1.67 of pre-tax income to pay $1 of dividends: $1 $1 Pre-tax income needed ¼ ¼ $1:67 ¼ to pay $1 of dividends 1 − Tax rate 0:60 Working backward, if a company has $1.67 in pre-tax income, it must pay $0.67 in taxes: (0.4)($1.67) = $0.67. This leaves the firm with after-tax income of $1.00. Of course, it is generally not possible to finance exclusively with debt capital, and the risk of doing so would offset the benefits of the higher expected income. Still, the fact that interest is a deductible expense has a profound effect on the way businesses are financed: Our corporate tax system favors debt financing over equity financing. This point is discussed in more detail in Chapters 9 and 15. Corporate Capital Gains. Before 1987, corporate long-term capital gains were taxed at lower rates than corporate ordinary income, so the situation was similar for corporations and individuals. Under current law, however, corporations’ capital gains are taxed at the same rates as their operating income. Corporate Loss Carryback and Carryforward. Ordinary corporate operating losses can be carried back (carryback) to each of the preceding 2 years and forward (carryforward) for the next 20 years and thus be used to offset taxable income in those years. For example, an operating loss in 2010 could be carried back and used to reduce taxable income in 2008 and 2009 as well as forward, if necessary, to reduce taxes in 2011, 2012, and so on, to the year 2030. After carrying back 2 years, any remaining loss is typically carried forward first to the next year, then to the one after that, and so on, until losses have been used up or the 20-year carryforward limit has been reached. To illustrate, suppose Apex Corporation had $2 million of pre-tax profits (taxable income) in 2008 and 2009, and then, in 2010, Apex lost $12 million. Also, assume that Apex’s federal-plus-state tax rate is 40%. As shown in Table 2-7, the company would use the carryback feature to recompute its taxes for 2008, using $2 million of the 2010 operating losses to reduce the 2008 pre-tax profit to zero. This would permit it to recover the taxes paid in 2008. Therefore, in 2010 Apex would receive a refund of its 2008 taxes because of the loss experienced in 2010. Because $10 million of the TABLE 2-7 A pe x C or p o r at i o n : C a l c u l a t i o n o f $12 Mi l l i on L o s s C ar r y b a c k a n d A m ou n t Av ai l ab l e f o r C a rr y fo r w ar d Original taxable income Carryback credit Adjusted profit Taxes previously paid (40%) Difference = Tax refund due Total tax refund received Amount of loss carryforward available Current loss Carryback losses used Carryforward losses still available PAST YEAR 2008 PAST YEAR 2009 CURRENT YEAR 2010 $2,000,000 2,000,000 $ 0 800,000 $ 800,000 $2,000,000 2,000,000 $ 0 800,000 $ 800,000 –$12,000,000 $ 1,600,000 –$12,000,000 4,000,000 −$ 8,000,000 74 Part 1: Fundamental Concepts of Corporate Finance resource See Ch02 Tool Kit.xls for details. unrecovered losses would still be available, Apex would repeat this procedure for 2009. Thus, in 2010 the company would pay zero taxes for 2010 and also would receive a refund for taxes paid in 2008 and 2009. Apex would still have $8 million of unrecovered losses to carry forward, subject to the 20-year limit. This $8 million could be used to offset future taxable income. The purpose of this loss treatment is to avoid penalizing corporations whose incomes fluctuate substantially from year to year. Improper Accumulation to Avoid Payment of Dividends. Corporations could refrain from paying dividends and thus permit their stockholders to avoid personal income taxes on dividends. To prevent this, the Tax Code contains an improper accumulation provision that states that earnings accumulated by a corporation are subject to penalty rates if the purpose of the accumulation is to enable stockholders to avoid personal income taxes. A cumulative total of $250,000 (the balance sheet item “retained earnings”) is by law exempted from the improper accumulation tax for most corporations. This is a benefit primarily to small corporations. The improper accumulation penalty applies only if the retained earnings in excess of $250,000 are shown by the IRS to be unnecessary to meet the reasonable needs of the business. A great many companies do indeed have legitimate reasons for retaining more than $250,000 of earnings. For example, earnings may be retained and used to pay off debt, to finance growth, or to provide the corporation with a cushion against possible cash drains caused by losses. How much a firm should be allowed to accumulate for uncertain contingencies is a matter of judgment. We shall consider this matter again in Chapter 14, which deals with corporate dividend policy. Consolidated Corporate Tax Returns. If a corporation owns 80% or more of another corporation’s stock, then it can aggregate income and file one consolidated tax return; thus, the losses of one company can be used to offset the profits of another. (Similarly, one division’s losses can be used to offset another division’s profits.) No business ever wants to incur losses (you can go broke losing $1 to save 35¢ in taxes), but tax offsets do help make it more feasible for large, multidivisional corporations to undertake risky new ventures or ventures that will suffer losses during a developmental period. Taxes on Overseas Income. Many U.S. corporations have overseas subsidiaries, and those subsidiaries must pay taxes in the countries where they operate. Often, foreign tax rates are lower than U.S. rates. As long as foreign earnings are reinvested overseas, no U.S. tax is due on those earnings. However, when foreign earnings are repatriated to the U.S. parent, they are taxed at the applicable U.S. rate, less a credit for taxes paid to the foreign country. As a result, U.S. corporations such as IBM, Coca-Cola, and Microsoft have been able to defer billions of dollars of taxes. This procedure has stimulated overseas investments by U.S. multinational firms—they can continue the deferral indefinitely, but only if they reinvest the earnings in their overseas operations.18 Taxation of Small Businesses: S Corporations The Tax Code provides that small businesses that meet certain restrictions may be set up as corporations and thus receive the benefits of the corporate form of organization— especially limited liability—yet still be taxed as proprietorships or partnerships rather This is a contentious political issue. U.S. corporations argue that our tax system is similar to systems in the rest of the world, and if they were taxed immediately on all overseas earnings then they would be at a competitive disadvantage vis-à-vis their global competitors. Others argue that taxation encourages overseas investments at the expense of domestic investments, contributing to the jobs outsourcing problem and also to the federal budget deficit. 18 Chapter 2: Financial Statements, Cash Flow, and Taxes 75 than as corporations. These corporations are called S corporations. (“Regular” corporations are called C corporations.) If a corporation elects S corporation status for tax purposes, then all of the business’s income is reported as personal income by its stockholders, on a pro rata basis, and thus is taxed at the rates that apply to individuals. This is an important benefit to the owners of small corporations in which all or most of the income earned each year will be distributed as dividends, because then the income is taxed only once, at the individual level. Personal Taxes resource See Web Extension 2A on the textbook’s Web site for details concerning personal taxation. Web Extension 2A provides a more detailed treatment of individual taxation, but the key elements are presented here. Ordinary income consists primarily of wages or profits from a proprietorship or partnership, plus investment income. For the 2009 tax year, individuals with less than $8,350 of taxable income are subject to a federal income tax rate of 10%. For those with higher income, tax rates increase and go up to 35%, depending on the level of income. This is called a progressive tax, because the higher one’s income, the larger the percentage paid in taxes. As noted before, individuals are taxed on investment income as well as earned income, but with a few exceptions and modifications. For example, interest received from most state and local government bonds, called municipals or “munis,” is not subject to federal taxation. However, interest earned on most other bonds or lending is taxed as ordinary income. This means that a lower-yielding muni can provide the same after-tax return as a higher-yielding corporate bond. For a taxpayer in the 35% marginal tax bracket, a muni yielding 5.5% provides the same after-tax return as a corporate bond with a pre-tax yield of 8.46%: 8.46%(1 − 0.35) = 5.5%. Assets such as stocks, bonds, and real estate are defined as capital assets. If you own a capital asset and its price goes up, then your wealth increases, but you are not liable for any taxes on your increased wealth until you sell the asset. If you sell the asset for more than you originally paid, the profit is called a capital gain; if you sell it for less, then you suffer a capital loss. The length of time you owned the asset determines the tax treatment. If held for less than one year, then your gain or loss is simply added to your other ordinary income. If held for more than a year, then gains are called long-term capital gains and are taxed at a lower rate. See Web Extension 2A for details, but the long-term capital gains rate is 15% for most situations. Under the 2003 tax law changes, dividends are now taxed as though they were capital gains. As stated earlier, corporations may deduct interest payments but not dividends when computing their corporate tax liability, which means that dividends are taxed twice, once at the corporate level and again at the personal level. This differential treatment motivates corporations to use debt relatively heavily and to pay small (or even no) dividends. The 2003 tax law did not eliminate the differential treatment of dividends and interest payments from the corporate perspective, but it did make the tax treatment of dividends more similar to that of capital gains from investors’ perspectives. To see this, consider a company that doesn’t pay a dividend but instead reinvests the cash it could have paid. The company’s stock price should increase, leading to a capital gain, which would be taxed at the same rate as the dividend. Of course, the stock price appreciation isn’t actually taxed until the stock is sold, whereas the dividend is taxed in the year it is paid, so dividends will still be more costly than capital gains for many investors. Finally, note that the income of S corporations and noncorporate businesses is reported as income by the firms’ owners. Since there are far more S corporations, 76 Part 1: Fundamental Concepts of Corporate Finance partnerships, and proprietorships than C corporations (which are subject to the corporate tax), individual tax considerations play an important role in business finance. Self-Test Explain what is meant by this statement: “Our tax rates are progressive.” If a corporation has $85,000 in taxable income, what is its tax liability? ($17,150) Explain the difference between marginal tax rates and average tax rates. What are municipal bonds, and how are these bonds taxed? What are capital gains and losses, and how are they taxed? How does the federal income tax system treat dividends received by a corporation versus those received by an individual? What is the difference in the tax treatment of interest and dividends paid by a corporation? Does this factor favor debt or equity financing? Briefly explain how tax loss carryback and carryforward procedures work. Summary The primary purposes of this chapter were (1) to describe the basic financial statements, (2) to present some background information on cash flows, and (3) to provide an overview of the federal income tax system. The key concepts covered are listed below. • • • • • • • • • • The four basic statements contained in the annual report are the balance sheet, the income statement, the statement of stockholders’ equity, and the statement of cash flows. The balance sheet shows assets on the left-hand side and liabilities and equity, or claims against assets, on the right-hand side. (Sometimes assets are shown at the top and claims at the bottom of the balance sheet.) The balance sheet may be thought of as a snapshot of the firm’s financial position at a particular point in time. The income statement reports the results of operations over a period of time, and it shows earnings per share as its “bottom line.” The statement of stockholders’ equity shows the change in retained earnings between balance sheet dates. Retained earnings represent a claim against assets, not assets per se. The statement of cash flows reports the effect of operating, investing, and financing activities on cash flows over an accounting period. Net cash flow differs from accounting profit because some of the revenues and expenses reflected in accounting profits may not have been received or paid out in cash during the year. Depreciation is typically the largest noncash item, so net cash flow is often expressed as net income plus depreciation. Operating current assets are the current assets that are used to support operations, such as cash, inventory, and accounts receivable. They do not include short-term investments. Operating current liabilities are the current liabilities that occur as a natural consequence of operations, such as accounts payable and accruals. They do not include notes payable or any other short-term debts that charge interest. Net operating working capital is the difference between operating current assets and operating current liabilities. Thus, it is the working capital acquired with investor-supplied funds. Operating long-term assets are the long-term assets used to support operations, such as net plant and equipment. They do not include any long-term investments that pay interest or dividends. Chapter 2: Financial Statements, Cash Flow, and Taxes • • • • • • • • • • • • • Total net operating capital (which means the same as operating capital and net operating assets) is the sum of net operating working capital and operating long-term assets. It is the total amount of capital needed to run the business. NOPAT is net operating profit after taxes. It is the after-tax profit a company would have if it had no debt and no investments in nonoperating assets. Because it excludes the effects of financial decisions, it is a better measure of operating performance than is net income. Free cash flow (FCF) is the amount of cash flow remaining after a company makes the asset investments necessary to support operations. In other words, FCF is the amount of cash flow available for distribution to investors, so the value of a company is directly related to its ability to generate free cash flow. FCF is defined as NOPAT minus the net investment in operating capital. Market Value Added (MVA) represents the difference between the total market value of a firm and the total amount of investor-supplied capital. If the market values of debt and preferred stock equal their values as reported on the financial statements, then MVA is the difference between the market value of a firm’s stock and the amount of equity its shareholders have supplied. Economic Value Added (EVA) is the difference between after-tax operating profit and the total dollar cost of capital, including the cost of equity capital. EVA is an estimate of the value created by management during the year, and it differs substantially from accounting profit because no charge for the use of equity capital is reflected in accounting profit. Interest income received by a corporation is taxed as ordinary income; however, 70% of the dividends received by one corporation from another are excluded from taxable income. Because interest paid by a corporation is a deductible expense whereas dividends are not, our tax system favors debt over equity financing. Ordinary corporate operating losses can be carried back to each of the preceding 2 years and carried forward for the next 20 years in order to offset taxable income in those years. S corporations are small businesses that have the limited-liability benefits of the corporate form of organization yet are taxed as partnerships or proprietorships. In the United States, tax rates are progressive—the higher one’s income, the larger the percentage paid in taxes. Assets such as stocks, bonds, and real estate are defined as capital assets. If a capital asset is sold for more than its cost, the profit is called a capital gain; if the asset is sold for a loss, it is called a capital loss. Assets held for more than a year provide long-term gains or losses. Dividends are taxed as though they were capital gains. Personal taxes are discussed in more detail in Web Extension 2A. Questions (2–1) 77 Define each of the following terms: a. Annual report; balance sheet; income statement b. Common stockholders’ equity, or net worth; retained earnings c. Statement of stockholders’ equity; statement of cash flows d. Depreciation; amortization; EBITDA e. Operating current assets; operating current liabilities; net operating working capital; total net operating capital 78 Part 1: Fundamental Concepts of Corporate Finance f. g. h. i. j. Accounting profit; net cash flow; NOPAT; free cash flow Market Value Added; Economic Value Added Progressive tax; taxable income; marginal and average tax rates Capital gain or loss; tax loss carryback and carryforward Improper accumulation; S corporation (2–2) What four statements are contained in most annual reports? (2–3) If a “typical” firm reports $20 million of retained earnings on its balance sheet, can the firm definitely pay a $20 million cash dividend? (2–4) Explain the following statement: “Whereas the balance sheet can be thought of as a snapshot of the firm’s financial position at a point in time, the income statement reports on operations over a period of time.” (2–5) What is operating capital, and why is it important? (2–6) Explain the difference between NOPAT and net income. Which is a better measure of the performance of a company’s operations? (2–7) What is free cash flow? Why is it the most important measure of cash flow? (2–8) If you were starting a business, what tax considerations might cause you to prefer to set it up as a proprietorship or a partnership rather than as a corporation? Self-Test Problem (ST–1) Net Income, Cash Flow, and EVA Solution Appears in Appendix A Last year Cole Furnaces had $5 million in operating income (EBIT). The company had a net depreciation expense of $1 million and an interest expense of $1 million; its corporate tax rate was 40%. The company has $14 million in operating current assets and $4 million in operating current liabilities; it has $15 million in net plant and equipment. It estimates that it has an after-tax cost of capital of 10%. Assume that Cole’s only noncash item was depreciation. a. b. c. d. What was the company’s net income for the year? What was the company’s net cash flow? What was the company’s net operating profit after taxes (NOPAT)? Calculate net operating working capital and total net operating capital for the current year. e. If total net operating capital in the previous year was $24 million, what was the company’s free cash flow (FCF) for the year? f. What was the company’s Economic Value Added (EVA)? Problems Answers Appear in Appendix B Note: By the time this book is published, Congress may have changed rates and/or other provisions of current tax law—as noted in the chapter, such changes occur fairly often. Work all problems on the assumption that the information in the chapter is applicable. EASY PROBLEMS 1–6 (2–1) Personal After-Tax Yield An investor recently purchased a corporate bond that yields 9%. The investor is in the 36% combined federal and state tax bracket. What is the bond’s after-tax yield? Chapter 2: Financial Statements, Cash Flow, and Taxes (2–2) Personal After-Tax Yield (2–3) Income Statement (2–4) Income Statement (2–5) Net Cash Flow (2–6) Statement of Retained Earnings 79 Corporate bonds issued by Johnson Corporation currently yield 8%. Municipal bonds of equal risk currently yield 6%. At what tax rate would an investor be indifferent between these two bonds? Little Books Inc. recently reported $3 million of net income. Its EBIT was $6 million, and its tax rate was 40%. What was its interest expense? (Hint: Write out the headings for an income statement and then fill in the known values. Then divide $3 million net income by 1 − T = 0.6 to find the pre-tax income. The difference between EBIT and taxable income must be the interest expense. Use this same procedure to work some of the other problems.) Pearson Brothers recently reported an EBITDA of $7.5 million and net income of $1.8 million. It had $2.0 million of interest expense, and its corporate tax rate was 40%. What was its charge for depreciation and amortization? Kendall Corners Inc. recently reported net income of $3.1 million and depreciation of $500,000. What was its net cash flow? Assume it had no amortization expense. In its most recent financial statements, Newhouse Inc. reported $50 million of net income and $810 million of retained earnings. The previous retained earnings were $780 million. How much in dividends was paid to shareholders during the year? INTERMEDIATE PROBLEMS 7–11 (2–7) Corporate Tax Liability (2–8) Corporate Tax Liability (2–9) Corporate After-Tax Yield (2–10) Cash Flows (2–11) Income and Cash Flow Analysis The Talley Corporation had a taxable income of $365,000 from operations after all operating costs but before (1) interest charges of $50,000, (2) dividends received of $15,000, (3) dividends paid of $25,000, and (4) income taxes. What are the firm’s income tax liability and its after-tax income? What are the company’s marginal and average tax rates on taxable income? The Wendt Corporation had $10.5 million of taxable income. a. What is the company’s federal income tax bill for the year? b. Assume the firm receives an additional $1 million of interest income from some bonds it owns. What is the tax on this interest income? c. Now assume that Wendt does not receive the interest income but does receive an additional $1 million as dividends on some stock it owns. What is the tax on this dividend income? The Shrieves Corporation has $10,000 that it plans to invest in marketable securities. It is choosing among AT&T bonds, which yield 7.5%, state of Florida muni bonds, which yield 5% (but are not taxable), and AT&T preferred stock, with a dividend yield of 6%. Shrieves’s corporate tax rate is 35%, and 70% of the dividends received are tax exempt. Find the after-tax rates of return on all three securities. The Moore Corporation has operating income (EBIT) of $750,000. The company’s depreciation expense is $200,000. Moore is 100% equity financed, and it faces a 40% tax rate. What is the company’s net income? What is its net cash flow? The Berndt Corporation expects to have sales of $12 million. Costs other than depreciation are expected to be 75% of sales, and depreciation is expected to be $1.5 million. All sales revenues will be collected in cash, and costs other than depreciation must be paid for during the year. Berndt’s federal-plus-state tax rate is 40%. Berndt has no debt. 80 Part 1: Fundamental Concepts of Corporate Finance a. Set up an income statement. What is Berndt’s expected net cash flow? b. Suppose Congress changed the tax laws so that Berndt’s depreciation expenses doubled. No changes in operations occurred. What would happen to reported profit and to net cash flow? c. Now suppose that Congress, instead of doubling Berndt’s depreciation, reduced it by 50%. How would profit and net cash flow be affected? d. If this were your company, would you prefer Congress to cause your depreciation expense to be doubled or halved? Why? CHALLENGING PROBLEMS 12–13 (2–12) Free Cash Flows Using Rhodes Corporation’s financial statements (shown below), answer the following questions. a. b. c. d. e. f. What is the net operating profit after taxes (NOPAT) for 2010? What are the amounts of net operating working capital for both years? What are the amounts of total net operating capital for both years? What is the free cash flow for 2010? What is the ROIC for 2010? How much of the FCF did Rhodes use for each of the following purposes: after-tax interest, net debt repayments, dividends, net stock repurchases, and net purchases of short-term investments? (Hint: Remember that a net use can be negative.) Rhodes Corporation: Income Statements for Year Ending December 31 (Millions of Dollars) 2 01 0 20 09 Sales Operating costs excluding depreciation Depreciation Earnings before interest and taxes Less interest Earnings before taxes Taxes (40%) Net income available to common stockholders $11,000 9,360 380 $ 1,260 120 $ 1,140 456 $ 684 $10,000 8,500 360 $ 1,140 100 $ 1,040 416 $ 624 Common dividends $ $ 220 200 Rhodes Corporation: Balance Sheets as of December 31 (Millions of Dollars) Assets Cash Short-term investments Accounts receivable Inventories Total current assets Net plant and equipment Total assets 2 01 0 2 0 09 $ 550 110 2,750 1,650 $5,060 3,850 $8,910 $ 500 100 2,500 1,500 $4,600 3,500 $8,100 Chapter 2: Financial Statements, Cash Flow, and Taxes 81 2010 2009 $1,100 550 384 $2,034 1,100 $3,134 4,312 1,464 $5,776 $8,910 $1,000 500 200 $1,700 1,000 $2,700 4,400 1,000 $5,400 $8,100 Liabilities and Equity Accounts payable Accruals Notes payable Total current liabilities Long-term debt Total liabilities Common stock Retained earnings Total common equity Total liabilities and equity (2–13) Loss Carryback and Carryforward The Bookbinder Company has made $150,000 before taxes during each of the last 15 years, and it expects to make $150,000 a year before taxes in the future. However, in 2010 the firm incurred a loss of $650,000. The firm will claim a tax credit at the time it files its 2010 income tax return, and it will receive a check from the U.S. Treasury. Show how it calculates this credit, and then indicate the firm’s tax liability for each of the next 5 years. Assume a 40% tax rate on all income to ease the calculations. SPREADSHEET PROBLEMS (2-14) Build a Model: Free Cash Flows, EVA, and MVA resource Begin with the partial model in the file Ch02 P14 Build a Model.xls on the textbook’s Web site. a. Cumberland Industries’s 2010 sales were $455,000,000; operating costs (excluding depreciation) were equal to 85% of sales; net fixed assets were $67,000,000; depreciation amounted to 10% of net fixed assets; interest expenses were $8,550,000; the state-plus-federal corporate tax rate was 40%; and Cumberland paid 25% of its net income out in dividends. Given this information, construct Cumberland’s 2010 income statement. Also calculate total dividends and the addition to retained earnings. (Hint: Start with the partial model in the file and report all dollar figures in thousands to reduce clutter.) b. Cumberland Industries’s partial balance sheets are shown below. Cumberland issued $10,000,000 of new common stock in 2010. Using this information and the results from part a, fill in the missing values for common stock, retained earnings, total common equity, and total liabilities and equity. Cumberland Industries: Balance Sheets as of December 31 (Thousands of Dollars) Assets Cash Short-term investments Accounts receivable Inventories Total current assets Net fixed assets Total assets 20 10 2 00 9 $ 91,450 11,400 108,470 38,450 $249,770 67,000 $316,770 $ 74,625 15,100 85,527 34,982 $210,234 42,436 $252,670 82 Part 1: Fundamental Concepts of Corporate Finance Liabilities and Equity Accounts payable Accruals Notes payable Total current liabilities Long-term debt Total liabilities Common stock Retained earnings Total common equity Total liabilities and equity 2010 2009 $ 30,761 30,405 12,717 $ 73,883 80,263 $154,146 ? ? ? ? $ 23,109 22,656 14,217 $ 59,982 63,914 $123,896 $ 90,000 38,774 $128,774 $252,670 c. Construct the statement of cash flows for 2010. (2–15) Build a Model: Free Cash Flows, EVA, and MVA resource Begin with the partial model in the file Ch02 P15 Build a Model.xls on the textbook’s Web site. a. Using the financial statements shown below for Lan & Chen Technologies, calculate net operating working capital, total net operating capital, net operating profit after taxes, free cash flow, and return on invested capital for 2010. (Hint: Start with the partial model in the file and report all dollar figures in thousands to reduce clutter.) b. Assume there were 15 million shares outstanding at the end of 2010, the year-end closing stock price was $65 per share, and the after-tax cost of capital was 8%. Calculate EVA and MVA for 2010. Lan & Chen Technologies: Income Statements for Year Ending December 31 (Thousands of Dollars) 2 0 10 2 00 9 Sales Expenses excluding depreciation and amortization EBITDA Depreciation and amortization EBIT Interest expense EBT Taxes (40%) Net income $945,000 812,700 $132,300 33,100 $ 99,200 10,470 $ 88,730 35,492 $ 53,238 $900,000 774,000 $126,000 31,500 $ 94,500 8,600 $ 85,900 34,360 $ 51,540 Common dividends Addition to retained earnings $ 43,300 $ 9,938 $ 41,230 $ 10,310 Chapter 2: Financial Statements, Cash Flow, and Taxes 83 Lan & Chen Technologies: December 31 Balance Sheets (Thousands of Dollars) 2 0 10 20 09 Assets Cash and cash equivalents Short-term investments Accounts receivable Inventories Total current assets Net fixed assets Total assets $ 47,250 3,800 283,500 141,750 $476,300 330,750 $807,050 $ 45,000 3,600 270,000 135,000 $453,600 315,000 $768,600 Liabilities and equity Accounts payable Accruals Notes payable Total current liabilities Long-term debt Total liabilities Common stock Retained earnings Total common equity Total liabilities and equity $ 94,500 47,250 26,262 $168,012 94,500 $262,512 444,600 99,938 $544,538 $807,050 $ 90,000 45,000 9,000 $144,000 90,000 $234,000 444,600 90,000 $534,600 $768,600 THOMSON ONE Business School Edition Problem Use the Thomson ONE—Business School Edition online database to work this chapter’s questions. EXPLORING STARBUCKS’S FINANCIAL STATEMENTS THOMSON ONE—BUSINESS SCHOOL EDITION WITH Over the past decade, Starbucks coffee shops have become an increasingly familiar part of the urban landscape. The Thomson ONE—Business School Edition online database can provide a wealth of financial information for companies such as Starbucks. Begin by entering the company’s ticker symbol, SBUX, and then selecting GO. The opening screen includes a summary of what Starbucks does, a chart of its recent stock price, EPS estimates, some recent news stories, and a list of key financial data and ratios. For recent stock price performance, look at the top of the Stock Price Chart and click on the section labeled Interactive Chart. From this point, we are able to obtain a chart of the company’s stock price performance relative to the overall market, as measured by the S&P 500. To obtain a 10-year chart, go to Time Frame, click on the down arrow, and select 10 years. Then click on Draw, and a 10-year price chart should appear. You can also find Starbucks’s recent financial statements. Near the top of your screen, click on the Financials tab to find the company’s balance sheet, income statement, and statement of cash flows for the past 5 years. Clicking on the Microsoft Excel icon downloads these statements directly to a spreadsheet. 84 Part 1: Fundamental Concepts of Corporate Finance Thomson ONE—BSE Discussion Questions 1. Looking at the most recent year available, what is the amount of total assets on Starbucks’s balance sheet? What percentage is fixed assets, such as plant and equipment, and what percentage is current assets? How much has the company grown over the years shown? 2. Does Starbucks have a lot of long-term debt? What are Starbucks’s primary sources of financing? 3. Looking at the statement of cash flows, what factors can explain the change in the company’s cash position over the last couple of years? 4. Looking at the income statement, what are the company’s most recent sales and net income? Over the past several years, what has been the sales growth rate? What has been the growth rate in net income? Mini Case Donna Jamison, a graduate of the University of Tennessee with four years of banking experience, was recently brought in as assistant to the chairman of the board of Computron Industries, a manufacturer of electronic calculators. The company doubled its plant capacity, opened new sales offices outside its home territory, and launched an expensive advertising campaign. Computron’s results were not satisfactory, to put it mildly. Its board of directors, which consisted of its president and vice-president plus its major stockholders (who were all local businesspeople), was most upset when directors learned how the expansion was going. Suppliers were being paid late and were unhappy, and the bank was complaining about the deteriorating situation and threatening to cut off credit. As a result, Al Watkins, Computron’s president, was informed that changes would have to be made—and quickly—or he would be fired. At the board’s insistence, Donna Jamison was given the job of assistant to Fred Campo, a retired banker who was Computron’s chairman and largest stockholder. Campo agreed to give up a few of his golfing days and to help nurse the company back to health, with Jamison’s assistance. Jamison began by gathering financial statements and other data. 20 09 20 10 Balance Sheets Assets Cash Short-term investments Accounts receivable Inventories Total current assets Gross fixed assets Less: Accumulated depreciation Net fixed assets Total assets $ 9,000 48,600 351,200 715,200 $1,124,000 491,000 146,200 $ 344,800 $1,468,800 7,282 20,000 632,160 1,287,360 $1,946,802 1,202,950 263,160 $ 939,790 $2,886,592 Liabilities and Equity Accounts payable Notes payable Accruals Total current liabilities $ 145,600 200,000 136,000 $ 481,600 $ 324,000 720,000 284,960 $1,328,960 $ Chapter 2: Financial Statements, Cash Flow, and Taxes Long-term debt Common stock (100,000 shares) Retained earnings Total equity Total liabilities and equity Income Statements Sales Cost of goods sold Other expenses Depreciation Total operating costs EBIT Interest expense EBT Taxes (40%) Net income Other Data Stock price Shares outstanding EPS DPS Tax rate 2009 323,432 460,000 203,768 $ 663,768 $1,468,800 2010 1,000,000 460,000 97,632 $ 557,632 $2,886,592 2 00 9 20 10 $3,432,000 2,864,000 340,000 18,900 $3,222,900 $ 209,100 62,500 $ 146,600 58,640 $ 87,960 $5,834,400 4,980,000 720,000 116,960 $5,816,960 $ 17,440 176,000 ($ 158,560) (63,424) ($ 95,136) $ 8.50 100,000 $ 0.880 $ 0.220 40% $ 6.00 100,000 ($ 0.951) $ 0.110 40% 2 01 0 Statement of Cash Flows Operating Activities Net income Adjustments: Noncash adjustments: Depreciation Changes in working capital: Change in accounts receivable Change in inventories Change in accounts payable Change in accruals Net cash provided (used) by operating activities Investing Activities Cash used to acquire fixed assets Change in short-term investments Net cash provided (used) by investing activities 85 ($ 95,136) 116,960 (280,960) (572,160) 178,400 148,960 ($ 503,936) ($ 711,950) 28,600 ($ 683,350) 86 Part 1: Fundamental Concepts of Corporate Finance 2010 Financing Activities Change in notes payable Change in long-term debt Change in common stock Payment of cash dividends Net cash provided (used) by financing activities Summary Net change in cash Cash at beginning of year Cash at end of year $ 520,000 676,568 — (11,000) $ 1,185,568 ($ $ 1,718) 9,000 7,282 Assume that you are Jamison’s assistant and that you must help her answer the following questions for Campo. a. What effect did the expansion have on sales and net income? What effect did the expansion have on the asset side of the balance sheet? What effect did it have on liabilities and equity? b. What do you conclude from the statement of cash flows? c. What is free cash flow? Why is it important? What are the five uses of FCF? d. What is Computron’s net operating profit after taxes (NOPAT)? What are operating current assets? What are operating current liabilities? How much net operating working capital and total net operating capital does Computron have? e. What is Computron’s free cash flow (FCF)? What are Computron’s “net uses” of its FCF? f. Calculate Computron’s return on invested capital. Computron has a 10% cost of capital (WACC). Do you think Computron’s growth added value? g. Jamison also has asked you to estimate Computron’s EVA. She estimates that the aftertax cost of capital was 10% in both years. h. What happened to Computron’s Market Value Added (MVA)? i. Assume that a corporation has $100,000 of taxable income from operations plus $5,000 of interest income and $10,000 of dividend income. What is the company’s federal tax liability? j. Assume that you are in the 25% marginal tax bracket and that you have $5,000 to invest. You have narrowed your investment choices down to California bonds with a yield of 7% or equally risky ExxonMobil bonds with a yield of 10%. Which one should you choose and why? At what marginal tax rate would you be indifferent to the choice between California and ExxonMobil bonds? CHAPTER 3 Analysis of Financial Statements T o guide or not to guide, that is the question. Or at least it’s the question many companies are wrestling with regarding earnings forecasts. Should a company provide earnings estimates to investors? In 2006, Best Buy answered this question by announcing that it would no longer provide quarterly earnings forecasts. It’s no coincidence that Best Buy’s decision came shortly after its actual earnings came in just 2 cents below the forecast, yet its stock price fell by 12%. Coca-Cola, Motorola, and Citigroup are among the growing number of companies that no longer provide quarterly earnings forecasts. Virtually no one disputes that investors need as much information as possible to accurately evaluate a company, and academic studies show that companies with greater transparency have higher valuations. However, greater disclosure often brings the possibility of lawsuits if investors have reason to believe that the disclosure is fraudulent. Although the Private Securities Litigation Reform Act of 1995 helped prevent “frivolous” lawsuits, many companies still chose not to provide information directly to all investors. Instead, before 2000, many companies provided earnings information to brokerage firms’ analysts, and the analysts then forecast their own earnings expectations. In 2000 the SEC adopted Reg FD (Regulation Fair Disclosure), which prevented companies from disclosing information only to select groups, such as analysts. Reg FD led many companies to begin providing quarterly earnings forecasts directly to the public, and a survey by the National Investors Relations Institute showed that 95% of respondents in 2006 provided either annual or quarterly earnings forecasts, up from 45% in 1999. Two trends are now in evidence. First, the number of companies reporting quarterly earnings forecasts is falling, but the number reporting annual forecasts is increasing. Second, many companies are providing other types of forward-looking information, including key operating ratios plus qualitative information about the company and its industry. Ratio analysis can help investors use such information, so keep that in mind as you read this chapter. Sources: Adapted from Joseph McCafferty, “Guidance Lite,” CFO, June 2006, 16–17, and William F. Coffin and Crocker Coulson, “Is Earnings Guidance Disappearing in 2006?” 2006, White Paper, available at http://www .ccgir.com/ccgir/white_papers/pdf/Earnings%20Guidance%202006.pdf. 87 88 Part 1: Fundamental Concepts of Corporate Finance Intrinsic Value and Analysis of Financial Statements The intrinsic value of a firm is determined by the present value of the expected future free cash flows (FCF) when discounted at the weighted average cost of capital (WACC). This chapter explains how to use financial statements to evaluate a company’s profitability, required capital investments, business risk, and mix of debt and equity. Net operating profit after taxes Free cash flow (FCF) FCF1 Value = (1 + WACC)1 Required investments in operating capital − + FCF2 (1 + WACC)2 = + …+ FCF∞ (1 + WACC)∞ Weighted average cost of capital (WACC) Market interest rates Cost of debt Cost of equity Market risk aversion resource The textbook’s Web site contains an Excel file that will guide you through the chapter’s calculations. The file for this chapter is Ch03 Tool Kit.xls, and we encourage you to open the file and follow along as you read the chapter. WWW See http://www.zacks.com for a source of standardized financial statements. Firm’s debt/equity mix Firm’s business risk Financial statement analysis involves (1) comparing a firm’s performance with that of other firms in the same industry and (2) evaluating trends in the firm’s financial position over time. Managers use financial analysis to identify situations needing attention; potential lenders use financial analysis to determine whether a company is creditworthy; and stockholders use financial analysis to help predict future earnings, dividends, and free cash flow. As we explain in this chapter, there are similarities and differences among these uses.1 3.1 FINANCIAL ANALYSIS When we perform a financial analysis, we conduct the following steps. Gather Data The first step in financial analysis is to gather data. As we discussed in Chapter 2, financial statements can be downloaded from many different Web sites. One of our favorites is Zacks Investment Research, which provides financial statements in Widespread accounting fraud has cast doubt on whether all firms’ published financial statements can be trusted. New regulations by the SEC and the exchanges, as well as new laws enacted by Congress, have improved oversight of the accounting industry and increased the criminal penalties on management for fraudulent reporting. 1 Chapter 3: Analysis of Financial Statements 89 a standardized format. If you cut and paste financial statements from Zacks into a spreadsheet and then perform a financial analysis, you can quickly repeat the analysis on a different company by simply pasting that company’s financial statements into the same cells as the original company’s statements. In other words, there is no need to reinvent the wheel each time you analyze a company. Examine the Statement of Cash Flows Some financial analysis can be done with virtually no calculations. For example, we always look to the statement of cash flows first, particularly the net cash provided by operating activities. Downward trends or negative net cash flow from operations almost always indicate problems. The statement of cash flows section on investing activities shows whether the company has made a big acquisition, especially when compared with the prior years’ net cash flows from investing activities. A quick look at the section on financing activities also reveals whether or not a company is issuing debt or buying back stock; in other words, is the company raising capital from investors or returning it to them? Calculate and Examine the Return on Invested Capital After examining the statement of cash flows, we calculate the return on invested capital (ROIC) as described in Chapter 2. The ROIC provides a vital measure of a firm’s overall performance. If ROIC is greater than the company’s weighted average cost of capital (WACC), then the company usually is adding value. If ROIC is less than WACC, then the company usually has serious problems. No matter what ROIC tells us about the firm’s overall performance, it is important to examine specific areas within the firm, and for that we use ratios. Begin Ratio Analysis Financial ratios are designed to extract important information that might not be obvious simply from examining a firm’s financial statements. For example, suppose Firm A owes $5 million of debt while Firm B owes $50 million of debt. Which company is in a stronger financial position? It is impossible to answer this question without first standardizing each firm’s debt relative to total assets, earnings, and interest. Such standardized comparisons are provided through ratio analysis. We will calculate the 2010 financial ratios for MicroDrive Inc., using data from the balance sheets and income statements given in Table 3-1. We will also evaluate the ratios in relation to the industry averages. Note that dollar amounts are in millions. 3.2 LIQUIDITY RATIOS resource See Ch03 Tool Kit.xls for all calculations. As shown in Table 3-1, MicroDrive has current liabilities of $310 million that must be paid off within the coming year. Will it have trouble satisfying those obligations? Liquidity ratios attempt to answer this type of question: We discuss two commonly used liquidity ratios in this section. The Current Ratio The current ratio is calculated by dividing current assets by current liabilities: Current ratio ¼ Current assets Current liabilities 90 Part 1: Fundamental Concepts of Corporate Finance T AB L E 3 - 1 M i c r o D r i v e I n c . : B a l a n c e S h e e t s a n d I n c o m e S t a t e m e n ts f o r Y ea r s E n d i n g D e c e m be r 31 ( M i l l i o n s o f D o l l a r s , E x c ep t fo r P e r S h ar e Da t a ) ASSETS 20 1 0 2 00 9 L I A B I L I T I E S A N D E Q U I TY Cash and equivalents Short-term investments Accounts receivable Inventories Total current assets Net plant and equipment $ 10 0 375 615 $1,000 1,000 $ 15 65 315 415 $ 810 870 Total assets $2,000 $1,680 Accounts payable Notes payable Accruals Total current liabilities Long-term bondsa Total liabilities Preferred stock (400,000 shares) Common stock (50,000,000 shares) Retained earnings Total common equity Total liabilities and equity 2010 $ 2009 60 110 140 $ 310 754 $1,064 40 130 766 $ 896 $2,000 $ 30 60 130 $ 220 580 $ 800 40 130 710 $ 840 $1,680 2010 2009 Net sales Operating costs excluding depreciation and amortizationb Earnings before interest, taxes, depreciation, and amortization (EBITDA) Depreciation Amortization Depreciation and amortization Earnings before interest and taxes (EBIT, or operating income) Less interest Earnings before taxes (EBT) Taxes (40%) Net income before preferred dividends Preferred dividends Net income $3,000.0 2,616.2 $ 383.8 100.0 0.0 $ 100.0 $ 283.8 88.0 $ 195.8 78.3 $ 117.5 4.0 $ 113.5 $2,850.0 2,497.0 $ 353.0 90.0 0.0 $ 90.0 $ 263.0 60.0 $ 203.0 81.2 $ 121.8 4.0 $ 117.8 Common dividends Addition to retained earnings $ $ 57.5 56.0 $ $ 53.0 64.8 Per-Share Data Common stock price Earnings per share (EPS) Book value per share (BVPS) Cash flow per share (CFPS) $ $ $ $ 23.00 2.27 17.92 4.27 $ $ $ $ 26.00 2.36 16.80 4.16 a The bonds have a sinking fund requirement of $20 million a year. The costs include lease payments of $28 million a year. b $1; 000 ¼ 3:2 $310 Industry average ¼ 4:2 ¼ Current assets normally include cash, marketable securities, accounts receivable, and inventories. Current liabilities consist of accounts payable, short-term notes payable, current maturities of long-term debt, accrued taxes, and other accrued expenses. Chapter 3: Analysis of Financial Statements 91 MicroDrive has a lower current ratio than the average for its industry. Is this good or bad? Sometimes the answer depends on who is asking the question. For example, suppose a supplier is trying to decide whether to extend credit to MicroDrive. In general, creditors like to see a high current ratio. If a company is getting into financial difficulty, it will begin paying its bills (accounts payable) more slowly, borrowing from its bank, and so on, so its current liabilities will be increasing. If current liabilities are rising faster than current assets then the current ratio will fall, and this could spell trouble. Because the current ratio provides the best single indicator of the extent to which the claims of short-term creditors are covered by assets that are expected to be converted to cash fairly quickly, it is the most commonly used measure of short-term solvency. Now consider the current ratio from the perspective of a shareholder. A high current ratio could mean that the company has a lot of money tied up in nonproductive assets, such as excess cash or marketable securities. Or perhaps the high current ratio is due to large inventory holdings, which might well become obsolete before they can be sold. Thus, shareholders might not want a high current ratio. An industry average is not a magic number that all firms should strive to maintain—in fact, some very well-managed firms will be above the average, while other good firms will be below it. However, if a firm’s ratios are far removed from the averages for its industry, this is a red flag, and analysts should be concerned about why the variance occurs. For example, suppose a low current ratio is traced to low inventories. Is this a competitive advantage resulting from the firm’s mastery of just-in-time inventory management, or is it an Achilles’ heel that is causing the firm to miss shipments and lose sales? Ratio analysis doesn’t answer such questions, but it does point to areas of potential concern. The Quick, or Acid Test, Ratio The quick, or acid test, ratio is calculated by deducting inventories from current assets and then dividing the remainder by current liabilities: Quick; or acid test; ratio ¼ Current assets − Inventories Current liabilities $385 ¼ 1:2 $310 Industry average ¼ 2:1 ¼ A liquid asset is one that trades in an active market and hence can be converted quickly to cash at the going market price. Inventories are typically the least liquid of a firm’s current assets; hence they are the current assets on which losses are most likely to occur in a bankruptcy. Therefore, a measure of the firm’s ability to pay off short-term obligations without relying on the sale of inventories is important. The industry average quick ratio is 2.1, so MicroDrive’s 1.2 ratio is low in comparison with other firms in its industry. Still, if the accounts receivable can be collected, the company can pay off its current liabilities without having to liquidate its inventory. Self-Test Identify two ratios that are used to analyze a firm’s liquidity position, and write out their equations. What are the characteristics of a liquid asset? Give some examples. Which current asset is typically the least liquid? A company has current liabilities of $800 million, and its current ratio is 2.5. What is its level of current assets? ($2,000 million) If this firm’s quick ratio is 2, how much inventory does it have? ($400 million) 92 Part 1: Fundamental Concepts of Corporate Finance 3.3 ASSET MANAGEMENT RATIOS Asset management ratios measure how effectively a firm is managing its assets. If a company has excessive investments in assets, then its operating capital will be unduly high, which will reduce its free cash flow and ultimately its stock price. On the other hand, if a company does not have enough assets then it will lose sales, which will hurt profitability, free cash flow, and the stock price. Therefore, it is important to have the right amount invested in assets. Ratios that analyze the different types of assets are described in this section. Evaluating Inventories: The Inventory Turnover Ratio The inventory turnover ratio is defined as sales divided by inventories: Inventory turnover ratio ¼ Sales Inventories $3;000 ¼ 4:9 $615 Industry average ¼ 9:0 ¼ As a rough approximation, each item of MicroDrive’s inventory is sold out and restocked, or “turned over,” 4.9 times per year.2 MicroDrive’s turnover of 4.9 is much lower than the industry average of 9.0. This suggests that MicroDrive is holding too much inventory. High levels of inventory add to net operating working capital (NOWC), which reduces FCF, which leads to lower stock prices. In addition, MicroDrive’s low inventory turnover ratio makes us wonder whether the firm is actually holding obsolete goods not worth their stated value.3 Note that sales occur over the entire year, whereas the inventory figure is measured at a single point in time. For this reason, it is better to use an average inventory measure.4 If the firm’s business is highly seasonal, or if there has been a strong upward or downward sales trend during the year, then it is especially useful to make some such adjustment. To maintain comparability with industry averages, however, we did not use the average inventory figure. “Turnover” is a term that originated many years ago with the old Yankee peddler who would load up his wagon with goods and then go off to peddle his wares. If he made 10 trips per year, stocked 100 pans, and made a gross profit of $5 per pan, his annual gross profit would be (100)($5)(10) = $5,000. If he “turned over” (i.e., sold) his inventory faster and made 20 trips per year, then his gross profit would double, other things held constant. So, his turnover directly affected his profits. 2 A problem arises when calculating and analyzing the inventory turnover ratio. Sales are stated at market prices, so if inventories are carried at cost, as they generally are, then the calculated turnover overstates the true turnover ratio. Therefore, it would be more appropriate to use cost of goods sold in place of sales in the formula’s numerator. However, established compilers of financial ratio statistics such as Dun & Bradstreet use the ratio of sales to inventories carried at cost. To develop a figure that can be compared with those published by Dun & Bradstreet and similar organizations, it is necessary to measure inventory turnover with sales in the numerator, as we do here. 3 Preferably, the average inventory value should be calculated by summing the monthly figures during the year and dividing by 12. If monthly data are not available, one can add the beginning and ending annual figures and divide by 2. However, most industry ratios are calculated as shown here, using end-of-year values. 4 Chapter 3: Analysis of Financial Statements 93 THE GLOBAL ECONOMIC CRISIS The Price is Right! (Or Wrong!) How much is an asset worth if no one is buying or selling? The answer to that question matters because an accounting practice called “mark to market” requires that some assets be adjusted on the balance sheet to reflect their “fair market value.” The accounting rules are complicated, but the general idea is that if an asset is available for sale, then the balance sheet would be most accurate if it showed the asset’s market value. For example, suppose a company purchased $100 million of Treasury bonds and the value of those bonds later fell to $90 million. With mark to market, the company would report the bonds’ value on the balance sheet as $90 million, not the original purchase price of $100 million. Notice that marking to market can have a significant impact on financial ratios and thus on investors’ perception of a firm’s financial health. But what if the assets are mortgage-backed securities that were originally purchased for $100 million? As defaults increased during 2008, the value of such securities fell rapidly, and then investors virtually stopped trading them. How should the company report them? At the $100 million original price, at a $60 million price that was observed before the market largely dried up, at $25 million when a hedge fund in desperate need for cash to avoid a costly default sold a few of these securities, or at $0, since there are no current quotes? Or should they be reported at a price generated by a computer model or in some other manner? The answer to this question has vital implications for the global financial crisis. In early 2009, Congress, the SEC, FASB, and the U.S. Treasury all are working to find the right answers. If they come up with a price that is too low, it could cause investors mistakenly to believe that some companies are worth much less than their intrinsic values, and this could trigger runs on banks and bankruptcies for companies that might otherwise survive. But if the price is too high, some “walking dead” or “zombie” companies could linger on and later cause even larger losses for investors, including the U.S. government, which is now the largest investor in many financial institutions. Either way, an error in pricing could perhaps trigger a domino effect that might topple the entire financial system. So let’s hope the price is right! Evaluating Receivables: The Days Sales Outstanding Days sales outstanding (DSO), also called the “average collection period” (ACP), is used to appraise accounts receivable, and it is calculated by dividing accounts receivable by average daily sales to find the number of days’ sales that are tied up in receivables.5 Thus, the DSO represents the average length of time that the firm must wait after making a sale before receiving cash, which is the average collection period. MicroDrive has 46 DSO, well above the 36-day industry average: DSO ¼ Receivables Receivables Days sales ¼ ¼ outstanding Average sales per day Annual sales=365 ¼ $375 $375 ¼ ¼ 45:6 days ≈ 46 days $3;000=365 $8:2192 Industry average ¼ 36 days MicroDrive’s sales terms call for payment within 30 days. The fact that 46 days of sales are outstanding indicates that customers, on average, are not paying their bills It would be better to use average receivables, but we have used year-end values for comparability with the industry average. 5 94 Part 1: Fundamental Concepts of Corporate Finance on time. As with inventory, high levels of accounts receivable cause high levels of NOWC, which hurts FCF and stock price. A customer who is paying late may well be in financial trouble, in which case MicroDrive may have a hard time ever collecting the receivable. Therefore, if the trend in DSO has been rising but the credit policy has not been changed, steps should be taken to review credit standards and to expedite the collection of accounts receivable. Evaluating Fixed Assets: The Fixed Assets Turnover Ratio The fixed assets turnover ratio measures how effectively the firm uses its plant and equipment. It is the ratio of sales to net fixed assets: Fixed assets turnover ratio ¼ Sales Net fixed assets $3;000 ¼ 3:0 $1;000 Industry average ¼ 3:0 ¼ MicroDrive’s ratio of 3.0 is equal to the industry average, indicating that the firm is using its fixed assets about as intensively as are other firms in its industry. Therefore, MicroDrive seems to have about the right amount of fixed assets in relation to other firms. A potential problem can exist when interpreting the fixed assets turnover ratio. Recall from accounting that fixed assets reflect the historical costs of the assets. Inflation has caused the current value of many assets that were purchased in the past to be seriously understated. Therefore, if we were comparing an old firm that had acquired many of its fixed assets years ago at low prices with a new company that had acquired its fixed assets only recently, we would probably find that the old firm had the higher fixed assets turnover ratio. However, this would be more reflective of the difficulty accountants have in dealing with inflation than of any inefficiency on the part of the new firm. You should be alert to this potential problem when evaluating the fixed assets turnover ratio. Evaluating Total Assets: The Total Assets Turnover Ratio The total assets turnover ratio is calculated by dividing sales by total assets: Total assets turnover ratio ¼ Sales Total assets $3;000 ¼ 1:5 $2;000 Industry average ¼ 1:8 ¼ MicroDrive’s ratio is somewhat below the industry average, indicating that the company is not generating a sufficient volume of business given its total asset investment. Sales should be increased, some assets should be sold, or a combination of these steps should be taken. Self-Test Identify four ratios that are used to measure how effectively a firm is managing its assets, and write out their equations. What problem might arise when comparing different firms’ fixed assets turnover ratios? A firm has annual sales of $200 million, $40 million of inventory, and $60 million of accounts receivable. What is its inventory turnover ratio? (5) What is its DSO based on a 365-day year? (109.5 days) Chapter 3: Analysis of Financial Statements 95 3.4 DEBT MANAGEMENT RATIOS The extent to which a firm uses debt financing, or financial leverage, has three important implications: (1) By raising funds through debt, stockholders can maintain control of a firm without increasing their investment. (2) If the firm earns more on investments financed with borrowed funds than it pays in interest, then its shareholders’ returns are magnified, or “leveraged,” but their risks are also magnified. (3) Creditors look to the equity, or owner-supplied funds, to provide a margin of safety, so the higher the proportion of funding supplied by stockholders, the less risk creditors face. Chapter 15 explains the first two points in detail, while the following ratios examine leverage from a creditor’s point of view. How the Firm is Financed: Total Liabilities to Total Assets The ratio of total liabilities to total assets is called the debt ratio, or sometimes the total debt ratio. It measures the percentage of funds provided by current liabilities and long-term debt: Debt ratio ¼ ¼ Total liabilities Total assets $310 þ $754 $1;064 ¼ ¼ 53:2% $2;000 $2;000 Industry average ¼ 40:0% Creditors prefer low debt ratios because the lower the ratio, the greater the cushion against creditors’ losses in the event of liquidation. Stockholders, on the other hand, may want more leverage because it magnifies their return, as we explain in Section 3.8 when we discuss the Du Pont model. MicroDrive’s debt ratio is 53.2% but its debt ratio in the previous year was 47.6%, which means that creditors are now supplying more than half the total financing. In addition to an upward trend, the level of the debt ratio is well above the industry average. Creditors may be reluctant to lend the firm more money because a high debt ratio is associated with a greater risk of bankruptcy. Some sources report the debt-to-equity ratio, defined as: Debt-to-equity ratio ¼ ¼ Total liabilities Total assets − Total liabilities $310 þ $754 $1;064 ¼ ¼ 1:14 $2;000 − ð$310 þ $754Þ $936 Industry average ¼ 0:67 The debt-to-equity ratio and the debt ratio contain the same information but present that information slightly differently.6 The debt-to-equity ratio shows that MicroDrive has $1.14 of debt for every dollar of equity, whereas the debt ratio shows that 53.2% of MicroDrive’s financing is in the form of liabilities. We find it more 6 The debt ratio and debt-to-equity ratios are simply transformations of each other: Debt-to-equity ¼ Debt ratio Debt-to-equity and Debt ratio ¼ 1  Debt ratio 1 þ Debt-to-equity 96 Part 1: Fundamental Concepts of Corporate Finance intuitive to think about the percentage of the firm that is financed with debt, so we usually use the debt ratio. However, the debt-to-equity ratio is also widely used, so you should know how to interpret it. Sometimes it is useful to express debt ratios in terms of market values. It is easy to calculate the market value of equity, which is equal to the stock price multiplied by the number of shares. MicroDrive’s market value of equity is $23(50) = $1,150. Often it is difficult to estimate the market value of liabilities, so many analysts define the market debt ratio as Market debt ratio ¼ ¼ Total liabilities Total liabilities þ Market value of equity $1;064 $1;064 ¼ ¼ 48:1% $1;064 þ ð$23 × 50Þ $2;214 MicroDrive’s market debt ratio in the previous year was 38.1%. The big increase was due to two major factors: Liabilities increased and the stock price fell. The stock price reflects a company’s prospects for generating future cash flows, so a decline in stock price indicates a likely decline in future cash flows. Thus, the market debt ratio reflects a source of risk that is not captured by the conventional book debt ratio. If you use a debt ratio that you did not calculate yourself, be sure to find out how the ratio was defined. Some sources provide the ratio of long-term debt to total assets, and some provide the ratio of all debt to equity, so be sure to check your source’s definition. Ability to Pay Interest: Times-Interest-Earned Ratio The times-interest-earned (TIE) ratio, also called the interest coverage ratio, is determined by dividing earnings before interest and taxes (EBIT in Table 3-1) by the interest expense: Times-interest-earned ðTIEÞ ratio ¼ EBIT Interest expense $283:8 ¼ 3:2 $88 Industry average ¼ 6:0 ¼ The TIE ratio measures the extent to which operating income can decline before the firm is unable to meet its annual interest costs. Failure to meet this obligation can bring legal action by the firm’s creditors, possibly resulting in bankruptcy. Note that earnings before interest and taxes, rather than net income, is used in the numerator. Because interest is paid with pre-tax dollars, the firm’s ability to pay current interest is not affected by taxes. MicroDrive’s interest is covered 3.2 times. The industry average is 6, so MicroDrive is covering its interest charges by a relatively low margin of safety. Thus, the TIE ratio reinforces the conclusion from our analysis of the debt ratio that MicroDrive would face difficulties if it attempted to borrow additional funds. Chapter 3: Analysis of Financial Statements 97 Ability to Service Debt: EBITDA Coverage Ratio The TIE ratio is useful for assessing a company’s ability to meet interest charges on its debt, but this ratio has two shortcomings: (1) Interest is not the only fixed financial charge—companies must also reduce debt on schedule, and many firms lease assets and thus must make lease payments. If they fail to repay debt or meet lease payments, they can be forced into bankruptcy. (2) EBIT does not represent all the cash flow available to service debt, especially if a firm has high depreciation and/or amortization charges. The EBITDA coverage ratio accounts for these deficiencies:7 EBITDA coverage ratio ¼ ¼ EBITDA þ Lease payments Interest þ Principal payments þ Lease payments $383:8 þ $28 $411:8 ¼ ¼ 3:0 $88 þ $20 þ $28 $136 Industry average ¼ 4:3 MicroDrive had $383.8 million of earnings before interest, taxes, depreciation, and amortization (EBITDA). Also, lease payments of $28 million were deducted while calculating EBITDA. That $28 million was available to meet financial charges; hence it must be added back, bringing the total available to cover fixed financial charges to $411.8 million. Fixed financial charges consisted of $88 million of interest, $20 million of sinking fund payments, and $28 million for lease payments, for a total of $136 million.8 Therefore, MicroDrive covered its fixed financial charges by 3.0 times. However, if EBITDA declines then the coverage will fall, and EBITDA certainly can decline. Moreover, MicroDrive’s ratio is well below the industry average, so again the company seems to have a relatively high level of debt. The EBITDA coverage ratio is most useful for relatively short-term lenders such as banks, which rarely make loans (except real estate-backed loans) for longer than about 5 years. Over a relatively short period, depreciation-generated funds can be used to service debt. Over a longer time, those funds must be reinvested to maintain the plant and equipment or else the company cannot remain in business. Therefore, banks and other relatively short-term lenders focus on the EBITDA coverage ratio, whereas long-term bondholders focus on the TIE ratio. Self-Test How does the use of financial leverage affect current stockholders’ control position? Explain the following statement: “Analysts look at both balance sheet and income statement ratios when appraising a firm’s financial condition.” Name three ratios that are used to measure the extent to which a firm uses financial leverage, and write out their equations. A company has EBITDA of $600 million, interest payments of $60 million, lease payments of $40 million, and required principal payments (due this year) of $30 million. What is its EBITDA coverage ratio? (4.9) 7 Different analysts define the EBITDA coverage ratio in different ways. For example: some omit the lease payment information; others “gross up” principal payments by dividing them by 1 – T since these payments are not tax deductions and hence must be made with after-tax cash flows. We included lease payments because for many firms they are quite important, and failing to make them can lead to bankruptcy just as surely as can failure to make payments on “regular” debt. We did not gross up principal payments because, if a company is in financial difficulty, then its tax rate will probably be zero; hence the gross up is not necessary whenever the ratio is really important. A sinking fund is a required annual payment designed to reduce the balance of a bond or preferred stock issue. 8 98 Part 1: Fundamental Concepts of Corporate Finance 3.5 PROFITABILITY RATIOS Profitability is the net result of a number of policies and decisions. The ratios examined thus far provide useful clues as to the effectiveness of a firm’s operations, but the profitability ratios go on to show the combined effects of liquidity, asset management, and debt on operating results. Net Profit Margin The net profit margin, which is also called the profit margin on sales, is calculated by dividing net income by sales. It gives the profit per dollar of sales: Net income available to common stockholders Net profit margin ¼ Sales $113:5 ¼ 3:8% $3;000 Industry average ¼ 5:0% ¼ MicroDrive’s net profit margin is below the industry average of 5%, but why is this so? Is it due to inefficient operations, high interest expenses, or both? Instead of just comparing net income to sales, many analysts also break the income statement into smaller parts to identify the sources of a low net profit margin. For example, the operating profit margin is defined as Operating profit margin ¼ EBIT Sales The operating profit margin identifies how a company is performing with respect to its operations before the impact of interest expenses is considered. Some analysts drill even deeper by breaking operating costs into their components. For example, the gross profit margin is defined as Gross profit margin ¼ Sales − Cost of goods sold Sales The gross profit margin identifies the gross profit per dollar of sales before any other expenses are deducted. Rather than calculate each type of profit margin here, later in the chapter we will use common size analysis and percent change analysis to focus on different parts of the income statement. In addition, we will use the Du Pont equation to show how the ratios interact with one another. Sometimes it is confusing to have so many different types of profit margins. To help simplify the situation, we will focus primarily on the net profit margin throughout the book and simply call it the “profit margin.” Basic Earning Power (BEP) Ratio The basic earning power (BEP) ratio is calculated by dividing earnings before interest and taxes (EBIT) by total assets: Chapter 3: Analysis of Financial Statements 99 The World Might be Flat, but Global Accounting is Bumpy! The Case of IFRS versus FASB In a flat world, distance is no barrier. Work flows to where it can be accomplished most efficiently, and capital flows to where it can be invested most profitably. If a radiologist in India is more efficient than one in the United States, then images will be e-mailed to India for diagnosis; if rates of return are higher in Brazil, then investors throughout the world will provide funding for Brazilian projects. One key to “flattening” the world is agreement on common standards. For example, there are common Internet standards so that users throughout the world are able to communicate. A glaring exception to standardization is in accounting. The Securities and Exchange Commission (SEC) in the United States requires firms to comply with standards set by the Financial Accounting Standards Board (FASB). But the European Union requires all EU-listed companies to comply with the International Financial Reporting Standards (IFRS) as defined by the International Accounting Standards Board (IASB). IFRS tends to rely on general principles, whereas FASB standards are rules-based. As the recent accounting scandals demonstrate, many U.S. companies have been able to comply with U.S. rules while violating the principle, or intent, underlying the rules. The United States is likely to adopt IFRS, or a slightly modified IFRS, but the question is “When?” The SEC estimated that a large company is likely to incur costs of up to $32 million when switching to IFRS. So even though a survey by the accounting firm KPMG indicates that most investors and analysts favor adoption of IFRS, the path to adoption is likely to be bumpy. Sources: See the Web sites of the IASB and the FASB, http:// www.iasb.org.uk and http://www.fasb.org. Also see David M. Katz and Sarah Johnson, “Top Obama Advisers Clash on Global Accounting Standards,” January 15, 2009, at http:// www.cfo.com; and “Survey Favors IFRS Adoption,” February 3, 2009, at http://www.webcpa.com. Basic earning power ðBEPÞ ratio ¼ EBIT Total assets $283:8 ¼ 14:2% $2;000 Industry average ¼ 17:2% ¼ This ratio shows the raw earning power of the firm’s assets before the influence of taxes and leverage, and it is useful for comparing firms with different tax situations and different degrees of financial leverage. Because of its low turnover ratios and low profit margin on sales, MicroDrive is not getting as high a return on its assets as is the average company in its industry.9 Return on Total Assets The ratio of net income to total assets measures the return on total assets (ROA) after interest and taxes. This ratio is also called the return on assets and is defined as follows: 9 Notice that EBIT is earned throughout the year, whereas the total assets figure is an end-of-the-year number. Therefore, it would be better, conceptually, to calculate this ratio as EBIT/(Average assets) = EBIT/[(Beginning assets + Ending assets)/2]. We have not made this adjustment because the published ratios used for comparative purposes do not include it. However, when we construct our own comparative ratios, we do make this adjustment. The same adjustment would also be appropriate for the next two ratios, ROA and ROE. 100 Part 1: Fundamental Concepts of Corporate Finance Net income available to common stockholders Return on ¼ ROA ¼ total assets Total assets $113:5 ¼ 5:7% $2;000 Industry average ¼ 9:0% ¼ MicroDrive’s 5.7% return is well below the 9% average for the industry. This low return is due to (1) the company’s low basic earning power and (2) high interest costs resulting from its above-average use of debt; both of these factors cause MicroDrive’s net income to be relatively low. Return on Common Equity The ratio of net income to common equity measures the return on common equity (ROE): Net income available to common stockholders Return on ¼ ROE ¼ common equity Common equity $113:5 ¼ 12:7% $896 Industry average ¼ 15:0% ¼ Stockholders invest to earn a return on their money, and this ratio tells how well they are doing in an accounting sense. MicroDrive’s 12.7% return is below the 15% industry average, but not as far below as its return on total assets. This somewhat better result is due to the company’s greater use of debt, a point that we explain in detail later in the chapter. Self-Test Identify and write out the equations for four profitability ratios. Why is the basic earning power ratio useful? Why does the use of debt lower ROA? What does ROE measure? A company has $200 billion of sales and $10 billion of net income. Its total assets are $100 billion, financed half by debt and half by common equity. What is its profit margin? (5%) What is its ROA? (10%) What is its ROE? (20%) Would ROA increase if the firm used less leverage? (Yes) Would ROE increase? (No) 3.6 MARKET VALUE RATIOS Market value ratios relate a firm’s stock price to its earnings, cash flow, and book value per share. Market value ratios are a way to measure the value of a company’s stock relative to that of another company. Price/Earnings Ratio The price/earnings (P/E) ratio shows how much investors are willing to pay per dollar of reported profits. MicroDrive’s stock sells for $23, so with an earnings per share (EPS) of $2.27 its P/E ratio is 10.1: Chapter 3: Analysis of Financial Statements Price=earnings ðP=EÞ ratio ¼ 101 Price per share Earnings per share $23:00 ¼ 10:1 $2:27 Industry average ¼ 12:5 ¼ Price/earnings ratios are higher for firms with strong growth prospects, other things held constant, but they are lower for riskier firms. Because MicroDrive’s P/E ratio is below the average, this suggests that the company is regarded as being somewhat riskier than most, as having poorer growth prospects, or both. In early 2009, the average P/E ratio for firms in the S&P 500 was 12.54, indicating that investors were willing to pay $12.54 for every dollar of earnings. Price/Cash Flow Ratio Stock prices depend on a company’s ability to generate cash flows. Consequently, investors often look at the price/cash flow ratio, where cash flow is defined as net income plus depreciation and amortization: Price=cash flow ratio ¼ Price per share Cash flow per share $23:00 ¼ 5:4 $4:27 Industry average ¼ 6:8 ¼ MicroDrive’s price/cash flow ratio is also below the industry average, once again suggesting that its growth prospects are below average, its risk is above average, or both. The price/EBITDA ratio is similar to the price/cash flow ratio, except the price/ EBITDA ratio measures performance before the impact of interest expenses and taxes, making it a better measure of operating performance. MicroDrive’s EBITDA per share is $383.8/50 = $7.676, so its price/EBITDA is $23/$7.676 = 3.0. The industry average price/EBITDA ratio is 4.6, so we see again that MicroDrive is below the industry average. Note that some analysts look at other multiples as well. For example, depending on the industry, some may look at measures such as price/sales or price/customers. Ultimately, though, value depends on free cash flows, so if these “exotic” ratios do not forecast future free cash flow, they may turn out to be misleading. This was true in the case of the dot-com retailers before they crashed and burned in 2000, costing investors many billions. Market/Book Ratio The ratio of a stock’s market price to its book value gives another indication of how investors regard the company. Companies with relatively high rates of return on equity generally sell at higher multiples of book value than those with low returns. First, we find MicroDrive’s book value per share: 102 Part 1: Fundamental Concepts of Corporate Finance Book value per share ¼ ¼ Common equity Shares outstanding $896 ¼ $17:92 50 Now we divide the market price by the book value to get a market/book (M/B) ratio of 1.3 times: Market=book ratio ¼ M=B ¼ Market price per share Book value per share $23:00 ¼ 1:3 $17:92 Industry average ¼ 1:7 ¼ Investors are willing to pay relatively little for a dollar of MicroDrive’s book value. The average company in the S&P 500 had a market/book ratio of about 2.50 in early 2009. Since M/B ratios typically exceed 1.0, this means that investors are willing to pay more for stocks than their accounting book values. The book value is a record of the past, showing the cumulative amount that stockholders have invested, either directly by purchasing newly issued shares or indirectly through retaining earnings. In contrast, the market price is forward-looking, incorporating investors’ expectations of future cash flows. For example, in early 2009 Alaska Air had a market/book ratio of only 0.81, reflecting the airline industry’s problems, whereas Apple’s market/book ratio was 3.45, indicating that investors expected Apple’s past successes to continue. Table 3-2 summarizes MicroDrive’s financial ratios. As the table indicates, the company has many problems. Self-Test Describe three ratios that relate a firm’s stock price to its earnings, cash flow, and book value per share, and write out their equations. What does the price/earnings (P/E) ratio show? If one firm’s P/E ratio is lower than that of another, what are some factors that might explain the difference? How is book value per share calculated? Explain why book values often deviate from market values. A company has $6 billion of net income, $2 billion of depreciation and amortization, $80 billion of common equity, and 1 billion shares of stock. If its stock price is $96 per share, what is its price/earnings ratio? (16) Its price/cash flow ratio? (12) Its market/book ratio? (1.2) 3.7 TREND ANALYSIS, COMMON SIZE ANALYSIS, AND PERCENTAGE CHANGE ANALYSIS Trends give clues as to whether a firm’s financial condition is likely to improve or deteriorate. To do a trend analysis, you examine a ratio over time, as shown in Figure 3-1. This graph shows that MicroDrive’s rate of return on common equity has been declining since 2007, even though the industry average has been relatively stable. All the other ratios could be analyzed similarly. In a common size analysis, all income statement items are divided by sales and all balance sheet items are divided by total assets. Thus, a common size income state- Chapter 3: Analysis of Financial Statements TABLE 3-2 103 M i c r o D r i v e I n c . : S u m m a r y o f F i n a n c i a l R a t i os ( M i l l i on s o f D o l l a r s ) RATIO F O R MU L A Liquidity CALCULATION RATIO I N DU S T R Y A V ER AG E COMME NT Current assets Current liabilities $1;000 ¼ $310 3.2 4.2 Poor Current assets − Inventories Current liabilities $385 ¼ $310 1.2 2.1 Poor Sales Inventories $3;000 ¼ $615 4.9 9.0 Poor Receivables Annual sales=365 $375 ¼ $8:219 45.6 36.0 Poor Fixed assets turnover Sales Net fixed assets $3;000 ¼ $1;000 3.0 3.0 OK Total assets turnover Sales Total assets $3;000 ¼ $2;000 1.5 1.8 Poor Total liabilities Total assets $1;064 ¼ $2;000 53.2% 40.0% High (risky) Earnings before interest and taxes ðEBITÞ Interest charges $283:8 ¼ $88 3.2 6.0 Low (risky) EBITDA þ Lease pmts: Interest þ Principal payments þ Lease pmts: $411:8 ¼ $136 3.0 4.3 Low (risky) Net income available to common stockholders Sales $113:5 ¼ $3;000 3.8% 5.0% Poor Earnings before interest and taxes ðEBITÞ Total assets $283:8 ¼ $2;000 14.2% 17.2% Poor Net income available to common stockholders Total assets $113:5 ¼ $2;000 5.7% 9.0% Poor Net income available to common stockholders Common equity $113:5 ¼ $896 12.7% 15.0% Poor Price/earnings (P/E) Price per share Earnings per share $23:00 ¼ $2:27 10.1 12.5 Low Price/cash flow Price per share Cash flow per share $23:00 ¼ $4:27 5.4 6.8 Low Market price per share Book value per share $23:00 ¼ $17:92 1.3 1.7 Low Current Quick Asset Management Inventory turnover Days sales outstanding (DSO) Debt Management Debt ratio Times-interest-earned (TIE) EBITDA coverage Profitability Profit margin on sales Basic earning power (BEP) Return on total assets (ROA) Return on common equity (ROE) Market Value Market/book (M/B) 104 Part 1: Fundamental Concepts of Corporate Finance FIGURE 3-1 Rate of Return on Common Equity, 2006–2010 ROE (%) 16.0% Industry MicroDrive 12.0% 8.0% 4.0% 0.0% 2006 resource See Ch03 Tool Kit.xls. 2007 2008 2009 2010 ment shows each item as a percentage of sales, and a common size balance sheet shows each item as a percentage of total assets.10 The advantage of common size analysis is that it facilitates comparisons of balance sheets and income statements over time and across companies. Common size statements are easy to generate if the financial statements are in a spreadsheet. In fact, if you obtain your data from a source that uses standardized financial statements, then it is easy to cut and paste the data for a new company over your original company’s data, and all of your spreadsheet formulas will be valid for the new company. We generated Figure 3-2 in the Excel file Ch03 Tool Kit.xls. Figure 3-2 shows MicroDrive’s 2009 and 2010 common size income statements, along with the composite statement for the industry. (Note: Rounding may cause addition/subtraction differences in Figures 3-2, 3-3, and 3-4.) MicroDrive’s EBIT is slightly below average, and its interest expenses are slightly above average. The net effect is a relatively low profit margin. Figure 3-3 shows MicroDrive’s common size balance sheets along with the industry composite. Its accounts receivable are significantly higher than the industry average, its inventories are significantly higher, and it uses much more debt than the average firm. In percentage change analysis, growth rates are calculated for all income statement items and balance sheet accounts relative to a base year. To illustrate, Figure 3-4 contains MicroDrive’s income statement percentage change analysis for 2010 relative to 2009. Sales increased at a 5.3% rate during 2010, but EBITDA increased by 8.7%. This “good news” was offset by a 46.7% increase in interest expense. The significant growth in interest expense caused growth in net income to be negative. Thus, the percentage change analysis points out that the decrease in net income in 2010 resulted almost exclusively from an increase in interest expense. This conclusion could be reached by analyzing dollar Some sources of industry data, such as Risk Management Associates (formerly known as Robert Morris Associates), are presented exclusively in common size form. 10 Chapter 3: Analysis of Financial Statements FIGURE 3-2 MicroDrive Inc.: Common Size Income Statement 167 168 169 170 171 172 173 174 175 176 177 178 179 180 FIGURE 3-3 105 Industry Composite 2010 Net sales 100.0% Operating costs 87.6% Earnings before interest, taxes, depr. & amort. (EBITDA) 12.4% Depreciation and amortization 2.8% Earnings before interest and taxes (EBIT) 9.6% Less interest 1.3% Earnings before taxes (EBT) 8.3% Taxes (40%) 3.3% Net income before preferred dividends 5.0% Preferred dividends 0.0% 5.0% Net income available to common stockholders (profit margin) MicroDrive 2010 2009 100.0% 100.0% 87.2% 87.6% 12.8% 12.4% 3.3% 3.2% 9.5% 9.2% 2.9% 2.1% 6.5% 7.1% 2.6% 2.8% 3.9% 4.3% 0.1% 0.1% 3.5% 4.1% MicroDrive Inc.: Common Size Balance Sheet 185 186 187 188 189 190 191 192 193 194 195 196 197 198 199 200 201 202 203 204 205 206 resource See Ch03 Tool Kit.xls for details. Self-Test Industry Composite 2010 MicroDrive 2010 2009 Assets Cash and equivalents Short-term investments Accounts receivable Inventories Total current assets Net plant and equipment Total assets 1.0% 2.2% 17.8% 19.8% 40.8% 59.2% 100.0% 0.5% 0.0% 18.8% 30.8% 50.0% 50.0% 100.0% 0.9% 3.9% 18.8% 24.7% 48.2% 51.8% 100.0% Liabilities and equity Accounts payable Notes payable Accruals Total current liabilities Long-term bonds Total liabilities Preferred stock Total common equity Total liabilities and equity 1.8% 4.4% 3.6% 9.8% 30.2% 40.0% 0.0% 60.0% 100.0% 3.0% 5.5% 7.0% 15.5% 37.7% 53.2% 2.0% 44.8% 100.0% 1.8% 3.6% 7.7% 13.1% 34.5% 47.6% 2.4% 50.0% 100.0% amounts, but percentage change analysis simplifies the task. We apply the same type of analysis to the balance sheets (see the file Ch03 Tool Kit.xls), which shows that inventories grew at a whopping 48.2% rate. With only a 5.3% growth in sales, the extreme growth in inventories should be of great concern to MicroDrive’s managers. What is a trend analysis, and what important information does it provide? What is common size analysis? What is percentage change analysis? 106 Part 1: Fundamental Concepts of Corporate Finance FIGURE 3-4 MicroDrive Inc.: Income Statement Percentage Change Analysis resource See Ch03 Tool Kit.xls for details. 213 214 215 216 217 218 219 220 221 222 223 224 225 226 Percent Change in 2010 Base Year = 2009 Net sales Operating costs Earnings before interest, taxes, depr. & amort. (EBITDA) Depreciation and amortization Earnings before interest and taxes (EBIT) Less interest Earnings before taxes (EBT) Taxes (40%) Net income before preferred dividends Preferred dividends Net income available to common stockholders 5.3% 4.8% 8.7% 11.1% 7.9% 46.7% (3.5%) (3.5%) (3.5%) 0.0% (3.7%) 3.8 TYING THE RATIOS TOGETHER: THE DU PONT EQUATION In ratio analysis, it is sometimes easy to miss the forest for all the trees. The Du Pont equation provides a framework that ties together a firm’s profitability, asset efficiency, and use of debt. The return on assets (ROA) can be expressed as the profit margin multiplied by the total assets turnover ratio: ROA ¼ Profit margin × Total assets turnover ¼ Net income Sales × Sales Total assets (3-1) For MicroDrive, the ROA is ROA = 3.8% × 1.5 = 5.7% MicroDrive made 3.8%, or 3.8 cents, on each dollar of sales, and its assets were turned over 1.5 times during the year. Therefore, the company earned a return of 5.7% on its assets. To find the return on equity (ROE), multiply the ROA by the equity multiplier, which is the ratio of assets to common equity: Equity multiplier ¼ Total assets Common equity (3-2) Firms that have a lot of leverage (i.e., a lot of liabilities or preferred stock) have a high equity multiplier because the assets are financed with a relatively smaller amount of equity. Therefore, the return on equity (ROE) depends on the ROA and the use of leverage: Chapter 3: Analysis of Financial Statements ROE ¼ ROA × Equity multiplier Net income Total assets × ¼ Total assets Common equity 107 (3-3) MicroDrive’s ROE is ROE ¼ 5:7% × $2;000=$896 ¼ 5:7% × 2:23 ¼ 12:7% Combining Equations 3-1 and 3-3 gives the extended, or modified, Du Pont equation, which shows how the profit margin, the total assets turnover ratio, and the equity multiplier combine to determine the ROE: ROE ¼ ðProfit marginÞðTotal assets turnoverÞðEquity multiplierÞ ¼ Net income Sales Total assets × × Sales Total assets Common equity (3-4) For MicroDrive, we have ROE ¼ ð3:8%Þð1:5Þð2:23Þ ¼ 12:7% The insights provided by the Du Pont model are valuable, and the model can be used for “quick and dirty” estimates of the impact that operating changes have on returns. For example, holding all else equal, if MicroDrive can implement lean production techniques and increase to 1.8 its ratio of sales to total assets, then its ROE will improve to (3.8%)(1.8)(2.23) = 15.25%. For a more complete “what if” analysis, most companies use a forecasting model such as the one described in Chapter 12. Self-Test Explain how the extended, or modified, Du Pont equation can be used to reveal the basic determinants of ROE. What is the equity multiplier? A company has a profit margin of 6%, a total asset turnover ratio of 2, and an equity multiplier of 1.5. What is its ROE? (18%) 3.9 COMPARATIVE RATIOS AND BENCHMARKING Ratio analysis involves comparisons. A company’s ratios are compared with those of other firms in the same industry—that is, with industry average figures. However, like most firms, MicroDrive’s managers go one step further: they also compare their ratios with those of a smaller set of the leading computer companies. This technique is called benchmarking, and the companies used for the comparison are called benchmark companies. For example, MicroDrive benchmarks against five other firms that its management considers to be the best-managed companies with operations similar to its own. Many companies also benchmark various parts of their overall operation against top companies, whether they are in the same industry or not. For example, MicroDrive has a division that sells hard drives directly to consumers through catalogs and the Internet. This division’s shipping department benchmarks against L.L.Bean, even though they are in different industries, because L.L.Bean’s shipping department is one of the best. MicroDrive wants its own shippers to strive to match L.L.Bean’s record for on-time shipments. 108 Part 1: Fundamental Concepts of Corporate Finance T AB L E 3 - 3 Co m pa r a t i v e R a t i os f o r A pp l e I n c ., t h e C o m p u t er Ha r d w a r e I n du s t r y , t h e T ec h n o l o g y S ec t or , an d t h e S & P 5 0 0 RATIO P/E ratio Market to book Price to tangible book Price to cash flow Net profit margin Quick ratio Current ratio Long-term debt to equity Total debt to equity Interest coverage (TIE)c Return on assets Return on equity Inventory turnover Asset turnover APPLE COMPUTER HARDWARE INDUSTRYa TECH NOL OGY SECTORb S&P 500 15.92 3.60 3.70 14.30 14.88 2.43 2.46 0.00 0.00 — 14.89 27.19 49.90 1.00 7.88 3.12 4.41 6.70 3.32 1.86 2.21 20.05 30.32 0.15 4.07 8.27 12.99 0.37 8.75 2.90 3.87 4.58 4.92 1.97 2.36 18.28 27.38 1.12 4.90 7.68 3.09 0.46 17.93 6.84 8.73 12.01 11.18 1.04 1.28 151.80 197.45 31.97 8.05 19.09 9.71 0.79 a The computer hardware industry is composed of fifty firms, including IBM, Dell, Apple, Sun Microsystems, Gateway, and Silicon Graphics. The technology sector contains eleven industries, including communications equipment, computer hardware, computer networks, semiconductors, and software and programming. c Apple had more interest income than interest expense. b Source: Adapted from http://www.investor.reuters.com, January 17, 2009. WWW To find quick information about a company, go to http://www.investor .reuters.com. Here you can find company profiles, stock price and share information, and several key ratios. Self-Test Comparative ratios are available from a number of sources, including Value Line, Dun and Bradstreet (D&B), and the Annual Statement Studies published by Risk Management Associates, which is the national association of bank loan officers. Table 3-3 reports selected ratios from Reuters for Apple and its industry, revealing that Apple has a much higher profit margin and lower debt ratio than its peers. Each data-supplying organization uses a somewhat different set of ratios designed for its own purposes. For example, D&B deals mainly with small firms, many of which are proprietorships, and it sells its services primarily to banks and other lenders. Therefore, D&B is concerned largely with the creditor’s viewpoint, and its ratios emphasize current assets and liabilities, not market value ratios. So, when you select a comparative data source, you should be sure that your own emphasis is similar to that of the agency whose ratios you plan to use. Additionally, there are often definitional differences in the ratios presented by different sources, so before using a source, be sure to verify the exact definitions of the ratios to ensure consistency with your own work. Differentiate between trend analysis and comparative ratio analysis. What is benchmarking? 3.10 USES AND LIMITATIONS OF RATIO ANALYSIS Ratio analysis provides useful information concerning a company’s operations and financial condition, but it has limitations that necessitate care and judgment. Some potential problems include the following. Chapter 3: Analysis of Financial Statements 109 Ratio Analysis on the Web A great source for comparative ratios is http://www .investor.reuters.com. You have to register to use the site, but registration is free. Once you register and log in, select Stocks; enter a company’s ticker symbol, select the Symbol ratio button, and then click the Go button. This brings up a table with the stock quote, company information, and additional links. Select Ratios, which brings up a page with a detailed ratio analysis for the company and includes comparative ratios for other companies in the same sector, the same industry, and the S&P 500. 1. Many large firms operate different divisions in different industries, and for such companies it is difficult to develop a meaningful set of industry averages. Therefore, industry averages are more applicable to small, narrowly focused firms than to large, multidivisional ones. 2. To set goals for high-level performance, it is best to benchmark on the industry leaders’ ratios rather than the industry average ratios. 3. Inflation may have badly distorted firms’ balance sheets—reported values are often substantially different from “true” values. Further, because inflation affects depreciation charges and inventory costs, reported profits are also affected. Thus, inflation can distort a ratio analysis for one firm over time or a comparative analysis of firms of different ages. 4. Seasonal factors can also distort a ratio analysis. For example, the inventory turnover ratio for a food processor will be radically different if the balance sheet figure used for inventory is the one just before versus the one just after the close of the canning season. This problem can be minimized by using monthly averages for inventory (and receivables) when calculating turnover ratios. 5. Firms can employ “window dressing” techniques to make their financial statements look stronger. To illustrate, suppose a company takes out a 2-year loan in late December. Because the loan is for more than one year, it is not included in current liabilities even though the cash received through the loan is reported as a current asset. This improves the current and quick ratios and makes the year-end balance sheet look stronger. If the company pays the loan back in January, then the transaction was strictly window dressing. 6. Companies’ choices of different accounting practices can distort comparisons. For example, choices of different inventory valuation and depreciation methods affect financial statements differently, making comparisons among companies less meaningful. As another example, if one firm leases a substantial amount of its productive equipment, then its assets may appear low relative to sales (because leased assets often do not appear on the balance sheet) and its debt may appear low (because the liability associated with the lease obligation may not be shown as debt).11 In summary, conducting ratio analysis in a mechanical, unthinking manner is dangerous, but when ratio analysis is used intelligently and with good judgment, it can provide useful insights into a firm’s operations and identify the right questions to ask. This may change when FASB and IASB complete their joint project on leasing. But it may be a while before this happens; in early 2009, the estimated project completion date was 2011. See http:// 72.3.243.42/project/leases.shtml for updates. 11 110 Part 1: Fundamental Concepts of Corporate Finance Self-Test List several potential problems with ratio analysis. 3.11 LOOKING BEYOND THE NUMBERS Sound financial analysis involves more than just calculating and comparing ratios— qualitative factors must be considered. Here are some questions suggested by the American Association of Individual Investors (AAII). 1. To what extent are the company’s revenues tied to one key customer or to one key product? To what extent does the company rely on a single supplier? Reliance on single customers, products, or suppliers increases risk. 2. What percentage of the company’s business is generated overseas? Companies with a large percentage of overseas business are exposed to risk of currency exchange volatility and political instability. 3. What are the probable actions of current competitors and the likelihood of additional new competitors? 4. Do the company’s future prospects depend critically on the success of products currently in the pipeline or on existing products? 5. How does the legal and regulatory environment affect the company? Self-Test What are some qualitative factors that analysts should consider when evaluating a company’s likely future financial performance? Summary This chapter explained techniques used by investors and managers to analyze financial statements. The key concepts covered are listed below. • • • • • • • Liquidity ratios show the relationship of a firm’s current assets to its current liabilities and thus its ability to meet maturing debts. Two commonly used liquidity ratios are the current ratio and the quick, or acid test, ratio. Asset management ratios measure how effectively a firm is managing its assets. These ratios include inventory turnover, days sales outstanding, fixed assets turnover, and total assets turnover. Debt management ratios reveal (1) the extent to which the firm is financed with debt and (2) its likelihood of defaulting on its debt obligations. They include the debt ratio, the times-interest-earned ratio, and the EBITDA coverage ratio. Profitability ratios show the combined effects of liquidity, asset management, and debt management policies on operating results. They include the net profit margin (also called the profit margin on sales), the basic earning power ratio, the return on total assets, and the return on common equity. Market value ratios relate the firm’s stock price to its earnings, cash flow, and book value per share, thus giving management an indication of what investors think of the company’s past performance and future prospects. These include the price/earnings ratio, the price/cash flow ratio, and the market/book ratio. Trend analysis, in which one plots a ratio over time, is important because it reveals whether the firm’s condition has been improving or deteriorating over time. The Du Pont system is designed to show how the profit margin on sales, the assets turnover ratio, and the use of debt all interact to determine the rate of Chapter 3: Analysis of Financial Statements • 111 return on equity. The firm’s management can use the Du Pont system to analyze ways of improving performance. Benchmarking is the process of comparing a particular company with a group of similar successful companies. Ratio analysis has limitations, but when used with care and judgment it can be very helpful. Questions (3–1) Define each of the following terms: a. Liquidity ratios: current ratio; quick, or acid test, ratio b. Asset management ratios: inventory turnover ratio; days sales outstanding (DSO); fixed assets turnover ratio; total assets turnover ratio c. Financial leverage ratios: debt ratio; times-interest-earned (TIE) ratio; coverage ratio d. Profitability ratios: profit margin on sales; basic earning power (BEP) ratio; return on total assets (ROA); return on common equity (ROE) e. Market value ratios: price/earnings (P/E) ratio; price/cash flow ratio; market/ book (M/B) ratio; book value per share f. Trend analysis; comparative ratio analysis; benchmarking g. Du Pont equation; window dressing; seasonal effects on ratios (3–2) Financial ratio analysis is conducted by managers, equity investors, long-term creditors, and short-term creditors. What is the primary emphasis of each of these groups in evaluating ratios? (3–3) Over the past year, M. D. Ryngaert & Co. has realized an increase in its current ratio and a drop in its total assets turnover ratio. However, the company’s sales, quick ratio, and fixed assets turnover ratio have remained constant. What explains these changes? (3–4) Profit margins and turnover ratios vary from one industry to another. What differences would you expect to find between a grocery chain such as Safeway and a steel company? Think particularly about the turnover ratios, the profit margin, and the Du Pont equation. (3–5) How might (a) seasonal factors and (b) different growth rates distort a comparative ratio analysis? Give some examples. How might these problems be alleviated? (3–6) Why is it sometimes misleading to compare a company’s financial ratios with those of other firms that operate in the same industry? Self-Test Problems (ST–1) Debt Ratio Solutions Appear in Appendix A Argent Corporation had earnings per share of $4 last year, and it paid a $2 dividend. Total retained earnings increased by $12 million during the year, and book value per share at year-end was $40. Argent has no preferred stock, and no new common stock was issued during the year. If Argent’s year-end debt (which equals its total liabilities) was $120 million, what was the company’s year-end debt/assets ratio? 112 Part 1: Fundamental Concepts of Corporate Finance (ST–2) The following data apply to Jacobus and Associates (millions of dollars): Ratio Analysis Cash and marketable securities Fixed assets Sales Net income Quick ratio Current ratio DSO ROE $ 100.00 $ 283.50 $1,000.00 $ 50.00 2.0 3.0 40.55 days 12% Jacobus has no preferred stock—only common equity, current liabilities, and longterm debt. a. Find Jacobus’s (1) accounts receivable, (2) current liabilities, (3) current assets, (4) total assets, (5) ROA, (6) common equity, and (7) long-term debt. b. In part a, you should have found Jacobus’s accounts receivable = $111.1 million. If Jacobus could reduce its DSO from 40.55 days to 30.4 days while holding other things constant, how much cash would it generate? If this cash were used to buy back common stock (at book value), thus reducing the amount of common equity, how would this affect (1) the ROE, (2) the ROA, and (3) the ratio of total debt to total assets? Problems Answers Appear in Appendix B EASY PROBLEMS 1–5 (3–1) Days Sales Outstanding (3–2) Debt Ratio (3–3) Market/Book Ratio (3–4) Price/Earnings Ratio (3–5) ROE Greene Sisters has a DSO of 20 days. The company’s average daily sales are $20,000. What is the level of its accounts receivable? Assume there are 365 days in a year. Vigo Vacations has an equity multiplier of 2.5. The company’s assets are financed with some combination of long-term debt and common equity. What is the company’s debt ratio? Winston Washers’s stock price is $75 per share. Winston has $10 billion in total assets. Its balance sheet shows $1 billion in current liabilities, $3 billion in long-term debt, and $6 billion in common equity. It has 800 million shares of common stock outstanding. What is Winston’s market/book ratio? A company has an EPS of $1.50, a cash flow per share of $3.00, and a price/cash flow ratio of 8.0. What is its P/E ratio? Needham Pharmaceuticals has a profit margin of 3% and an equity multiplier of 2.0. Its sales are $100 million and it has total assets of $50 million. What is its ROE? INTERMEDIATE PROBLEMS 6–10 (3–6) Du Pont Analysis (3–7) Current and Quick Ratios Donaldson & Son has an ROA of 10%, a 2% profit margin, and a return on equity equal to 15%. What is the company’s total assets turnover? What is the firm’s equity multiplier? Ace Industries has current assets equal to $3 million. The company’s current ratio is 1.5, and its quick ratio is 1.0. What is the firm’s level of current liabilities? What is the firm’s level of inventories? Chapter 3: Analysis of Financial Statements (3–8) Profit Margin and Debt Ratio 113 Assume you are given the following relationships for the Clayton Corporation: Sales/total assets Return on assets (ROA) Return on equity (ROE) 1.5 3% 5% Calculate Clayton’s profit margin and debt ratio. (3–9) Current and Quick Ratios (3–10) Times-Interest-Earned Ratio The Nelson Company has $1,312,500 in current assets and $525,000 in current liabilities. Its initial inventory level is $375,000, and it will raise funds as additional notes payable and use them to increase inventory. How much can Nelson’s short-term debt (notes payable) increase without pushing its current ratio below 2.0? What will be the firm’s quick ratio after Nelson has raised the maximum amount of short-term funds? The Manor Corporation has $500,000 of debt outstanding, and it pays an interest rate of 10% annually: Manor’s annual sales are $2 million, its average tax rate is 30%, and its net profit margin on sales is 5%. If the company does not maintain a TIE ratio of at least 5 to 1, then its bank will refuse to renew the loan and bankruptcy will result. What is Manor’s TIE ratio? CHALLENGING PROBLEMS 11–14 (3–11) Balance Sheet Analysis Complete the balance sheet and sales information in the table that follows for Hoffmeister Industries using the following financial data: Debt ratio: 50% Quick ratio: 0.80 Total assets turnover: 1.5 Days sales outstanding: 36.5 daysa Gross profit margin on sales: (Sales – Cost of goods sold)/Sales = 25% Inventory turnover ratio: 5.0 a Calculation is based on a 365-day year. Balance Sheet Cash Accounts receivable Inventories Fixed assets Total assets Sales (3–12) Comprehensive Ratio Calculations (3–13) Comprehensive Ratio Analysis $300,000 Accounts payable Long-term debt Common stock Retained earnings Total liabilities and equity Cost of goods sold 60,000 97,500 The Kretovich Company had a quick ratio of 1.4, a current ratio of 3.0, an inventory turnover of 6 times, total current assets of $810,000, and cash and marketable securities of $120,000. What were Kretovich’s annual sales and its DSO? Assume a 365-day year. Data for Morton Chip Company and its industry averages follow. a. b. c. d. Calculate the indicated ratios for Morton. Construct the extended Du Pont equation for both Morton and the industry. Outline Morton’s strengths and weaknesses as revealed by your analysis. Suppose Morton had doubled its sales as well as its inventories, accounts receivable, and common equity during 2010. How would that information affect the validity of your ratio analysis? (Hint: Think about averages and the effects of rapid growth on ratios if averages are not used. No calculations are needed.) 114 Part 1: Fundamental Concepts of Corporate Finance M o r t o n C h i p C o m p a n y : B alanc e S heet a s of D e c e m b e r 3 1 , 2 0 1 0 (Thousan ds of D ollars) Cash Receivables Inventories Total current assets Net fixed assets $ 77,500 336,000 241,500 $655,000 292,500 Total assets $947,500 Accounts payable Notes payable Other current liabilities Total current liabilities Long-term debt Common equity Total liabilities and equity $129,000 84,000 117,000 $330,000 256,500 361,000 $947,500 Morton Chi p C ompany: Income Statement for Year Ended December 31, 2010 (Thousands of Dollars) Sales Cost of goods sold Selling, general, and administrative expenses Earnings before interest and taxes (EBIT) Interest expense Earnings before taxes (EBT) Federal and state income taxes (40%) Net income Ratio Current assets/Current liabilities Days sales outstandinga Sales/Inventory Sales/Fixed assets Sales/Total assets Net income/Sales Net income/Total assets Net income/Common equity Total debt/Total assets a (3–14) Comprehensive Ratio Analysis M or ton $1,607,500 1,392,500 145,000 $ 70,000 24,500 $ 45,500 18,200 $ 27,300 Indu str y Avera ge 2.0 35.0 days 6.7 12.1 3.0 1.2% 3.6% 9.0% 60.0% Calculation is based on a 365-day year. The Jimenez Corporation’s forecasted 2011 financial statements follow, along with some industry average ratios. a. Calculate Jimenez’s 2011 forecasted ratios, compare them with the industry average data, and comment briefly on Jimenez’s projected strengths and weaknesses. b. What do you think would happen to Jimenez’s ratios if the company initiated cost-cutting measures that allowed it to hold lower levels of inventory and substantially decreased the cost of goods sold? No calculations are necessary: Think about which ratios would be affected by changes in these two accounts. Chapter 3: Analysis of Financial Statements Jimenez Corporati on: Foreca sted Balance Sheet as of De ce mb er 3 1, 201 1 Assets Cash Accounts receivable Inventories Total current assets Fixed assets Total assets Liabilities and Equity Accounts and notes payable Accruals Total current liabilities Long-term debt Common stock Retained earnings Total liabilities and equity $ 72,000 439,000 894,000 $1,405,000 431,000 $1,836,000 $ 432,000 170,000 $ 602,000 404,290 575,000 254,710 $1,836,000 J i m e n e z C o r p o r at i on: F or ec as t ed Income Statement for 2011 Sales Cost of goods sold Selling, general, and administrative expenses Depreciation Earnings before taxes (EBT) Taxes (40%) Net income Per Share Data EPS Cash dividends per share P/E ratio Market price (average) Number of shares outstanding Industry Financial Ratios (2010)a Quick ratio Current ratio Inventory turnoverb Days sales outstandingc Fixed assets turnoverb Total assets turnoverb Return on assets Return on equity Debt ratio Profit margin on sales P/E ratio Price/Cash flow ratio a Industry average ratios have been constant for the past 4 years. Based on year-end balance sheet figures. c Calculation is based on a 365-day year. b $4,290,000 3,580,000 370,320 159,000 $ 180,680 72,272 $ 108,408 $ $ $ 4.71 0.95 5 23.57 23,000 1.0 2.7 7.0 32 days 13.0 2.6 9.1% 18.2% 50.0% 3.5% 6.0 3.5 115 116 Part 1: Fundamental Concepts of Corporate Finance SPREADSHEET PROBLEM (3-15) Build a Model: Ratio Analysis resource Start with the partial model in the file Ch03 P15 Build a Model.xls from the textbook’s Web site. Joshua & White (J&W) Technologies’s financial statements are also shown below. Answer the following questions. (Note: Industry average ratios are provided in Ch03 P15 Build a Model.xls.) a. b. c. d. Has J&W’s liquidity position improved or worsened? Explain. Has J&W’s ability to manage its assets improved or worsened? Explain. How has J&W’s profitability changed during the last year? Perform an extended Du Pont analysis for J&W for 2009 and 2010. What do these results tell you? e. Perform a common size analysis. What has happened to the composition (that is, percentage in each category) of assets and liabilities? f. Perform a percentage change analysis. What does this tell you about the change in profitability and asset utilization? J o s h u a & W h i t e T e c h n o l o g i e s : D e c e m b e r 31 B a l a n c e S h e e t s (Thousan ds of D ollars) Assets Cash and cash equivalents Short-term investments Accounts receivable Inventories Total current assets 2010 2009 $ 21,000 $ 20,000 3,759 3,240 52,500 48,000 84,000 56,000 $161,259 $127,240 Net fixed assets 218,400 Total assets 200,000 $379,659 $327,240 L i a bi l i t i e s & Equ i ty 2010 2009 $ 33,600 $ 32,000 Accruals 12,600 12,000 Notes payable 19,929 6,480 $ 66,129 $ 50,480 67,662 58,320 Total liabilities $133,791 $108,800 Accounts payable Total current liabilities Long-term debt Common stock Retained earnings Total common equity Total liabilities & equity 183,793 178,440 62,075 40,000 $245,868 $218,440 $379,659 $327,240 Joshua & White Technologies December 31 Income Statements (Thousan ds of D ollars) Sales Expenses excluding depr. & amort. EBITDA Depreciation and amortization EBIT 2010 2009 $420,000 327,600 $ 92,400 19,660 $ 72,740 $400,000 320,000 $ 80,000 18,000 $ 62,000 Chapter 3: Analysis of Financial Statements 117 2010 2009 Interest expense EBT Taxes (40%) Net income 5,740 $67,000 26,800 $40,200 4,460 $57,540 23,016 $34,524 Common dividends $18,125 $17,262 O th er Dat a 2010 200 9 $ 90.00 4,052 $20,000 $ 0 $ 96.00 4,000 $20,000 $ 0 Year-end stock price Number of shares (Thousands) Lease payment (Thousands of Dollars) Sinking fund payment (Thousands of Dollars) THOMSON ONE Business School Edition Problem Use the Thomson ONE—Business School Edition online database to work this chapter’s questions. ANALYSIS OF FORD’S FINANCIAL STATEMENTS WITH THOMSON ONE—BUSINESS SCHOOL EDITION Use Thomson ONE to analyze Ford Motor Company. Enter Ford’s ticker symbol (F) and select GO. By selecting the tab at the top labeled Financials, you can find Ford’s key financial statements for the past several years. At the Financials screen on the second line of tabs, select the Fundamental Ratios tab. If you then select the SEC Database Ratios from the pull-down menu, you can select either annual or quarterly ratios. Under annual ratios, there is an in-depth summary of Ford’s various ratios over the past three years. Click on the Peers tab (on the first line of tabs) near the top of the screen for a summary of financial information for Ford and a few of its peers. If you click on the Peer Sets tab (second line of tabs), you can modify the list of peer firms. The default setup is “Peers set by SIC Code.” To obtain a comparison of many of the key ratios presented in the text, just click on Financials (second line of tabs) and select Key Financial Ratios from the drop-down menu. Thomson ONE—BSE Discussion Questions 1. What has happened to Ford’s liquidity position over the past 3 years? How does Ford’s liquidity compare with its peers? (Hint: You may use both the peer key financial ratios and liquidity comparison to answer this question.) 2. Take a look at Ford’s inventory turnover ratio. How does this ratio compare with its peers? Have there been any interesting changes over time in this measure? Do you consider Ford’s inventory management to be a strength or a weakness? 3. Construct a simple Du Pont analysis for Ford and its peers. What are Ford’s strengths and weaknesses relative to its competitors? 118 Part 1: Fundamental Concepts of Corporate Finance Mini Case The first part of the case, presented in Chapter 2, discussed the situation of Computron Industries after an expansion program. A large loss occurred in 2010, rather than the expected profit. As a result, its managers, directors, and investors are concerned about the firm’s survival. Donna Jamison was brought in as assistant to Fred Campo, Computron’s chairman, who had the task of getting the company back into a sound financial position. Computron’s 2009 and 2010 balance sheets and income statements, together with projections for 2011, are shown in the following tables. The tables also show the 2009 and 2010 financial ratios, along with industry average data. The 2011 projected financial statement data represent Jamison’s and Campo’s best guess for 2011 results, assuming that some new financing is arranged to get the company “over the hump.” Balance Sheets 20 09 20 10 20 11 E A s s et s Cash Short-term investments Accounts receivable Inventories Total current assets Gross fixed assets Less: Accumulated depreciation Net fixed assets Total assets $ 9,000 48,600 351,200 715,200 $1,124,000 491,000 146,200 $ 344,800 $1,468,800 $ 7,282 20,000 632,160 1,287,360 $ 1,946,802 1,202,950 263,160 $ 939,790 $ 2,886,592 $ 14,000 71,632 878,000 1,716,480 $2,680,112 1,220,000 383,160 $ 836,840 $3,516,952 Liabilities and Equity Accounts payable Notes payable Accruals Total current liabilities Long-term debt Common stock (100,000 shares) Retained earnings Total equity Total liabilities and equity $ 145,600 200,000 136,000 $ 481,600 323,432 460,000 203,768 $ 663,768 $1,468,800 $ 324,000 720,000 284,960 $ 1,328,960 1,000,000 460,000 97,632 $ 557,632 $ 2,886,592 $ 359,800 300,000 380,000 $1,039,800 500,000 1,680,936 296,216 $1,977,152 $3,516,952 Note: “E” denotes “estimated”; the 2011 data are forecasts. Jamison must prepare an analysis of where the company is now, what it must do to regain its financial health, and what actions should be taken. Your assignment is to help her answer the following questions. Provide clear explanations, not yes or no answers. a. Why are ratios useful? What three groups use ratio analysis and for what reasons? b. Calculate the 2011 current and quick ratios based on the projected balance sheet and income statement data. What can you say about the company’s liquidity position in 2009, 2010, and as projected for 2011? We often think of ratios as being useful (1) to managers to help run the business, (2) to bankers for credit analysis, and (3) to stockholders for stock valuation. Would these different types of analysts have an equal interest in the liquidity ratios? c. Calculate the 2011 inventory turnover, days sales outstanding (DSO), fixed assets turnover, and total assets turnover. How does Computron’s utilization of assets stack up against that of other firms in its industry? Chapter 3: Analysis of Financial Statements 119 I n c o m e S ta t e m e n t s 2 00 9 Sales Cost of goods sold Other expenses Depreciation Total operating costs EBIT Interest expense EBT Taxes (40%) Net income Other Data Stock price Shares outstanding EPS DPS Tax rate Book value per share Lease payments 20 10 $3,432,000 2,864,000 340,000 18,900 $3,222,900 $ 209,100 62,500 $ 146,600 58,640 $ 87,960 $ $ $ $ $ $5,834,400 4,980,000 720,000 116,960 $5,816,960 $ 17,440 176,000 ($ 158,560) (63,424) ($ 95,136) 8.50 100,000 0.880 0.220 40% 6.638 40,000 $ 6.00 100,000 ($ 0.951) 0.110 40% $ 5.576 $ 40,000 2 01 1E $7,035,600 5,800,000 612,960 120,000 $6,532,960 $ 502,640 80,000 $ 422,640 169,056 $ 253,584 $ 12.17 250,000 $ 1.014 0.220 40% $ 7.909 $ 40,000 Note: “E” denotes “estimated”; the 2011 data are forecasts. R at i o A n a l y s i s 20 09 Current Quick Inventory turnover Days sales outstanding Fixed assets turnover Total assets turnover Debt ratio TIE EBITDA coverage Profit margin Basic earning power ROA ROE Price/Earnings (P/E) Price/Cash flow Market/Book 2.3 0.8 4.8 37.3 10.0 2.3 54.8% 3.3 2.6 2.6% 14.2% 6.0% 13.3% 9.7 8.0 1.3 2 01 0 1.5 0.5 4.5 39.6 6.2 2.0 80.7% 0.1 0.8 −1.6% 0.6% −3.3% −17.1% −6.3 27.5 1.1 20 11 E I nd us tr y A v er ag e 2.7 1.0 6.1 32.0 7.0 2.5 50.0% 6.2 8.0 3.6% 17.8% 9.0% 17.9% 16.2 7.6 2.9 Note: “E” denotes “estimated.” d. Calculate the 2011 debt, times-interest-earned, and EBITDA coverage ratios. How does Computron compare with the industry with respect to financial leverage? What can you conclude from these ratios? e. Calculate the 2011 profit margin, basic earning power (BEP), return on assets (ROA), and return on equity (ROE). What can you say about these ratios? f. Calculate the 2011 price/earnings ratio, price/cash flow ratio, and market/book ratio. Do these ratios indicate that investors are expected to have a high or low opinion of the company? 120 Part 1: Fundamental Concepts of Corporate Finance g. Perform a common size analysis and percentage change analysis. What do these analyses tell you about Computron? h. Use the extended Du Pont equation to provide a summary and overview of Computron’s financial condition as projected for 2011. What are the firm’s major strengths and weaknesses? i. What are some potential problems and limitations of financial ratio analysis? j. What are some qualitative factors that analysts should consider when evaluating a company’s likely future financial performance? Selected Additional Cases The following cases from Textchoice, Cengage Learning’s online library, cover many of the concepts discussed in this chapter and are available at http://www.textchoice2.com. Klein-Brigham Series: Case 35, “Mark X Company (A),” which illustrates the use of ratio analysis in the evaluation of a firm’s existing and potential financial positions; Case 36, “Garden State Container Corporation,” which is similar in content to Case 35; Case 51, “Safe Packaging Corporation,” which updates Case 36; Case 68, “Sweet Dreams Inc.,” which also updates Case 36; and Case 71, “Swan-Davis, Inc.,” which illustrates how financial analysis—based on both historical statements and forecasted statements—is used for internal management and lending decisions. PART 2 Fixed Income Securities Chapter 4 Time Value of Money Chapter 5 Bonds, Bond Valuation, and Interest Rates 121 This page intentionally left blank CHAPTER 4 Time Value of Money W hen you graduate and go to work, either a defined benefit (DB) or a defined contribution (DC) pension plan will almost certainly be part of your compensation package. Under a DB plan, the company will put funds into its pension fund, which will then invest in stocks, bonds, real estate, and so forth and then use those funds to make the promised payments after you retire. Under a DC plan, the company will put money into your 401(k) plan (which is essentially a mutual fund), you will decide what type of assets to buy, and you will withdraw money after you retire. The analysis required to set up a good retirement program is based on the subject of this chapter, the time value of money (TVM). How do you suppose a stock market crash like we had in 2008, with the average stock down about 40%, will affect DB and DC retirement plans? If you have a 401(k) plan that holds stocks, as most people do, TVM analysis would show clearly that you will have to work longer than you expected, reduce your post-retirement standard of living, or both. With a DB plan, a stock market decline reduces the value of the investments set aside for you by the company. If there is also a decline in interest rates, as there was in 2008, TVM analysis shows that the amount of money the company should set aside for you goes up. Thus, the company’s pension funding status, which is the difference between the value of the pension plan’s investments and the amount the plan should have on hand to cover the future obligations, becomes severely underfunded if the market crashes and interest rates fall. This can even lead to bankruptcy, in which case you might end up with retirement payments from the government instead of from the company, with the government’s payments a lot lower than those promised by the company’s plan. If you don’t believe us, ask someone who recently retired from a bankrupt airline or auto company.1 If you want to see something alarming, apply the procedures set forth in this chapter to the pension plan of almost any municipal government. Politicians find it a lot easier to promise high future benefits than to raise taxes to pay for those benefits. Of course, the federal government is doing the same thing with Social Security, Medicare, and Medicaid, and with federal employees. Politicians need to study this chapter! 123 1 124 Part 2: Fixed Income Securities Corporate Valuation and the Time Value of Money In Chapter 1 we explained (1) that managers should strive to make their firms more valuable and (2) that the value of a firm is determined by the size, timing, and risk of its free cash flows (FCF). Recall from Chapter 2 that free cash flows are the cash flows available for distribution to all of a firm’s investors (stockholders and creditors). We explain how to calculate the weighted average cost of capital (WACC) in Chapter 9, but it is enough for now to think of the WACC as the average rate of return required by all of the firm’s investors. The intrinsic value of a company is given by the following diagram. Net operating profit after taxes Free cash flow (FCF) Value = FCF1 (1 + WACC)1 Required investments in operating capital − + FCF2 (1 + WACC)2 = + …+ FCF∞ (1 + WACC)∞ Weighted average cost of capital (WACC) Market interest rates Market risk aversion Cost of debt Cost of equity Firm’s debt/equity mix Firm’s business risk In Chapter 1 we saw that the primary objective of financial management is to maximize the intrinsic value of a firm’s stock. We also saw that stock values depend on the timing of the cash flows investors expect from an investment—a dollar expected sooner is worth more than a dollar expected further in the future. Therefore, it is essential for financial managers to understand the time value of money and its impact on stock prices. In this chapter we will explain exactly how the timing of cash flows affects asset values and rates of return. The principles of time value analysis have many applications, including retirement planning, loan payment schedules, and decisions to invest (or not) in new equipment. In fact, of all the concepts used in finance, none is more important than the time value of money (TVM), also called discounted cash flow (DCF) analysis. Time value concepts are used throughout the remainder of the book, so it is vital that you understand the material in Chapter 4 and be able to work the chapter’s problems before you move on to other topics.2 The problems can be worked with either a calculator or an Excel spreadsheet. Calculator manuals tend to be long and complicated, partly because they cover a number of topics that aren’t used in the basic finance course. Therefore, on this textbook’s Web site we provide tutorials for the most commonly used calculators. The tutorials are keyed to this chapter, and they show exactly how to do the calculations used in the chapter. If you don’t know how to use your calculator, go to the Web site, get the relevant tutorial, and go through it as you study the chapter. The chapter’s Tool Kit also explains how to do all of the within-chapter calculations using Excel. The Tool Kit, along with an Excel tutorial designed for this book, is provided on the book’s Web site. 2 Chapter 4: Time Value of Money 125 4.1 TIME LINES resource The textbook’s Web site contains an Excel file that will guide you through the chapter’s calculations. The file for this chapter is Ch04 Tool Kit.xls, and we encourage you to open the file and follow along as you read the chapter. Self-Test The first step in a time value analysis is to set up a time line to help you visualize what’s happening in the particular problem. To illustrate, consider the following diagram, where PV represents $100 that is in a bank account today and FV is the value that will be in the account at some future time (3 years from now in this example): Periods Cash 0 PV=$100 5% 1 2 3 FV=? The intervals from 0 to 1, 1 to 2, and 2 to 3 are time periods such as years or months. Time 0 is today, and it is the beginning of Period 1; Time 1 is one period from today, and it is both the end of Period 1 and the beginning of Period 2; and so on. In our example the periods are years, but they could also be quarters or months or even days. Note again that each tick mark corresponds to both the end of one period and the beginning of the next one. Thus, if the periods are years, the tick mark at Time 2 represents both the end of Year 2 and the beginning of Year 3. Cash flows are shown directly below the tick marks, and the relevant interest rate is shown just above the time line. Unknown cash flows, which you are trying to find, are indicated by question marks. Here the interest rate is 5%; a single cash outflow, $100, is invested at Time 0; and the Time-3 value is unknown and must be found. In this example, cash flows occur only at Times 0 and 3, with no flows at Times 1 or 2. We will, of course, deal with situations where multiple cash flows occur. Note also that in our example the interest rate is constant for all 3 years. The interest rate is generally held constant, but if it varies then in the diagram we show different rates for the different periods. Time lines are especially important when you are first learning time value concepts, but even experts use them to analyze complex problems. Throughout the book, our procedure is to set up a time line to show what’s happening, provide an equation that must be solved to find the answer, and then explain how to solve the equation with a regular calculator, a financial calculator, and a computer spreadsheet. Do time lines deal only with years, or could other periods be used? Set up a time line to illustrate the following situation: You currently have $2,000 in a 3-year certificate of deposit (CD) that pays a guaranteed 4% annually. You want to know the value of the CD after 3 years. 4.2 FUTURE VALUES A dollar in hand today is worth more than a dollar to be received in the future—if you had the dollar now you could invest it, earn interest, and end up with more than one dollar in the future. The process of going forward, from present values (PVs) to future values (FVs), is called compounding. To illustrate, refer back to our 3-year time line and assume that you have $100 in a bank account that pays a guaranteed 5% interest each year. How much would you have at the end of Year 3? We first define some terms, after which we set up a time line and show how the future value is calculated. 126 Part 2: Fixed Income Securities PV = Present value, or beginning amount. In our example, PV = $100. FVN = Future value, or ending amount, in the account after N periods. Whereas PV is the value now, or the present value, FVN is the value N periods into the future, after interest earned has been added to the account. CFt = Cash flow. Cash flows can be positive or negative. For a borrower, the first cash flow is positive and the subsequent cash flows are negative, and the reverse holds for a lender. The cash flow for a particular period is often given a subscript, CFt, where t is the period. Thus, CF0 = PV = the cash flow at Time 0, whereas CF3 would be the cash flow at the end of Period 3. In this example the cash flows occur at the ends of the periods, but in some problems they occur at the beginning. I = Interest rate earned per year. (Sometimes a lowercase i is used.) Interest earned is based on the balance at the beginning of each year, and we assume that interest is paid at the end of the year. Here I = 5% or, expressed as a decimal, 0.05. Throughout this chapter, we designate the interest rate as I (or I/YR, for interest rate per year) because that symbol is used on most financial calculators. Note, though, that in later chapters we use the symbol “r” to denote the rate because r (for rate of return) is used more often in the finance literature. Also, in this chapter we generally assume that interest payments are guaranteed by the U.S. government and hence are riskless (i.e., certain). In later chapters we will deal with risky investments, where the rate actually earned might be different from its expected level. INT = Dollars of interest earned during the year = (Beginning amount) × I. In our example, INT = $100(0.05) = $5 for Year 1, but it rises in subsequent years as the amount at the beginning of each year increases. N = Number of periods involved in the analysis. In our example, N = 3. Sometimes the number of periods is designated with a lowercase n, so both N and n indicate number of periods. We can use four different procedures to solve time value problems.3 These methods are described next. Step-by-Step Approach The time line itself can be modified and used to find the FV of $100 compounded for 3 years at 5%, as shown below: Time Amount at beginning of period 0 $100.00 5% 1 2 3 $105.00 $110.25 $115.76 3 A fifth procedure is called the tabular approach, which uses tables that provide “interest factors;” this procedure was used before financial calculators and computers became available. Now, though, calculators and spreadsheets such as Excel are programmed to calculate the specific factor needed for a given problem, which is then used to find the FV. This is much more efficient than using the tables. Also, calculators and spreadsheets can handle fractional periods and fractional interest rates. For these reasons, tables are not used in business today; hence we do not discuss them in the text. However, because some professors cover the tables for pedagogic purposes, we discuss them in Web Extension 4A, on the textbook’s Web site. Chapter 4: Time Value of Money 127 We start with $100 in the account, which is shown at t = 0. We then multiply the initial amount, and each succeeding beginning-of-year amount, by (1 + I) = (1.05). • You earn $100(0.05) = $5 of interest during the first year, so the amount at the end of Year 1 (or at t = 1) is FV1 ¼ PV þ INT ¼ PV þ PVðIÞ ¼ PVð1 þ IÞ ¼ $100ð1 þ 0:05Þ ¼ $100ð1:05Þ ¼ $105 • • • We begin the second year with $105, earn 0.05($105) = $5.25 on the now larger beginning-of-period amount, and end the year with $110.25. Interest during Year 2 is $5.25, and it is higher than the first year’s interest, $5, because we earned $5(0.05) = $0.25 interest on the first year’s interest. This is called “compounding,” and interest earned on interest is called “compound interest.” This process continues, and because the beginning balance is higher in each successive year, the interest earned each year increases. The total interest earned, $15.76, is reflected in the final balance, $115.76. The step-by-step approach is useful because it shows exactly what is happening. However, this approach is time-consuming, especially if the number of years is large and you are using a calculator rather than Excel, so streamlined procedures have been developed. Formula Approach In the step-by-step approach, we multiplied the amount at the beginning of each period by (1 + I) = (1.05). Notice that the value at the end of Year 2 is FV2 ¼ FV1 ð1 þ IÞ ¼ PVð1 þ IÞð1 þ IÞ ¼ PVð1 þ IÞ2 ¼ 100ð1:05Þ2 ¼ $110:25 If N = 3, then we multiply PV by (1 + I) three different times, which is the same as multiplying the beginning amount by (1 + I)3. This concept can be extended, and the result is this key equation: FVN ¼ PVð1 þ IÞN (4-1) We can apply Equation 4-1 to find the FV in our example: FV3 ¼ $100ð1:05Þ3 ¼ $115:76 Equation 4-1 can be used with any calculator, even a nonfinancial calculator that has an exponential function, making it easy to find FVs no matter how many years are involved. Financial Calculators Financial calculators were designed specifically to solve time value problems. First, note that financial calculators have five keys that correspond to the five variables in the basic time value equations. Equation 4-1 has only four variables, but we will shortly deal with situations where a fifth variable (a set of periodic additional 128 Part 2: Fixed Income Securities payments) is involved. We show the inputs for our example above their keys in the following diagram, and the output, which is the FV, below its key. Since in this example there are no periodic payments, we enter 0 for PMT. We describe the keys in more detail below the diagram. Inputs: Output: 3 5 –100 0 N I/YR PV PMT FV 115.76 N = Number of periods = 3. Some calculators use n rather than N. I/YR = Interest rate per period = 5. Some calculators use i or I rather than I/YR. Calculators are programmed to automatically convert the 5 to the decimal 0.05 before doing the arithmetic. PV = Present value = 100. In our example we begin by making a deposit, which is an outflow of 100, so the PV is entered with a negative sign. On most calculators you must enter the 100, then press the +/– key to switch from +100 to –100. If you enter –100 directly, this will subtract 100 from the last number in the calculator, which will give you an incorrect answer unless the last number was zero. PMT = Payment. This key is used if we have a series of equal, or constant, payments. Since there are no such payments in our current problem, we enter PMT = 0. We will use the PMT key later in this chapter. FV = Future value. In our example, the calculator automatically shows the FV as a positive number because we entered the PV as a negative number. If we had entered the 100 as a positive number, then the FV would have been negative. Calculators automatically assume that either the PV or the FV must be negative. As noted in our example, you first enter the four known values (N, I/YR, PMT, and PV) and then press the FV key to get the answer, FV = 115.76. Spreadsheets resource See Ch04 Tool Kit.xls for all calculations. Spreadsheets are ideally suited for solving many financial problems, including those dealing with the time value of money.4 Spreadsheets are obviously useful for calculations, but they can also be used like a word processor to create exhibits like our Figure 4-1, which includes text, drawings, and calculations. We use this figure to show that four methods can be used to find the FV of $100 after 3 years at an interest rate of 5%. The time line on Rows 43 to 45 is useful for visualizing the problem, after which the spreadsheet calculates the required answer. Note that the letters across the top designate columns, the numbers down the left column designate rows, and the rows and columns jointly designate cells. Thus, cell C39 shows the amount of the investment, $100, and it is given a minus sign because it is an outflow. The file Ch04 Tool Kit.xls on the book’s Web site does the calculations in the chapter using Excel. We highly recommend that you go through this Tool Kit. This will give you practice with Excel, and that will help tremendously in later courses, in the job market, and in the workplace. Also, going through the models will improve your understanding of financial concepts. 4 Chapter 4: Time Value of Money 129 Hints on Using Financial Calculators When using a financial calculator, make sure your machine is set up as indicated below. Refer to your calculator manual or to our calculator tutorial on the text’s Web site for information on setting up your calculator. ◆ One payment per period. Many calculators “come out of the box” assuming that 12 payments are made per year; that is, they assume monthly payments. However, in this book we generally deal with problems in which only one payment is made each year. Therefore, you should set your calculator at one payment per year and leave it there. See our tutorial or your calculator manual if you need assistance. We will show you how to solve problems with more than 1 payment per year in Section 4.15. ◆ End mode. With most contracts, payments are Notice that for reasonable values of I, either PV or FVN must be negative, and the other one must be positive to make the equation equal 0. This is reasonable because, in all realistic situations, one cash flow is an outflow (which should have a negative sign) and one is an inflow (which should have a positive sign). For example, if you make a deposit (which is an outflow, and hence should have a negative sign) then you will expect to make a later withdrawal (which is an inflow with a positive sign). The bottom line is that one of your inputs for a cash flow must be negative and one must be positive. This generally means typing the outflow as a positive number and then pressing the +/– key to convert from + to – before hitting the enter key. made at the end of each period. However, some contracts call for payments at the beginning of each period. You can switch between “End Mode” and “Begin Mode” depending on the problem you are solving. Because most of the problems in this book call for end-of-period payments, you should return your calculator to End Mode after you work a problem in which payments are made at the beginning of periods. ◆ Decimal places. When doing arithmetic, calcula- ◆ Negative sign for outflows. When first learning ◆ Interest rates. For arithmetic operations with a how to use financial calculators, students often forget that one cash flow must be negative. Mathematically, financial calculators solve a version of this equation: nonfinancial calculator, the rate 5.25% must be stated as a decimal, .0525. However, with a financial calculator you must enter 5.25, not .0525, because financial calculators are programmed to assume that rates are stated as percentages. PVð1 þ IÞN þ FVN ¼ 0 (4-2) tors use a great many decimal places. However, they allow you to show from 0 to 11 decimal places on the display. When working with dollars, we generally specify two decimal places. When dealing with interest rates, we generally specify two places if the rate is expressed as a percentage, like 5.25%, but we specify four places if the rate is expressed as a decimal, like 0.0525. It is useful to put all of the problem’s inputs in a section of the spreadsheet designated “Inputs.” In Figure 4-1 we put the inputs in the range A38:C41, with C39 being the cell where we specify the investment, C40 the interest rate, and C41 the number of periods. We can use these three cell references, rather than the fixed numbers themselves, in the formulas in the remainder of the model. This makes it easy to modify the problem by changing the inputs and then having the new data automatically used in the calculations. Time lines are important for solving finance problems because they help us visualize what’s happening. When we work a problem by hand we usually draw a time line, and when we work a problem with Excel, we actually set the model up as a time line. For example, in Figure 4-1 Rows 43 to 45 are indeed a time line. It’s easy 130 Part 2: Fixed Income Securities FIGURE 4-1 Alternative Procedures for Calculating Future Values INPUTS: Investment = CF0 = PV = Interest rate = No. of periods = I = –$100.00 5.00% 3 N = Setup of the problem as a Periods: 0 5% 1 2 3 –$100 0 0 FV = ? $100 $105.00 $110.25 $115.76 = $115.76 Time Line Cash Flow: 1.Step-by-Step: Multiply $100 by (1 + I) 2. Formula: FVN = PV(1 + I)N 3. Financial Calculator: FV3 = $100(1.05)3 3 5 –$100.00 $0 N I/YR PV PMT FV $115.76 4. Excel Spreadsheet: FVN = = FV(I,N,0,PV) Fixed inputs: FVN = = FV(0.05,3,0,–100) $115.76 Cell references: FVN = = FV(C40,C41,0,C39) $115.76 FV Function: In the Excel formula, the terms are entered in the sequence: interest, periods, 0 to indicate no periodic cash flows, and then the PV. The data can be entered as fixed numbers or, better yet, as cell references. to construct time lines with Excel, with each column designating a different period on the time line. On Row 47 we use Excel to go through the step-by-step calculations, multiplying the beginning-of-year values by (1 + I) to find the compounded value at the end of each period. Cell G47 shows the final result of the step-by-step approach. We illustrate the formula approach in Row 49, using Excel to solve Equation 4-1 to find the FV. Cell G49 shows the formula result, $115.76. As it must, it equals the step-by-step result. Rows 51 to 53 illustrate the financial calculator approach, which again produces the same answer, $115.76. The last section, in Rows 55 to 58, illustrates Excel’s future value (FV) function. You can access the function wizard by clicking the fx symbol in Excel’s formula bar. Then select the category for Financial functions, and then the FV function, which is =FV(I,N,0,PV), as shown in Cell E55.5 Cell E56 shows how the formula would look with numbers as inputs; the actual function itself is entered in Cell G56, but it shows up in the table as the answer, $115.76. If you access the model and put the pointer on Cell G56, you will see the full formula. Finally, Cell E57 shows how the formula would look with cell references rather than fixed values as inputs, with All functions begin with an equal sign. The third entry is zero in this example, which indicates that there are no periodic payments. Later in this chapter we will use the FV function in situations where we have nonzero periodic payments. Also, for inputs we use our own notation, which is similar but not identical to Excel’s notation. 5 Chapter 4: Time Value of Money 131 the actual function again in Cell G57. We generally use cell references as function inputs because this makes it easy to change inputs and see how those changes affect the output. This is called “sensitivity analysis.” Many real-world financial applications use sensitivity analysis, so it is useful to get in the habit of setting up an input data section and then using cell references rather than fixed numbers in the functions. When entering interest rates in Excel, you can use either actual numbers or percentages, depending on how the cell is formatted. For example, in cell C40, we first formatted to Percentage, and then typed in 5, which showed up as 5%. However, Excel uses 0.05 for the arithmetic. Alternatively, we could have formatted C40 as a Number, in which case we would have typed “0.05.” If C40 is formatted to Number and you enter 5, then Excel would think you meant 500%. Thus, Excel’s procedure is quite different from the convention used in financial calculators. Comparing the Procedures The first step in solving any time value problem is to understand what is happening and then to diagram it on a time line. Woody Allen said that 90% of success is just showing up. With time value problems, 90% of success is correctly setting up the time line. After you diagram the problem on a time line, your next step is to pick one of the four approaches shown in Figure 4-1 to solve the problem. Any may be used, but your choice of method will depend on the particular situation. All business students should know Equation 4-1 by heart and should also know how to use a financial calculator. So, for simple problems such as finding the future value of a single payment, it is generally easiest and quickest to use either the formula approach or a financial calculator. However, for problems that involve several cash flows, the formula approach usually is time-consuming, so either the calculator or spreadsheet approach would generally be used. Calculators are portable and quick to set up, but if many calculations of the same type must be done, or if you want to see how changes in an input such as the interest rate affect the future value, then the spreadsheet approach is generally more efficient. If the problem has many irregular cash flows, or if you want to analyze alternative scenarios using different cash flows or interest rates, then the spreadsheet approach definitely is the most efficient procedure. Spreadsheets have two additional advantages over calculators. First, it is easier to check the inputs with a spreadsheet—they are visible, whereas with a calculator they are buried somewhere in the machine. Thus, you are less likely to make a mistake in a complex problem when you use the spreadsheet approach. Second, with a spreadsheet, you can make your analysis much more transparent than you can when using a calculator. This is not necessarily important when all you want is the answer, but if you need to present your calculations to others, like your boss, it helps to be able to show intermediate steps, which enables someone to go through your exhibit and see exactly what you did. Transparency is also important when you must go back, sometime later, and reconstruct what you did. You should understand the various approaches well enough to make a rational choice, given the nature of the problem and the equipment you have available. In any event, you must understand the concepts behind the calculations, and you must also know how to set up time lines in order to work complex problems. This is true for stock and bond valuation, capital budgeting, lease analysis, and many other important financial problems. 132 Part 2: Fixed Income Securities The Power of Compound Interest Assume that you are 26 and just received your MBA. After reading the introduction to this chapter, you decide to start investing in the stock market for your retirement. Your goal is to have $1 million when you retire at age 65. Assuming you earn 10% annually on your stock investments, how much must you invest at the end of each year in order to reach your goal? The answer is $2,491, but this amount depends critically on the return earned on your investments. If your return drops to 8%, the required annual contribution would rise to $4,185. On the other hand, if the return rises to 12%, you would need to put away only $1,462 per year. What if you are like most 26-year-olds and wait until later to worry about retirement? If you wait until age 40, you will need to save $10,168 per year to reach your $1 million goal, assuming you can earn 10%, but $13,679 per year if you earn only 8%. If you wait until age 50 and then earn 8%, the required amount will be $36,830 per year! Although $1 million may seem like a lot of money, it won’t be when you get ready to retire. If inflation averages 5% a year over the next 39 years, then your $1 million nest egg would be worth only $149,148 in today’s dollars. If you live for 20 years after retirement and earn a real 3% rate of return, your annual retirement income in today’s dollars would be only $9,733 before taxes. So, after celebrating your graduation and new job, start saving! Graphic View of the Compounding Process resource See Ch04 Tool Kit.xls for all calculations. Figure 4-2 shows how a $100 investment grows (or declines) over time at different interest rates. Interest rates are normally positive, but the “growth” concept is broad enough to include negative rates. We developed the curves by solving Equation 4-1 with different values for N and I. The interest rate is a growth rate: If money is deposited and earns 5% per year, then your funds will grow by 5% per year. Note also that time value concepts can be applied to anything that grows—sales, population, earnings per share, or your future salary. Also, as noted before, the “growth rate” can be negative, as was sales growth for a number of auto companies in recent years. Simple Interest versus Compound Interest As explained earlier, when interest is earned on the interest earned in prior periods, we call it compound interest. If interest is earned only on the principal, we call it simple interest. The total interest earned with simple interest is equal to the principal multiplied by the interest rate times the number of periods: PV(I)(N). The future value is equal to the principal plus the interest: FV = PV + PV(I)(N). For example, suppose you deposit $100 for 3 years and earn simple interest at an annual rate of 5%. Your balance at the end of 3 years would be: FV ¼ PV þ PVðIÞðNÞ ¼ $100 þ $100ð5%Þð3Þ ¼ $100 þ $15 ¼ $115 Notice that this is less than the $115.76 we calculated earlier using compound interest. Most applications in finance are based on compound interest, but you should be aware that simple interest is still specified in some legal documents. Self-Test Explain why this statement is true: “A dollar in hand today is worth more than a dollar to be received next year, assuming interest rates are positive." What is compounding? What would the future value of $100 be after 5 years at 10% compound interest? ($161.05) Chapter 4: Time Value of Money FIGURE 4-2 133 Growth of $100 at Various Interest Rates and Time Periods FV of $100 After N Years $600 $500 I = 20% $400 $300 I = 10% $200 I = 5% I = 0% $100 I = –20% $0 0 1 2 3 4 5 6 7 8 9 10 Years Suppose you currently have $2,000 and plan to purchase a 3-year certificate of deposit (CD) that pays 4% interest, compounded annually. How much will you have when the CD matures? ($2,249.73) How would your answer change if the interest rate were 5%, or 6%, or 20%? (Hint: With a calculator, enter N = 3, I/YR = 4, PV = −2000, and PMT = 0; then press FV to get 2,249.73. Then, enter I/YR = 5 to override the 4% and press FV again to get the second answer. In general, you can change one input at a time to see how the output changes.) ($2,315.25; $2,382.03; $3,456.00) A company’s sales in 2009 were $100 million. If sales grow by 8% annually, what will they be 10 years later? ($215.89 million) What would they be if they decline by 8% per year for 10 years? ($43.44 million) How much would $1, growing at 5% per year, be worth after 100 years? ($131.50) What would FV be if the growth rate were 10%? ($13,780.61) 4.3 PRESENT VALUES Suppose you have some extra money and want to make an investment. A broker offers to sell you a bond that will pay a guaranteed $115.76 in 3 years. Banks are currently offering a guaranteed 5% interest on 3-year certificates of deposit (CDs), and if you don’t buy the bond you will buy a CD. The 5% rate paid on the CD is defined as your opportunity cost, or the rate of return you would earn on an alternative investment of similar risk if you don’t invest in the security under consideration. Given these conditions, what’s the most you should pay for the bond? First, recall from the future value example in the last section that if you invested $100 at 5% in a CD, it would grow to $115.76 in 3 years. You would also have $115.76 after 3 years if you bought the bond. Therefore, the most you should pay for the bond is $100—this is its “fair price,” which is also its intrinsic, or fundamental, value. If you 134 Part 2: Fixed Income Securities could buy the bond for less than $100, then you should buy it rather than invest in the CD. Conversely, if its price were more than $100, you should buy the CD. If the bond’s price were exactly $100, you should be indifferent between the bond and the CD. The $100 is defined as the present value, or PV, of $115.76 due in 3 years when the appropriate interest rate is 5%. In general, the present value of a cash flow due N years in the future is the amount which, if it were on hand today, would grow to equal the given future amount. Since $100 would grow to $115.76 in 3 years at a 5% interest rate, $100 is the present value of $115.76 due in 3 years at a 5% rate. Finding present values is called discounting, and as previously noted, it is the reverse of compounding: If you know the PV, you can compound to find the FV; or if you know the FV, you can discount to find the PV. Indeed, we simply solve Equation 4-1, the formula for the future value, for the PV to produce the present value equation as follows. Compounding to find future values : Discounting to find present values : resource See Ch04 Tool Kit.xls for all calculations. FIGURE 4-3 Future value ¼ FVN ¼ PVð1 þ IÞN Present value ¼ PV ¼ FVN ð1 þ IÞN (4-1) (4-3) The top section of Figure 4-3 shows inputs and a time line for finding the present value of $115.76 discounted back for 3 years. We first calculate the PV using the stepby-step approach. When we found the FV in the previous section, we worked from left to right, multiplying the initial amount and each subsequent amount by (1 + I). To find Alternative Procedures for Calculating Present Values INPUTS: Future payment = CFN = FV = Interest rate = I = 5.00% No. of periods = N = 3 Problem as a Time Line $115.76 Periods: Cash Flow Time Line: 1.Step-by-Step: 0 2 PV = ? $100.00 2. Formula: FVN = PV/(1+I)N 3. Financial Calculator: 1 $115.76 $105.00 PV = $115.76(1.05)3 3 5 N I/YR 3 PV $110.25 $115.76 = $100.00 $0 $115.76 PMT FV –$100.00 4. Excel Spreadsheet: PV Function: PV = = PV(I,N,0,FV) Fixed inputs: PV = = PV(0.05,3,0,115.76) = –$100.00 Cell references: PV = = PV(C111,C112,0,C110) = –$100.00 In the Excel formula, the terms are entered in the sequence: interest, periods, 0 to indicate no periodic cash flows, and then the FV. The data can be entered as fixed numbers or, better yet, as cell references. Chapter 4: Time Value of Money 135 present values, we work backwards, or from right to left, dividing the future value and each subsequent amount by (1 + I), with the present value of $100 shown in Cell D118. The step-by-step procedure shows exactly what’s happening, and that can be quite useful when you are working complex problems or trying to explain a model to others. However, it’s inefficient, especially if you are dealing with more than a year or two. A more efficient procedure is to use the formula approach in Equation 4-3, simply dividing the future value by (1 + I)N. This gives the same result, as we see in Figure 43, Cell G120. Equation 4-2 is actually programmed into financial calculators. As shown in Figure 4-3, Rows 122 to 124, we can find the PV by entering values for N=3, I/YR=5, PMT=0, and FV=115.76, and then pressing the PV key to get −100. Excel also has a function that solves Equation 4-3—this is the PV function, and it is written as =PV(I,N,0,FV).6 Cell E126 shows the inputs to this function. Next, Cell E127 shows the Excel function with fixed numbers as inputs, with the actual function and the resulting −$100 in Cell G127. Cell E128 shows the Excel function using cell references, with the actual function and the resulting −$100 in Cell G128. The fundamental goal of financial management is to maximize the firm’s intrinsic value, and the intrinsic value of a business (or any asset, including stocks and bonds) is the present value of its expected future cash flows. Because present value lies at the heart of the valuation process, we will have much more to say about it in the remainder of this chapter and throughout the book. Graphic View of the Discounting Process Figure 4-4 shows that the present value of a sum to be received in the future decreases and approaches zero as the payment date is extended further and further into the future; it also shows that, the higher the interest rate, the faster the present value falls. At relatively high rates, funds due in the future are worth very little today, and even at relatively FIGURE 4-4 Present Value of $1 at Various Interest Rates and Time Periods Present Value of $1 resource I = 0% 1.00 See Ch04 Tool Kit.xls for all calculations. 0.80 I = 5% 0.60 I = 10% 0.40 I = 20% 0.20 0.00 0 10 20 30 40 50 Years The third entry in the PV function is zero to indicate that there are no intermediate payments in this particular example. 6 136 Part 2: Fixed Income Securities low rates present values of sums due in the very distant future are quite small. For example, at a 20% discount rate, $1 million due in 100 years would be worth just over 1 cent today. (However, 1 cent would grow to almost $1 million in 100 years at 20%.) Self-Test What is “discounting,” and how is it related to compounding? How is the future value equation (4-1) related to the present value equation (4-3)? How does the present value of a future payment change as the time to receipt is lengthened? As the interest rate increases? Suppose a risk-free bond promises to pay $2,249.73 in 3 years. If the going risk-free interest rate is 4%, how much is the bond worth today? ($2,000) How would your answer change if the bond matured in 5 rather than 3 years? ($1,849.11) If the risk-free interest rate is 6% rather than 4%, how much is the 5-year bond worth today? ($1,681.13) How much would $1 million due in 100 years be worth today if the discount rate were 5%? ($7,604.49) What if the discount rate were 20%? ($0.0121) 4.4 FINDING THE INTEREST RATE, I Thus far we have used Equations 4-1, 4-2, and 4-3 to find future and present values. Those equations have four variables, and if we know three of them, then we (or our calculator or Excel) can solve for the fourth. Thus, if we know PV, I, and N, we can solve Equation 4-1 for FV, or if we know FV, I, and N, we can solve Equation 4-3 to find PV. That’s what we did in the preceding two sections. Now suppose we know PV, FV, and N, and we want to find I. For example, suppose we know that a given security has a cost of $100 and that it will return $150 after 10 years. Thus, we know PV, FV, and N, and we want to find the rate of return we will earn if we buy the security. Here’s the solution using Equation 4-1: FV ¼ PVð1 þ IÞN $150 ¼ $100ð1 þ IÞ10 $150=$100 ¼ ð1 þ IÞ10 ð1 þ IÞ10 ¼ 1:5 ð1 þ IÞ ¼ 1:5ð1=10Þ 1 þ I ¼ 1:0414 I ¼ 0:0414 ¼ 4:14%: Finding the interest rate by solving the formula takes a little time and thought, but financial calculators and spreadsheets find the answer almost instantly. Here’s the calculator setup: Inputs: 10 N Output: I/YR –100 0 150 PV PMT FV 4.14 Enter N=10, PV= −100, PMT= 0 (because there are no payments until the security matures), and FV=150. Then, when you press the I/YR key, the calculator gives the answer, 4.14%. Notice that the PV is a negative value because it is a cash outflow (an investment) and the FV is positive because it is a cash inflow (a return of the investment). If you enter both PV and FV as positive numbers (or both as negative numbers), you will get an error message rather than the answer. Chapter 4: Time Value of Money resource See Ch04 Tool Kit.xls for all calculations. Self-Test 137 In Excel, the RATE function can be used to find the interest rate: =RATE(N,PMT, PV,FV). For this example, the interest rate is found as =RATE(10,0,−100,150) = 0.0414 = 4.14%. See the file Ch04 Tool Kit.xls on the textbook’s Web site for an example. Suppose you can buy a U.S. Treasury bond that makes no payments until the bond matures 10 years from now, at which time it will pay you $1,000.7 What interest rate would you earn if you bought this bond for $585.43? (5.5%) What rate would you earn if you could buy the bond for $550? (6.16%) For $600? (5.24%) Microsoft earned $0.33 per share in 1997. Ten years later, in 2007, it earned $1.42. What was the growth rate in Microsoft’s earnings per share (EPS) over the 10-year period? (15.71%) If EPS in 2007 had been $1.00 rather than $1.42, what would the growth rate have been? (11.72%) 4.5 FINDING THE NUMBER OF YEARS, N We sometimes need to know how long it will take to accumulate a specific sum of money, given our beginning funds and the rate we will earn. For example, suppose we now have $500,000 and the interest rate is 4.5%. How long will it be before we have $1 million? Here’s Equation 4-1, showing all the known variables. $1;000;000 ¼ $500;000ð1 þ 0:045ÞN (4-1) We need to solve for N, and we can use three procedures: a financial calculator, Excel (or some other spreadsheet), or by working with natural logs. As you might expect, the calculator and spreadsheet approaches are easier.8 Here’s the calculator setup: Inputs: N Output: resource See Ch04 Tool Kit.xls for all calculations. Self-Test 4.5 –500000 0 1000000 I/YR PV PMT FV 15.7473 Enter I/YR = 4.5, PV = −500000, PMT = 0, and FV = 1000000. We press the N key to get the answer, 15.7473 years. In Excel, we would use the NPER function: =NPER (I,PMT,PV,FV). Inserting data, we have =NPER(0.045,0,−500000,1000000) = 15.7473. The chapter’s tool kit, Ch04 Tool Kit.xls, shows this example. How long would it take $1,000 to double if it were invested in a bank that pays 6% per year? (11.9 years) How long would it take if the rate were 10%? (7.27 years) Microsoft’s 2007 earnings per share were $1.42, and its growth rate during the prior 10 years was 15.71% per year. If that growth rate were maintained, how long would it take for Microsoft’s EPS to double? (4.75 years) 7 This is a STRIP bond, which we explain in Chapter 5. Here’s the setup for the log solution. First, transform Equation 4-1 as indicated, then find the natural logs using a financial calculator, and then solve for N: 8 $1; 000; 000 ¼ $500; 000ð1 þ 0:045ÞN 2 ¼ ð1 þ 0:045ÞN lnð2Þ ¼ N½lnð1:045Þ N ¼ 0:6931=0:0440 ¼ 15:7473 years 138 Part 2: Fixed Income Securities 4.6 ANNUITIES Thus far we have dealt with single payments, or “lump sums.” However, assets such as bonds provide a series of cash inflows over time, and obligations such as auto loans, student loans, and mortgages call for a series of payments. If the payments are equal and are made at fixed intervals, then we have an annuity. For example, $100 paid at the end of each of the next 3 years is a 3-year annuity. If payments occur at the end of each period, then we have an ordinary (or deferred) annuity. Payments on mortgages, car loans, and student loans are generally made at the ends of the periods and thus are ordinary annuities. If the payments are made at the beginning of each period, then we have an annuity due. Rental lease payments, life insurance premiums, and lottery payoffs (if you are lucky enough to win one!) are examples of annuities due. Ordinary annuities are more common in finance, so when we use the term “annuity” in this book, you may assume that the payments occur at the ends of the periods unless we state otherwise. Next we show the time lines for a $100, 3-year, 5%, ordinary annuity and for the same annuity on an annuity due basis. With the annuity due, each payment is shifted back (to the left) by 1 year. In our example, we assume that a $100 payment will be made each year, so we show the payments with minus signs. Ordinary Annuity: Periods 0 5% Payments Annuity Due: Periods Payments 0 5% −$100 1 2 3 −$100 −$100 −$100 1 2 −$100 −$100 3 As we demonstrate in the following sections, we can find an annuity’s future value, present value, the interest rate built into the contracts, how long it takes to reach a financial goal using the annuity, and, if we know all of those values, the size of the annuity payment. Keep in mind that annuities must have constant payments and a fixed number of periods. If these conditions don’t hold, then the series is not an annuity. Self-Test What’s the difference between an ordinary annuity and an annuity due? Why should you prefer to receive an annuity due with payments of $10,000 per year for 10 years than an otherwise similar ordinary annuity? 4.7 FUTURE VALUE OF AN ORDINARY ANNUITY Consider the ordinary annuity whose time line was shown previously, where you deposit $100 at the end of each year for 3 years and earn 5% per year. Figure 4-5 shows how to calculate the future value of the annuity, FVAN, using the same approaches we used for single cash flows. As shown in the step-by-step section of Figure 4-5, we compound each payment out to Time 3, then sum those compounded values in Cell F226 to find the annuity’s FV, FVA3 = $315.25. The first payment earns interest for two periods, the second for Chapter 4: Time Value of Money FIGURE 4-5 139 Summary: Future Value of an Ordinary Annuity INPUTS: Payment amount = PMT = Interest rate = I = 5.00% No. of periods = N = 3 –$100.00 1 2 3 –$100 –$100 –$100 0 1. Step-by-Step: $100.00 $105.00 $110.25 $315.25 Multiply each payment by (1+I)N-t and sum these FVs to find FVAN: 2. Formula: FVAN 3. Financial Calculator: = PMT × (1+I)N 1 – I I = $315.25 3 5 $0 –$100.00 N I/YR PV PMT FV $315.25 4. Excel Spreadsheet: FVAN = = FV(I,N,PMT,PV) Fixed inputs: FVAN = = FV(0.05,3,–100,0) $315.25 Cell references: FVAN = = FV(C216,C217,C215,0) $315.25 FV Function: one period, and the third earns no interest because it is made at the end of the annuity’s life. This approach is straightforward, but if the annuity extends out for many years, it is cumbersome and time-consuming. As you can see from the time line diagram, with the step-by-step approach we apply the following equation with N = 3 and I = 5%: FVAN ¼ PMTð1 þ IÞN−1 þ PMTð1 þ IÞN−2 þ PMTð1 þ IÞN−3 ¼ $100ð1:05Þ2 þ $100ð1:05Þ1 þ $100ð1:05Þ0 ¼ $315:25 For the general case, the future value of an annuity is FVAN ¼ PMTð1 þ IÞN−1 þ PMTð1 þ IÞN−2 þ PMTð1 þ IÞN−3 þ … þ PMTð1 þ IÞ0 As shown in Web Extension 4B on the textbook’s Web site, the future value of an annuity can be written as follows:9 9 Section 4.11 shows that the present value of an infinitely long annuity, called a perpetuity, is equal to PMT/I. The cash flows of an ordinary annuity of N periods are equal to the cash flows of a perpetuity minus the cash flows of a perpetuity that begins at year N+1. Therefore, the future value of an N-period annuity is equal to the future value (as of year N) of a perpetuity minus the value (as of year N) of a perpetuity that begins at year N+1. See Web Extension 4B on the textbook’s Web site for details regarding derivations of Equation 4-4. 140 Part 2: Fixed Income Securities " ð1 þ IÞN 1 − FVAN ¼ PMT I I # (4-4) Using Equation 4-4, the future value of the annuity is found to be $315.25:   ð1 þ 0:05Þ3 1 ¼ $315:25 FVA3 ¼ $100 − 0:05 0:05 As you might expect, annuity problems can be solved easily using a financial calculator or a spreadsheet, most of which have the following formula built into them: " # ð1 þ IÞN 1 PVð1 þ IÞ þ PMT − þ FV ¼ 0 I I N (4-5) The procedure when dealing with annuities is similar to what we have done thus far for single payments, but the presence of recurring payments means that we must use the PMT key. Here’s the calculator setup for our illustrative annuity: Inputs: Output: resource See Ch04 Tool Kit.xls for all calculations. Self-Test 3 5 0 –100 N I/YR PV PMT End Mode FV 315.25 We enter PV = 0 because we start off with nothing, and we enter PMT = –100 because we will deposit this amount in the account at the end of each of the 3 years. The interest rate is 5%, and when we press the FV key we get the answer, FVA3 = 315.25. Since this is an ordinary annuity, with payments coming at the end of each year, we must set the calculator appropriately. As noted earlier, most calculators “come out of the box” set to assume that payments occur at the end of each period—that is, to deal with ordinary annuities. However, there is a key that enables us to switch between ordinary annuities and annuities due. For ordinary annuities, the designation “End Mode” or something similar is used, while for annuities due the designator is “Begin,” “Begin Mode,” “Due,” or something similar. If you make a mistake and set your calculator on Begin Mode when working with an ordinary annuity, then each payment will earn interest for one extra year, which will cause the compounded amounts, and thus the FVA, to be too large. The spreadsheet approach uses Excel’s FV function, =FV(I,N,PMT,PV). In our example, we have =FV(0.05,3,−100,0), and the result is again $315.25. For an ordinary annuity with 5 annual payments of $100 and a 10% interest rate, for how many years will the first payment earn interest, and what is the compounded value of this payment at the end? (4 years, $146.41) Answer this same question for the fifth payment. (0 years, $100) Assume that you plan to buy a condo 5 years from now, and you estimate that you can save $2,500 per year toward a down payment. You plan to deposit the money in a bank that pays 4% interest, and you will make the first deposit at the end of this year. How much will you have after 5 years? ($13,540.81) How would your answer change if the bank’s interest rate were increased to 6%, or decreased to 3%? ($14,092.73; $13,272.84) Chapter 4: Time Value of Money 4.8 FUTURE VALUE OF AN 141 ANNUITY DUE Because each payment occurs one period earlier with an annuity due, the payments will all earn interest for one additional period. Therefore, the FV of an annuity due will be greater than that of a similar ordinary annuity. If you went through the step-by-step procedure, you would see that our illustrative annuity due has a FV of $331.01 versus $315.25 for the ordinary annuity. See Ch04 Tool Kit.xls on the textbook’s Web site for a summary of future value calculations. With the formula approach, we first use Equation 4-4, but since each payment occurs one period earlier, we multiply the Equation 4-4 result by (1 + I): FVAdue ¼ FVAordinary ð1 þ IÞ (4-6) Thus, for the annuity due, FVAdue = $315.25(1.05) = $331.01, which is the same result as found with the step-by-step approach. With a calculator we input the variables just as we did with the ordinary annuity, but we now set the calculator to Begin Mode to get the answer, $331.01. Inputs: 3 5 0 –100 N I/YR PV PMT resource Self-Test FV 331.01 Output: See Ch04 Tool Kit.xls for all calculations. Begin Mode In Excel, we still use the FV function, but we must indicate that we have an annuity due. The function is =FV(I,N,PMT,PV,Type), where “Type” indicates the type of annuity. If Type is omitted then Excel assumes that it is 0, which indicates an ordinary annuity. For an annuity due, Type = 1. As shown in Ch04 Tool Kit.xls, the function is =FV(0.05,3,−100,0,1) = $331.01. Why does an annuity due always have a higher future value than an ordinary annuity? If you know the value of an ordinary annuity, explain why you could find the value of the corresponding annuity due by multiplying by (1+I). Assume that you plan to buy a condo 5 years from now and that you need to save for a down payment. You plan to save $2,500 per year, with the first payment being made immediately and deposited in a bank that pays 4%. How much will you have after 5 years? ($14,082.44) How much would you have if you made the deposits at the end of each year? ($13,540.81) 4.9 PRESENT VALUE AND ANNUITIES DUE OF ORDINARY ANNUITIES The present value of any annuity, PVAN, can be found using the step-by-step, formula, calculator, or spreadsheet methods. We begin with ordinary annuities. Present Value of an Ordinary Annuity See Figure 4-6 for a summary of the different approaches for calculating the present value of an ordinary annuity. 142 Part 2: Fixed Income Securities FIGURE 4-6 Summary: Present Value of an Ordinary Annuity INPUTS: Payment amount = PMT = Interest rate = I = 5.00% No. of periods = N = 3 Periods: 0 –$100.00 Cash Flow Time Line: 1 2 3 –$100 –$100 –$100 $95.24 1. Step-by-Step: Divide each payment by (1+I)t and sum these PVs to find PVAN: $90.70 $86.38 $272.32 2. Formula: PVAN 3. Financial Calculator: 4. Excel Spreadsheet: resource See Ch04 Tool Kit.xls for all calculations. = PMT × 1 1 – I I(1+1)N 3 5 N I FV Function: = $272.32 –100.00 0 PMT FV PV 272.32 PVAN = = PV(I,N,PMT,FV) Fixed inputs: PVAN = = PV(0.05,3,–100,0,) Cell references: PVAN = = PV(C285,C286,C284,0) = = $272.32 $272.32 As shown in the step-by-step section of Figure 4-6, we discount each payment back to Time 0, then sum those discounted values to find the annuity’s PV, PVA3 = $272.32. This approach is straightforward, but if the annuity extends out for many years, it is cumbersome and time-consuming. The time line diagram shows that with the step-by-step approach we apply the following equation with N = 3 and I = 5%: PVAN ¼ PMT=ð1 þ IÞ1 þ PMT=ð1 þ IÞ2 þ … þ PMT=ð1 þ IÞN The present value of an annuity can be written as10 " 1 1 PVAN ¼ PMT − I Ið1 þ IÞN # For our illustrative annuity, the present value is " # 1 1 − PVA3 ¼ PMT ¼ $272:32 0:05 0:05ð1 þ 0:05Þ3 10 See Web Extension 4B on the textbook’s Web site for details of this derivation. (4-7) Chapter 4: Time Value of Money 143 Financial calculators are programmed to solve Equation 4-7, so we merely input the variables and press the PV key, first making sure the calculator is set to End Mode. The calculator setup is shown below: Inputs: 3 5 N I/YR resource 0 PMT FV End Mode (Ordinary Annuity) 272.32 Output: See Ch04 Tool Kit.xls for all calculations. PV –100 Section 4 of Figure 4-6 shows the spreadsheet solution using Excel’s built-in PV function: =PV(I,N,PMT,FV). In our example, we have =PV(0.05,3,−100,0) with a resulting value of $272.32. Present Value of Annuities Due Because each payment for an annuity due occurs one period earlier, the payments will all be discounted for one less period. Therefore, the PV of an annuity due must be greater than that of a similar ordinary annuity. If you went through the step-by-step procedure, you would see that our illustrative annuity due has a PV of $285.94 versus $272.32 for the ordinary annuity. See Ch04 Tool Kit.xls for this and the other calculations. With the formula approach, we first use Equation 4-7 to find the value of the ordinary annuity and then, since each payment now occurs one period earlier, we multiply the Equation 4-7 result by (1 + I): PVAdue ¼ PVAordinary ð1 þ IÞ (4-8) PVAdue ¼ $272:32ð1:05Þ ¼ $285:94 With a financial calculator, the inputs are the same as for an ordinary annuity, except you must set the calculator to Begin Mode: Inputs: Output: resource See Ch04 Tool Kit.xls for all calculations. Self-Test 3 5 N I/YR PV –100 0 PMT FV Begin Mode (Annuity Due) 285.94 In Excel, we again use the PV function, but now we must indicate that we have an annuity due. The function is now =PV(I,N,PMT,FV,Type), where “Type” is the type of annuity. If Type is omitted then Excel assumes that it is 0, which indicates an ordinary annuity; for an annuity due, Type = 1. As shown in Ch04 Tool Kit.xls, the function for this example is =PV(0.05,3,−100,0,1) = $285.94. Why does an annuity due have a higher present value than an ordinary annuity? If you know the present value of an ordinary annuity, what’s an easy way to find the PV of the corresponding annuity due? What is the PVA of an ordinary annuity with 10 payments of $100 if the appropriate interest rate is 10%? ($614.46) What would the PVA be if the interest rate were 4%? ($811.09) What if the interest rate were 0%? ($1,000.00) What would the PVAs be if we were dealing with annuities due? ($675.90, $843.53, and $1,000.00) 144 Part 2: Fixed Income Securities Variable Annuities: Good or Bad? Retirees appreciate stable, predictable income, so they often buy annuities. Insurance companies have been the traditional suppliers, using the payments they receive to buy high-grade bonds, whose interest is then used to make the promised payments. Such annuities were quite safe and stable and provided returns of around 7.5%. However, returns on stocks (dividends plus capital gains) have historically exceeded bonds’ returns (interest). Therefore, some insurance companies in the 1990s began to offer variable annuities, which were backed by stocks instead of bonds. If stocks earned in the future as much as they had in the past, then variable annuities could offer returns of about 9%, better than the return on a fixed rate annuities. If stock returns turned out to be lower in the future than they had been in the past (or even had negative returns), then the variable annuities promised a guaranteed minimum payment of about 6.5%. Variable annuities appealed to many retirees, so companies that offered them had a significant competitive advantage. The insurance company that pioneered variable annuities, The Hartford, tried to hedge its position with derivatives that paid off if stocks went down. But like so many other derivatives-based risk management programs, this one went awry in 2008 because stock losses exceeded the assumed worst-case scenario. The Hartford, which was founded in 1810 and was one of the oldest and largest U.S. insurance companies at the beginning of 2008, saw its stock fall from $85.54 to $4.16. Because of the general stock market crash, investors feared that The Hartford would be unable to make good on its variable annuity promises, which would lead to bankruptcy. The company was bailed out by the economic stimulus package, but this 199-year-old firm will never be the same again. Source: Leslie Scism and Liam Pleven, “Hartford Aims to Take Risk Out of Annuities,” Online Wall Street Journal, January 13, 2009. Assume that you are offered an annuity that pays $100 at the end of each year for 10 years. You could earn 8% on your money in other equally risky investments. What is the most you should pay for the annuity? ($671.01) If the payments began immediately, then how much would the annuity be worth? ($724.69) 4.10 FINDING ANNUITY PAYMENTS, PERIODS, AND INTEREST RATES In the three preceding sections we discussed how to find the FV and PV of ordinary annuities and annuities due, using these four methods: step-by-step, formula, financial calculator, and Excel. Five variables are involved—N, I, PMT, FV, and PV—and if you know any four, you can find the fifth by solving either 4-4 (4-6 for annuities due) or 4-7 (4-8 for annuities due). However, a trial-and-error procedure is generally required to find N or I, and that can be quite tedious. Therefore, we discuss only the financial calculator and spreadsheet approaches for finding N and I. Finding Annuity Payments, PMT We need to accumulate $10,000 and have it available 5 years from now. We can earn 6% on our money. Thus, we know that FV = 10,000, PV = 0, N = 5, and I/YR = 6. We can enter these values in a financial calculator and then press the PMT key to find our required deposits. However, the answer depends on whether we make deposits at the end of each year (ordinary annuity) or at the beginning (annuity due), so the mode must be set properly. Here are the results for each type of annuity: Chapter 4: Time Value of Money Inputs: 5 6 0 N I/YR PV 10000 End Mode (Ordinary Annuity) FV PMT 145 –1773.96 Output: Inputs: 5 6 0 N I/YR PV 10000 FV PMT Begin Mode (Annuity Due) –1673.55 Output: Thus, you must put away $1,773.96 per year if you make payments at the end of each year, but only $1,673.55 if the payments begin immediately. Finally, note that the required payment for the annuity due is the ordinary annuity payment divided by (1 + I): $1,773.96/1.06 = $1,673.55. Excel can also be used to find annuity payments, as shown below for the two types of annuities. For end-of-year (ordinary) annuities, “Type” can be left blank or a 0 can be inserted. For beginning-of-year annuities (annuities due), the same function is used but now Type is designated as 1. Here is the setup for the two types of annuities. resource See Ch04 Tool Kit.xls for all calculations. Function : ¼ PMTðI; N; PV; FV; TypeÞ Ordinary annuity : ¼ PMTð0:06; 5; 0; 10000Þ Annuity due ¼ $1; 773:96 ¼ PMTð0:06; 5; 0; 10000; 1Þ ¼ $1; 673:55 Finding the Number of Periods, N Suppose you decide to make end-of-year deposits, but you can save only $1,200 per year. Again assuming that you would earn 6%, how long would it take you to reach your $10,000 goal? Here is the calculator setup: Inputs: N Output: resource See Ch04 Tool Kit.xls for all calculations. 6 0 –1200 10000 I/YR PV PMT FV End Mode 6.96 With these smaller deposits, it would take 6.96 years, not 5 years, to reach the $10,000 target. If you began the deposits immediately, then you would have an annuity due and N would be slightly less, 6.63 years. With Excel, you can use the NPER function: =NPER(I,PMT,PV,FV, Type). For our ordinary annuity example, Type is left blank (or 0 is inserted) and the function is =NPER(0.06,−1200,0,10000) = 6.96. If we put in 1 for type, we would find N = 6.63. Finding the Interest Rate, I Now suppose you can save only $1,200 annually, but you still need to have the $10,000 in 5 years. What rate of return would you have to earn to reach your goal? Here is the calculator setup: 146 Part 2: Fixed Income Securities Inputs: 5 N Output: I/YR 0 –1200 10000 PV PMT FV End Mode 25.78 Thus, you would need to earn a whopping 25.78%! About the only way to earn such a high return would be either to invest in speculative stocks or head to a Las Vegas casino. Of course, speculative stocks and gambling aren’t like making deposits in a bank with a guaranteed rate of return, so there would be a high probability that you’d end up with nothing. So, you should probably save more, lower your $10,000 target, or extend your time horizon. It might be appropriate to seek a somewhat higher return, but trying to earn 25.78% in a 6% market would involve speculation, not investing. In Excel, you can use the RATE function: =RATE(N,PMT,PV,FV,Type). For our example, the function is =RATE(5,−1200,0,10000) = 0.2578 = 25.78%. If you decide to make the payments beginning immediately then the required rate of return would decline sharply, to 17.54%. Self-Test Suppose you inherited $100,000 and invested it at 7% per year. How large of a withdrawal could you make at the end of each of the next 10 years and end up with zero? ($14,237.75) How would your answer change if you made withdrawals at the beginning of each year? ($13,306.31) If you had $100,000 that was invested at 7% and you wanted to withdraw $10,000 at the end of each year, how long would your funds last? (17.8 years) How long would they last if you earned 0%? (10 years) How long would they last if you earned the 7% but limited your withdrawals to $7,000 per year? (forever) Your rich uncle named you as the beneficiary of his life insurance policy. The insurance company gives you a choice of $100,000 today or a 12-year annuity of $12,000 at the end of each year. What rate of return is the insurance company offering? (6.11%) Assume that you just inherited an annuity that will pay you $10,000 per year for 10 years, with the first payment being made today. A friend of your mother offers to give you $60,000 for the annuity. If you sell it to him, what rate of return will your mother’s friend earn on the investment? (13.70%) If you think a “fair” rate of return would be 6%, how much should you ask for the annuity? ($78,016.92) 4.11 PERPETUITIES In the previous section we dealt with annuities whose payments continue for a specific number of periods—for example, $100 per year for 10 years. However, some securities promise to make payments forever. For example, in the mid-1700s the British government issued some bonds that never matured and whose proceeds were used to pay off other British bonds. Since this action consolidated the government’s debt, the new bonds were called “consols.” The term stuck, and now any bond that promises to pay interest perpetually is called a consol, or a perpetuity. The interest rate on the consols was 2.5%, so a consol with a face value of $1,000 would pay $25 per year in perpetuity.11 11 The consols actually pay interest in pounds, but we discuss them in dollar terms for simplicity. Chapter 4: Time Value of Money 147 Using the Internet for Personal Financial Planning People continually face important financial decisions that require an understanding of the time value of money. Should we buy or lease a car? How much and how soon should we begin to save for our children’s education? How expensive a house can we afford? Should we refinance our home mortgage? How much must we save each year if we are to retire comfortably? The answers to these questions are often complicated, and they depend on a number of factors, such as projected housing and education costs, interest rates, inflation, expected family income, and stock mar- ket returns. Hopefully, after completing this chapter, you will have a better idea of how to answer such questions. Note, though, that a number of online resources are available to help with financial planning. A good place to start is http://www.smartmoney.com. Smartmoney is a personal finance magazine produced by the publishers of The Wall Street Journal. If you go to Smartmoney’s Web site you will find a section entitled “Tools.” This section has a number of financial calculators, spreadsheets, and descriptive materials that cover a wide range of personal finance issues. A consol, or perpetuity, is simply an annuity whose promised payments extend out forever. Since the payments go on forever, you can’t apply the step-by-step approach. However, it’s easy to find the PV of a perpetuity with the following formula:12 PV of a perpetuity ¼ PMT I (4-9) We can use Equation 4-9 to find the value of a British consol with a face value of $1,000 that pays $25 per year in perpetuity. The answer depends on the interest rate being earned on investments of comparable risk at the time the consol is being valued. Originally, the “going rate” as established in the financial marketplace was 2.5%, so originally the consol’s value was $1,000: Consol's valueOriginally ¼ $25=0:025 ¼ $1; 000 The annual payment is still $25 today, but the going interest rate has risen to about 5.2%, causing the consol’s value to fall to $480.77: Consol's valueToday ¼ $25=0:052 ¼ $480:77 Note, though, that if interest rates decline in the future, say to 2%, then the value of the consol will rise to $1,250.00: Consol's value if rates decline to 2% ¼ $25=0:02 ¼ $1; 250:00 These examples demonstrate an important point: When interest rates change, the prices of outstanding bonds also change, but inversely to the change in rates. Thus, bond prices decline if rates rise, and prices increase if rates fall. This holds for all bonds, both consols and those with finite maturities. We will discuss this point in more detail in Chapter 5, where we cover bonds in depth. Self-Test What is the present value of a perpetuity that pays $1,000 per year, beginning 1 year from now, if the appropriate interest rate is 5%? ($20,000) What would the value be if 12 See Web Extension 4B on the textbook’s Web site for a derivation of the perpetuity formula. 148 Part 2: Fixed Income Securities the annuity began its payments immediately? ($21,000) (Hint: Just add the $1,000 to be received immediately to the formula value of the annuity.) Do bond prices move directly or inversely with interest rates—that is, what happens to the value of a bond if interest rates increase or decrease? 4.12 UNEVEN, OR IRREGULAR, CASH FLOWS The definition of an annuity includes the term constant payment—in other words, annuities involve a set of identical payments over a given number of periods. Although many financial decisions do involve constant payments, many others involve cash flows that are uneven or irregular. For example, the dividends on common stocks are typically expected to increase over time, and investments in capital equipment almost always generate cash flows that vary from year to year. Throughout the book, we use the term payment (PMT) in situations where the cash flows are constant and thus an annuity is involved; we use the term cash flow (CFt), where the t designates the period in which the particular cash flow occurs, if the cash flows are irregular. There are two important classes of uneven cash flows: (1) those in which the cash flow stream consists of a series of annuity payments plus an additional final lump sum in Year N, and (2) all other uneven streams. Bonds are an instance of the first type, while stocks and capital investments illustrate the second type. Here’s an example of each type. Stream 1. Annuity plus additional final payment: Periods 0 I = 12% 1 2 3 Cash flows $0 $100 4 5 $100 $100 $100 $ 100 $ 1,000 $1,100 2 3 4 5 $300 $300 $300 $500 Stream 2. Irregular cash flows: Periods 0 I = 12% 1 Cash flows $0 $100 Equation 4-10 can be used, following the step-by-step procedure, to find the PV of either stream. However, as we shall see, the solution process differs significantly for the two types. N PV ¼ ∑ CF1 CF2 CFN CFt þ þ…þ ¼ t ð1 þ IÞ1 ð1 þ IÞ2 ð1 þ IÞN t¼1 ð1 þ IÞ (4-10) Annuity Plus Additional Final Payment First, consider Stream 1 and notice that it is a 5-year, 12%, ordinary annuity plus a final payment of $1,000. We can find the PV of the annuity, find the PV of the final payment, and then sum them to get the PV of the stream. Financial calculators are programmed do this for us—we use all five time value of money (TVM) keys, entering the data for the four known values as shown below, and then pressing the PV key to get the answer, $927.90: Chapter 4: Time Value of Money Inputs: 5 12 N I/YR PV 100 1000 PMT FV 149 –927.90 Output: Similarly, we could use Excel’s PV function, =PV(I,N,PMT,FV) = PV(0.12,5,100,1000) = −$927.90. Note that the process is similar to that for annuities, except we now have a nonzero value for FV. Irregular Cash Flow Stream resource See Ch04 Tool Kit.xls for all calculations. FIGURE 4-7 Now consider the irregular stream, which is analyzed in Figure 4-7. The top section shows the basic time line, which contains the inputs, and we first use the step-by-step approach to find PV = $1,016.35. Note that we show the PV of each cash flow directly below the cash flow, and then we sum these PVs to find the PV of the stream. This setup saves space as compared with showing the individual PVs in a column, and it is also transparent and thus easy to understand. Now consider the financial calculator approach. The cash flows don’t form an annuity, so you can’t use the annuity feature on the calculator. You could, of course, use the calculator in the step-by-step procedure, but financial calculators have a feature—the cash flow register—that allows you to find the present value more efficiently. First, you input the individual cash flows, in chronological Present Value of an Irregular Cash Flow Stream Step-by-step: Interest rate Periods: = I CF Time Line: PVs of the CFs: $0.00 ∑ C477:G477 = $1,016.35 0 = 12% 1 2 3 4 5 $100.00 $89.29 $300.00 $239.16 $300.00 $213.53 $300.00 $190.66 $500.00 $283.71 = Sum of the individual PVs = PV of the irregular CF stream. Here we put the PVs of each individual CF under the CF itself and then summed them to find the PV of the entire stream, rather than show them all in Column C as was done in Figure 4-6. This setup takes up less space and also makes the calculations quite transparent, which is useful, especially when the table must be explained to people who did not develop it. People appreciate transparency and clarity. Calculator: Excel Function: You could enter the cash flows into the cash flow register of a financial calculator, enter I/YR, and then press the NPV key to find the answer. Fixed inputs: Cell references: $1,016.35 NPV = =NPV(0.12,100,300,300,300,500) $1,016.35 NPV = =NPV(C471,C474:G474) $1,016.35 Our Excel formula ignores the initial cash flow (in Year 0). When entering a cash flow range, Excel assumes that the first value occurs at the end of the first year. As we will see later, if there is an initial cash flow, it must be added seperately to complete the NPV formula result. Notice too that you can enter cash flows one-by-one, but if the cash flows appear in consecutive cells, you can enter the cell range, as we did here. 150 Part 2: Fixed Income Securities order, into the cash flow register.13 Cash flows are designated CF0, CF1, CF2, CF3,…, CFN. Next, you enter the interest rate, I. At this point, you have substituted in all the known values of Equation 4-10, so when you press the NPV key you get the PV of the stream. The calculator finds the PV of each cash flow and sums them to find the PV of the entire stream. To input the cash flows for this problem, enter 0 (because CF0 = 0), 100, 300, 300, 300, and 500 in that order into the cash flow register, enter I=12, and then press NPV to obtain the answer, $1,016.35. Two points should be noted. First, when dealing with the cash flow register, the calculator uses the term “NPV” rather than “PV.” The N stands for “net,” so NPV is the abbreviation for “net present value,” which is simply the net present value of a series of positive and negative cash flows, including any cash flow at time zero. The NPV function will be used extensively when we get to capital budgeting, where CF0 is generally the cost of the project. The second point to note is that repeated cash flows with identical values can be entered into the cash flow register more efficiently by using the Nj key. In this illustration, you would enter CF0 =0, CF1 =100, CF2 =300, Nj=3 (which tells the calculator that the 300 occurs 3 times), and CF5 =500.14 Then enter I=12, press the NPV key, and 1,016.35 will appear in the display. Also, note that numbers entered into the cash flow register remain in the register until they are cleared. Thus, if you previously worked a problem with eight cash flows, then moved to one with only four cash flows, the calculator would simply add the cash flows from the second problem to those of the first problem, and you would get an incorrect answer. Therefore, you must be sure to clear the cash flow register before starting a new problem. Spreadsheets are especially useful for solving problems with uneven cash flows. You enter the cash flows in the spreadsheet as shown in Figure 4-7 on Row 474. To find the PV of these cash flows without going through the stepby-step process, you would use the NPV function. First put the cursor on the cell where you want the answer to appear, Cell G486, click Financial, scroll down to NPV, and click OK to get the dialog box. Then enter C471 (or 0.12) for Rate and enter either the individual cash flows or the range of cells containing the cash flows, C474:G474, for Value 1. Be very careful when entering the range of cash flows. With a financial calculator, you begin by entering the Time-0 cash flow. With Excel, you do not include the Time-0 cash flow; instead, you begin with the Year-1 cash flow. Now, when you click OK, you get the PV of the stream, $1,016.35. Note that you can use the PV function if the payments are constant, but you must use the NPV function if the cash flows are not constant. Finally, note that Excel has a major advantage over financial calculators in that you can see the cash flows, which makes it easy to spot data entry errors. With a calculator, the numbers are buried in the machine, making it harder to check your work. We cover the calculator mechanics in the tutorial, and we discuss the process in more detail in Chapter 10, where we use the NPV calculation to analyze proposed projects. If you don’t know how to use the cash flow register of your calculator, you should to go to our tutorial or your calculator manual, learn the steps, and be sure you can make this calculation. You will have to know how to do it eventually, and now is a good time to learn. 13 On some calculators, instead of entering CF5 = 500, you enter CF3 = 500, because this is the next cash flow different from 300. 14 Chapter 4: Time Value of Money Self-Test 151 Could you use Equation 4-3, once for each cash flow, to find the PV of an uneven stream of cash flows? What is the present value of a 5-year ordinary annuity of $100 plus an additional $500 at the end of Year 5 if the interest rate is 6%? ($794.87) How would the PV change if the $100 payments occurred in Years 1 through 10 and the $500 came at the end of Year 10? ($1,015.21) What is the present value of the following uneven cash flow stream: $0 at Time 0, $100 at the end of Year 1 (or at Time 1), $200 at the end of Year 2, $0 at the end of Year 3, and $400 at the end of Year 4—assuming the interest rate is 8%? ($558.07) Would a “typical” common stock provide cash flows more like an annuity or more like an uneven cash flow stream? 4.13 FUTURE VALUE OF AN UNEVEN CASH FLOW STREAM The future value of an uneven cash flow stream (sometimes called the terminal, or horizon, value) is found by compounding each payment to the end of the stream and then summing the future values: FV¼ CF0 ð1 þ IÞN þ CF1 ð1 þ IÞN−1 þ CF2 ð1 þ IÞN−2 þ … þ CFN−1 ð1 þ IÞ þ CFN N (4-11) ¼ ∑ CFt ð1 þ IÞN−t t¼0 The future value of our illustrative uneven cash flow stream is $1,791.15, as shown in Figure 4-8. Most financial calculators have a net future value (NFV) key which, after the cash flows and interest rate have been entered, can be used to obtain the future value of an uneven cash flow stream. If your calculator doesn’t have the NFV feature, you can first find the net present value of the stream, then find its net future value as NFV = NPV(1 + I)N. In the illustrative problem, we find PV = 1,016.35 using the cash flow register and I=12. Then we use the TVM register, entering N=5, I=12, PV = −1016.35, and PMT = 0. When we press FV, we find FV = 1,791.15, which is FIGURE 4-8 Future Value of an Irregular Cash Flow Stream Step-by-step: Periods: Interest rate 0 = I 1 2 3 4 5 $300 $300 $300 $500 = 12% CF Time Line: $0 $100 FV of each CF: $0.00 $157.35 Calculator: Excel: $421.48 $376.32 $336.00 Sum of the Cash Flows’ FVs = FV of the stream = You could enter the cash flows into the cash flow register of a financial Step 1. Find NPV: Step 2. Compound NPV to find NFV: =NPV(C505,C507:G507) =FV(C505,G504,0,–G513) $500 $1,791.15 $1,791.15 $1,016.35 $1,791.15 152 Part 2: Fixed Income Securities the same as the value shown on the time line in Figure 4-8. As Figure 4-8 also shows, this same procedure can be used with Excel. Self-Test What is the future value of this cash flow stream: $100 at the end of 1 year, $150 after 2 years, and $300 after 3 years, assuming the appropriate interest rate is 15%? ($604.75) 4.14 SOLVING FOR I WITH IRREGULAR CASH FLOWS Before financial calculators and spreadsheets existed, it was extremely difficult to find I if the cash flows were uneven. However, with spreadsheets and financial calculators it’s easy to find I. If you have an annuity plus a final lump sum, you can input values for N, PV, PMT, and FV into the calculator’s TVM registers and then press the I/YR key. Here’s the setup for Stream 1 from Section 4.12, assuming we must pay $927.90 to buy the asset: Inputs: 5 N I/YR –927.90 100 1000 PV PMT FV 12.00 Output: The rate of return on the $927.90 investment is 12%. Finding the interest rate for an irregular cash flow stream with a calculator is a bit more complicated. Figure 4-9 shows Stream 2 from Section 4.12, assuming a required investment of CF0 = −$1,000. First, note that there is no simple step-by-step method for finding the rate of return—finding the rate for this investment requires a trial-and-error process, which is terribly time consuming. Therefore, we really need a financial calculator or a spreadsheet. With a calculator, we would enter the CFs into the cash flow register and then press the IRR key to get the answer. IRR stands for “internal rate of return,” and it is the rate of return the investment provides. The investment is the cash flow at Time 0, and it must be entered as a negative number. When we enter those cash flows in the calculator’s cash flow register and press the IRR key, we get the rate of return on the $1,000 investment, 12.55%. Finally, note that once you have entered the cash flows in the calculator’s register, you can find both the investment’s net present value (NPV) and its internal rate of return. For investment decisions, we typically want both of these numbers. Therefore, we generally enter the data once and then find both the NPV and the IRR. You would get the same answer using Excel’s IRR function, as shown in Figure 4-9. Notice that when using the IRR—unlike using the NPV function—you must include all cash flows, including the Time-0 cash flow. FIGURE 4-9 IRR of an Uneven Cash Flow Stream Periods: 0 Periods: 01 12 23 34 45 CF Time Line: CF Time $100 $300 $300 $300 $500 –$1,000 Line: $100 $300 $300 $300 –$1,000 You could enterYou the could cash flows the cash flow of a financial enter into the cash flows intoregister the cash flow register of a financial Calculator: 12.55% Calculator: calculator and then press and the IRR to find the answer. calculator thenkey press the IRR key to find the answer. Excel IRR Function: Cell references:Cell references: IRR = Excel IRR Function: =IRR(B549:G549) =IRR(B549:G549) 12.55% IRR = 5 $500 12.55% 12.55% Chapter 4: Time Value of Money Self-Test 153 An investment costs $465 now and is expected to produce cash flows of $100 at the end of each of the next 4 years, plus an extra lump-sum payment of $200 at the end of the fourth year. What is the expected rate of return on this investment? (9.05%) An investment costs $465 and is expected to produce cash flows of $100 at the end of Year 1, $200 at the end of Year 2, and $300 at the end of Year 3. What is the expected rate of return on this investment? (11.71%) 4.15 SEMIANNUAL AND OTHER COMPOUNDING PERIODS In most of our examples thus far, we assumed that interest is compounded once a year, or annually. This is annual compounding. Suppose, however, that you put $1,000 into a bank that pays a 6% annual interest rate but credits interest each 6 months. This is semiannual compounding. If you leave your funds in the account, how much would you have at the end of 1 year under semiannual compounding? Note that you will receive $60 of interest for the year, but you will receive $30 of it after only 6 months and the other $30 at the end of the year. You will earn interest on the first $30 during the second 6 months, so you will end the year with more than the $60 you would have had under annual compounding. You would be even better off under quarterly, monthly, weekly, or daily compounding. Note also that virtually all bonds pay interest semiannually; most stocks pay dividends quarterly; most mortgages, student loans, and auto loans involve monthly payments; and most money fund accounts pay interest daily. Therefore, it is essential that you understand how to deal with nonannual compounding. Types of Interest Rates When we move beyond annual compounding, we must deal with the following four types of interest rates: • • • • Nominal annual rates, given the symbol INOM Annual percentage rates, termed APR rates Periodic rates, denoted as IPER Effective annual rates, given the symbol EAR or EFF% Nominal (or Quoted) Rate, INOM.15 This is the rate quoted by banks, brokers, and other financial institutions. So, if you talk with a banker, broker, mortgage lender, auto finance company, or student loan officer about rates, the nominal rate is the one he or she will normally quote you. However, to be meaningful, the quoted nominal rate must also include the number of compounding periods per year. For example, a bank might offer you a CD at 6% compounded daily, while a credit union might offer 6.1% compounded monthly. Note that the nominal rate is never shown on a time line, and it is never used as an input in a financial calculator (except when compounding occurs only once a year). If more frequent compounding occurs, you must use periodic rates. Periodic Rate, IPER. This is the rate charged by a lender or paid by a borrower each period. It can be a rate per year, per 6 months (semiannually), per quarter, per month, per day, or per any other time interval. For example, a bank might charge 1.5% per month on 15 The term nominal rate as it is used here has a different meaning than the way it was used in Chapter 1. There, nominal interest rates referred to stated market rates as opposed to real (zero-inflation) rates. In this chapter, the term nominal rate means the stated, or quoted, annual rate as opposed to the effective annual rate, which we explain later. In both cases, though, nominal means stated, or quoted, as opposed to some sort of adjusted rate. 154 Part 2: Fixed Income Securities its credit card loans, or a finance company might charge 3% per quarter on installment loans. We find the periodic rate as follows: Periodic rate IPER = INOM/M (4-12) where INOM is the nominal annual rate and M is the number of compounding periods per year. Thus, a 6% nominal rate with semiannual payments results in a periodic rate of Periodic rate IPER = 6%/2 = 3.00%. If only one payment is made per year then M = 1, in which case the periodic rate would equal the nominal rate: 6%/1 = 6%. The periodic rate is the rate shown on time lines and used in calculations.16 To illustrate, suppose you invest $100 in an account that pays a nominal rate of 12%, compounded quarterly, or 3% per period. How much would you have after 2 years if you leave the funds on deposit? First, here is the time line for the problem: 0 3% 1 2 3 4 5 6 7 8 −100 FV=? Quarters To find the FV, we would use this modified version of Equation 4-1: FVN ¼ PVð1 þ IPER ÞNumber of periods   INOM MN ¼ PV 1 þ M (4-13)   0:12 4 × 2 ¼ $100ð1 þ 0:03Þ8 ¼ $126:68: ¼ $100 1 þ 4 (4-13) With a financial calculator, we find the FV using these inputs: N = 4 × 2 = 8, I = 12/4 = 3, PV = −100, and PMT = 0. The result is again FV = $126.68.17 Inputs: Output: 8 3 –100 0 N I/YR PV PMT FV 126.68 The only exception is in cases where (1) annuities are involved and (2) the payment periods do not correspond to the compounding periods. In such cases—for example, if you are making quarterly payments into a bank account to build up a specified future sum but the bank pays interest on a daily basis—then the calculations are more complicated. For such problems, the simplest procedure is to determine the periodic (daily) interest rate by dividing the nominal rate by 365 (or by 360 if the bank uses a 360-day year), then compound each payment over the exact number of days from the payment date to the terminal point, and then sum the compounded payments to find the future value of the annuity. This is what is generally done in the real world, because with a computer it’s a simple process. 16 Most financial calculators have a feature that allows you to set the number of payments per year and then use the nominal annual interest rate. However, students tend to make fewer errors when using the periodic rate with their calculators set for one payment per year (i.e., per period), so this is what we recommend. Note also that a normal time line cannot be used unless you use the periodic rate. 17 Chapter 4: Time Value of Money 155 Effective (or Equivalent) Annual Rate (EAR or EFF%). This is the annual (interest once a year) rate that produces the same final result as compounding at the periodic rate for M times per year. The EAR, also called EFF% (for effective percentage rate), is found as follows:18  EAR ¼ EFF% ¼ 1þ INOM M M −1:0 (4-14) Here INOM/M is the periodic rate and M is the number of periods per year. If a bank would lend you money at a nominal rate of 12%, compounded quarterly, then the EFF% rate would be 12.5509%: Rate on bank loan : EFF% ¼ ð1 þ 0:03Þ4 −1:0 ¼ ð1:03Þ4 −1:0 ¼ 1:125509−1:0 ¼ 0:125509 ¼ 12:5509% To see the importance of the EFF%, suppose that—as an alternative to the bank loan— you could borrow on a credit card that charges 1% per month. Would you be better off using the bank loan or credit card loan? To answer this question, the cost of each alternative must be expressed as an EFF%. We just saw that the bank loan’s effective cost is 12.5509%. The cost of the credit card loan, with monthly payments, is slightly higher, 12.6825%: Credit card loan : EFF% ¼ ð1 þ 0:01Þ12 −1:0 ¼ ð1:01Þ12 −1:0 ¼ 1:126825−1:0 ¼ 0:126825 ¼ 12:6825% This result is logical: Both loans have the same 12% nominal rate, yet you would have to make the first payment after only one month on the credit card versus three months under the bank loan. The EFF% rate is rarely used in calculations. However, it must be used to compare the effective costs of different loans or rates of return on different investments when payment periods differ, as in our example of the credit card versus a bank loan. The Result of Frequent Compounding What would happen to the future value of an investment if interest were compounded annually, semiannually, quarterly, or some other less-than-annual period? Because interest will be earned on interest more often, you should expect higher future values the more frequently compounding occurs. Similarly, you should expect the effective annual rate to increase with more frequent compounding. As Figure 4-10 shows, these results do occur—the future value and the EFF% do increase as the frequency of compounding increases. Notice that the biggest increase in FV (and in EFF%) occurs when compounding goes from annual to semiannual, and notice also that moving from monthly to daily compounding has a relatively small impact. Although Figure 4-10 shows daily compounding as the smallest interval, it is possible to compound even more frequently. At the limit, compounding can occur continuously. This is explained in Web Extension 4C on the textbook’s Web site. 18 You could also use the “interest conversion feature” of a financial calculator. Most financial calculators are programmed to find the EFF% or, given the EFF%, to find the nominal rate; this is called “interest rate conversion.” You enter the nominal rate and the number of compounding periods per year, then press the EFF% key to find the effective annual rate. However, we generally use Equation 4-14 because it’s easy and because using the equation reminds us of what we are really doing. If you do use the interest rate conversion feature on your calculator, don’t forget to reset your settings afterward. Interest conversion is discussed in our calculator tutorials. 156 Part 2: Fixed Income Securities Truth in Lending: What Loans Really Cost Congress passed the Consumer Credit Protection Act in 1968. The Truth in Lending provisions in the Act require banks and other lenders to disclose the annual percentage rate (APR) they are charging. For example, suppose you plan to buy a fancy TV set that costs $3,000, and the store offers you credit for one year at an “add-on” quoted rate of 8%. Here we first find the total dollars of interest by multiplying the $3,000 you are borrowing times 8%, getting $240. This interest is then added to the $3,000 cost of the TV, resulting in a total loan of $3,240. The total loan is divided by 12 to get the monthly payments: $3,240/12 = $270 per month, with the first payment made at the time of purchase. Therefore, we have a 12-month annuity due with payments of $270. Is your cost really the 8% that you were quoted? FIGURE 4-10 To find the APR, you first set your calculator to Begin Mode, then enter N = 12, PV = 3000, PMT = –270, and FV = 0. Then, when you press the I/YR key, you get the periodic rate, 1.4313%. You then multiply by 12 to get the APR, 17.1758%. You could also find the EFF%, which is 18.5945%. We show these calculations using both the calculator and Excel, along with a time line that helps us visualize what’s happening, in the chapter’s Excel Tool Kit. The 17.1758% APR that the dealer is required to report is a much better indicator of the loan’s cost than the 8% nominal rate, but it still does not reflect the true cost, which is the 18.5945% effective annual rate. Thus, buying the TV on time would really cost you 18.5945%. If you don’t know what’s happening when you buy on time or borrow, you may pay a lot more than you think! Effect on $100 of Compounding More Frequently Than Once a Year Frequency of Compounding Annual Semiannual Quarterly Monthly Daily Nominal Annual Rate 12% 12% 12% 12% 12% Number of periods per year (M)a 1 2 4 12 365 Periodic Interest Rate 12.0000% 6.0000% 3.0000% 1.0000% 0.0329% Effective Annual Rateb 12.0000% 12.3600% 12.5509% 12.6825% 12.7475% Future Valuec $112.00 $112.36 $112.55 $112.68 $112.75 Percentage increase in FV 0.32% 0.17% 0.12% 0.06% a We used 365 days per year in the calculations. The EFF% is calculated using text Equation 4-14. c The Future Value is calculated using text Equation 4-1. b Self-Test Would you rather invest in an account that pays a 7% nominal rate with annual compounding or with monthly compounding? If you borrowed at a nominal rate of 7%, would you rather make annual or rather monthly payments? Why? What is the future value of $100 after 3 years if the appropriate interest rate is 8%, compounded annually? ($125.97) Compounded monthly? ($127.02) What is the present value of $100 due in 3 years if the appropriate interest rate is 8%, compounded annually? ($79.38) Compounded monthly? ($78.73) Define the following terms: “annual percentage rate, or APR”; “effective annual rate, or EFF%”; and “nominal interest rate, INOM.“ A bank pays 5% with daily compounding on its savings accounts. Should it advertise the nominal or effective rate if it is seeking to attract new deposits? Credit card issuers must by law print their annual percentage rate on their monthly statements. A common APR is 18%, with interest paid monthly. What is the EFF% on such a loan? (19.56%) Chapter 4: Time Value of Money 157 Some years ago banks weren’t required to reveal the rate they charged on credit cards. Then Congress passed a “truth in lending” law that required them to publish their APR rate. Is the APR rate really the “most truthful” rate, or would the EFF% be even “more truthful”? 4.16 FRACTIONAL TIME PERIODS19 Thus far we have assumed that payments occur at either the beginning or the end of periods, but not within periods. However, we occasionally encounter situations that require compounding or discounting over fractional periods. For example, suppose you deposited $100 in a bank that pays a nominal rate of 10%, compounded daily, based on a 365-day year. How much would you have after 9 months? The answer of $107.79 is found as follows:20 Periodic rate ¼ IPER ¼ 0:10=365 ¼ 0:000273973 per day Number of days ¼ ð9=12Þð365Þ ¼ 0:75ð365Þ ¼ 273:75 days; rounded to 274 Ending amount ¼ $100ð1:000273973Þ274 ¼ $107:79 Now suppose that instead you borrow $100 at a nominal rate of 10% per year, simple interest, which means that interest is not earned on interest. If the loan is outstanding for 274 days (or 9 months), how much interest would you have to pay? The interest owed is equal to the principal multiplied by the interest rate times the number of periods. In this case, the number of periods is equal to a fraction of a year: N = 274/365 = 0.7506849. Interest owed = $100(10%)(0.7506849) = $7.51 Another approach would be to use the daily rate rather than the annual rate and thus to use the exact number of days rather than the fraction of the year: Interest owed = $100(0.000273973)(274) = $7.51 You would owe the bank a total of $107.51 after 274 days. This is the procedure most banks actually use to calculate interest on loans, except that they generally require borrowers to pay the interest on a monthly basis rather than after 274 days; this more frequent compounding raises the EFF% and thus the total amount of interest paid. Self-Test Suppose a company borrowed $1 million at a rate of 9%, simple interest, with interest paid at the end of each month. The bank uses a 360-day year. How much interest would the firm have to pay in a 30-day month? ($7,500.00) What would the interest be if the bank used a 365-day year? ($7,397.26) Suppose you deposited $1,000 in a credit union that pays 7% with daily compounding and a 365-day year. What is the EFF%? (7.250098%) How much could you withdraw after 7 months, assuming this is 7/12 of a year? ($1,041.67) This section is interesting and useful, but relatively technical. It can be omitted, at the option of the instructor, without loss of continuity. 19 We assume that these 9 months constitute 9/12 of a year. Also, bank deposit and loan contracts specifically state whether they are based on a 360-day or a 365-day year. If a 360-day year is used, then the daily rate is higher, so the effective rate is also higher. Here we assumed a 365-day year. Finally, note that banks’ computers, like Excel, have built-in calendars, so they can calculate the exact number of days. Note also that banks often treat such loans as follows. (1) They require monthly payments, and they figure the interest for the month as the periodic rate multiplied by the beginning-of-month balance times the number of days in the month. This is called “simple interest.” (2) The interest for the month is either added to the next beginning of month balance, or else the borrower must actually pay the earned interest. In this case, the EFF% is based on 12 compounding periods, not 365 as is assumed in our example. 20 158 Part 2: Fixed Income Securities 4.17 AMORTIZED LOANS An extremely important application of compound interest involves loans that are paid off in installments over time. Included are automobile loans, home mortgage loans, student loans, and many business loans. A loan that is to be repaid in equal amounts on a monthly, quarterly, or annual basis is called an amortized loan.21 For example, suppose a company borrows $100,000, with the loan to be repaid in 5 equal payments at the end of each of the next 5 years. The lender charges 6% on the balance at the beginning of each year. Here’s a picture of the situation: 0 I = 6% $100,000 1 2 3 4 5 PMT PMT PMT PMT PMT Our task is to find the amount of the payment, PMT, such that the sum of their PVs equals the amount of the loan, $100,000: 5 ∑ PMT PMT PMT PMT PMT PMT $100; 000 ¼ 5 ¼ 1þ 2þ 3þ 4þ t¼1 ð1:06Þt ð1:06Þ ð1:06Þ ð1:06Þ ð1:06Þ ð1:06Þ It is possible to solve the annuity formula, Equation 4-7, for PMT, but it is much easier to use a financial calculator or spreadsheet. With a financial calculator, we insert values as shown below to get the required payments, $23,739.64. Inputs: Output: 5 6 100000 N I/YR PV 0 PMT FV –23739.64 With Excel, you would use the PMT function: =PMT(I,N,PV,FV) = PMT(0.06, 5,100000,0) = −$23,739.64. Thus, we see that the borrower must pay the lender $23,739.64 per year for the next 5 years. Each payment will consist of two parts—part interest and part repayment of principal. This breakdown is shown in the amortization schedule given in Figure 4-11. The interest component is relatively high in the first year, but it declines as the loan balance decreases. For tax purposes, the borrower would deduct the interest component while the lender would report the same amount as taxable income. Over the 5 years, the lender will earn 6% on its investment and also recover the amount of its investment. Self-Test Consider again the example in Figure 4-11. If the loan were amortized over 5 years with 60 equal monthly payments, how much would each payment be, and how would the first payment be divided between interest and principal? (Each payment would be $1,933.28; the first payment would have $500 of interest and $1,433.28 of principal repayment.) Suppose you borrowed $30,000 on a student loan at a rate of 8% and now must repay it in three equal installments at the end of each of the next 3 years. How large would your payments be, how much of the first payment would represent 21 The word amortized comes from the Latin mors, meaning “death,” so an amortized loan is one that is “killed off” over time. Chapter 4: Time Value of Money FIGURE 4-11 159 Loan Amortization Schedule, $100,000 at 6% for 5 Years Year 1 2 3 4 5 Amount borrowed: Years: Rate: PMT: Beginning Amount (1) $100,000.00 $82,260.36 $63,456.34 $43,524.08 $22,395.89 $100,000 5 6% $23,739.64 = PMT(C646,C645,-C644) Repayment of Principalb Payment Interesta (2) (2) - (3) = (4) (3) $23,739.64 $23,739.64 $23,739.64 $23,739.64 $23,739.64 $6,000.00 $4,935.62 $3.807.38 $2,611.44 $1,343.75 Ending Balance (1) - (4) = (5) $17,739.64 $18,804.02 $19,932.26 $21,128.20 $22,395.89 $82,260.36 $63,456.34 $43,524.08 $22,395.89 $0.00 a Interest in each period is calculated by multiplying the loan balance at the beginning of the year by the interest rate. Therefore, interest in Year 1 is $100,000(0.06) = $6,000; in Year 2 it is $82,260.36(0.06) = $4,935.62; and so on. b Repayment of principal is the $23,739.64 annual payment minus the interest charges for the year, $17,739.64 for Year 1. interest and how much would be principal, and what would your ending balance be after the first year? (PMT = $11,641.01; interest = $2,400; principal = $9,241.01; balance at end of Year 1 = $20,758.99) 4.18 GROWING ANNUITIES22 Normally, an annuity is defined as a series of constant payments to be received over a specified number of periods. However, the term growing annuity is used to describe a series of payments that grow at a constant rate. Example 1: Finding a Constant Real Income resource See Ch04 Tool Kit.xls for all calculations. Growing annuities are often used in the area of financial planning, where a prospective retiree wants to determine the maximum constant real, or inflation-adjusted, withdrawals that he or she can make over a specified number of years. For example, suppose a 65-year-old is contemplating retirement, expects to live for another 20 years, has a $1 million nest egg, expects the investments to earn a nominal annual rate of 6%, expects inflation to average 3% per year, and wants to withdraw a constant real amount annually over the next 20 years so as to maintain a constant standard of living. If the first withdrawal is to be made today, what is the amount of that initial withdrawal? This problem can be solved in three ways. (1) Set up a spreadsheet model that is similar to an amortization table, where the account earns 6% per year, withdrawals rise at the 3% inflation rate, and Excel’s Goal Seek function is used to find the initial inflation-adjusted withdrawal. A zero balance will be shown at the end of the twentieth year. (2) Use a financial calculator, where we first calculate the real rate of return, This section is interesting and useful, but relatively technical. It can be omitted, at the option of the instructor, without loss of continuity. 22 160 Part 2: Fixed Income Securities THE GLOBAL ECONOMIC CRISIS An Accident Waiting to Happen: Option Reset Adjustable Rate Mortgages Option reset adjustable rate mortgages (ARMs) give the borrower some choices regarding the initial monthly payment. One popular option ARM allowed borrowers to make a monthly payment equal to only half of the interest due in the first month. Because the monthly payment was less than the interest charge, the loan balance grew each month. When the loan balance exceeded 110% of the original principal, the monthly payment was reset to fully amortize the now-larger loan at the prevailing market interest rates. Here’s an example. Someone borrows $325,000 for 30 years at an initial rate of 7%. The interest accruing in the first month is (7%/12)($325,000) = $1,895.83. Therefore, the initial monthly payment is 50%($1,895.83) = $947.92. Another $947.92 of deferred interest is added to the loan balance, taking it up to $325,000 + $947.92 = $325,947.82. Because the loan is now larger, interest in the second month is higher, and both interest and the loan balance will continue to rise each month. The first month after the loan balance exceeds 110%($325,000) = $357,500, the contract calls for the payment to be reset so as to fully amortize the loan at the then-prevailing interest rate. First, how long would it take for the balance to exceed $357,500? Consider this from the lender’s perspective: the lender initially pays out $325,000, receives $947.92 each month, and then would receive a payment of $357,500 if the loan were payable when the balance hit that amount, with interest accruing at a 7% annual rate and with monthly compounding. We enter these values into a financial calculator: I = 7%/12, PV = −325000, PMT = 947.92, and FV = 357500. We solve for N = 31.3 months, rounded up to 32 months. Thus, the borrower will make 32 payments of $947.92 before the ARM resets. The payment after the reset depends upon the terms of the original loan and the market interest rate at the time of the reset. For many borrowers, the initial rate was a lower-than-market “teaser” rate, so a higherthan-market rate would be applied to the remaining balance. For this example, we will assume that the original rate wasn’t a teaser and that the rate remains at 7%. Keep in mind, though, that for many borrowers the reset rate was higher than the initial rate. The balance after the 32nd payment can be found as the future value of the original loan and the 32 monthly payments, so we enter these values in the financial calculator: N = 32, I = 7%/12, PMT = 947.92, PV = −325000, and then solve for FV = $358,242.84. The number of remaining payments to amortize the $358,424.84 loan balance is 360 − 32 = 328, so the amount of each payment is found by setting up the calculator as: N = 328, I = 7%/12, PV = 358242.84 and FV = 0. Solving, we find that PMT = $2,453.94. Even if interest rates don’t change, the monthly payment jumps from $947.92 to $2,453.94 and would increase even more if interest rates were higher at the reset. This is exactly what happened to millions of American homeowners who took out option reset ARMS in the early 2000s. When large numbers of resets began in 2007, defaults ballooned. The accident caused by option reset ARMs didn’t wait very long to happen! adjusted for inflation, and use it for I/YR when finding the payment for an annuity due. (3) Use a relatively complicated and obtuse formula to find this same amount.23 We will focus on the first two approaches. For example, the formula used to find the payment of a growing annuity due is shown below. If g = annuity growth rate and r = nominal rate of return on investment, then 23 PVIF of a growing annuity due ¼ PVIFGADue ¼ f1  ½ð1 þ gÞ=ð1 þ rÞN g½ð1 þ rÞ=ðr  gÞ PMT ¼ PV=PVIFGADue where PVIF denotes “present value interest factor.” Similar formulas are available for growing ordinary annuities. Chapter 4: Time Value of Money 161 We illustrate the spreadsheet approach in the chapter model, Ch04 Tool Kit.xls. The spreadsheet model provides the most transparent picture of what’s happening, since it shows the value of the retirement portfolio, the portfolio’s annual earnings, and each withdrawal over the 20-year planning horizon—especially if you include a graph. A picture is worth a thousand numbers, and graphs make it easy to explain the situation to people who are planning their financial futures. To implement the calculator approach, we first find the expected real rate of return, where rr is the real rate of return and rNOM the nominal rate of return. The real rate of return is the return that we would see if there were no inflation. We calculate the real rate as: Real rate ¼ rr ¼ ½ð1 þ rNOM Þ=ð1 þ InflationÞ − 1:0 (4-15) ¼ ½1:06=1:03 − 1:0 ¼ 0:029126214 ¼ 2:9126214% (4-15) Using this real rate of return, we solve the annuity due problem exactly as we did earlier in the chapter. We set the calculator to Begin Mode, after which we input N=20, I/YR = real rate=2.9126214, PV=−1,000,000, and FV=0; then we press PMT to get $64,786.88. This is the amount of the initial withdrawal at Time 0 (today), and future withdrawals will increase at the inflation rate of 3%. These withdrawals, growing at the inflation rate, will provide the retiree with a constant real income over the next 20 years—provided the inflation rate and the rate of return do not change. In our example we assumed that the first withdrawal would be made immediately. The procedure would be slightly different if we wanted to make end-of-year withdrawals. First, we would set the calculator to End Mode. Second, we would enter the same inputs into the calculator as just listed, including the real interest rate for I/YR. The calculated PMT would be $66,673.87. However, that value is in beginning-of-year terms, and since inflation of 3% will occur during the year, we must make the following adjustment to find the inflation-adjusted initial withdrawal: Initial end-of -year withdrawal ¼ $66;673:87ð1 þ InflationÞ ¼ $66;673:87ð1:03Þ ¼ $68;674:09: Thus the first withdrawal at the end of the year would be $68,674.09; it would grow by 3% per year; and after the 20th withdrawal (at the end of the 20th year) the balance in the retirement fund would be zero. We also demonstrate the solution for this end-of-year payment example in Ch04 Tool Kit.xls. There we set up a table showing the beginning balance, the annual withdrawals, the annual earnings, and the ending balance for each of the 20 years. This analysis confirms the $68,674.09 initial end-of-year withdrawal derived previously. Example 2: Initial Deposit to Accumulate a Future Sum As another example of growing annuities, suppose you need to accumulate $100,000 in 10 years. You plan to make a deposit in a bank now, at Time 0, and then make 9 more deposits at the beginning of each of the following 9 years, for a total of 10 deposits. The bank pays 6% interest, you expect inflation to be 2% per year, and you plan to increase your annual deposits at the inflation rate. How much must you deposit initially? First, we calculate the real rate: Real rate = rr = [1.06/1.02] − 1.0 = 0.0392157 = 3.9215686% 162 Part 2: Fixed Income Securities Next, since inflation is expected to be 2% per year, in 10 years the target $100,000 will have a real value of $100,000/(1 + 0.02)10 = $82,034.83. Now we can find the size of the required initial payment by setting a financial calculator to the Begin Mode and then inputting N = 10, I/YR = 3.9215686, PV = 0, and FV = 82,034.83. Then, when we press the PMT key, we get PMT = −6,598.87. Thus, a deposit of $6,598.87 made at time 0 and growing by 2% per year will accumulate to $100,000 by Year 10 if the interest rate is 6%. Again, this result is confirmed in the chapter’s Tool Kit. The key to this analysis is to express I/YR, FV, and PMT in real, not nominal, terms. Self-Test Differentiate between a “regular” and a “growing” annuity. What three methods can be used to deal with growing annuities? If the nominal interest rate is 10% and the expected inflation rate is 5%, what is the expected real rate of return? (4.7619%) Summary Most financial decisions involve situations in which someone makes a payment at one point in time and receives money later. Dollars paid or received at two different points in time are different, and this difference is dealt with using time value of money (TVM) analysis. • • • • • • • • • Compounding is the process of determining the future value (FV) of a cash flow or a series of cash flows. The compounded amount, or future value, is equal to the beginning amount plus interest earned. Future value of a single payment = FVN = PV(1 + I)N. Discounting is the process of finding the present value (PV) of a future cash flow or a series of cash flows; discounting is the reciprocal, or reverse, of compounding. FVN Present value of a payment received at the end of Time N ¼ PV ¼ : ðI þ IÞN An annuity is defined as a series of equal periodic payments (PMT) for a specified number of periods. An annuity whose payments occur at the end of each period is called an ordinary   annuity. ð1 þ IÞN 1  : Future value of an (ordinary) annuity FVAN ¼ PMT I I   1 1 Present value of an (ordinary) annuity PVAN ¼ PMT  : I Ið1 þ IÞN If payments occur at the beginning of the periods rather than at the end, then we have an annuity due. The PV of each payment is larger, because each payment is discounted back one year less, so the PV of the annuity is also larger. Similarly, the FV of the annuity due is larger because each payment is compounded for an extra year. The following formulas can be used to convert the PV and FV of an ordinary annuity to an annuity due: Chapter 4: Time Value of Money 163 PVAdue ¼ PVAordinary ð1 þ IÞ FVAdue ¼ FVAordinary ð1 þ IÞ • A perpetuity is an annuity with an infinite number of payments. Value of a perpetuity ¼ • • • PMT I To find the PV or FV of an uneven series, find the PV or FV of each individual cash flow and then sum them. If you know the cash flows and the PV (or FV) of a cash flow stream, you can determine its interest rate. When compounding occurs more frequently than once a year, the nominal rate must be converted to a periodic rate, and the number of years must be converted to periods: Periodic rateðIPER Þ ¼ Nominal annual rate ÷ Periods per year Number of Periods ¼ Years × Periods per year • • The periodic rate and number of periods is used for calculations and is shown on time lines. If you are comparing the costs of alternative loans that require payments more than once a year, or the rates of return on investments that pay interest more than once a year, then the comparisons should be based on effective (or equivalent) rates of return. Here is the formula:   INOM M −1:0 EAR ¼ EFF% ¼ 1 þ M The general equation for finding the future value of a current cash flow (PV) for any number of compounding periods per year is   INOM MN FVN ¼ PVð1 þ IPER ÞNumber of periods ¼ PV 1 þ M where INOM ¼ Nominal quoted interest rate M ¼ Number of compounding periods per year N ¼ Number of years • • • • • An amortized loan is one that is paid off with equal payments over a specified period. An amortization schedule shows how much of each payment constitutes interest, how much is used to reduce the principal, and the unpaid balance at the end of each period. The unpaid balance at Time N must be zero. A “Growing Annuity” is a stream of cash flows that grows at a constant rate for a specified number of years. The present and future values of growing annuities can be found with relatively complicated formulas or, more easily, with an Excel model. Web Extension 4A explains the tabular approach. Web Extension 4B provides derivations of the annuity formulas. Web Extension 4C explains continuous compounding. 164 Part 2: Fixed Income Securities Questions (4–1) Define each of the following terms: a. PV; I; INT; FVN; PVAN; FVAN; PMT; M; INOM b. Opportunity cost rate c. Annuity; lump-sum payment; cash flow; uneven cash flow stream d. Ordinary (or deferred) annuity; annuity due e. Perpetuity; consol f. Outflow; inflow; time line; terminal value g. Compounding; discounting h. Annual, semiannual, quarterly, monthly, and daily compounding i. Effective annual rate (EAR or EFF%); nominal (quoted) interest rate; APR; periodic rate j. Amortization schedule; principal versus interest component of a payment; amortized loan (4–2) What is an opportunity cost rate? How is this rate used in discounted cash flow analysis, and where is it shown on a time line? Is the opportunity rate a single number that is used to evaluate all potential investments? (4–3) An annuity is defined as a series of payments of a fixed amount for a specific number of periods. Thus, $100 a year for 10 years is an annuity, but $100 in Year 1, $200 in Year 2, and $400 in Years 3 through 10 does not constitute an annuity. However, the entire series does contain an annuity. Is this statement true or false? (4–4) If a firm’s earnings per share grew from $1 to $2 over a 10-year period, the total growth would be 100%, but the annual growth rate would be less than 10%. True or false? Explain. (4–5) Would you rather have a savings account that pays 5% interest compounded semiannually or one that pays 5% interest compounded daily? Explain. Self-Test Problems (ST–1) Future Value Solutions Appear in Appendix A Assume that 1 year from now you plan to deposit $1,000 in a savings account that pays a nominal rate of 8%. a. If the bank compounds interest annually, how much will you have in your account 4 years from now? b. What would your balance be 4 years from now if the bank used quarterly compounding rather than annual compounding? c. Suppose you deposited the $1,000 in 4 payments of $250 each at the end of Years 1, 2, 3, and 4. How much would you have in your account at the end of Year 4, based on 8% annual compounding? d. Suppose you deposited 4 equal payments in your account at the end of Years 1, 2, 3, and 4. Assuming an 8% interest rate, how large would each of your payments have to be for you to obtain the same ending balance as you calculated in part a? (ST–2) Time Value of Money Assume that 4 years from now you will need $1,000. Your bank compounds interest at an 8% annual rate. Chapter 4: Time Value of Money 165 a. How much must you deposit 1 year from now to have a balance of $1,000 at Year 4? b. If you want to make equal payments at the end of Years 1 through 4 to accumulate the $1,000, how large must each of the 4 payments be? c. If your father were to offer either to make the payments calculated in part b ($221.92) or to give you a lump sum of $750 one year from now, which would you choose? d. If you will have only $750 at the end of Year 1, what interest rate, compounded annually, would you have to earn to have the necessary $1,000 at Year 4? e. Suppose you can deposit only $186.29 each at the end of Years 1 through 4, but you still need $1,000 at the end of Year 4. What interest rate, with annual compounding, is required to achieve your goal? f. To help you reach your $1,000 goal, your father offers to give you $400 one year from now. You will get a part-time job and make 6 additional deposits of equal amounts each 6 months thereafter. If all of this money is deposited in a bank that pays 8%, compounded semiannually, how large must each of the 6 deposits be? g. What is the effective annual rate being paid by the bank in part f? (ST–3) Effective Annual Rates Problems Bank A pays 8% interest, compounded quarterly, on its money market account. The managers of Bank B want its money market account’s effective annual rate to equal that of Bank A, but Bank B will compound interest on a monthly basis. What nominal, or quoted, rate must Bank B set? Answers Appear in Appendix B EASY PROBLEMS 1–8 (4–1) Future Value of a Single Payment (4–2) Present Value of a Single Payment (4–3) Interest Rate on a Single Payment (4–4) Number of Periods of a Single Payment (4–5) Number of Periods for an Annuity (4–6) Future Value: Ordinary Annuity versus Annuity Due (4–7) Present and Future Value of an Uneven Cash Flow Stream If you deposit $10,000 in a bank account that pays 10% interest annually, how much will be in your account after 5 years? What is the present value of a security that will pay $5,000 in 20 years if securities of equal risk pay 7% annually? Your parents will retire in 18 years. They currently have $250,000, and they think they will need $1 million at retirement. What annual interest rate must they earn to reach their goal, assuming they don’t save any additional funds? If you deposit money today in an account that pays 6.5% annual interest, how long will it take to double your money? You have $42,180.53 in a brokerage account, and you plan to deposit an additional $5,000 at the end of every future year until your account totals $250,000. You expect to earn 12% annually on the account. How many years will it take to reach your goal? What is the future value of a 7%, 5-year ordinary annuity that pays $300 each year? If this were an annuity due, what would its future value be? An investment will pay $100 at the end of each of the next 3 years, $200 at the end of Year 4, $300 at the end of Year 5, and $500 at the end of Year 6. If other investments of equal risk earn 8% annually, what is this investment’s present value? Its future value? 166 Part 2: Fixed Income Securities (4–8) Annuity Payment and EAR You want to buy a car, and a local bank will lend you $20,000. The loan would be fully amortized over 5 years (60 months), and the nominal interest rate would be 12%, with interest paid monthly. What is the monthly loan payment? What is the loan’s EFF%? INTERMEDIATE PROBLEMS 9–29 (4–9) Present and Future Values of Single Cash Flows for Different Periods Find the following values, using the equations, and then work the problems using a financial calculator to check your answers. Disregard rounding differences. (Hint: If you are using a financial calculator, you can enter the known values and then press the appropriate key to find the unknown variable. Then, without clearing the TVM register, you can “override” the variable that changes by simply entering a new value for it and then pressing the key for the unknown variable to obtain the second answer. This procedure can be used in parts b and d, and in many other situations, to see how changes in input variables affect the output variable.) a. b. c. d. (4–10) Present and Future Values of Single Cash Flows for Different Interest Rates (4–11) Time for a Lump Sum to Double Use both the TVM equations and a financial calculator to find the following values. See the Hint for Problem 4-9. a. b. c. d. (4–12) An initial $500 compounded for 10 years at 6% An initial $500 compounded for 10 years at 12% The present value of $500 due in 10 years at a 6% discount rate The present value of $500 due in 10 years at a 12% discount rate To the closest year, how long will it take $200 to double if it is deposited and earns the following rates? [Notes: (1) See the Hint for Problem 4-9. (2) This problem cannot be solved exactly with some financial calculators. For example, if you enter PV = –200, PMT = 0, FV = 400, and I = 7 in an HP-12C and then press the N key, you will get 11 years for part a. The correct answer is 10.2448 years, which rounds to 10, but the calculator rounds up. However, the HP-10B gives the exact answer.] a. b. c. d. Future Value of an Annuity An initial $500 compounded for 1 year at 6% An initial $500 compounded for 2 years at 6% The present value of $500 due in 1 year at a discount rate of 6% The present value of $500 due in 2 years at a discount rate of 6% 7% 10% 18% 100% Find the future value of the following annuities. The first payment in these annuities is made at the end of Year 1, so they are ordinary annuities. (Notes: See the Hint to Problem 4-9. Also, note that you can leave values in the TVM register, switch to Begin Mode, press FV, and find the FV of the annuity due.) a. b. c. d. $400 per year for 10 years at 10% $200 per year for 5 years at 5% $400 per year for 5 years at 0% Now rework parts a, b, and c assuming that payments are made at the beginning of each year; that is, they are annuities due. Chapter 4: Time Value of Money (4–13) Present Value of an Annuity (4–14) Uneven Cash Flow Stream 167 Find the present value of the following ordinary annuities (see the Notes to Problem 4-12). a. b. c. d. $400 per year for 10 years at 10% $200 per year for 5 years at 5% $400 per year for 5 years at 0% Now rework parts a, b, and c assuming that payments are made at the beginning of each year; that is, they are annuities due. Find the present values of the following cash flow streams. The appropriate interest rate is 8%. (Hint: It is fairly easy to work this problem dealing with the individual cash flows. However, if you have a financial calculator, read the section of the manual that describes how to enter cash flows such as the ones in this problem. This will take a little time, but the investment will pay huge dividends throughout the course. Note that, when working with the calculator’s cash flow register, you must enter CF0 = 0. Note also that it is quite easy to work the problem with Excel, using procedures described in the Chapter 4 Tool Kit.) Year 1 2 3 4 5 Cash Stream A $100 400 400 400 300 Cash Stream B $300 400 400 400 100 b. What is the value of each cash flow stream at a 0% interest rate? (4–15) Effective Rate of Interest (4–16) Future Value for Various Compounding Periods (4–17) Present Value for Various Compounding Periods (4–18) Future Value of an Annuity for Various Compounding Periods Find the interest rate (or rates of return) in each of the following situations. a. b. c. d. You borrow $700 and promise to pay back $749 at the end of 1 year. You lend $700 and receive a promise to be paid $749 at the end of 1 year. You borrow $85,000 and promise to pay back $201,229 at the end of 10 years. You borrow $9,000 and promise to make payments of $2,684.80 at the end of each of the next 5 years. Find the amount to which $500 will grow under each of the following conditions. a. b. c. d. 12% 12% 12% 12% compounded compounded compounded compounded annually for 5 years semiannually for 5 years quarterly for 5 years monthly for 5 years Find the present value of $500 due in the future under each of the following conditions. a. 12% nominal rate, semiannual compounding, discounted back 5 years b. 12% nominal rate, quarterly compounding, discounted back 5 years c. 12% nominal rate, monthly compounding, discounted back 1 year Find the future values of the following ordinary annuities. a. FV of $400 each 6 months for 5 years at a nominal rate of 12%, compounded semiannually b. FV of $200 each 3 months for 5 years at a nominal rate of 12%, compounded quarterly 168 Part 2: Fixed Income Securities c. The annuities described in parts a and b have the same total amount of money paid into them during the 5-year period, and both earn interest at the same nominal rate, yet the annuity in part b earns $101.75 more than the one in part a over the 5 years. Why does this occur? (4–19) Effective versus Nominal Interest Rates Universal Bank pays 7% interest, compounded annually, on time deposits. Regional Bank pays 6% interest, compounded quarterly. a. Based on effective interest rates, in which bank would you prefer to deposit your money? b. Could your choice of banks be influenced by the fact that you might want to withdraw your funds during the year as opposed to at the end of the year? In answering this question, assume that funds must be left on deposit during an entire compounding period in order for you to receive any interest. (4–20) Amortization Schedule (4–21) Growth Rates a. Set up an amortization schedule for a $25,000 loan to be repaid in equal installments at the end of each of the next 5 years. The interest rate is 10%. b. How large must each annual payment be if the loan is for $50,000? Assume that the interest rate remains at 10% and that the loan is still paid off over 5 years. c. How large must each payment be if the loan is for $50,000, the interest rate is 10%, and the loan is paid off in equal installments at the end of each of the next 10 years? This loan is for the same amount as the loan in part b, but the payments are spread out over twice as many periods. Why are these payments not half as large as the payments on the loan in part b? Sales for Hanebury Corporation’s just-ended year were $12 million. Sales were $6 million 5 years earlier. a. At what rate did sales grow? b. Suppose someone calculated the sales growth for Hanebury in part a as follows: “Sales doubled in 5 years. This represents a growth of 100% in 5 years; dividing 100% by 5 results in an estimated growth rate of 20% per year.” Explain what is wrong with this calculation. (4–22) Expected Rate of return (4–23) Effective Rate of Interest (4–24) Required Lump-Sum Payment Washington-Pacific invested $4 million to buy a tract of land and plant some young pine trees. The trees can be harvested in 10 years, at which time W-P plans to sell the forest at an expected price of $8 million. What is W-P’s expected rate of return? A mortgage company offers to lend you $85,000; the loan calls for payments of $8,273.59 at the end of each year for 30 years. What interest rate is the mortgage company charging you? To complete your last year in business school and then go through law school, you will need $10,000 per year for 4 years, starting next year (that is, you will need to withdraw the first $10,000 one year from today). Your rich uncle offers to put you through school, and he will deposit in a bank paying 7% interest a sum of money that is sufficient to provide the 4 payments of $10,000 each. His deposit will be made today. a. How large must the deposit be? b. How much will be in the account immediately after you make the first withdrawal? After the last withdrawal? Chapter 4: Time Value of Money (4–25) Repaying a Loan (4–26) Reaching a Financial Goal (4–27) Present Value of a Perpetuity (4–28) PV and Effective Annual Rate (4–29) Loan Amortization 169 While Mary Corens was a student at the University of Tennessee, she borrowed $12,000 in student loans at an annual interest rate of 9%. If Mary repays $1,500 per year, then how long (to the nearest year) will it take her to repay the loan? You need to accumulate $10,000. To do so, you plan to make deposits of $1,250 per year—with the first payment being made a year from today—into a bank account that pays 12% annual interest. Your last deposit will be less than $1,250 if less is needed to round out to $10,000. How many years will it take you to reach your $10,000 goal, and how large will the last deposit be? What is the present value of a perpetuity of $100 per year if the appropriate discount rate is 7%? If interest rates in general were to double and the appropriate discount rate rose to 14%, what would happen to the present value of the perpetuity? Assume that you inherited some money. A friend of yours is working as an unpaid intern at a local brokerage firm, and her boss is selling securities that call for 4 payments of $50 (1 payment at the end of each of the next 4 years) plus an extra payment of $1,000 at the end of Year 4. Your friend says she can get you some of these securities at a cost of $900 each. Your money is now invested in a bank that pays an 8% nominal (quoted) interest rate but with quarterly compounding. You regard the securities as being just as safe, and as liquid, as your bank deposit, so your required effective annual rate of return on the securities is the same as that on your bank deposit. You must calculate the value of the securities to decide whether they are a good investment. What is their present value to you? Assume that your aunt sold her house on December 31, and to help close the sale she took a second mortgage in the amount of $10,000 as part of the payment. The mortgage has a quoted (or nominal) interest rate of 10%; it calls for payments every 6 months, beginning on June 30, and is to be amortized over 10 years. Now, 1 year later, your aunt must inform the IRS and the person who bought the house about the interest that was included in the two payments made during the year. (This interest will be income to your aunt and a deduction to the buyer of the house.) To the closest dollar, what is the total amount of interest that was paid during the first year? CHALLENGING PROBLEMS 30–34 (4–30) Loan Amortization (4–31) Nonannual Compounding (4–32) Nominal Rate of return Your company is planning to borrow $1 million on a 5-year, 15%, annual payment, fully amortized term loan. What fraction of the payment made at the end of the second year will represent repayment of principal? a. It is now January 1. You plan to make a total of 5 deposits of $100 each, one every 6 months, with the first payment being made today. The bank pays a nominal interest rate of 12% but uses semiannual compounding. You plan to leave the money in the bank for 10 years. How much will be in your account after 10 years? b. You must make a payment of $1,432.02 in 10 years. To get the money for this payment, you will make 5 equal deposits, beginning today and for the following 4 quarters, in a bank that pays a nominal interest rate of 12% with quarterly compounding. How large must each of the 5 payments be? Anne Lockwood, manager of Oaks Mall Jewelry, wants to sell on credit, giving customers 3 months to pay. However, Anne will have to borrow from her bank to carry the accounts receivable. The bank will charge a nominal rate of 15% and will 170 Part 2: Fixed Income Securities compound monthly. Anne wants to quote a nominal rate to her customers (all of whom are expected to pay on time) that will exactly offset her financing costs. What nominal annual rate should she quote to her credit customers? (4–33) Required Annuity Payments (4–34) Growing Annuity Payments Assume that your father is now 50 years old, that he plans to retire in 10 years, and that he expects to live for 25 years after he retires—that is, until age 85. He wants his first retirement payment to have the same purchasing power at the time he retires as $40,000 has today. He wants all of his subsequent retirement payments to be equal to his first retirement payment. (Do not let the retirement payments grow with inflation: Your father realizes that the real value of his retirement income will decline year by year after he retires.) His retirement income will begin the day he retires, 10 years from today, and he will then receive 24 additional annual payments. Inflation is expected to be 5% per year from today forward. He currently has $100,000 saved up; and he expects to earn a return on his savings of 8% per year with annual compounding. To the nearest dollar, how much must he save during each of the next 10 years (with equal deposits being made at the end of each year, beginning a year from today) to meet his retirement goal? (Note: Neither the amount he saves nor the amount he withdraws upon retirement is a growing annuity.) You want to accumulate $1 million by your retirement date, which is 25 years from now. You will make 25 deposits in your bank, with the first occurring today. The bank pays 8% interest, compounded annually. You expect to get annual raises of 3%, which will offset inflation, and you will let the amount you deposit each year also grow by 3% (i.e., your second deposit will be 3% greater than your first, the third will be 3% greater than the second, etc.). How much must your first deposit be if you are to meet your goal? SPREADSHEET PROBLEM (4-35) Build a Model: The Time Value of Money resource See Ch04 Tool Kit.xls for all calculations. Start with the partial model in the file Ch04 P35 Build a Model.xls from the textbook’s Web site. Answer the following questions, using a spreadsheet model to do the calculations. a. Find the FV of $1,000 invested to earn 10% annually 5 years from now. Answer this question first by using a math formula and then by using the Excel function wizard. b. Now create a table that shows the FV at 0%, 5%, and 20% for 0, 1, 2, 3, 4, and 5 years. Then create a graph with years on the horizontal axis and FV on the vertical axis to display your results. c. Find the PV of $1,000 due in 5 years if the discount rate is 10% per year. Again, work the problem with a formula and also by using the function wizard. d. A security has a cost of $1,000 and will return $2,000 after 5 years. What rate of return does the security provide? e. Suppose California’s population is 30 million people and its population is expected to grow by 2% per year. How long would it take for the population to double? f. Find the PV of an ordinary annuity that pays $1,000 at the end of each of the next 5 years if the interest rate is 15%. Then find the FV of that same annuity. g. How would the PV and FV of the above annuity change if it were an annuity due rather than an ordinary annuity? h. What would the FV and PV for parts a and c be if the interest rate were 10% with semiannual compounding rather than 10% with annual compounding? Chapter 4: Time Value of Money 171 i. Find the PV and FV of an investment that makes the following end-of-year payments. The interest rate is 8%. Payment $100 200 400 Year 1 2 3 j. Suppose you bought a house and took out a mortgage for $50,000. The interest rate is 8%, and you must amortize the loan over 10 years with equal end-of-year payments. Set up an amortization schedule that shows the annual payments and the amount of each payment that repays the principal and the amount that constitutes interest expense to the borrower and interest income to the lender. 1. 2. Create a graph that shows how the payments are divided between interest and principal repayment over time. Suppose the loan called for 10 years of monthly payments, 120 payments in all, with the same original amount and the same nominal interest rate. What would the amortization schedule show now? Mini Case Assume that you are nearing graduation and have applied for a job with a local bank. As part of the bank’s evaluation process, you have been asked to take an examination that covers several financial analysis techniques. The first section of the test addresses discounted cash flow analysis. See how you would do by answering the following questions. a. Draw time lines for (1) a $100 lump sum cash flow at the end of Year 2, (2) an ordinary annuity of $100 per year for 3 years, and (3) an uneven cash flow stream of –$50, $100, $75, and $50 at the end of Years 0 through 3. b. 1. What’s the future value of an initial $100 after 3 years if it is invested in an account paying 10% annual interest? 2. What’s the present value of $100 to be received in 3 years if the appropriate interest rate is 10%? c. We sometimes need to find out how long it will take a sum of money (or anything else) to grow to some specified amount. For example, if a company’s sales are growing at a rate of 20% per year, how long will it take sales to double? d. If you want an investment to double in 3 years, what interest rate must it earn? e. What’s the difference between an ordinary annuity and an annuity due? What type of annuity is shown below? How would you change the time line to show the other type of annuity? 0 f. 1. 1 2 3 100 100 100 What’s the future value of a 3-year ordinary annuity of $100 if the appropriate interest rate is 10%? 2. What’s the present value of the annuity? 3. What would the future and present values be if the annuity were an annuity due? 172 Part 2: Fixed Income Securities g. What is the present value of the following uneven cash flow stream? The appropriate interest rate is 10%, compounded annually. 0 1 2 3 4 0 100 300 300 –50 h. 1. Define the stated (quoted) or nominal rate INOM as well as the periodic rate IPER. 2. Will the future value be larger or smaller if we compound an initial amount more often than annually—for example, every 6 months, or semiannually—holding the stated interest rate constant? Why? 3. What is the future value of $100 after 5 years under 12% annual compounding? Semiannual compounding? Quarterly compounding? Monthly compounding? Daily compounding? 4. What is the effective annual rate (EAR or EFF%)? What is the EFF% for a nominal rate of 12%, compounded semiannually? Compounded quarterly? Compounded monthly? Compounded daily? i. Will the effective annual rate ever be equal to the nominal (quoted) rate? j. 1. Construct an amortization schedule for a $1,000, 10% annual rate loan with 3 equal installments. 2. During Year 2, what is the annual interest expense for the borrower, and what is the annual interest income for the lender? k. Suppose that on January 1 you deposit $100 in an account that pays a nominal (or quoted) interest rate of 11.33463%, with interest added (compounded) daily. How much will you have in your account on October 1, or 9 months later? l. 1. What is the value at the end of Year 3 of the following cash flow stream if the quoted interest rate is 10%, compounded semiannually? 0 1 2 3 100 100 100 Years 2. What is the PV of the same stream? 3. Is the stream an annuity? 4. An important rule is that you should never show a nominal rate on a time line or use it in calculations unless what condition holds? (Hint: Think of annual compounding, when INOM = EFF% = IPER.) What would be wrong with your answers to parts (1) and (2) if you used the nominal rate of 10% rather than the periodic rate, INOM/2 = 10%/2 = 5%? m. Suppose someone offered to sell you a note calling for the payment of $1,000 in 15 months. They offer to sell it to you for $850. You have $850 in a bank time deposit that pays a 6.76649% nominal rate with daily compounding, which is a 7% effective annual interest rate, and you plan to leave the money in the bank unless you buy the note. The note is not risky—you are sure it will be paid on schedule. Should you buy the note? Check the decision in three ways: (1) by comparing your future value if you buy the note versus leaving your money in the bank; (2) by comparing the PV of the note with your current bank account; and (3) by comparing the EFF% on the note with that of the bank account. CHAPTER 5 Bonds, Bond Valuation, and Interest Rates A lot of U.S. bonds have been issued, and we mean a lot! According to the Federal Reserve, there are about $5.7 trillion of outstanding U.S. Treasury securities, more than $2.6 trillion of municipal securities, $3.7 trillion of corporate bonds, and more than $1.4 billion of foreign bonds held in the United States. Not only is the dollar amount mind-boggling, but so is the variety. Bonds come in many shapes and flavors, and one even has a negative interest rate. How can a bond have a negative rate? First, consider a bond that makes no payments before it comes due. For example, an investor might buy a bond today for $558 in exchange for the promise of $1,000 in 10 years. The investor would not receive any cash interest payments, but the 10-year increase from the original purchase price to the $1,000 repayment would provide a 6% annual return on the investment. Although there are no annual cash interest payments, the government still allows corporate issuers to deduct an imputed annual interest expense from their taxable income based on the bond’s annual appreciation in value. Thus, the company gets a tax deduction each year, even though it isn’t making actual interest payments. Berkshire Hathaway (chaired by Warren Buffett) issued bonds with a negative interest rate in 2002. Technically, Berkshire’s bonds called for a 3% interest payment, but they also had an attached warrant that would allow an investor to purchase shares of Berkshire Hathaway stock at a fixed price in the future. If the stock price rises above the specified price, then investors can profit by exercising the warrants. However, Berkshire Hathaway didn’t just give away the warrants—it required investors to make annual installment payments equal to 3.75% of the bond’s face value. Thus, investors receive a 3% interest payment but must then pay a 3.75% warrant fee, for a net interest rate of negative 0.75%. Berkshire Hathaway can deduct the 3% interest payment for tax purposes, but the 3.75% warrant fee is not taxable, further increasing Berkshire Hathaway’s annual after-tax cash flow. Think about the implications of these and other bonds as you read this chapter. Source: “Flow of Funds Accounts of the United States, Section L.2, Credit Market Debt Owed by Nonfinancial Sectors,” http://www.federalreserve.gov/releases/Z1/current/. 173 174 Part 2: Fixed Income Securities Intrinsic Value and the Cost of Debt This chapter explains bond pricing and bond risk, which affect the return demanded by a firm’s bondholders. A bondholder’s return is a cost from the company’s point of view. This cost of debt affects the firm’s weighted average cost of capital (WACC), which in turn affects the company’s intrinsic value. Therefore, it is important for all managers to understand the cost of debt, which we explain in this chapter. Net operating profit after taxes – Free cash flow (FCF) Value = FCF1 (1 + WACC)1 + FCF2 Required investments in operating capital = +… + (1 + WACC)2 FCF∞ (1 + WACC)∞ Weighted average cost of capital (WACC) Market interest rates Cost of debt Cost of equity Market risk aversion Firm’s debt/equity mix Firm’s business risk Growing companies must acquire land, buildings, equipment, inventory, and other operating assets. The debt markets are a major source of funding for such purchases. Therefore, every manager should have a working knowledge of the types of bonds that companies and government agencies issue, the terms that are contained in bond contracts, the types of risks to which both bond investors and issuers are exposed, and procedures for determining the values of and rates of return on bonds. 5.1 WHO ISSUES BONDS? resource The textbook’s Web site contains an Excel file that will guide you through the chapter’s calculations. The file for this chapter is Ch05 Tool Kit.xls, and we encourage you to open the file and follow along as you read the chapter. A bond is a long-term contract under which a borrower agrees to make payments of interest and principal, on specific dates, to the holders of the bond. For example, on January 5, 2011, MicroDrive Inc. borrowed $50 million by issuing $50 million of bonds. For convenience, we assume that MicroDrive sold 50,000 individual bonds for $1,000 each. Actually, it could have sold one $50 million bond, 10 bonds with a $5 million face value, or any other combination that totals to $50 million. In any event, MicroDrive received the $50 million, and in exchange it promised to make annual interest payments and to repay the $50 million on a specified maturity date. Chapter 5: Bonds, Bond Valuation, and Interest Rates 175 Investors have many choices when investing in bonds, but bonds are classified into four main types: Treasury, corporate, municipal, and foreign. Each type differs with respect to expected return and degree of risk. Treasury bonds, sometimes referred to as government bonds, are issued by the U.S. federal government.1 It is reasonable to assume that the federal government will make good on its promised payments, so these bonds have almost no default risk. However, Treasury bond prices decline when interest rates rise, so they are not free of all risks. Federal agencies and other government-sponsored entities (GSEs) include the Tennessee Valley Authority, the Small Business Administration, Fannie Mae, Freddie Mac, and the Federal Home Loan Bank System, among others. Agency debt and GSE debt are not officially backed by the full faith and credit of the U.S. government, but investors assume that the government implicitly guarantees this debt, so these bonds carry interest rates only slightly higher than Treasury bonds. In 2008, the implicit guarantee became much more explicit as the government placed several GSEs into conservatorship, including Fannie Mae and Freddie Mac. Corporate bonds, as the name implies, are issued by corporations. Unlike Treasury bonds, corporate bonds are exposed to default risk—if the issuing company gets into trouble, it may be unable to make the promised interest and principal payments. Different corporate bonds have different levels of default risk, depending on the issuing company’s characteristics and the terms of the specific bond. Default risk is often referred to as “credit risk,” and the larger the credit risk, the higher the interest rate the issuer must pay. Municipal bonds, or “munis,” are issued by state and local governments. Like corporate bonds, munis have default risk. However, munis offer one major advantage: The interest earned on most municipal bonds is exempt from federal taxes and also from state taxes if the holder is a resident of the issuing state. Consequently, municipal bonds carry interest rates that are considerably lower than those on corporate bonds with the same default risk. Foreign bonds are issued by foreign governments or foreign corporations. Foreign corporate bonds are, of course, exposed to default risk, and so are some foreign government bonds. An additional risk exists if the bonds are denominated in a currency other than that of the investor’s home currency. For example, if a U.S. investor purchases a corporate bond denominated in Japanese yen and if the yen subsequently falls relative to the dollar, then the investor will lose money even if the company does not default on its bonds. Self-Test What is a bond? What are the four main types of bonds? Why are U.S. Treasury bonds not riskless? To what types of risk are investors of foreign bonds exposed? 5.2 KEY CHARACTERISTICS OF BONDS Although all bonds have some common characteristics, they do not always have identical contractual features, as described below. 1 The U.S. Treasury actually issues three types of securities: “bills,” “notes,” and “bonds.” A bond makes an equal payment every 6 months until it matures, at which time it makes an additional lump-sum payment. If the maturity at the time of issue is less than 10 years, the security is called a note rather than a bond. A T-bill has a maturity of 52 weeks or less at the time of issue, and it makes no payments at all until it matures. Thus, T-bills are sold initially at a discount to their face, or maturity, value. 176 Part 2: Fixed Income Securities THE GLOBAL ECONOMIC CRISIS Betting With or Against the U.S. Government: The Case of Treasury Bond Credit Default Swaps It might be hard to believe, but there is actually a market for U.S. Treasury bond insurance. In early 2009, a credit default swap (CDS) on a 5-year T-bond was selling for 71 basis points (a basis point is 1 percentage point). This means that you could pay $7.10 a year to a counterparty who would promise to insure $1,000 of T-bond principal against default. Considering that the T-bond pays only $18.75 a year in interest, the insurance would eat up a lot of the annual interest for an investor who owned the bond. However, most of the trading in this CDS is by speculators and hedgers who don’t even own the T-bond but are simply betting for or against the financial soundness of the U.S. government. But it does make you wonder: “If the United States fails, who will be around to pay off the CDS?"! Par Value The par value is the stated face value of the bond; for illustrative purposes, we generally assume a par value of $1,000. In practice, some bonds have par values that are multiples of $1,000 (for example, $5,000) and some have par values of less than $1,000 (Treasury bonds can be purchased in multiples of $100). The par value generally represents the amount of money the firm borrows and promises to repay on the maturity date. Coupon Interest Rate resource An excellent site for information on many types of bonds is the Yahoo! Finance bond site, which can be found at http:// finance.yahoo.com/ bonds. The site has a great deal of information about corporates, municipals, Treasuries, and bond funds. It includes free bond searches, through which the user specifies the attributes desired in a bond and then the search returns the publicly traded bonds meeting the criteria. The site also includes a bond calculator and an excellent glossary of bond terminology. MicroDrive’s bonds require the company to pay a fixed number of dollars of interest every year (or, more typically, every 6 months). When this coupon payment, as it is called, is divided by the par value, the result is the coupon interest rate. For example, MicroDrive’s bonds have a $1,000 par value, and they pay $100 in interest each year. The bond’s coupon interest is $100, so its coupon interest rate is $100/$1,000 = 10%. The coupon payment, which is fixed at the time the bond is issued, remains in force during the life of the bond.2 Typically, at the time a bond is issued, its coupon payment is set at a level that will enable the bond to be issued at or near its par value. In some cases, a bond’s coupon payment will vary over time. For these floatingrate bonds, the coupon rate is set for, say, the initial 6-month period, after which it is adjusted every 6 months based on some market rate. Some corporate issues are tied to the Treasury bond rate; other issues are tied to other rates, such as LIBOR (the London Interbank Offered Rate). Many additional provisions can be included in floating-rate issues. For example, some are convertible to fixed-rate debt, whereas others have upper and lower limits (“caps” and “floors”) on how high or low the rate can go. Floating-rate debt is popular with investors who are worried about the risk of rising interest rates, since the interest paid on such bonds increases whenever market At one time, bonds literally had a number of small coupons attached to them, and on each interest payment date the owner would clip off the coupon for that date and either cash it at the bank or mail it to the company’s paying agent, who would then mail back a check for the interest. For example, a 30-year, semiannual bond would start with 60 coupons. Today, most new bonds are registered—no physical coupons are involved, and interest checks are mailed automatically to the registered owners. 2 Chapter 5: Bonds, Bond Valuation, and Interest Rates resource For more on zero coupon bonds, including U.S. Treasury STRIP bonds, see Web Extension 5A on the textbook’s Web site. 177 rates rise. This causes the market value of the debt to be stabilized, and it also provides institutional buyers, such as banks, with income that is better geared to their own obligations. Banks’ deposit costs rise with interest rates, so the income on floating-rate loans they have made rises at the same time as their deposit costs rise. The savings and loan industry was almost destroyed as a result of its former practice of making fixed-rate mortgage loans but borrowing on floating-rate terms. If you are earning 6% fixed but paying 10% floating (which they were), you will soon go bankrupt (which they did). Moreover, floating-rate debt appeals to corporations that want to issue long-term debt without committing themselves to paying a historically high interest rate for the entire life of the loan. Some bonds pay no coupons at all but are offered at a substantial discount below their par values and hence provide capital appreciation rather than interest income. These securities are called zero coupon bonds (“zeros”). Most zero coupon bonds are Treasury bonds, although a few corporations, such as Coca-Cola, have zero coupon bonds outstanding. Some bonds are issued with a coupon rate too low for the bond to be issued at par, so the bond is issued at a price less than its par value. In general, any bond originally offered at a price significantly below its par value is called an original issue discount (OID) bond. Some bonds don’t pay cash coupons but pay coupons consisting of additional bonds (or a percentage of an additional bond). These are called payment-in-kind bonds, or just PIK bonds. PIK bonds are usually issued by companies with cash flow problems, which makes them risky. Some bonds have a step-up provision: If the company’s bond rating is downgraded, then it must increase the bond’s coupon rate. Step-ups are more popular in Europe than in the United States, but that is beginning to change. Note that a stepup is quite dangerous from the company’s standpoint. The downgrade means that it is having trouble servicing its debt, and the step-up will exacerbate the problem. This combination has led to a number of bankruptcies. Maturity Date Bonds generally have a specified maturity date on which the par value must be repaid. MicroDrive bonds issued on January 5, 2011, will mature on January 5, 2026; thus, they have a 15-year maturity at the time they are issued. Most bonds have original maturities (the maturity at the time the bond is issued) ranging from 10 to 40 years, but any maturity is legally permissible.3 Of course, the effective maturity of a bond declines each year after it has been issued. Thus, MicroDrive’s bonds have a 15-year original maturity, but in 2012, a year later, they will have a 14-year maturity, and so on. Provisions to Call or Redeem Bonds Most corporate bonds contain a call provision, which gives the issuing corporation the right to call the bonds for redemption.4 The call provision generally states that the company must pay the bondholders an amount greater than the par value if they are called. The additional sum, which is termed a call premium, is often set equal to 1 year’s interest if the bonds are called during the first year, and the In July 1993, Walt Disney Co., attempting to lock in a low interest rate, issued the first 100-year bonds to be sold by any borrower in modern times. Soon after, Coca-Cola became the second company to stretch the meaning of “long-term bond” by selling $150 million of 100-year bonds. 3 A majority of municipal bonds also contain call provisions. Although the U.S. Treasury no longer issues callable bonds, some past Treasury issues were callable. 4 178 Part 2: Fixed Income Securities premium declines at a constant rate of INT/N each year thereafter (where INT = annual interest and N = original maturity in years). For example, the call premium on a $1,000 par value, 10-year, 10% bond would generally be $100 if it were called during the first year, $90 during the second year (calculated by reducing the $100, or 10%, premium by one-tenth), and so on. However, bonds are often not callable until several years (generally 5 to 10) after they are issued. This is known as a deferred call, and the bonds are said to have call protection. Suppose a company sold bonds when interest rates were relatively high. Provided the issue is callable, the company could sell a new issue of low-yielding securities if and when interest rates drop. It could then use the proceeds of the new issue to retire the high-rate issue and thus reduce its interest expense. This process is called a refunding operation. A call provision is valuable to the firm but potentially detrimental to investors. If interest rates go up, the company will not call the bond, and the investor will be stuck with the original coupon rate on the bond, even though interest rates in the economy have risen sharply. However, if interest rates fall, the company will call the bond and pay off investors, who then must reinvest the proceeds at the current market interest rate, which is lower than the rate they were getting on the original bond. In other words, the investor loses when interest rates go up but doesn’t reap the gains when rates fall. To induce an investor to take this type of risk, a new issue of callable bonds must provide a higher coupon rate than an otherwise similar issue of noncallable bonds. Bonds that are redeemable at par at the holder’s option protect investors against a rise in interest rates. If rates rise, the price of a fixed-rate bond declines. However, if holders have the option of turning their bonds in and having them redeemed at par, then they are protected against rising rates. If interest rates have risen, holders will turn in the bonds and reinvest the proceeds at a higher rate. Event risk is the chance that some sudden event will occur and increase the credit risk of a company, hence lowering the firm’s bond rating and the value of its outstanding bonds. Investors’ concern over event risk means that those firms deemed most likely to face events that could harm bondholders must pay extremely high interest rates. To reduce this interest rate, some bonds have a covenant called a super poison put, which enables a bondholder to turn in, or “put,” a bond back to the issuer at par in the event of a takeover, merger, or major recapitalization. Some bonds have a make-whole call provision. This allows a company to call the bond, but it must pay a call price that is essentially equal to the market value of a similar noncallable bond. This provides companies with an easy way to repurchase bonds as part of a financial restructuring, such as a merger. Sinking Funds Some bonds include a sinking fund provision that facilitates the orderly retirement of the bond issue. On rare occasions the firm may be required to deposit money with a trustee, which invests the funds and then uses the accumulated sum to retire the bonds when they mature. Usually, though, the sinking fund is used to buy back a certain percentage of the issue each year. A failure to meet the sinking fund requirement causes the bond to be thrown into default, which may force the company into bankruptcy. In most cases, the firm is given the right to administer the sinking fund in either of two ways. Chapter 5: Bonds, Bond Valuation, and Interest Rates 179 1. The company can call in for redemption (at par value) a certain percentage of the bonds each year; for example, it might be able to call 5% of the total original amount of the issue at a price of $1,000 per bond. The bonds are numbered serially, and those called for redemption are determined by a lottery administered by the trustee. 2. The company may buy the required number of bonds on the open market. The firm will choose the least-cost method. If interest rates have risen, causing bond prices to fall, then it will buy bonds in the open market at a discount; if interest rates have fallen, it will call the bonds. Note that a call for sinking fund purposes is quite different from a refunding call as discussed previously. A sinking fund call typically requires no call premium, but only a small percentage of the issue is normally callable in any one year.5 Although sinking funds are designed to protect bondholders by ensuring that an issue is retired in an orderly fashion, you should recognize that sinking funds can work to the detriment of bondholders. For example, suppose that the bond carries a 10% interest rate but that yields on similar bonds have fallen to 7.5%. A sinking fund call at par would require an investor to give up a bond that pays $100 of interest and then to reinvest in a bond that pays only $75 per year. This obviously harms those bondholders whose bonds are called. On balance, however, bonds that have a sinking fund are regarded as being safer than those without such a provision, so at the time they are issued sinking fund bonds have lower coupon rates than otherwise similar bonds without sinking funds. Other Provisions and Features Owners of convertible bonds have the option to convert the bonds into a fixed number of shares of common stock. Convertibles offer investors the chance to share in the upside if a company does well, so investors are willing to accept a lower coupon rate on convertibles than on an otherwise identical but nonconvertible bond. Warrants are options that permit the holder to buy stock at a fixed price, thereby providing a gain if the price of the stock rises. Some bonds are issued with warrants. As with convertibles, bonds with warrants have lower coupon rates than straight bonds. An income bond is required to pay interest only if earnings are high enough to cover the interest expense. If earnings are not sufficient, then the company is not required to pay interest and the bondholders do not have the right to force the company into bankruptcy. Therefore, from an investor’s standpoint, income bonds are riskier than “regular” bonds. Indexed bonds, also called purchasing power bonds, first became popular in Brazil, Israel, and a few other countries plagued by high inflation rates. The interest payments and maturity payment rise automatically when the inflation rate rises, thus protecting the bondholders against inflation. In January 1997, the U.S. Treasury began issuing indexed bonds called TIPS, short for Treasury Inflation-Protected Securities. Later in this chapter we show how TIPS can be used to estimate the risk-free rate. Bond Markets Corporate bonds are traded primarily in electronic/telephone markets rather than in organized exchanges. Most bonds are owned by and traded among a relatively small 5 Some sinking funds require the issuer to pay a call premium. 180 Part 2: Fixed Income Securities number of very large financial institutions, including banks, investment banks, life insurance companies, mutual funds, and pension funds. Although these institutions buy and sell very large blocks of bonds, it is relatively easy for bond dealers to arrange transactions because there are relatively few players in this market as compared with stock markets. Information on bond trades is not widely published, but a representative group of bonds is listed and traded on the bond division of the NYSE and is reported on the bond market page of The Wall Street Journal. Bond data are also available on the Internet at sites such as http://finance.yahoo.com. Self-Test Define “floating-rate bonds” and “zero coupon bonds.” Why is a call provision advantageous to a bond issuer? What are the two ways a sinking fund can be handled? Which method will be chosen by the firm if interest rates have risen? If interest rates have fallen? Are securities that provide for a sinking fund regarded as being riskier than those without this type of provision? Explain. What are income bonds and indexed bonds? Why do bonds with warrants and convertible bonds have lower coupons than similarly rated bonds that do not have these features? 5.3 BOND VALUATION The value of any financial asset—a stock, a bond, a lease, or even a physical asset such as an apartment building or a piece of machinery—is simply the present value of the cash flows the asset is expected to produce. The cash flows from a specific bond depend on its contractual features as described in the previous section. For a standard coupon-bearing bond such as the one issued by MicroDrive, the cash flows consist of interest payments during the life of the bond plus the amount borrowed when the bond matures (usually a $1,000 par value): 0 rd% Bond’s Va lue 1 2 3 INT INT INT ... N INT M The notation in the time line is explained below. rd = The bond’s required rate of return, which is the market rate of interest for that type of bond. This is the discount rate that is used to calculate the present value of the bond’s cash flows. It is also called the “yield” or “going rate of interest.” Note that rd is not the coupon interest rate. It is equal to the coupon rate only if (as in this case) the bond is selling at par. Generally, most coupon bonds are issued at par, which implies that the coupon rate is set at rd. Thereafter, interest rates, as measured by rd, will fluctuate, but the coupon rate is fixed, so rd will equal the coupon rate only by chance. We use the term “i” or “I” to designate the interest rate for many calculations because those terms are used on financial calculators, but “r,” with the subscript “d” to designate the rate on a debt security, is normally used in finance. Chapter 5: Bonds, Bond Valuation, and Interest Rates 181 N = Number of years before the bond matures. Note that N declines each year after the bond was issued, so a bond that had a maturity of 15 years when it was issued (original maturity = 15) will have N = 14 after 1 year, N = 13 after 2 years, and so on. Note also that for the sake of simplicity we assume the bond pays interest once a year, or annually, so N is measured in years. We consider bonds with semiannual payment bonds later in the chapter. INT = Dollars of interest paid each year = (Coupon rate)(Par value). For a bond with a 10% coupon and a $1,000 par value, the annual interest is 0.10($1,000) = $100. In calculator terminology, INT = PMT = 100. If the bond had been a semiannual payment bond, the payment would have been $50 every 6 months. M = Par, or maturity, value of the bond. This amount must be paid off at maturity, and it is often equal to $1,000. The following general equation, written in several forms, can be used to find the value of any bond, VB: VB ¼ INT INT M … þ INT þ 1þ 2þ N ð1 þ rd Þ ð1 þ rd Þ ð1 þ rd Þ ð1 þ rd ÞN N ¼ M ∑ ð1INT þ þ r Þ ð1 þ r Þ t¼1 " d t d N (5-1) # 1 1 M ¼ INT − þ N rd rd ð1 þ rd Þ ð1 þ rd ÞN Observe that the cash flows consist of an annuity of N years plus a lump-sum payment at the end of Year N. Equation 5-1 can be solved by using (1) a formula, (2) a financial calculator, or (3) a spreadsheet. Solving for the Bond Price Recall that MicroDrive issued a 15-year bond with an annual coupon rate of 10% and a par value of $1,000. To find the value of MicroDrive’s bond by using a formula, insert values for MicroDrive’s bond into Equation 5-1: 15 ∑ $100 $1; 000 þ þ 0:10Þt ð1 þ 0:10Þ15 " # 1 1 $1; 000 − þ ¼ $100 0:10 0:10ð1 þ 0:10Þ15 ð1 þ 0:10Þ15 VB ¼ t¼1 ð1 (5-1a) ¼ $1; 000 You could use the first line of Equation 5-1a to discount each cash flow back to the present and then sum these PVs to find the bond’s value; see Figure 5-1. This procedure is not very efficient, especially if the bond has many years to maturity. 182 Part 2: Fixed Income Securities FIGURE 5-1 Time Line for MicroDrive Inc.’s Bonds, 10% Interest Rate 1 2 3 4 5 6 7 8 9 10 11 12 13 14 15 Payments 100 100 100 100 100 100 100 100 100 100 100 100 100 100 100 + 1,000 90.91 82.64 75.13 68.30 62.09 56.45 51.32 46.65 42.41 38.55 35.05 31.86 28.97 26.33 23.94 239.39 Present = . Value 1,000.00 where rd = 10% resource Alternatively, you could use the formula in the second line of Equation 5-1a with a simple or scientific calculator, although this would still be somewhat cumbersome. A financial calculator is ideally suited for finding bond values. Here is the setup for MicroDrive’s bond: See Ch05 Tool Kit.xls on the textbook’s Web site. Inputs: Output: resource See Ch05 Tool Kit.xls on the textbook’s Web site. 15 10 N I/YR PV 100 1000 PMT FV –1000 Input N = 15, I/YR = rd = 10, INT = PMT = 100, and M = FV = 1000; then press the PV key to find the value of the bond, $1,000. Since the PV is an outflow to the investor, it is shown with a negative sign. The calculator is programmed to solve Equation 5-1: It finds the PV of an annuity of $100 per year for 15 years, discounted at 10%, then it finds the PV of the $1,000 maturity payment, and then it adds these two PVs to find the value of the bond. Notice that even though the time line in Figure 5-1 shows a total of $1,100 at Year 15, you should not enter FV = 1100! When you entered N = 15 and PMT = 100, you told the calculator that there is a $100 payment at Year 15. Thus, setting FV = 1000 accounts for any extra payment at Year 15, above and beyond the $100 payment. With Excel, it is easiest to use the PV function: =PV(I,N,PMT,FV,0). For MicroDrive’s bond, the function is =PV(0.10,15,100,1000,0) with a result of −$1,000. Like the financial calculator solution, the bond value is negative because PMT and FV are positive. Excel also provides specialized functions for bond prices based on actual dates. For example, in Excel you could find the MicroDrive bond value as of the date it was issued by using the function wizard to enter this formula: == PRICEðDATEð2011;1;5Þ;DATEð2026;1;5Þ;10%;10%;100;1;1Þ Chapter 5: Bonds, Bond Valuation, and Interest Rates 183 The first two arguments in the function are Excel’s DATE function. The DATE function takes the year, month, and day as inputs and converts them into a date. The first argument is the date on which you want to find the price, and the second argument is the maturity date. The third argument in the PRICE function is the bond’s coupon rate, followed by the required return on the bond, rd. The fifth argument, 100, is the redemption value of the bond at maturity per $100 of face value; entering “100” means that the bond pays 100% of its face value when it matures. The sixth argument is the number of payments per year. The last argument, 1, tells the program to base the price on the actual number of days in each month and year. This function produces a result based upon a face value of $100. In other words, if the bond pays $100 of face value at maturity, then the PRICE function result is the price of the bond. Because MicroDrive’s bond pays $1,000 of face value at maturity, we must multiply the PRICE function’s result by 10. In this example, the PRICE function returns a result of $100. When we multiply it by 10, we get the actual price of $1,000. This function is essential if a bond is being evaluated between coupon payment dates. See Ch05 Tool Kit.xls on the textbook’s Web site for an example.6 Interest Rate Changes and Bond Prices In this example, the bond is selling at a price equal to its par value. Whenever the going market rate of interest, rd, is equal to the coupon rate, a fixed-rate bond will sell at its par value. Normally, the coupon rate is set at the going rate when a bond is issued, causing it to sell at par initially. The coupon rate remains fixed after the bond is issued, but interest rates in the market move up and down. Looking at Equation 5-1, we see that an increase in the market interest rate (rd) will cause the price of an outstanding bond to fall, whereas a decrease in rates will cause the bond’s price to rise. For example, if the market interest rate on MicroDrive’s bond increased to 15% immediately after it was issued, we would recalculate the price with the new market interest rate as follows: Inputs: Output: 15 15 N I/YR PV 100 1000 PMT FV –707.63 The price would fall to $707.63. Notice that the bond would then sell at a price below its par value. Whenever the going rate of interest rises above the coupon rate, a fixed-rate bond’s price will fall below its par value, and it is called a discount bond. 6 The bond prices quoted by brokers are calculated as described. However, if you bought a bond between interest payment dates, you would have to pay the basic price plus accrued interest. Thus, if you purchased a MicroDrive bond 6 months after it was issued, your broker would send you an invoice stating that you must pay $1,000 as the basic price of the bond plus $50 interest, representing one-half the annual interest of $100. The seller of the bond would receive $1,050. If you bought the bond the day before its interest payment date, you would pay $1,000 + (364/365)($100) = $1,099.73. Of course, you would receive an interest payment of $100 at the end of the next day. For more on the valuation of bonds between payment dates, see Richard Taylor, “The Valuation of Semiannual Bonds between Interest Payment Dates,” The Financial Review, August 1988, pp. 365–368, and K. S. Maurice Tse and Mark A. White, “The Valuation of Semiannual Bonds between Interest Payment Dates: A Correction,” Financial Review, November 1990, pp. 659–662. 184 Part 2: Fixed Income Securities On the other hand, bond prices rise when market interest rates fall. For example, if the market interest rate on MicroDrive’s bond decreased to 5%, then we would once again recalculate its price: Inputs: 15 5 N I/YR PV 100 1000 PMT FV –1518.98 Output: In this case, the price rises to $1,518.98. In general, whenever the going interest rate falls below the coupon rate, a fixed-rate bond’s price will rise above its par value, and it is called a premium bond. Self-Test Why do the prices of fixed-rate bonds fall if expectations for inflation rise? What is a discount bond? A premium bond? A bond that matures in 6 years has a par value of $1,000, an annual coupon payment of $80, and a market interest rate of 9%. What is its price? ($955.14) A bond that matures in 18 years has a par value of $1,000, an annual coupon of 10%, and a market interest rate of 7%. What is its price? ($1,301.77) 5.4 CHANGES IN BOND VALUES OVER TIME At the time a coupon bond is issued, the coupon is generally set at a level that will cause the market price of the bond to equal its par value. If a lower coupon were set then investors would not be willing to pay $1,000 for the bond, and if a higher coupon were set then investors would clamor for the bond and bid its price up over $1,000. Investment bankers can judge quite precisely the coupon rate that will cause a bond to sell at its $1,000 par value. A bond that has just been issued is known as a new issue. (Investment bankers classify a bond as a new issue for about a month after it has first been issued. New issues are usually actively traded and are called “on-the-run” bonds.) Once the bond has been on the market for a while, it is classified as an outstanding bond, also called a seasoned issue. Newly issued bonds generally sell very close to par, but the prices of seasoned bonds vary widely from par. Except for floating-rate bonds, coupon payments are constant, so when economic conditions change, a 10% coupon bond with a $100 coupon that sold at par when it was issued will sell for more or less than $1,000 thereafter. MicroDrive’s bonds with a 10% coupon rate were originally issued at par. If rd remained constant at 10%, what would the value of the bond be 1 year after it was issued? Now the term to maturity is only 14 years—that is, N = 14. With a financial calculator, just override N = 15 with N = 14, press the PV key, and you find a value of $1,000. If we continued, setting N = 13, N = 12, and so forth, we would see that the value of the bond will remain at $1,000 as long as the going interest rate remains constant at the coupon rate, 10%. Now suppose interest rates in the economy fell after the MicroDrive bonds were issued and, as a result, rd fell below the coupon rate, decreasing from 10% to 5%. Both the coupon interest payments and the maturity value remain constant, but now 5% would have to be used for rd in Equation 5-1. The value of the bond at the end of the first year would be $1,494.93: Chapter 5: Bonds, Bond Valuation, and Interest Rates 185 14 VB ¼ $1; 000 ∑ ð1 þ$100 þ 0:05Þ ð1 þ 0:05Þ t¼1 t 14 " # 1 1 $1; 000 − ¼ $100 þ 0:05 0:05ð1 þ 0:05Þ14 ð1 þ 0:05Þ14 ¼ $1; 494:93 With a financial calculator, just change rd = I/YR from 10 to 5, and then press the PV key to get the answer, $1,494.93. Thus, if rd fell below the coupon rate, the bond would sell above par, or at a premium. The arithmetic of the bond value increase should be clear, but what is the logic behind it? Because rd has fallen to 5%, with $1,000 to invest you could buy new bonds like MicroDrive’s (every day some ten to twelve companies sell new bonds), except that these new bonds would pay $50 of interest each year rather than $100. Naturally, you would prefer $100 to $50, so you would be willing to pay more than $1,000 for a MicroDrive bond to obtain its higher coupons. All investors would react similarly; as a result, the MicroDrive bonds would be bid up in price to $1,494.93, at which point they would provide the same 5% rate of return to a potential investor as the new bonds. Assuming that interest rates remain constant at 5% for the next 14 years, what would happen to the value of a MicroDrive bond? It would fall gradually from $1,494.93 at present to $1,000 at maturity, when MicroDrive will redeem each bond for $1,000. This point can be illustrated by calculating the value of the bond 1 year later, when it has 13 years remaining to maturity. With a financial calculator, simply input the values for N, I/YR, PMT, and FV, now using N = 13, and press the PV key to find the value of the bond, $1,469.68. Thus, the value of the bond will have fallen from $1,494.93 to $1,469.68, or by $25.25. If you were to calculate the value of the bond at other future dates, the price would continue to fall as the maturity date approached. Note that if you purchased the bond at a price of $1,494.93 and then sold it 1 year later with rd still at 5%, you would have a capital loss of $25.25, or a total return of $100.00 − $25.25 = $74.75. Your percentage rate of return would consist of the rate of return due to the interest payment (called the current yield) and the rate of return due to the price change (called the capital gains yield). This total rate of return is often called the bond yield, and it is calculated as follows: Interest; or current; yield ¼ $100=$1; 494:93 ¼ 0:0669 ¼ 6:69% Capital gains yield ¼ −$25:25=$1; 494:93 ¼ −0:0169 ¼ −1:69% Total rate of return; or yield ¼ $74:75=$1; 494:93 ¼ 0:0500 ¼ 5:00% Had interest rates risen from 10% to 15% during the first year after issue (rather than falling from 10% to 5%), then you would enter N = 14, I/YR = 15, PMT = 100, and FV = 1000, and then press the PV key to find the value of the bond, $713.78. In this case, the bond would sell below its par value, or at a discount. The total expected future return on the bond would again consist of an expected return due to interest and an expected return due to capital gains or capital losses. In this situation, the capital gains yield would be positive. The total return would be 15%. To see this, calculate the price of the bond with 13 years left to maturity, assuming that interest rates remain at 15%. With a calculator, enter N = 13, I/YR = 15, PMT = 100, and FV = 1000; then press PV to obtain the bond’s value, $720.84. 186 Part 2: Fixed Income Securities Note that the capital gain for the year is the difference between the bond’s value at Year 2 (with 13 years remaining) and the bond’s value at Year 1 (with 14 years remaining), or $720.84 − $713.78 = $7.06. The interest yield, capital gains yield, and total yield are calculated as follows: Interest; or current; yield ¼ $100=$713:78 ¼ 0:1401 ¼ 14:01% Capital gains yield ¼ $7:06=$713:78 ¼ 0:0099 ¼ 0:99% Total rate of return; or yield ¼ $107:06=$713:78 ¼ 0:1500 ¼ 15:00% Figure 5-2 graphs the value of the bond over time, assuming that interest rates in the economy (1) remain constant at 10%, (2) fall to 5% and then remain constant at that level, or (3) rise to 15% and remain constant at that level. Of course, if interest rates do not remain constant, then the price of the bond will fluctuate. However, regardless of what future interest rates do, the bond’s price will approach $1,000 as it nears the maturity date (barring bankruptcy, in which case the bond’s value might fall dramatically). Figure 5-2 illustrates the following key points. resource See Ch05 Tool Kit.xls for all calculations. FIGURE 5-2 FIGURE 5 - 2 1. Whenever the going rate of interest, rd, is equal to the coupon rate, a fixed-rate bond will sell at its par value. Normally, the coupon rate is set equal to the going rate when a bond is issued, causing it to sell at par initially. 2. Interest rates do change over time, but the coupon rate remains fixed after the bond has been issued. Whenever the going rate of interest rises above the coupon rate, a fixed-rate bond’s price will fall below its par value. Such a bond is called a discount bond. 3. Whenever the going rate of interest falls below the coupon rate, a fixed-rate bond’s price will rise above its par value. Such a bond is called a premium bond. 4. Thus, an increase in interest rates will cause the prices of outstanding bonds to fall, whereas a decrease in rates will cause bond prices to rise. Time Path of the Value of a 10% Coupon, $1,000 Par Value Bond When Interest Rates Are 5%, 10%, and 15% Bond Value ($) Time Path of 10% Coupon Bond’s Value When rd Falls to 5% and Remains There (Premium Bond) 1,495 M = 1,000 Time Path of Bond Value When rd = Coupon Rate = 10% (Par Bond) M 714 Time Path of 10% Coupon Bond’s Value When rd Rises to 15% and Remains There (Discount Bond) 0 1 2 3 Note: The curves for 5% and 15% have a slight bow. 4 5 6 7 8 9 10 11 12 13 14 15 Years Chapter 5: Bonds, Bond Valuation, and Interest Rates 187 Drinking Your Coupons In 1996, Chateau Teyssier, an English vineyard, was looking for some cash to purchase additional vines and to modernize its production facilities. Their solution? With the assistance of a leading underwriter, Matrix Securities, the vineyard issued 375 bonds, each costing 2,650 British pounds. The issue raised nearly 1 million pounds, or roughly $1.5 million. What makes these bonds interesting is that, instead of paying with something boring like money, they paid their investors back with wine. Each June until 2002, when the bond matured, investors received their “coupons.” Between 1997 and 2001, each bond provided six cases of the vineyard’s rosé or claret. Starting in 1998 and continuing through maturity in 2002, investors also received four cases of its prestigious Saint Emilion Grand Cru. Then, in 2002, they got their money back. The bonds were not without risk. The vineyard’s owner, Jonathan Malthus, acknowledged that the quality of the wine “is at the mercy of the gods." Source: Steven Irvine, “My Wine Is My Bond, and I Drink My Coupons,” Euromoney, July 1996, p. 7. 5. The market value of a bond will always approach its par value as its maturity date approaches, provided the firm does not go bankrupt. These points are very important, for they show that bondholders may suffer capital losses or make capital gains depending on whether interest rates rise or fall after the bond is purchased. Self-Test What is meant by the terms “new issue” and “seasoned issue"? Last year, a firm issued 30-year, 8% annual coupon bonds at a par value of $1,000. (1) Suppose that 1 year later the going rate drops to 6%. What is the new price of the bonds, assuming that they now have 29 years to maturity? ($1,271.81) (2) Suppose instead that 1 year after issue the going interest rate increases to 10% (rather than dropping to 6%). What is the price? ($812.61) 5.5 BONDS WITH SEMIANNUAL COUPONS Although some bonds pay interest annually, the vast majority actually pay interest semiannually. To evaluate semiannual payment bonds, we must modify the valuation model as follows. 1. Divide the annual coupon interest payment by 2 to determine the dollars of interest paid every 6 months. 2. Multiply the years to maturity, N, by 2 to determine the number of semiannual periods. 3. Divide the nominal (quoted) interest rate, rd, by 2 to determine the periodic (semiannual) interest rate. By making these changes, we obtain the following equation for finding the value of a bond that pays interest semiannually: 2N VB ¼ M ∑ ð1INT=2 þ þ r =2Þ ð1 þ r =2Þ t¼1 d t d 2N (5-2) To illustrate, assume now that MicroDrive’s bonds pay $50 interest every 6 months rather than $100 at the end of each year. Each semiannual interest payment is only 188 Part 2: Fixed Income Securities half as large, but there are twice as many of them. The nominal, or quoted, coupon rate is “10%, semiannual payments."7 When the going (nominal) rate of interest is 5% with semiannual compounding, the value of this 15-year bond is found as follows: Inputs: 30 2.5 N I/YR PV 50 1000 PMT FV –1523.26 Output: Enter N = 30, rd = I/YR = 2.5, PMT = 50, FV = 1000, and then press the PV key to obtain the bond’s value, $1,523.26. The value with semiannual interest payments is slightly larger than $1,518.98, the value when interest is paid annually. This higher value occurs because interest payments are received somewhat faster under semiannual compounding. Self-Test Describe how the annual bond valuation formula is changed to evaluate semiannual coupon bonds. Write out the revised formula. A bond has a 25-year maturity, an 8% annual coupon paid semiannually, and a face value of $1,000. The going nominal annual interest rate (rd) is 6%. What is the bond’s price? ($1,257.30) 5.6 BOND YIELDS Unlike the coupon interest rate, which is fixed, the bond’s yield varies from day to day depending on current market conditions. Moreover, the yield can be calculated in three different ways, and three “answers” can be obtained. These different yields are described in the following sections. Yield to Maturity Suppose you were offered a 14-year, 10% annual coupon, $1,000 par value bond at a price of $1,494.93. What rate of interest would you earn on your investment if you bought the bond and held it to maturity? This rate is called the bond’s yield to maturity (YTM), and it is the interest rate generally discussed by investors when they talk about rates of return. The yield to maturity is usually the same as the market rate of interest, rd. To find the YTM for a bond with annual interest payments, you must solve Equation 5-1 for rd:8 N Bond price ¼ M ∑ ð1 þINT þ YTMÞ ð1 þ YTMÞ t¼1 t N (5-3) In this situation, the coupon rate of “10% paid semiannually” is the rate that bond dealers, corporate treasurers, and investors generally would discuss. Of course, if this bond were issued at par, then its effective annual rate would be higher than 10%:   rNOM M 0:10 2 EAR ¼ EFF% ¼ 1 þ 1¼ 1þ  1 ¼ ð1:05Þ2  1 ¼ 10:25% M 2 7 Because 10% with annual payments is quite different from 10% with semiannual payments, we have assumed a change in effective rates in this section from the situation described in Section 5.3, where we assumed 10% with annual payments. 8 If the bond has semiannual payments, you must solve Equation 5-2 for rd. Chapter 5: Bonds, Bond Valuation, and Interest Rates 189 For MicroDrive’s yield, you must solve this equation: $1; 494:93 ¼ $100 $100 $1; 000 þ…þ þ ð1 þ rd Þ1 ð1 þ rd Þ14 ð1 þ rd Þ14 You could substitute values for rd until you found a value that “works” and forces the sum of the PVs on the right side of the equal sign to equal $1,494.93, but this would be tedious and time-consuming.9 As you might guess, it is much easier with a financial calculator. Here is the setup: Inputs: 14 N Output: resource See Ch05 Tool Kit.xls on the textbook’s Web site. I/YR –1494.93 100 1000 PV PMT FV 5 Simply enter N = 14, PV = −1494.93, PMT = 100, and FV = 1000, and then press the I/YR key for the answer of 5%. You could also find the YTM with a spreadsheet. In Excel, you would use the RATE function for this bond, inputting N = 14, PMT = 100, PV = −1494.93, FV = 1000, 0 for Type, and leave Guess blank: =RATE(14,100,−1494.93,1000,0). The result is 5%. The RATE function works only if the current date is immediately after either the issue date or a coupon payment date. To find bond yields on other dates, use Excel’s YIELD function. See the Ch05 Tool Kit.xls file for an example. The yield to maturity can be viewed as the bond’s promised rate of return, which is the return that investors will receive if all the promised payments are made. However, the yield to maturity equals the expected rate of return only if (1) the probability of default is zero and (2) the bond cannot be called. If there is some default risk or if the bond may be called, then there is some probability that the promised payments to maturity will not be received, in which case the calculated yield to maturity will differ from the expected return. The YTM for a bond that sells at par consists entirely of an interest yield, but if the bond sells at a price other than its par value then the YTM will consist of the interest yield plus a positive or negative capital gains yield. Note also that a bond’s yield to maturity changes whenever interest rates in the economy change, and this is almost daily. If you purchase a bond and hold it until it matures, you will receive the YTM that existed on the purchase date but the bond’s calculated YTM will change frequently between the purchase date and the maturity date.10 9 Alternatively, you can substitute values of rd into the third form of Equation 5-1 until you find a value that works. We often are asked by students if the purchaser of a bond will receive the YTM if interest rates subsequently change. The answer is definitely “yes” provided the question means “Is the realized rate of return on the investment in the bond equal to the YTM?” This is because the realized rate of return on an investment is the rate that sets the present value of the realized cash flows equal to the price. If instead the question means “Is the realized rate of return on the investment in the bond and the subsequent reinvestment of the coupons equal to the YTM?” then the answer is definitely “no.” Thus, the question really is one about strategy and timing. The bond, in combination with a reinvestment strategy, is really two investments, and clearly the realized rate on this combined strategy depends on the reinvestment rate (see Web Extension 5C for more on investing for a target future value). For the rest of the book, we assume that an investment in a bond is just that, an investment only in the bond, and not a combination of the bond and a reinvestment strategy; this means the investor earns the expected YTM if the bond is held to maturity. 10 190 Part 2: Fixed Income Securities Yield to Call If you purchased a bond that was callable and the company called it, you would not have the option of holding the bond until it matured. Therefore, the yield to maturity would not be earned. For example, if MicroDrive’s 10% coupon bonds were callable and if interest rates fell from 10% to 5%, then the company could call in the 10% bonds, replace them with 5% bonds, and save $100 − $50 = $50 interest per bond per year. This would be good for the company but not for the bondholders. If current interest rates are well below an outstanding bond’s coupon rate, then a callable bond is likely to be called, and investors will estimate its expected rate of return as the yield to call (YTC) rather than as the yield to maturity. To calculate the YTC, solve this equation for rd: N Price of callable bond ¼ Call price ∑ ð1INT þ þr Þ ð1 þ r Þ t¼1 d t d (5-4) N Here N is the number of years until the company can call the bond, rd is the YTC, and “Call price” is the price the company must pay in order to call the bond (it is often set equal to the par value plus 1 year’s interest). To illustrate, suppose MicroDrive’s bonds had a provision that permitted the company, if it desired, to call the bonds 10 years after the issue date at a price of $1,100. Suppose further that 1 year after issuance the going interest rate had declined, causing the price of the bonds to rise to $1,494.93. Here is the time line and the setup for finding the bond’s YTC with a financial calculator: 0 YTC = ? − 1,494.93 Inputs: 2 100 100 100 100 1,100 –1494.93 100 1100 PV PMT FV 9 N Output: 1 I/YR ... 8 9 4.21 = YTC The YTC is 4.21%—this is the return you would earn if you bought the bond at a price of $1,494.93 and it was called 9 years from today. (The bond could not be called until 10 years after issuance, and 1 year has gone by, so there are 9 years left until the first call date.) Do you think MicroDrive will call the bonds when they become callable? MicroDrive’s actions depend on the going interest rate when the bonds become callable. If the going rate remains at rd = 5%, then MicroDrive could save 10% − 5% = 5%, or $50 per bond per year, by calling them and replacing the 10% bonds with a new 5% issue. There would be costs to the company to refund the issue, but the interest savings would probably be worth the cost, so MicroDrive would probably refund the bonds. Therefore, you would probably earn YTC = 4.21% rather than YTM = 5% if you bought the bonds under the indicated conditions. In the balance of this chapter, we assume that bonds are not callable unless otherwise noted. However, some of the end-of-chapter problems deal with yield to call. Chapter 5: Bonds, Bond Valuation, and Interest Rates 191 Current Yield If you examine brokerage house reports on bonds, you will often see reference to a bond’s current yield. The current yield is the annual interest payment divided by the bond’s current price. For example, if MicroDrive’s bonds with a 10% coupon were currently selling at $985, then the bond’s current yield would be $100/$985 = 0.1015 = 10.15%. Unlike the yield to maturity, the current yield does not represent the rate of return that investors should expect on the bond. The current yield provides information regarding the amount of cash income that a bond will generate in a given year, but it does not provide an accurate measure of the bond’s total expected return, the yield to maturity. In fact, here is the relation between current yield, capital gains yield (which can be negative for a capital loss), and the yield to maturity: Current yield þ Capital gains yield ¼ Yield to maturity (5-5) The Cost of Debt and Intrinsic Value The “Intrinsic Value Box” at the beginning of this chapter highlights the cost of debt, which affects the weighted average cost of capital (WACC), which in turn affects the company’s intrinsic value. The pre-tax cost of debt from the company’s perspective is the required return from the debtholder’s perspective. Therefore, the pretax cost of debt is the yield to maturity (or the yield to call if a call is likely). But why do different bonds have different yields to maturity? The following sections answer this question. Self-Test Explain the difference between the yield to maturity and the yield to call. How does a bond’s current yield differ from its total return? Could the current yield exceed the total return? A bond currently sells for $850. It has an 8-year maturity, an annual coupon of $80, and a par value of $1,000. What is its yield to maturity? (10.90%) What is its current yield? (9.41%) A bond currently sells for $1,250. It pays a $110 annual coupon and has a 20-year maturity, but it can be called in 5 years at $1,110. What are its YTM and its YTC? (8.38%, 6.85%) Is the bond likely to be called if interest rates don’t change? 5.7 THE PRE-TAX COST OF DEBT: DETERMINANTS OF MARKET INTEREST RATES Up until now we have given you rd, the going market rate. But as we showed in Chapter 1, different debt securities often have very different market rates. What explains these differences? In general, the quoted (or nominal) interest rate on a debt security, rd, is composed of a real risk-free rate of interest, r*, plus several premiums that reflect inflation, the risk of the security, and the security’s marketability (or liquidity). A conceptual framework is shown below: Quoted market interest rate ¼ rd ¼ r þ IP þ DRP þ LP þ MRP ¼ rRF þ DRP þ LP þ MRP (5-6) 192 Part 2: Fixed Income Securities Here are definitions of the variables in Equation 5-6: rd = Quoted, or nominal, rate of interest on a given security.11 There are many different securities and hence many different quoted interest rates. r* = Real risk-free rate of interest. Pronounced “r-star,” r* is the rate that would exist on a riskless security if zero inflation were expected. IP = Inflation premium, which is equal to the average expected inflation rate over the life of the security. The expected future inflation rate is not necessarily equal to the current inflation rate, so IP is not necessarily equal to current inflation. rRF = r* + IP, and it is the quoted risk-free rate of interest on a security such as a U.S. Treasury bill, which is very liquid and also free of most risks. Note that rRF includes the premium for expected inflation because rRF = r* + IP. DRP = Default risk premium. This premium reflects the possibility that the issuer will not pay interest or principal at the stated time and in the stated amount. The DRP is zero for U.S. Treasury securities, but it rises as the riskiness of issuers increases. LP = Liquidity, or marketability, premium. This is a premium charged by lenders to reflect the fact that some securities cannot be converted to cash on short notice at a “reasonable” price. The LP is very low for Treasury securities and for securities issued by large, strong firms, but it is relatively high on securities issued by very small firms. MRP = Maturity risk premium. As we will explain later, longer-term bonds (even Treasury bonds) are exposed to a significant risk of price declines, and a maturity risk premium is charged by lenders to reflect this risk. We discuss the components whose sum makes up the quoted, or nominal, rate on a given security in the following sections. Self-Test WWW See http://www.bloom berg.com and select MARKET DATA. Then select RATES AND BONDS for a partial listing of indexed Treasury bonds and their interest rates. See http:// online.wsj.com for a complete set of Treasury quotes. See http://www .treasurydirect.gov/indiv/ products/products.htmfor a complete listing of all Treasury securities. Write out an equation for the nominal interest rate on any debt security. 5.8 THE REAL RISK-FREE RATE OF INTEREST, r* The real risk-free rate of interest, r*, is defined as the interest rate that would exist on a riskless security if no inflation were expected, and it may be thought of as the rate of interest on short-term U.S. Treasury securities in an inflation-free world. The real risk-free rate is not static—it changes over time depending on economic The term nominal as it is used here means the stated rate as opposed to the real rate, which is adjusted to remove inflation effects. Suppose you bought a 10-year Treasury bond with a quoted, or nominal, rate of about 4.6%. If inflation averages 2.5% over the next 10 years, then the real rate would be about 4.6% − 2.5% = 2.1%. To be technically correct, we should find the real rate by solving for r* in the following equation: (1 + r*)(1 + 0.025) = (1 + 0.046). Solving the equation, we find r* = 2.05%. Since this is very close to the 2.1% just calculated, we will continue to approximate the real rate in this chapter by subtracting inflation from the nominal rate. 11 Chapter 5: Bonds, Bond Valuation, and Interest Rates 193 conditions, especially (1) the rate of return corporations and other borrowers expect to earn on productive assets and (2) people’s time preferences for current versus future consumption.12 In addition to its regular bond offerings, in 1997 the U.S. Treasury began issuing indexed bonds, with payments linked to inflation. These bonds are called TIPS, short for Treasury Inflation-Protected Securities. Because the payments (including the principal) are tied to inflation, the yield on TIPS is a good estimate of the risk-free rate. In early 2009, the TIPS with about 1 year remaining until maturity had a 1.54% yield.13 This is a pretty good estimate of the real risk-free rate, r*, although ideally we would prefer a TIPS with an even shorter time until maturity. We will have more to say about how to use TIPS when we discuss the inflation premium in the next section. For details on how TIPS are adjusted to protect against inflation, see Web Extension 5B on the textbook’s Web site. Self-Test What security provides a good estimate of the real risk-free rate? 5.9 THE INFLATION PREMIUM (IP) Inflation has a major effect on interest rates because it erodes the purchasing power of the dollar and lowers the real rate of return on investments. To illustrate, suppose you invest $3,000 in a default-free zero coupon bond that matures in 1 year and pays a 5% interest rate. At the end of the year, you will receive $3,150—your original $3,000 plus $150 of interest. Now suppose that the inflation rate during the year is 10% and that it affects all items equally. If gas had cost $3 per gallon at the beginning of the year, it would cost $3.30 at the end of the year. Therefore, your $3,000 would have bought $3,000/$3 = 1,000 gallons at the beginning of the year but only $3,150/$3.30 = 955 gallons at the end. In real terms, you would be worse off—you would receive $150 of interest, but it would not be sufficient to offset inflation. You would thus be better off buying 1,000 gallons of gas (or some other storable asset) than buying the default-free bond. Investors are well aware of inflation’s effects on interest rates, so when they lend money, they build in an inflation premium (IP) equal to the average expected inflation rate over the life of the security. For a short-term, default-free U.S. Treasury bill, the actual interest rate charged, rT-bill, would be the real risk-free rate, r*, plus the inflation premium (IP): rT-bill ¼ rRF ¼ r þ IP 12 The real rate of interest as discussed here is different from the current real rate as often discussed in the press. The current real rate is often estimated as the current interest rate minus the current (or most recent) inflation rate, whereas the real rate, as used here (and in the fields of finance and economics generally) without the word “current,” is the current interest rate minus the expected future inflation rate over the life of the security. For example, suppose the current quoted rate for a 1-year Treasury bill is 5%, inflation during the previous year was 2%, and inflation expected for the coming year is 4%. Then the current real rate would be approximately 5% – 2% = 3%, but the expected real rate would be approximately 5% – 4% = 1%. 13 Negative nominal rates are pretty much impossible—investors would just hold cash instead of investing in a negative-rate bond. But negative real rates are possible. In spring 2008, the combination of stagnant economic growth, a high level of investor uncertainty, fears of inflation, and the Federal Reserve’s reduction in nominal short-term interest rates caused the real rate to fall below zero, as measured by negative yields on several short-term TIPS. 194 Part 2: Fixed Income Securities Therefore, if the real short-term risk-free rate of interest were r* = 0.6% and if inflation were expected to be 1.0% (and hence IP = 1.0%) during the next year, then the quoted rate of interest on 1-year T-bills would be 0.6% + 1.0% = 1.6%. It is important to note that the inflation rate built into interest rates is the inflation rate expected in the future, not the rate experienced in the past. Thus, the latest reported figures might show an annual inflation rate of 2%, but that is for the past year. If people on average expect a 6% inflation rate in the future, then 6% would be built into the current interest rate. Note also that the inflation rate reflected in the quoted interest rate on any security is the average rate of inflation expected over the security’s life. Thus, the inflation rate built into a 1-year bond is the expected inflation rate for the next year, but the inflation rate built into a 30-year bond is the average rate of inflation expected over the next 30 years. If It is the expected inflation during year t, then the inflation premium for an N-year bond’s yield (IPN) can be approximated as IPN ¼ I1 þ I2 þ … þ IN N (5-7) For example, if investors expect inflation to average 3% during Year 1 and 5% during Year 2, then the inflation premium built into a 2-year bond’s yield can be approximated by14 IP2 ¼ I1 þ I2 3% þ 5% ¼ 4% ¼ 2 2 In the previous section, we saw that the yield on an inflation-indexed Treasury bond (TIPS) is a good estimate of the real interest rate. We can also use TIPS to estimate inflation premiums. For example, in early 2009 the yield on a 5-year nonindexed T-bond was 1.91% and the yield on a 5-year TIPS was 1.41%. Thus, the 5-year inflation premium was 1.91% − 1.41% = 0.50%, implying that investors expected inflation to average 0.50% over the next 5 years.15 Similarly, the rate on a 20-year nonindexed T-bond was 3.93% and the rate on a 20-year indexed T-bond was 2.44%. Thus, the 20-year inflation premium was approximately 3.93% − 2.44% = 1.49%, implying that investors expected inflation to average 1.49% over the long term.16 These calculations are summarized below: To be theoretically correct, we should take the geometric average: (1 + IP2)2 = (1 + I1)(1 + I2). In this example, we have (1 + IP2)2 = (1 + 0.03)(1 + 0.05). Solving for IP2 yields 3.9952, which is close to our approximation of 4%. 14 15 To be theoretically precise, we should use a geometric average by solving the following equation: (1 + IP)(1.0141) = 1.0191. Solving for IP gives IP = 0.493%, which is the same as our approximation. Note, though, that the difference in yield between a T-bond and a TIPS of the same maturity reflects both the expected inflation and any risk premium for bearing inflation risk. So the difference in yields is really an upper limit on the expected inflation. There are several other sources for the estimated inflation premium. The Congressional Budget Office regularly updates the estimates of inflation that it uses in its forecasted budgets; see http://www.cbo.gov/; select Economic Projections. A second source is the University of Michigan’s Institute for Social Research, which regularly polls consumers regarding their expectations for price increases during the next year; see http://www.isr.umich.edu/home/; select Inst for Social Research; then search for Consumers to get the survey. We prefer using inflation premiums derived from indexed and nonindexed Treasury securities, as described in the text, since these are based on how investors actually spend their money, not on theoretical models or opinions. 16 Chapter 5: Bonds, Bond Valuation, and Interest Rates Yield on nonindexed U.S. Treasury bond Yield on TIPS Inflation premium 195 Maturity 5 Years 20 Years 1.91% 3.93% 1.41 2.44 0.50% 1.49% Expectations for future inflation are closely, but not perfectly, correlated with rates experienced in the recent past. Therefore, if the inflation rate reported for last month increases, people often raise their expectations for future inflation, and this change in expectations will cause an increase in interest rates. Note that Germany, Japan, and Switzerland have, over the past several years, had lower inflation rates than the United States, so their interest rates have generally been lower than ours. South Africa, Brazil, and most South American countries have experienced higher inflation, which is reflected in their interest rates. Self-Test Explain how a TIPS and a nonindexed Treasury security can be used to estimate the inflation premium. The yield on a 15-year TIPS is 3% and the yield on a 15-year Treasury bond is 5%. What is the inflation premium for a 15-year security? (2%) 5.10 THE NOMINAL, RATE OF INTEREST, rRF OR QUOTED, RISK-FREE The nominal, or quoted, risk-free rate, rRF, is the real risk-free rate plus a premium for expected inflation: rRF = r* + IP. To be strictly correct, the risk-free rate should mean the interest rate on a totally risk-free security—one that has no risk of default, no maturity risk, no liquidity risk, no risk of loss if inflation increases, and no risk of any other type. There is no such security, so there is no observable truly riskfree rate. When the term “risk-free rate” is used without either the modifier “real” or the modifier “nominal,” people generally mean the quoted (nominal) rate, and we will follow that convention in this book. Therefore, when we use the term “risk-free rate, rRF,” we mean the nominal risk-free rate, which includes an inflation premium equal to the average expected inflation rate over the life of the security. In general, we use the T-bill rate to approximate the short-term risk-free rate and use the T-bond rate to approximate the long-term risk-free rate (even though it also includes a maturity premium). So, whenever you see the term “risk-free rate,” assume that we are referring either to the quoted U.S. T-bill rate or to the quoted T-bond rate. Since rRF = r* + IP, we can express the quoted rate as Nominal; or quoted; rate ¼ rd ¼ rRF þ DRP þ LP þ MRP(5-7) Self-Test What security is a good approximation of the nominal risk-free rate? 5.11 THE DEFAULT RISK PREMIUM (DRP) If the issuer defaults on a payment, investors receive less than the promised return on the bond. The quoted interest rate includes a default risk premium (DRP)—the greater the default risk, the higher the bond’s yield to maturity.17 The default risk Suppose two bonds have the same promised cash flows, coupon rate, maturity, liquidity, and inflation exposure, but one bond has more default risk than the other. Investors will naturally pay less for the bond with the greater chance of default. As a result, bonds with higher default risk will have higher interest rates. 17 196 Part 2: Fixed Income Securities on Treasury securities is virtually zero, but default risk can be substantial for corporate and municipal bonds. In this section, we consider some issues related to default risk. Bond Contract Provisions That Influence Default Risk Default risk is affected by both the financial strength of the issuer and the terms of the bond contract, especially whether collateral has been pledged to secure the bond. Several types of contract provisions are discussed next. Bond Indentures. An indenture is a legal document that spells out the rights of both bondholders and the issuing corporation, and a trustee is an official (usually a bank) who represents the bondholders and makes sure the terms of the indenture are carried out. The indenture may be several hundred pages in length, and it will include restrictive covenants that cover such points as the conditions under which the issuer can pay off the bonds prior to maturity, the levels at which certain ratios must be maintained if the company is to issue additional debt, and restrictions against the payment of dividends unless earnings meet certain specifications. The Securities and Exchange Commission (1) approves indentures and (2) makes sure that all indenture provisions are met before allowing a company to sell new securities to the public. A firm will have different indentures for each of the major types of bonds it issues, but a single indenture covers all bonds of the same type. For example, one indenture will cover a firm’s first mortgage bonds, another its debentures, and a third its convertible bonds. Mortgage Bonds. A corporation pledges certain assets as security for a mortgage bond. The company might also choose to issue second-mortgage bonds secured by the same assets that were secured by a previously issued mortgage bond. In the event of liquidation, the holders of these second mortgage bonds would have a claim against the property, but only after the first mortgage bondholders had been paid off in full. Thus, second mortgages are sometimes called junior mortgages, because they are junior in priority to the claims of senior mortgages, or first-mortgage bonds. All mortgage bonds are subject to an indenture that usually limits the amount of new bonds that can be issued. Debentures and Subordinated Debentures. A debenture is an unsecured bond, and as such it provides no lien against specific property as security for the obligation. Debenture holders are, therefore, general creditors whose claims are protected by property not otherwise pledged. The term subordinate means “below,” or “inferior to"; thus, in the event of bankruptcy, subordinated debt has claims on assets only after senior debt has been paid off. Subordinated debentures may be subordinated either to designated notes payable (usually bank loans) or to all other debt. In the event of liquidation or reorganization, holders of subordinated debentures cannot be paid until all senior debt, as named in the debentures’ indentures, has been paid. Development Bonds. Some companies may be in a position to benefit from the sale of either development bonds or pollution control bonds. State and local governments may set up both industrial development agencies and pollution control agencies. These agencies are allowed, under certain circumstances, to sell taxexempt bonds and then make the proceeds available to corporations for specific uses deemed (by Congress) to be in the public interest. For example, a Detroit pollution control agency might sell bonds to provide Ford with funds for purchasing pollution control equipment. Because the income from the bonds would be tax exempt, the bonds would have a relatively low interest rates. Note, how- Chapter 5: Bonds, Bond Valuation, and Interest Rates 197 THE GLOBAL ECONOMIC CRISIS Insuring with Credit Default Swaps: Let the Buyer Beware! Recall that a credit default swap (CDS) is like an insurance policy. The purchaser of the CDS agrees to make annual payments to a counterparty that agrees to pay if a particular bond defaults. During the 2000s, investment banks often would purchase CDS for the mortgage-backed securities (MBS) they were creating in order to make the securities more attractive to in- vestors. But how good was this type of insurance? As it turned out, not very. For example, Lehman Brothers might have bought a CDS from AIG in order to sell a Lehman-created MBS to an investor. But when the MBS began defaulting, neither Lehman nor AIG was capable of making full restitution to the investor. ever, that these bonds are guaranteed by the corporation that will use the funds, not by a governmental unit, so their rating reflects the credit strength of the corporation using the funds. Municipal Bond Insurance. Municipalities can have their bonds insured, which means that an insurance company guarantees to pay the coupon and principal payments should the issuer default. This reduces risk to investors, who will thus accept a lower coupon rate for an insured bond than for a comparable but uninsured one. Even though the municipality must pay a fee to have its bonds insured, its savings due to the lower coupon rate often make insurance cost effective. Keep in mind that the insurers are private companies, and the value added by the insurance depends on the creditworthiness of the insurer. The larger insurers are strong companies, and their own ratings are AAA. Bond Ratings Since the early 1900s, bonds have been assigned quality ratings that reflect their probability of going into default. The three major rating agencies are Moody’s Investors Service (Moody’s), Standard & Poor’s Corporation (S&P), and Fitch Ratings. As shown in Columns (3) and (4) of Table 5-1, triple-A and double-A bonds are extremely safe, rarely defaulting even within 5 years of being assigned a rating. SingleA and triple-B bonds are also strong enough to be called investment-grade bonds, and they are the lowest-rated bonds that many banks and other institutional investors are permitted by law to hold. Double-B and lower bonds are speculative, or junk bonds. These bonds have a significant probability of defaulting. Bond Rating Criteria, Upgrades, and Downgrades Bond ratings are based on both quantitative and qualitative factors, as we describe below. 1. Financial Ratios. Many ratios potentially are important, but the return on assets, debt ratio, and interest coverage ratio are particularly valuable for predicting financial distress. For example, Columns 5 and 6 in Table 5-1 show a strong relationship between ratings and the return on capital and the debt ratio. 2. Bond Contract Terms. Important provisions for determining the bond’s rating include whether the bond is secured by a mortgage on specific assets, whether the bond is subordinated to other debt, any sinking fund provisions, guarantees by some other party with a high credit ranking, and restrictive covenants such as requirements that the firm keep its debt ratio below a given level or that it keep its times interest earned ratio above a given level. 198 Part 2: Fixed Income Securities T AB L E 5 - 1 Bond Ratings, D efault Risk, and Yields P E RC E N T D E F A UL T IN G R AT I N G A G E N C Y a WITHIN: b P E RC E N T U PG R A D E D MEDIAN RATIOS c O R D O W N G R AD ED I N 2 0 0 8 b R E TU R N O N T O T A L D E B T/ FITCH MO ODY ’ S 1 YEAR 5 Y E A RS CAP ITAL TO TA L C A PI T AL DO WN UP Y IE L D (1) ( 2) (3) (4) ( 5) (6) (7) (8) (9) 0.0% 0.0 0.1 0.3 0.0% 0.1 0.6 2.9 S& P A N D Investment-grade bonds AAA Aaa AA Aa A A BBB Baa Junk bonds BB Ba B B CCC Caa 1.4 1.8 22.3 8.2 9.2 36.9 27.6% 27.0 17.5 13.4 11.3 8.7 3.2 12.4% 28.3 37.5 42.5 53.7 75.9 113.5 13.6% 21.8 8.0 6.4 NA 0.0 1.8 2.6 15.1 10.8 26.1 6.8 5.6 8.7 d 5.50% 5.62 5.79 7.53 11.62 13.70 26.30 Notes: aThe ratings agencies also use “modifiers” for bonds rated below triple-A. S&P and Fitch use a plus and minus system; thus, A+ designates the strongest A-rated bonds and A− the weakest. Moody’s uses a 1, 2, or 3 designation, with 1 denoting the strongest and 3 the weakest; thus, within the double-A category, Aa1 is the best, Aa2 is average, and Aa3 is the weakest. b Default data are from Fitch Ratings Global Corporate Finance 2008 Transition and Default Study, March 5, 2009: see http://www.fitchratings.com/corporate/reports/report_frame.cfm?rpt_id=428182. c Median ratios are from Standard & Poor’s 2006 Corporate Ratings Criteria, April 23, 2007: see http://www2.standardand poors .com/spf/pdf/fixedincome/Corporate_Ratings_2006.pdf. d Composite yields for AAA, AA, and A bonds can be found at http://finance.yahoo.com/bonds/composite_bond_rates. Representative yields for BBB, BB, B, and CCC bonds can be found using the bond screener at http://screen.yahoo .com/bonds.html. 3. Qualitative Factors. Included here would be such factors as sensitivity of the firm’s earnings to the strength of the economy, how it is affected by inflation, whether it is having or is likely to have labor problems, the extent of its international operations (including the stability of the countries in which it operates), potential environmental problems, potential antitrust problems, and so on. Today (2009), a critical factor is exposure to sub-prime loans, including the difficulty of determining the extent of this exposure owing to the complexity of the assets backed by such loans. Rating agencies review outstanding bonds on a periodic basis and re-rate if necessary. Columns (7) and (8) in Table 5-1 show the percentages of companies in each rating category that were downgraded or upgraded in 2008 by Fitch Ratings. The year 2008 was a difficult one, as more bonds were downgraded than upgraded. Over the long run, ratings agencies have done a reasonably good job of measuring the average credit risk of bonds and of changing ratings whenever there is a significant change in credit quality. However, it is important to understand that ratings do not adjust immediately to changes in credit quality, and in some cases there can be a considerable lag between a change in credit quality and a change in rating. For example, Enron’s bonds still carried an investment-grade rating on a Friday in December 2001, but the company declared bankruptcy two days later, on Sunday. Many other abrupt downgrades occurred in 2007 and 2008, leading to calls by Congress and the SEC for changes in rating agencies and the way they rate bonds. Clearly, improvements can be made, but there will always be occasions when completely unexpected information about a company is released, leading to a sudden change in its rating. Chapter 5: Bonds, Bond Valuation, and Interest Rates 199 THE GLOBAL ECONOMIC CRISIS Might the U.S. Treasury Bond Be Downgraded? The worsening recession that began at the end of 2007 led Congress to pass a huge economic stimulus package in early 2009. The combination of the stimulus package and the government’s bailouts of financial institutions is causing the U.S. government to increase its borrowing. Moody’s, the bond rating agency, projects that federal debt will rise from $5.8 trillion (41% of GDP) at the beginning of 2009 to $9 trillion (62% of GDP) by the end of 2010. This is a lot of money, even by Washington standards! Moody’s rates sovereign debt as well as corporate debt, and the United States has always ranked right at the top. However, Moody’s essentially put America on “credit watch” starting February 13, 2009. Moody’s predicted that the U.S. ranking will go from the strongest to near the bottom of the most developed nations’ rankings (Canada, Britain, France, Germany, and the Scandinavian countries). It even stated that the U.S.’s “triple A rating isn’t assured forever." A downgrade might cause lenders in China and elsewhere around the globe to sell many of their T-bonds. Massive selling pressure would cause T-bond interest rates to go up, which would increase the government’s cost of borrowing. The sellers would want to convert into their own currency the dollars they receive from selling T-bonds. This means they would “sell” dollars for their own currency, and this selling pressure would cause the value of the dollar to fall. If the Chinese and others didn’t want to hold dollar denominated debt, who would buy the extra $3.2 trillion of bonds the United States must sell to fund the stimulus package? A downgrading would also have a huge effect on the finance profession—we would no longer have a proxy for the riskless rate, which is an important element of financial theory and practice. Clearly, though, that would be a very small problem compared to the damage a downgrade would do to our economy. Bond Ratings and the Default Risk Premium Why are bond ratings so important? First, most bonds are purchased by institutional investors rather than individuals, and many institutions are restricted to investmentgrade securities. Thus, if a firm’s bonds fall below BBB, it will have a difficult time selling new bonds because many potential purchasers will not be allowed to buy them. Second, many bond covenants stipulate that the coupon rate on the bond automatically increases if the rating falls below a specified level. Third, because a bond’s rating is an indicator of its default risk, the rating has a direct, measurable influence on the bond’s yield. Column (9) of Table 5-1 shows that a AAA bond has a yield of 5.50% and that yields increase as the rating falls. In fact, an investor would earn 26.3% on a CCC bond if it didn’t default! A bond spread is the difference between a bond’s yield and the yield on some other security of the same maturity. Unless specified differently, the term “spread” generally means the difference between a bond’s yield and the yield on a Treasury bond of similar maturity. Figure 5-3 shows the spreads between an index of AAA bonds and a 10-year Treasury bond; it also shows spreads for an index of BBB bonds relative to the T-bond. Figure 5-3 illustrates three important points. First, the BAA spread always is greater than the AAA spread. This is because a BAA bond is riskier than an AAA bond, so BAA investors require extra compensation for their extra risk. The same is true for other ratings: Lower-rated bonds have higher yields. 200 Part 2: Fixed Income Securities FIGURE 5-3 Bond Spreads Spread (%) 7.00 6.00 BAA − T-bond 5.00 4.00 3.00 2.00 1.00 AAA − T-bond 2009-01 2008-07 2008-01 2007-07 2007-01 2006-07 2006-01 2005-07 2005-01 2004-07 2004-01 2003-07 2003-01 2002-07 2002-01 2001-07 2001-01 2000-07 2000-01 1999-07 1999-01 0.00 Note: All data are from the Federal Reserve Bank of St. Louis’s Economic Database, FRED: http://research.stlouisfed.org/fred2/. The spreads are defined as the yield on the risky bond (AAA or BAA) minus the yield on a 10-year Treasury bond. Second, the spreads are not constant over time. For example, look at the AAA spread. It was exceptionally low during the boom years of 2005–2007 but rose dramatically as the economy subsequently declined. Third, the difference between the BAA spread and the AAA spread isn’t constant over time. The two spreads were quite close to one another in early 2000 but were very far apart in early 2009. In other words, BAA investors didn’t require much extra return over that of an AAA bond to induce them to take on that extra risk for most of the decade, but now (2009) they are requiring a very large risk premium. Not only do spreads vary with the rating of the security, they also usually increase as maturity increases. This should make sense. If a bond matures soon, investors are able to forecast the company’s performance fairly well. But if a bond has a long time until it matures, investors have a difficult time forecasting the likelihood that the company will fall into financial distress. This extra uncertainty creates additional risk, so investors demand a higher required return. Self-Test Differentiate between mortgage bonds and debentures. Name the major rating agencies, and list some factors that affect bond ratings. What is a bond spread? How do bond ratings affect the default risk premium? A 10-year T-bond has a yield of 6%. A 10-year corporate bond with a rating of AA has a yield of 7.5%. If the corporate bond has excellent liquidity, what is an estimate of the corporate bond’s default risk premium? (1.5%) Chapter 5: Bonds, Bond Valuation, and Interest Rates 201 THE GLOBAL ECONOMIC CRISIS Are Investors Rational? The figure in this box shows the yield on junk bonds during the recent past. Observe that the yields were less than 10% from mid-2003 through the end of 2008. During this period, 10-year T-bonds yielded an average of about 4.4%. Thus, the spread on junk bonds over Treasuries was only about 5.4% during these boom years, which is exceptionally low by historic standards. In other words, investors had a voracious appetite for risk and simply didn’t require much extra return to induce them to buy very risky securities—investors almost ignored risk during the boom years. But as the economy began to deteriorate in 2008, investors reversed course and became extremely risk averse, with junk-bond yields climbing as high as 25% and the spread exceeding 21%. Such drastic changes in investors’ risk aversion are hard to reconcile with careful, deliberate, and rational behavior! 3/27/2009 30% 25% 20% 15% 10% 03 04 05 06 07 08 09 Source: Financial Industry Regulatory Authority, March 27, 2009, High Yield Index (NBBH) Yields: see http://cxa. marketwatch.com/finra/BondCenter/ActiveUSCorpBond. aspx. Copyright © 2009 FINRA. All rights reserved. FINRA is a registered trademark of the Financial Industry Regulatory Authority, Inc. Reprinted with permission from FINRA. 5.12 THE LIQUIDITY PREMIUM (LP) A “liquid” asset can be converted to cash quickly and at a “fair market value.” Financial assets are generally more liquid than real assets. Because liquidity is important, investors include liquidity premiums (LPs) when market rates of securities are established. Although it is difficult to measure liquidity premiums accurately, a differential of at least 2 percentage points (and perhaps up to 4 or 5 percentage points) exists between the least liquid and the most liquid financial assets of similar default risk and maturity. Corporate bonds issued by small companies are traded less frequently than those issued by large companies, so small-company bonds tend to have a higher liquidity premium. As discussed in Chapter 1, liquidity in the market for mortgage-backed securities evaporated in 2008 and early 2009. The few transactions that occurred were priced such that the yields on these MBS were extremely high, which was partially due to a much higher liquidity premium caused by the extremely low liquidity of MBS. Self-Test Which bond usually will have a higher liquidity premium: one issued by a large company or one issued by a small company? 5.13 THE MATURITY RISK PREMIUM (MRP) All bonds, even Treasury bonds, are exposed to two additional sources of risk: interest rate risk and reinvestment risk. The net effect of these two sources of risk upon a bond’s yield is called the maturity risk premium, MRP. The following sections explain how interest rate risk and reinvestment risk affect a bond’s yield. Interest Rate Risk Interest rates go up and down over time, and an increase in interest rates leads to a decline in the value of outstanding bonds. This risk of a decline in bond values due to 202 Part 2: Fixed Income Securities resource For more on bond risk, including duration analysis, see Web Extension 5C on the textbook’s Web site. rising interest rates is called interest rate risk. To illustrate, suppose you bought some 10% MicroDrive bonds at a price of $1,000 and then interest rates rose in the following year to 15%. As we saw earlier, the price of the bonds would fall to $713.78, so you would have a loss of $286.22 per bond.18 Interest rates can and do rise, and rising rates cause a loss of value for bondholders. Thus, bond investors are exposed to risk from changing interest rates. One’s exposure to interest rate risk is higher on bonds with long maturities than on those maturing in the near future.19 This point can be demonstrated by showing how the value of a 1-year bond with a 10% annual coupon fluctuates with changes in rd and then comparing these changes with those on a 25-year bond. The 1-year bond’s value for rd = 5% is shown below: Inputs: 1 5 N I/YR Output (Bond Value): PV 100 1000 PMT FV –1047.62 Using either a calculator or a spreadsheet, you could calculate the bond values for a 1-year and a 25-year bond at several current market interest rates; these results are plotted in Figure 5-4. Note how much more sensitive the price of the 25-year bond is to changes in interest rates. At a 10% interest rate, both the 25-year and the 1-year bonds are valued at $1,000. When rates rise to 15%, the 25-year bond falls to $676.79 but the 1-year bond falls only to $956.52. For bonds with similar coupons, this differential sensitivity to changes in interest rates always holds true: The longer the maturity of the bond, the more its price changes in response to a given change in interest rates. Thus, even if the risk of default on two bonds is exactly the same, the one with the longer maturity is exposed to more risk from a rise in interest rates. The explanation for this difference in interest rate risk is simple. Suppose you bought a 25-year bond that yielded 10%, or $100 a year. Now suppose interest rates on bonds of comparable risk rose to 15%. You would be stuck with only $100 of interest for the next 25 years. On the other hand, had you bought a 1-year bond, you would have a low return for only 1 year. At the end of the year, you would get your $1,000 back, and you could then reinvest it and receive a 15% return ($150) for the next year. Thus, interest rate risk reflects the length of time one is committed to a given investment. 18 You would have an accounting (and tax) loss only if you sold the bond; if you held it to maturity, you would not have such a loss. However, even if you did not sell, you would still have suffered a real economic loss in an opportunity cost sense because you would have lost the opportunity to invest at 15% and would be stuck with a 10% bond in a 15% market. In an economic sense, “paper losses” are just as bad as realized accounting losses. Actually, a bond’s maturity and coupon rate each affect interest rate risk. Low coupons mean that most of the bond’s return will come from repayment of principal, whereas on a high-coupon bond with the same maturity, more of the cash flows will come in during the early years because of the relatively large coupon payments. A measurement called “duration,” which finds the average number of years that the bond’s PV of cash flows remains outstanding, has been developed to combine maturity and coupons. A zero coupon bond, which has no interest payments and whose payments all come at maturity, has a duration equal to the bond’s maturity. Coupon bonds all have durations that are shorter than maturity, and the higher the coupon rate, the shorter the duration. Bonds with longer duration are exposed to more interest rate risk. Excel’s DURATION function provides an easy way to calculate a bond’s duration. See Web Extension 5C and Ch05 Tool Kit.xls for more on duration. 19 Chapter 5: Bonds, Bond Valuation, and Interest Rates FIGURE 5-4 203 Value of Long- and Short-Term 10% Annual Coupon Bonds at Different Market Interest Rates Bond Value ($) 1,800 resource 1,600 25-Year Bond 1,400 See Ch05 Tool Kit.xls 1,200 1-Year Bond 1,000 800 600 400 200 0 0% 5% 10% 15% 20% 25% Interest Rate, rd Reinvestment Rate Risk As we saw in the preceding section, an increase in interest rates will hurt bondholders because it will lead to a decline in the value of a bond portfolio. But can a decrease in interest rates also hurt bondholders? The answer is “yes,” because if interest rates fall then a bondholder may suffer a reduction in his or her income. For example, consider a retiree who has a portfolio of bonds and lives off the income they produce. The bonds, on average, have a coupon rate of 10%. Now suppose that interest rates decline to 5%. The short-term bonds will mature, and when they do, they will have to be replaced with lower-yielding bonds. In addition, many of the remaining longterm bonds may be called, and as calls occur, the bondholder will have to replace 10% bonds with 5% bonds. Thus, our retiree will suffer a reduction of income. The risk of an income decline due to a drop in interest rates is called reinvestment rate risk. Reinvestment rate risk is obviously high on callable bonds. It is also high on short-maturity bonds, because the shorter the maturity of a bond, the fewer the years when the relatively high old interest rate will be earned and the sooner the funds will have to be reinvested at the new low rate. Thus, retirees whose primary holdings are short-term securities, such as bank CDs and short-term bonds, are hurt badly by a decline in rates, but holders of long-term bonds continue to enjoy their old high rates. Comparing Interest Rate Risk and Reinvestment Rate Risk: The Maturity Risk Premium Note that interest rate risk relates to the value of the bonds in a portfolio, while reinvestment rate risk relates to the income the portfolio produces. If you hold long-term bonds then you will face a lot of interest rate risk, because the value of your bonds will decline if interest rates rise; but you will not face much reinvestment rate risk, so your income will be stable. On the other hand, if you hold short-term bonds, 204 Part 2: Fixed Income Securities you will not be exposed to much interest rate risk because the value of your portfolio will be stable, but you will be exposed to considerable reinvestment rate risk because your income will fluctuate with changes in interest rates. We see, then, that no fixedrate bond can be considered totally riskless—even most Treasury bonds are exposed to both interest rate risk and reinvestment rate risk.20 Bond prices reflect the trading activities of the marginal investors, defined as those who trade often enough and with large enough sums to determine bond prices. Although one particular investor might be more averse to reinvestment risk than to interest rate risk, the data suggest that the marginal investor is more averse to interest rate risk than to reinvestment risk. To induce the marginal investor to take on interest rate risk, long-term bonds must have a higher expected rate of return than short-term bonds. Holding all else equal, this additional return is the maturity risk premium (MRP). Self-Test Differentiate between interest rate risk and reinvestment rate risk. To which type of risk are holders of long-term bonds more exposed? Short-term bondholders? Assume that the real risk-free rate is r* = 3% and that the average expected inflation rate is 2.5% for the foreseeable future. The DRP and LP for a bond are each 1%, and the applicable MRP is 2%. What is the bond’s yield? (9.5%) 5.14 THE TERM STRUCTURE OF INTEREST RATES The term structure of interest rates describes the relationship between long-term and short-term rates. The term structure is important both to corporate treasurers deciding whether to borrow by issuing long-term or short-term debt and to investors who are deciding whether to buy long-term or short-term bonds. Interest rates for bonds with different maturities can be found in a variety of publications, including The Wall Street Journal and the Federal Reserve Bulletin, as well as on a number of Web sites, including Bloomberg, Yahoo!, CNN Financial, and the Federal Reserve Board. Using interest rate data from these sources, we can determine the term structure at any given point in time. For example, Figure 5-5 presents interest rates for different maturities on three different dates. The set of data for a given date, when plotted on a graph such as Figure 5-5, is called the yield curve for that date. As the figure shows, the yield curve changes both in position and in slope over time. In March 1980, all rates were quite high because high inflation was expected. However, the rate of inflation was expected to decline, so the inflation premium (IP) was larger for short-term bonds than for long-term bonds. This caused shortterm yields to be higher than long-term yields, resulting in a downward-sloping yield curve. By February 2000, inflation had indeed declined and thus all rates were lower. The yield curve had become humped—medium-term rates were higher than either short- or long-term rates. By March 2009, all rates had fallen below the 2000 levels. Because short-term rates had dropped below long-term rates, the yield curve was upward sloping. Historically, long-term rates are generally higher than short-term rates owing to the maturity risk premium, so the yield curve usually slopes upward. For this reason, people often call an upward-sloping yield curve a “normal” yield curve and a yield Although indexed Treasury bonds are almost riskless, they pay a relatively low real rate. Note also that risks have not disappeared—they have simply been transferred from bondholders to taxpayers. 20 Chapter 5: Bonds, Bond Valuation, and Interest Rates FIGURE 5-5 205 U.S. Treasury Bond Interest Rates on Different Dates Interest Rate (%) 16% 14% Yield Curve for March 1980 12% 10% 8% Yield Curve for February 2000 6% 4% Yield Curve for March 2009 2% 0% resource For a discussion of the expectations theory, see Web Extension 5D on the textbook’s Web site. Self-Test 0 5 10 15 20 25 30 Years to Maturity curve that slopes downward an inverted, or “abnormal,” curve. Thus, in Figure 5-5 the yield curve for March 1980 was inverted whereas the yield curve in March 2009 was normal. As stated above, the February 2000 curve was humped. A few academics and practitioners contend that large bond traders who buy and sell securities of different maturities each day dominate the market. According to this view, a bond trader is just as willing to buy a 30-year bond to pick up a shortterm profit as to buy a 3-month security. Strict proponents of this view argue that the shape of the yield curve is therefore determined only by market expectations about future interest rates, a position that is called the pure expectations theory, or sometimes just the expectations theory. If this were true, then the maturity risk premium (MRP) would be zero and long-term interest rates would simply be a weighted average of current and expected future short-term interest rates. See Web Extension 5D for a more detailed discussion of the expectations theory. What is a yield curve, and what information would you need to draw this curve? Distinguish among the shapes of a “normal” yield curve, an “abnormal” curve, and a “humped” curve. If the interest rates on 1-, 5-, 20-, and 30-year bonds are (respectively) 4%, 5%, 6%, and 7%, then how would you describe the yield curve? How would you describe it if the rates were reversed? 5.15 FINANCING WITH JUNK BONDS Recall that bonds rated less than BBB are noninvestment-grade debt, also called junk bonds or high-yield debt. There are two ways that a bond can become a junk bond. First, the bond might have been investment-grade debt when it was issued but its rating declined because the issuing corporation had fallen on hard times. Such bonds are called “fallen angels,” and there are many such bonds as we write this in 2009. 206 Part 2: Fixed Income Securities Some bonds are junk bonds at the time they are issued, but this was not always true. Prior to the 1980s, fixed-income investors such as pension funds and insurance companies were generally unwilling to buy risky bonds, so it was almost impossible for risky companies to raise capital in the public bond markets. Then, in the late 1970s, Michael Milken of the investment banking firm Drexel Burnham Lambert, relying on historical studies that showed risky bonds yielded more than enough to compensate for their risk, convinced institutional investors that junk bond yields were worth their risk. Thus was born the junk bond market. In the 1980s, large investors like T. Boone Pickens and Henry Kravis thought that certain old-line, established companies were run inefficiently and were financed too conservatively. These corporate raiders were able to put in some of their own money, borrow the rest via junk bonds, and take over the target company, usually taking the company private. The fact that interest on the bonds was tax deductible, combined with the much higher debt ratios of the restructured firms, also increased after-tax cash flows and helped make the deals feasible. Because these deal used lots of debt, they were called leveraged buyouts (LBOs). In recent years, private equity firms have conducted transactions very similar to the LBOs of the 1980s, taking advantage of historically low junk-bond rates to help finance their purchases. For example, in 2007 the private equity firm Kohlberg Kravis Roberts and Company (KKR) took the discount retailer Dollar General private in a $6.9 billion deal. As part of the transaction, Dollar General issued $1.9 billion in junk bonds. So KKR financed approximately 73% of the deal with its own cash (coming from its own equity and from money it had borrowed on its own account) and about 27% of the deal with money that Dollar General raised. Dollar General’s sales have been soaring during the early part of the current recession, making KKR’s purchase look like a winner. Self-Test What are junk bonds? 5.16 BANKRUPTCY AND REORGANIZATION When a business becomes insolvent, it does not have enough cash to meet its interest and principal payments. A decision must then be made whether to dissolve the firm through liquidation or to permit it to reorganize and thus stay alive. These issues are addressed in Chapters 7 and 11 of the federal bankruptcy statutes, and the final decision is made by a federal bankruptcy court judge. The decision to force a firm to liquidate versus permit it to reorganize depends on whether the value of the reorganized firm is likely to be greater than the value of the firm’s assets if they are sold off piecemeal. In a reorganization, the firm’s creditors negotiate with management on the terms of a potential reorganization. The reorganization plan may call for a restructuring of the firm’s debt, in which case the interest rate may be reduced, the term to maturity lengthened, or some of the debt may be exchanged for equity. The point of the restructuring is to reduce the financial charges to a level that the firm’s cash flows can support. Of course, the common stockholders also have to give up something: they often see their position diluted as a result of additional shares being given to debtholders in exchange for accepting a reduced amount of debt principal and interest. In fact, the original common stockholders often end up with nothing. A trustee may be appointed by the court to oversee the reorganization, but generally the existing management is allowed to retain control. Liquidation occurs if the company is deemed to be too far gone to be saved—if it is worth more dead than alive. If the bankruptcy court orders liquidation, then assets are sold off and the cash obtained is distributed as specified in Chapter 7 of Chapter 5: Bonds, Bond Valuation, and Interest Rates 207 the Bankruptcy Act. Here is the priority of claims: (1) past-due property taxes; (2) secured creditors who are entitled to the proceeds from the sale of collateral; (3) the trustee’s costs of administering and operating the bankrupt firm; (4) expenses incurred after bankruptcy was filed; (5) wages due workers, up to a limit of $2,000 per worker; (6) claims for unpaid contributions to employee benefit plans (with wages and claims not to exceed $2,000 per worker); (7) unsecured claims for customer deposits up to $900 per customer; (8) federal, state, and local taxes due; (9) unfunded pension plan liabilities (although some limitations exist); (10) general unsecured creditors; (11) preferred stockholders (up to the par value of their stock); and (12) common stockholders (although usually nothing is left). The key points for you to know are: (1) the federal bankruptcy statutes govern both reorganization and liquidation, (2) bankruptcies occur frequently, and (3) a priority of the specified claims must be followed when distributing the assets of a liquidated firm. Self-Test Differentiate between a Chapter 7 liquidation and a Chapter 11 reorganization. List the priority of claims for the distribution of a liquidated firm’s assets. Summary This chapter described the different types of bonds that governments and corporations issue, explained how bond prices are established, and discussed how investors estimate the rates of return they can expect to earn. The rate of return required by debtholders is the company’s pre-tax cost of debt, and this rate depends on the risk that investors face when they buy bonds. • • • • • • A bond is a long-term promissory note issued by a business or governmental unit. The issuer receives money in exchange for promising to make interest payments and to repay the principal on a specified future date. Some special types of long-term financing include zero coupon bonds, which pay no annual interest but are issued at a discount; see Web Extension 5A for more on zero coupon bonds. Other types are floating-rate debt, whose interest payments fluctuate with changes in the general level of interest rates; and junk bonds, which are high-risk, high-yield instruments issued by firms that use a great deal of financial leverage. A call provision gives the issuing corporation the right to redeem the bonds prior to maturity under specified terms, usually at a price greater than the maturity value (the difference is a call premium). A firm will typically call a bond if interest rates fall substantially below the coupon rate. A sinking fund is a provision that requires the corporation to retire a portion of the bond issue each year. The purpose of the sinking fund is to provide for the orderly retirement of the issue. A sinking fund typically requires no call premium. The value of a bond is found as the present value of an annuity (the interest payments) plus the present value of a lump sum (the principal). The bond is evaluated at the appropriate periodic interest rate over the number of periods for which interest payments are made. The equation used to find the value of an annual coupon bond is N VB ¼ • M ∑ ð1INT þ þ r Þ ð1 þ r Þ t¼1 d t d N An adjustment to the formula must be made if the bond pays interest semiannually: divide INT and rd by 2, and multiply N by 2. 208 Part 2: Fixed Income Securities • The expected rate of return on a bond held to maturity is defined as the bond’s yield to maturity (YTM): N Bond price ¼ • • M ∑ ð1 þINT þ YTMÞ ð1 þ YTMÞ t¼1 t N The expected rate of return on a callable bond held to its call date is defined as the yield to call (YTC). The nominal (or quoted) interest rate on a debt security, rd, is composed of the real risk-free rate, r*, plus premiums that reflect inflation (IP), default risk (DRP), liquidity (LP), and maturity risk (MRP): rd ¼ r þ IP þ DRP þ LP þ MRP • • • • • • • • • • The risk-free rate of interest, rRF, is defined as the real risk-free rate, r*, plus an inflation premium, IP: rRF = r* + IP. Treasury Inflation-Protected Securities (TIPS) are U.S. Treasury bonds that have no inflation risk. See Web Extension 5B for more discussion of TIPS. The longer the maturity of a bond, the more its price will change in response to a given change in interest rates; this is called interest rate risk. However, bonds with short maturities expose investors to high reinvestment rate risk, which is the risk that income from a bond portfolio will decline because cash flows received from bonds will be rolled over at lower interest rates. Duration is a measure of interest rate risk. See Web Extension 5C for a discussion of duration. Corporate and municipal bonds have default risk. If an issuer defaults, investors receive less than the promised return on the bond. Therefore, investors should evaluate a bond’s default risk before making a purchase. Bonds are assigned ratings that reflect the probability of their going into default. The highest rating is AAA, and they go down to D. The higher a bond’s rating, the lower its risk and therefore its interest rate. The relationship between the yields on securities and the securities’ maturities is known as the term structure of interest rates, and the yield curve is a graph of this relationship. The shape of the yield curve depends on two key factors: (1) expectations about future inflation and (2) perceptions about the relative risk of securities with different maturities. The yield curve is normally upward sloping—this is called a normal yield curve. However, the curve can slope downward (an inverted yield curve) if the inflation rate is expected to decline. The yield curve also can be humped, which means that interest rates on medium-term maturities are higher than rates on both short- and long-term maturities. The expectations theory states that yields on long-term bonds reflect expected future interest rates. Web Extension 5D discusses this theory. Questions (5–1) Define each of the following terms: a. Bond; Treasury bond; corporate bond; municipal bond; foreign bond b. Par value; maturity date; coupon payment; coupon interest rate c. Floating-rate bond; zero coupon bond; original issue discount bond (OID) Chapter 5: Bonds, Bond Valuation, and Interest Rates d. e. f. g. h. i. j. k. l. m. n. 209 Call provision; redeemable bond; sinking fund Convertible bond; warrant; income bond; indexed, or purchasing power, bond Premium bond; discount bond Current yield (on a bond); yield to maturity (YTM); yield to call (YTC) Indentures; mortgage bond; debenture; subordinated debenture Development bond; municipal bond insurance; junk bond; investment-grade bond Real risk-free rate of interest, r*; nominal risk-free rate of interest, rRF Inflation premium (IP); default risk premium (DRP); liquidity; liquidity premium (LP) Interest rate risk; maturity risk premium (MRP); reinvestment rate risk Term structure of interest rates; yield curve “Normal” yield curve; inverted (“abnormal”) yield curve (5–2) “Short-term interest rates are more volatile than long-term interest rates, so shortterm bond prices are more sensitive to interest rate changes than are long-term bond prices.” Is this statement true or false? Explain. (5–3) The rate of return you would get if you bought a bond and held it to its maturity date is called the bond’s yield to maturity. If interest rates in the economy rise after a bond has been issued, what will happen to the bond’s price and to its YTM? Does the length of time to maturity affect the extent to which a given change in interest rates will affect the bond’s price? (5–4) If you buy a callable bond and interest rates decline, will the value of your bond rise by as much as it would have risen if the bond had not been callable? Explain. (5–5) A sinking fund can be set up in one of two ways. Discuss the advantages and disadvantages of each procedure from the viewpoint of both the firm and its bondholders. Self-Test Problem (ST–1) Bond Valuation Solution Appears in Appendix A The Pennington Corporation issued a new series of bonds on January 1, 1987. The bonds were sold at par ($1,000), had a 12% coupon, and matured in 30 years on December 31, 2016. Coupon payments are made semiannually (on June 30 and December 31). a. What was the YTM on the date the bonds were issued? b. What was the price of the bonds on January 1, 1992 (5 years later), assuming that interest rates had fallen to 10%? c. Find the current yield, capital gains yield, and total yield on January 1, 1992, given the price as determined in part b. d. On July 1, 2010 (6.5 years before maturity), Pennington’s bonds sold for $916.42. What are the YTM, the current yield, and the capital gains yield for that date? e. Now assume that you plan to purchase an outstanding Pennington bond on March 1, 2010, when the going rate of interest given its risk is 15.5%. How large a check must you write to complete the transaction? (Hint: Don’t forget the accrued interest.) 210 Part 2: Fixed Income Securities Problems Answers Appear in Appendix B EASY PROBLEMS 1–6 (5–1) Bond Valuation with Annual Payments (5–2) Yield to Maturity for Annual Payments (5–3) Current Yield for Annual Payments (5–4) Determinant of Interest Rates (5–5) Default Risk Premium (5–6) Maturity Risk Premium Jackson Corporation’s bonds have 12 years remaining to maturity. Interest is paid annually, the bonds have a $1,000 par value, and the coupon interest rate is 8%. The bonds have a yield to maturity of 9%. What is the current market price of these bonds? Wilson Wonders’s bonds have 12 years remaining to maturity. Interest is paid annually, the bonds have a $1,000 par value, and the coupon interest rate is 10%. The bonds sell at a price of $850. What is their yield to maturity? Heath Foods’s bonds have 7 years remaining to maturity. The bonds have a face value of $1,000 and a yield to maturity of 8%. They pay interest annually and have a 9% coupon rate. What is their current yield? The real risk-free rate of interest is 4%. Inflation is expected to be 2% this year and 4% during the next 2 years. Assume that the maturity risk premium is zero. What is the yield on 2-year Treasury securities? What is the yield on 3-year Treasury securities? A Treasury bond that matures in 10 years has a yield of 6%. A 10-year corporate bond has a yield of 9%. Assume that the liquidity premium on the corporate bond is 0.5%. What is the default risk premium on the corporate bond? The real risk-free rate is 3%, and inflation is expected to be 3% for the next 2 years. A 2-year Treasury security yields 6.3%. What is the maturity risk premium for the 2-year security? INTERMEDIATE PROBLEMS 7–20 (5–7) Bond Valuation with Semiannual Payments (5–8) Yield to Maturity and Call with Semiannual Payments (5–9) Bond Valuation and Interest Rate Risk Renfro Rentals has issued bonds that have a 10% coupon rate, payable semiannually. The bonds mature in 8 years, have a face value of $1,000, and a yield to maturity of 8.5%. What is the price of the bonds? Thatcher Corporation’s bonds will mature in 10 years. The bonds have a face value of $1,000 and an 8% coupon rate, paid semiannually. The price of the bonds is $1,100. The bonds are callable in 5 years at a call price of $1,050. What is their yield to maturity? What is their yield to call? The Garraty Company has two bond issues outstanding. Both bonds pay $100 annual interest plus $1,000 at maturity. Bond L has a maturity of 15 years, and Bond S has a maturity of 1 year. a. What will be the value of each of these bonds when the going rate of interest is (1) 5%, (2) 8%, and (3) 12%? Assume that there is only one more interest payment to be made on Bond S. b. Why does the longer-term (15-year) bond fluctuate more when interest rates change than does the shorter-term bond (1 year)? (5–10) Yield to Maturity and Required Returns The Brownstone Corporation’s bonds have 5 years remaining to maturity. Interest is paid annually, the bonds have a $1,000 par value, and the coupon interest rate is 9%. Chapter 5: Bonds, Bond Valuation, and Interest Rates 211 a. What is the yield to maturity at a current market price of (1) $829 or (2) $1,104? b. Would you pay $829 for one of these bonds if you thought that the appropriate rate of interest was 12%—that is, if rd = 12%? Explain your answer. (5–11) Yield to Call and Realized Rates of Return (5–12) Bond Yields and Rates of Return Seven years ago, Goodwynn & Wolf Incorporated sold a 20-year bond issue with a 14% annual coupon rate and a 9% call premium. Today, G&W called the bonds. The bonds originally were sold at their face value of $1,000. Compute the realized rate of return for investors who purchased the bonds when they were issued and who surrender them today in exchange for the call price. A 10-year, 12% semiannual coupon bond with a par value of $1,000 may be called in 4 years at a call price of $1,060. The bond sells for $1,100. (Assume that the bond has just been issued.) a. b. c. d. (5–13) Yield to Maturity and Current Yield (5–14) Current Yield with Semiannual Payments (5–15) Yield to Call, Yield to Maturity, and Market Rates (5–16) Interest Rate Sensitivity What What What What is is is is the the the the bond’s bond’s bond’s bond’s yield to maturity? current yield? capital gain or loss yield? yield to call? You just purchased a bond that matures in 5 years. The bond has a face value of $1,000 and has an 8% annual coupon. The bond has a current yield of 8.21%. What is the bond’s yield to maturity? A bond that matures in 7 years sells for $1,020. The bond has a face value of $1,000 and a yield to maturity of 10.5883%. The bond pays coupons semiannually. What is the bond’s current yield? Absalom Motors’s 14% coupon rate, semiannual payment, $1,000 par value bonds that mature in 30 years are callable 5 years from now at a price of $1,050. The bonds sell at a price of $1,353.54, and the yield curve is flat. Assuming that interest rates in the economy are expected to remain at their current level, what is the best estimate of the nominal interest rate on new bonds? A bond trader purchased each of the following bonds at a yield to maturity of 8%. Immediately after she purchased the bonds, interest rates fell to 7%. What is the percentage change in the price of each bond after the decline in interest rates? Fill in the following table: Pric e @ 8% Price @ 7% Per centag e C han ge 10-year, 10% annual coupon 10-year zero 5-year zero 30-year zero $100 perpetuity (5–17) Bond Value as Maturity Approaches An investor has two bonds in his portfolio. Each bond matures in 4 years, has a face value of $1,000, and has a yield to maturity equal to 9.6%. One bond, Bond C, pays an annual coupon of 10%; the other bond, Bond Z, is a zero coupon bond. Assuming that the yield to maturity of each bond remains at 9.6% over the next 4 years, what will be the price of each of the bonds at the following time periods? Fill in the following table: 212 Part 2: Fixed Income Securities t P rice of Bon d C Price of Bo nd Z 0 1 2 3 4 (5–18) Determinants of Interest Rates (5–19) Maturity Risk Premiums (5–20) Inflation Risk Premiums The real risk-free rate is 2%. Inflation is expected to be 3% this year, 4% next year, and then 3.5% thereafter. The maturity risk premium is estimated to be 0.0005 × (t − 1), where t = number of years to maturity. What is the nominal interest rate on a 7-year Treasury security? Assume that the real risk-free rate, r*, is 3% and that inflation is expected to be 8% in Year 1, 5% in Year 2, and 4% thereafter. Assume also that all Treasury securities are highly liquid and free of default risk. If 2-year and 5-year Treasury notes both yield 10%, what is the difference in the maturity risk premiums (MRPs) on the two notes; that is, what is MRP5 minus MRP2? Because of a recession, the inflation rate expected for the coming year is only 3%. However, the inflation rate in Year 2 and thereafter is expected to be constant at some level above 3%. Assume that the real risk-free rate is r* = 2% for all maturities and that there are no maturity premiums. If 3-year Treasury notes yield 2 percentage points more than 1-year notes, what inflation rate is expected after Year 1? CHALLENGING PROBLEMS 21–23 (5–21) Bond Valuation and Changes in Maturity and Required Returns (5–22) Yield to Maturity and Yield to Call Suppose Hillard Manufacturing sold an issue of bonds with a 10-year maturity, a $1,000 par value, a 10% coupon rate, and semiannual interest payments. a. Two years after the bonds were issued, the going rate of interest on bonds such as these fell to 6%. At what price would the bonds sell? b. Suppose that, 2 years after the initial offering, the going interest rate had risen to 12%. At what price would the bonds sell? c. Suppose, as in part a, that interest rates fell to 6% 2 years after the issue date. Suppose further that the interest rate remained at 6% for the next 8 years. What would happen to the price of the bonds over time? Arnot International’s bonds have a current market price of $1,200. The bonds have an 11% annual coupon payment, a $1,000 face value, and 10 years left until maturity. The bonds may be called in 5 years at 109% of face value (call price = $1,090). a. b. c. d. What is the yield to maturity? What is the yield to call if they are called in 5 years? Which yield might investors expect to earn on these bonds, and why? The bond’s indenture indicates that the call provision gives the firm the right to call them at the end of each year beginning in Year 5. In Year 5, they may be called at 109% of face value, but in each of the next 4 years the call percentage will decline by 1 percentage point. Thus, in Year 6 they may be called at 108% of face value, in Year 7 they may be called at 107% of face value, and so on. If the yield curve is horizontal and interest rates remain at their current level, when is the latest that investors might expect the firm to call the bonds? Chapter 5: Bonds, Bond Valuation, and Interest Rates (5–23) Determinants of Interest Rates 213 Suppose you and most other investors expect the inflation rate to be 7% next year, to fall to 5% during the following year, and then to remain at a rate of 3% thereafter. Assume that the real risk-free rate, r*, will remain at 2% and that maturity risk premiums on Treasury securities rise from zero on very short-term securities (those that mature in a few days) to a level of 0.2 percentage points for 1-year securities. Furthermore, maturity risk premiums increase 0.2 percentage points for each year to maturity, up to a limit of 1.0 percentage point on 5-year or longer-term T-notes and T-bonds. a. Calculate the interest rate on 1-, 2-, 3-, 4-, 5-, 10-, and 20-year Treasury securities, and plot the yield curve. b. Now suppose ExxonMobil’s bonds, rated AAA, have the same maturities as the Treasury bonds. As an approximation, plot an ExxonMobil yield curve on the same graph with the Treasury bond yield curve. (Hint: Think about the default risk premium on ExxonMobil’s long-term versus its short-term bonds.) c. Now plot the approximate yield curve of Long Island Lighting Company, a risky nuclear utility. SPREADSHEET PROBLEM (5–24) Build a Model: Bond Valuation resource Start with the partial model in the file Ch05 P24 Build a Model.xls on the textbook’s Web site. A 20-year, 8% semiannual coupon bond with a par value of $1,000 may be called in 5 years at a call price of $1,040. The bond sells for $1,100. (Assume that the bond has just been issued.) a. b. c. d. e. What is the bond’s yield to maturity? What is the bond’s current yield? What is the bond’s capital gain or loss yield? What is the bond’s yield to call? How would the price of the bond be affected by a change in the going market interest rate? (Hint: Conduct a sensitivity analysis of price to changes in the going market interest rate for the bond. Assume that the bond will be called if and only if the going rate of interest falls below the coupon rate. This is an oversimplification, but assume it anyway for purposes of this problem.) f. Now assume the date is October 25, 2010. Assume further that a 12%, 10-year bond was issued on July 1, 2010, pays interest semiannually (on January 1 and July 1), and sells for $1,100. Use your spreadsheet to find the bond’s yield. Mini Case Sam Strother and Shawna Tibbs are vice presidents of Mutual of Seattle Insurance Company and co-directors of the company’s pension fund management division. An important new client, the North-Western Municipal Alliance, has requested that Mutual of Seattle present an investment seminar to the mayors of the represented cities, and Strother and Tibbs, who will make the actual presentation, have asked you to help them by answering the following questions a. What are the key features of a bond? b. What are call provisions and sinking fund provisions? Do these provisions make bonds more or less risky? 214 Part 2: Fixed Income Securities c. How does one determine the value of any asset whose value is based on expected future cash flows? d. How is the value of a bond determined? What is the value of a 10-year, $1,000 par value bond with a 10% annual coupon if its required rate of return is 10%? e. (1) What would be the value of the bond described in part d if, just after it had been issued, the expected inflation rate rose by 3 percentage points, causing investors to require a 13% return? Would we now have a discount or a premium bond? (2) What would happen to the bond’s value if inflation fell and rd declined to 7%? Would we now have a premium or a discount bond? (3) What would happen to the value of the 10-year bond over time if the required rate of return remained at 13%? If it remained at 7%? (Hint: With a financial calculator, enter PMT, I/YR, FV, and N, and then change N to see what happens to the PV as the bond approaches maturity.) f. (1) What is the yield to maturity on a 10-year, 9% annual coupon, $1,000 par value bond that sells for $887.00? That sells for $1,134.20? What does the fact that a bond sells at a discount or at a premium tell you about the relationship between rd and the bond’s coupon rate? (2) What are the total return, the current yield, and the capital gains yield for the discount bond? (Assume the bond is held to maturity and the company does not default on the bond.) g. How does the equation for valuing a bond change if semiannual payments are made? Find the value of a 10-year, semiannual payment, 10% coupon bond if the nominal rd = 13%. h. Suppose a 10-year, 10% semiannual coupon bond with a par value of $1,000 is